Download Neuroanatomy (Duane E. Haines)

Document related concepts

Drosophila embryogenesis wikipedia , lookup

Primate basal ganglia system wikipedia , lookup

Cell nucleus wikipedia , lookup

Anatomical terms of location wikipedia , lookup

Transcript
Duane E. Haines
Neuroanatomy
An Atlas of
Structures, Sections,
and Systems.
SIXTH EDITION
Contents
Preface to Sixth Edition . . . . . . . . . . . . . . . . . . . . . . . . . . . . . . . . . . . . . . . . . . . . . . . . . . . . . . . . . . . . . . . . . . . . . . . . . . . .
Preface to the First Edition . . . . . . . . . . . . . . . . . . . . . . . . . . . . . . . . . . . . . . . . . . . . . . . . . . . . . . . . . . . . . . . . . . . . . . . . . .
Acknowledgments . . . . . . . . . . . . . . . . . . . . . . . . . . . . . . . . . . . . . . . . . . . . . . . . . . . . . . . . . . . . . . . . . . . . . . . . . . . . . . .
Chapter 1
Introduction and Reader’s Guide . . . . . . . . . . . . . . . . . . . . . . . . . . . . . . . . . . . . . . . . . . . . . .
Including Rationale for Labels and Abbreviations . . . . . . . . . . . . . . . . . . . . . . . . . . . . .
Chapter 2
External Morphology of the Central Nervous System . . . . . . . . . . . . . . . . . . . . . . . . . . .
The Spinal Cord: Gross Views and Vasculature . . . . . . . . . . . . . . . . . . . . . . . . . . . . . .
The Brain: Lobes, Principle Brodmann Areas, Sensory-Motor Somatotopy . . . . . . . . . . .
The Brain: Gross Views, Vasculature, and MRI . . . . . . . . . . . . . . . . . . . . . . . . . . . . . .
The Cranial Nerves in MRI . . . . . . . . . . . . . . . . . . . . . . . . . . . . . . . . . . . . . . . . . . . .
The Insula: Gross View and MRI . . . . . . . . . . . . . . . . . . . . . . . . . . . . . . . . . . . . . . . .
The Meninges, Cisterns, and Meningeal and Cisternal Hemorrhages . . . . . . . . . . . . . . . .
The Ventricles and Ventricular Hemorrhages . . . . . . . . . . . . . . . . . . . . . . . . . . . . . . .
Chapter 3
Dissections of the Central Nervous System . . . . . . . . . . . . . . . . . . . . . . . . . . . . . . . . . . . . .
Lateral, Medial, and Ventral Aspects . . . . . . . . . . . . . . . . . . . . . . . . . . . . . . . . . . . . .
Overall Views . . . . . . . . . . . . . . . . . . . . . . . . . . . . . . . . . . . . . . . . . . . . . . . . . . . . .
Chapter 4
Internal Morphology of the Brain in Slices and MRI . . . . . . . . . . . . . . . . . . . . . . . . . . .
Brain Slices in the Coronal Plane Correlated with MRI . . . . . . . . . . . . . . . . . . . . . . . . .
Brain Slices in the Axial Plane Correlated with MRI . . . . . . . . . . . . . . . . . . . . . . . . . . .
Chapter 5
Internal Morphology of the Spinal Cord and Brain in Stained Sections . . . . . . . . .
The Spinal Cord with CT and MRI . . . . . . . . . . . . . . . . . . . . . . . . . . . . . . . . . . . . . . .
Arterial Patterns Within the Spinal Cord With Vascular Syndromes . . . . . . . . . . . . . . . .
The Degenerated Corticospinal Tract . . . . . . . . . . . . . . . . . . . . . . . . . . . . . . . . . . . . .
The Medulla Oblongata with MRI and CT . . . . . . . . . . . . . . . . . . . . . . . . . . . . . . . . .
Arterial Patterns Within the Medulla Oblongata With Vascular Syndromes . . . . . . . . . . .
The Cerebellar Nuclei . . . . . . . . . . . . . . . . . . . . . . . . . . . . . . . . . . . . . . . . . . . . . . .
The Pons with MRI and CT . . . . . . . . . . . . . . . . . . . . . . . . . . . . . . . . . . . . . . . . . . . .
Arterial Patterns Within the Pons With Vascular Syndromes . . . . . . . . . . . . . . . . . . . . .
The Midbrain with MRI and CT . . . . . . . . . . . . . . . . . . . . . . . . . . . . . . . . . . . . . . . .
Arterial Patterns Within the Midbrain With Vascular Syndromes . . . . . . . . . . . . . . . . . .
The Diencephalon and Basal Nuclei with MRI . . . . . . . . . . . . . . . . . . . . . . . . . . . . . . .
Arterial Patterns Within the Forebrain With Vascular Syndromes . . . . . . . . . . . . . . . . .
Chapter 6
Internal Morphology of the Brain in Stained Sections:
Axial–Sagittal Correlations with MRI . . . . . . . . . . . . . . . . . . . . . . . . . . . . . . . . . . . . . . . . . .
Axial–Sagittal Correlations . . . . . . . . . . . . . . . . . . . . . . . . . . . . . . . . . . . . . . . . . . . .
Chapter 7
Synopsis of Functional Components, Tracts, Pathways, and Systems . . . . . . . . . . . .
Components of Cranial and Spinal Nerves . . . . . . . . . . . . . . . . . . . . . . . . . . . . . . . . . .
Orientation . . . . . . . . . . . . . . . . . . . . . . . . . . . . . . . . . . . . . . . . . . . . . . . . . . . . . . .
Sensory Pathways . . . . . . . . . . . . . . . . . . . . . . . . . . . . . . . . . . . . . . . . . . . . . . . . . .
Motor Pathways . . . . . . . . . . . . . . . . . . . . . . . . . . . . . . . . . . . . . . . . . . . . . . . . . . .
Cerebellum and Basal Nuclei . . . . . . . . . . . . . . . . . . . . . . . . . . . . . . . . . . . . . . . . . . .
Optic, Auditory, and Vestibular Systems . . . . . . . . . . . . . . . . . . . . . . . . . . . . . . . . . .
Limbic System . . . . . . . . . . . . . . . . . . . . . . . . . . . . . . . . . . . . . . . . . . . . . . . . . . . . .
v
vii
ix
1
8
9
10
13
16
38
45
46
52
55
56
59
63
63
73
83
84
94
96
98
110
112
116
124
126
136
138
158
161
162
173
174
176
178
190
204
220
232
xi
xii Contents
Chapter 8
Anatomical–Clinical Correlations: Cerebral Angiogram, MRA, and MRV . . . . . .
Cerebral Angiogram, MRA, and MRV . . . . . . . . . . . . . . . . . . . . . . . . . . . . . . . . . . . .
Blood Supply to the Choroid Plexi . . . . . . . . . . . . . . . . . . . . . . . . . . . . . . . . . . . . . . .
Overview of Vertebral and Carotid Arteries . . . . . . . . . . . . . . . . . . . . . . . . . . . . . . . .
239
240
251
252
Q&A’s: A Sampling of Study and Review Questions, Many in the USMLE Style,
All With Explained Answers . . . . . . . . . . . . . . . . . . . . . . . . . . . . . . . . . . . . . . . . . . . . . . . . . . .
253
Sources and Suggested Readings . . . . . . . . . . . . . . . . . . . . . . . . . . . . . . . . . . . . . . . . . . . . . . . . . . . . . . . . . . . . . . . . . . . . . .
Index . . . . . . . . . . . . . . . . . . . . . . . . . . . . . . . . . . . . . . . . . . . . . . . . . . . . . . . . . . . . . . . . . . . . . . . . . . . . . . . . . . . . . . . .
297
301
Chapter 9
Preface to the Sixth Edition
revious editions of Neuroanatomy have endeavored 1) to provide a
structural basis for understanding the function of the central nervous system; 2) to emphasize points of clinical relevance through use
of appropriate terminology and examples; and 3) to integrate neuroanatomical and clinical information in a format that will meet the educational needs of the user. The goal of the sixth edition is to continue
this philosophy and to present structural information and concepts in
an even more clinically useful and relevant format. Information learned
in the basic science setting should flow as seamlessly as possible into the
clinical setting.
I have received many constructive suggestions and comments from
my colleagues and students. This is especially the case for the modifications made in Chapters 2, 5, 7, 8, and 9 in this new edition. The
names of the individuals who have provided suggestions or comments
are given in the Acknowledgments. This thoughtful and helpful input
is greatly appreciated and has influenced the preparation of this new
edition.
The major changes made in the sixth edition of Neuroanatomy are as
follows:
First, recognizing that brain anatomy is seen in clear and elegant detail in MRI and CT, and that this is the primary way the brain is viewed
in the health care setting, additional new images have been incorporated
into this new edition. Every effort has been made to correlate the MRI
or CT with brain or spinal cord anatomy by relating these images on the
same page or on facing pages. New MRI or CT have been introduced
into chapter 2 (spinal cord, meningeal hemorrhages correlated with the
meninges, cisterns, hemorrhage into the brain, hemorrhage into the
ventricles correlated with the structure of the ventricles), chapter 5
(spinal cord and brainstem), and chapter 8 (vascular).
Second, the structure of the central nervous system should be available to the student (or the medical professional for that matter) in a format that makes this information immediately accessible, and applicable, to the requirements of the clinical experience. It is commonplace
to present brain structure in an anatomical orientation (e.g., the colliculi are “up” in the image and the interpeduncular fossa is “down”).
However, when the midbrain is viewed in an axial MRI or CT, the reverse is true: the colliculi are “down” in the image and the interpeduncular fossa is “up”. There are many good reasons for making brainstem
images available in an anatomical orientation and for teaching this view
in the academic setting. These reasons are recognized in this book. On
the other hand, the extensive use of MRI or CT in all areas of medicine, not just the clinical neurosciences, requires that students be
clearly aware of how brain and spinal cord structure is viewed, and used, in
the clinical environment. To address this important question, a series of
illustrations, including MRI or CT, are introduced in the spinal cord
and brainstem sections of chapter 5. These images are arranged to show
1) the small colorized version of the spinal cord or brainstem in an
anatomical orientation; 2) the same image flipped bottom-to-top into
a clinical orientation; and 3) the clinical orientation of the colorized
line drawing followed by T1 and T2 MRI and/or CT at levels comparable to the line drawing and corresponding stained section. This approach retains the inherent strengths of the full-page, colorized line
P
drawing and its companion stained section in the anatomical orientation. At the same time, it introduces, on the same set of pages, the important concept that CNS anatomy, both external and internal, is oriented differently in MRI or CT. It is the clinical orientation issue that
will confront the student/clinician in the clinical setting. It is certainly
appropriate to introduce, and even stress, this view of the brain and
spinal cord in the basic science years.
Third, new images have been included in chapter 8. These include,
but are not limited to, new examples of general vessel arrangement in
MRA, examples of specific vessels in MRI, and some additional examples of hemorrhage.
Fourth, additional examples of cranial nerves traversing the subarachnoid space are included. In fact, the number of MRI showing cranial nerves has been doubled. In addition, each new plate starts with a
gross anatomical view of the nerve (or nerves) shown in the succeeding MRI in that figure.
Fifth, additional clinical information and correlations have been included. These are in the form of new images, new and/or modified figure descriptions, and changes in other portions of the textual elements.
Sixth, in some instances, existing figures have been relocated to improve their correlation with other images. In other instances, existing
figures have been repeated and correlated with newly added MRI or
CT so as to more clearly illustrate an anatomical-clinical correlation.
Seventh, a new chapter (chapter 9), consisting of approximately 240
study and review questions and answers in the USMLE style, has been
added. All of these questions have explained answers keyed to specific
pages in the Atlas. Although not designed to be an exhaustive set, this
new chapter should give the user of this atlas a unique opportunity for
self-assessment.
Two further issues figured prominently in the development of this
new edition. First, the question of whether to use eponyms in their
possessive form. To paraphrase one of my clinical colleagues “Parkinson did not die of his disease (Parkinson disease), he died of a stroke;
it was never his own personal disease.” There are rare exceptions, such
as Lou Gehrig’s disease, but the point is well taken. McKusick
(1998a,b) has also made compelling arguments in support of using the
non-possessive form of eponyms. It is, however, acknowledged that
views differ on this question—much like debating how many angels
can dance on the head of a pin. Consultation with my neurology and
neurosurgery colleagues, a review of some of the more comprehensive
neurology texts (e.g., Rowland, 2000; Victor and Ropper, 2001), and
the standards established in The Council of Biology Editors Manual for
Authors, Editors, and Publishers (1994) and the American Medical Association’s Manual of Style (1998) clearly indicate an overwhelming
preference for the non possessive form. Recognizing that many users
of this book will enter clinical training, it was deemed appropriate to
encourage a contemporary approach. Consequently, the non possessive form of the eponym is used.
The second issue concerns use of the most up-to-date anatomical
terminology. With the publication of Terminologia Anatomica (Thieme,
New York, 1998), a new official international list of anatomical terms
for neuroanatomy is available. This new publication, having been
v
vi Preface to the Sixth Edition
adopted by the International Federation of Associations of Anatomists,
supersedes all previous terminology lists. Every effort has been made
to incorporate any applicable new or modified terms into this book.
The number of changes is modest and related primarily to directional
terms: posterior for dorsal, anterior for ventral, etc. In most cases, the
previous term appears in parentheses following the official term, i.e.,
posterior (dorsal) cochlear nucleus. It is almost certain that some changes
have eluded detection; these will be caught in subsequent printings.
Last, but certainly not least, the sixth edition is a few pages
longer than was the fifth edition. This results exclusively from the
inclusion of more MRI and CT, a better integration of anatomicalclinical information, including more clinical examples (text and illustrations), and the inclusion of Study/Review and USMLE style
questions with explained answers.
Duane E. Haines
Jackson, Mississippi
References:
Council of Biology Editions Style Manual Committee. Scientific Style and
Format—The CBE Manual for Authors, Editors, and Publishers. 6th Ed.
Cambridge: Cambridge University Press, 1994.
Federative Committee on Anatomical Terminology. Terminologia Anatomica. Thieme, Stuttgart and New York, 1998.
Iverson, MA et al. American Medical Association Manual of Style—A Guide
for Authors and Editors. 9th Ed. Baltimore: Williams & Wilkins, 1998.
McKusick, VA. On the naming of clinical disorders, with particular reference to eponyms. Medicine 1998;77: 1–2.
McKusick, VA. Mendelian Inheritance in Man, A Catalog of Human Genes
and Genetic Disorders. 12th Ed. Baltimore: The Johns Hopkins University Press, 1998.
Rowland, LP. Merritt’s Neurology. 10th Ed. Baltimore: Lippincott Williams
& Wilkins, 2000.
Victor, M and Ropper, AH. Adams and Victor’s Principles of Neurology. 7th
Ed. New York: McGraw-Hill, Medical Publishing Division, 2001.
Preface to the First Edition
his atlas is a reflection of, and a response to, suggestions from professional and graduate students over the years I have taught human
neurobiology. Admittedly, some personal philosophy, as regards
teaching, has crept into all parts of the work.
The goal of this atlas is to provide a maximal amount of useful information, in the form of photographs and drawings, so that the initial
learning experience will be pleasant, logical, and fruitful, and the review process effective and beneficial to longterm professional goals. To
this end several guiding principles have been followed. First, the entire
anatomy of the central nervous system (CNS), external and internal,
has been covered in appropriate detail. Second, a conscientious effort
has been made to generate photographs and drawings of the highest
quality: illustrations that clearly relay information to the reader. Third,
complementary information always appears on facing page. This may
take the form of two views of related structures such as brainstem or
successive brain slices or a list of abbreviations and description for a
full-page figure. Fourth, illustrations of blood supply have been included and integrated into their appropriate chapters. When gross
anatomy of the brain is shown, the patterns of blood vessels and relationships of sinuses appear on facing pages. The distribution pattern of
blood vessels to internal CNS structures is correlated with internal
morphology as seen in stained sections. Including information on external vascular patterns represents a distinct departure from what is
available in most atlases, and illustrations of internal vessel distribution
are unique to this atlas.
There are other features which, although not unique in themselves,
do not usually appear in atlas format. In the chapter containing crosssections, special effort has been made to provide figures that are accurate, clear, and allow considerable flexibility in how they can be used
for both teaching and learning. The use of illustrations that are one-half
photograph and one-half drawing is not entirely novel. In this atlas,
however, the sections are large, clearly labeled, and the drawing side
is a mirror-image of the photograph side. One section of the atlas is devoted to summaries of a variety of major pathways. Including this material in a laboratory atlas represents a distinct departure from the standard approach. However, feedback over the years strongly indicates
that this type of information in atlas format is extremely helpful to stu-
T
dents in the laboratory and greatly enhances their ability to grasp and
retain information on CNS connections. While this atlas does not attempt to teach clinical concepts, a chapter correlating selected views
of angiograms and CT scans with morphological relationships of cerebral arteries and internal brain structures is included. These examples
illustrate that a clear understanding of normal morphological relationships, as seen in the laboratory, can be directly transposed to clinical
situations.
This atlas was not conceived with a particular audience in mind. It
was designed to impart a clear and comprehensive understanding of
CNS morphology to its readers, whoever they may be. It is most obviously appropriate for human neurobiology courses as taught to medical, dental, and graduate students. In addition, students in nursing,
physical therapy, and other allied health curricula, and psychology as
well, may also find its contents helpful and applicable to their needs.
Inclusion and integration of blood vessel patterns, both external and internal, and the summary pathway drawings may be useful to the individual requiring a succinct, yet comprehensive review before taking
board exams in the neurological, neurosurgical, and psychiatric specialties.
The details in some portions of this atlas may exceed that found in
comparable parts of other atlases. If one is to err, it seems more judicious to err on the side of greater detail than on the side of inadequate
detail. If the student is confronted with more information on a particular point than is needed during the initial learning process, he or she
can simply bypass the extra information. However, once the initial
learning is completed, the additional information will be there to enhance the review process. If students have inadequate information in
front of them it may be difficult, or even impossible, to fill in missing
points that may not be part of their repertoire of knowledge. In addition, information may be inserted out of context, and, thereby, hinder
the learning experience.
A work such as this is bound to be subject to oversights, and for such
foibles, I am solely responsible. I welcome comments, suggestions, and
corrections from my colleagues and from students.
Duane E. Haines
vii
Acknowledgments
s was the case in previous editions of this book, my colleagues and
students in both medical and graduate programs have been most
gracious in offering their suggestions and comments. I greatly appreciate their time and interest in the continuing usefulness of this book.
As changes were being contemplated for this new edition, input on
potential modifications was solicited from faculty as well as students in
an effort to ascertain how these changes might impact on the usefulness
of this Atlas. These individuals went out of their way to review the documents that were provided and to give insightful, and sometimes
lengthy, comments on the pros and cons of the ideas being considered.
This input was taken into consideration as the initial plans were modified
and finalized by the author and then incorporated into this new edition.
The faculty who gave generously of their time and energy were Drs. A.
Agmon, C. Anderson, R. Baisden, S. Baldwin, J. L. Culberson, B. Hallas, J. B. Hutchins, T. Imig, G. R. Leichnetz, E. Levine, R. C. S. Lin, J.
C. Lynch, T. McGraw-Ferguson, G. F. Martin, G. A. Mihailoff, R. L.
Norman, R. E. Papka, H. J. Ralston, J. Rho, L. T. Robertson, J. D.
Schlag, K. L. Simpson, and C. Stefan. The students who offered helpful
and insightful comments were A. Alqueza (medical student, University
of Florida at Gainesville), A. S. Bristol (graduate student, University of
California at Irvine), L. Simmons (medical student, Vanderbilt University), J. A. Tucker (medical student, The University of Mississippi Medical Center), S. Thomas (graduate student, University of Maryland at
College Park), and M. Tomblyn (medical student, Rush Medical College). I greatly appreciate their comments and suggestions.
I would also like to thank my colleagues in the Department of
Anatomy at The University of Mississippi Medical Center (UMMC) for
their many helpful suggestions and comments. My colleagues in the
Department of Neurosurgery at UMMC (Drs. A. Parent [Chairman],
L. Harkey, J. Lancon, J. Ross, D. Esposito, and G. Mandybur) and in
the Department of Neurology at UMMC (especially Drs. J. Corbett
[Chairman], S. Subramony, H. Uschmann, and M. Santiago) have offered valuable input on a range of clinical issues. I am especially indebted to Dr. J. A. Lancon (Neurosurgery) for his significant contributions to this new edition. These include his willingness to participate
as co-author of Chapter 9 and his careful review of all new clinical information added to the book. I would also like to thank Ms. Amanda
Ellis, B.S.N., for keeping my friend John on track.
I am indebted to the following individuals for their careful review
of previous editions of the book: Drs. B. Anderson, R. Borke, Patricia
Brown, Paul Brown, T. Castro, B. Chronister, A. Craig, E. Dietrichs,
J. Evans, B. Falls, C. Forehand, R. Frederickson, E. Garcis-Rill, G.
Grunwald, J. King, A. Lamperti, K. Peusner, C. Phelps, D. Rosene,
A
A. Rosenquist, M. Schwartz, J. Scott, V. Seybold, D. Smith, S. Stensaas, D. Tolbert, F. Walberg, S. Walkley, M. Woodruff, M. Wyss,
and B. Yezierski. The stained sections used in this atlas are from the
teaching collection in the Department of Anatomy at West Virginia
University School of Medicine.
Dr. R. Brent Harrison (former Chairman of Radiology, UMMC),
Dr. Robert D. Halpert (current Chairman of Radiology, UMMC) and
Dr. Gurmett Dhilon (Neuroradiology) generously continue to give me
full access to all their facilities. I would like to express a special thanks
to Mr. W. (Eddie) Herrington (Chief CT/MRI Technologist) and Mr.
Joe Barnes (Senior MRI Technologist) for their outstanding efforts to
supply new images and their special efforts to generate images at specific planes for this new edition. In the same vein, Drs. G. Dhilon and
S. Crawford also made special attempts to get specific MRI at special
planes. I am also deeply appreciative to several technologists and nurses
in the CT/MRI suite, and particularly to Master Johnathan Barnes, for
being such cooperative “patients” as we worked to generate scans that
matched stained sections in the Atlas as closely as possible.
Modifications, both great and small, to the artwork and labeling
scheme, as well as some new renderings, were the work of Mr.
Michael Schenk (Director of Biomedical Illustration Services). Mr. Bill
Armstrong (Director of Biomedical Photography) produced outstanding photographs of gross specimens and slices, CTs, MRIs, and MRAs.
I am very appreciative of the time, effort, and dedication of these individuals to create the very best artwork and photographs possible for
this new edition. Ms. Katherine Squires did all the typing for the sixth
edition. Her excellent cooperation, patience, and good-natured repartee with the author were key elements in completing the final draft in
a timely manner.
This sixth edition would not have been possible without the interest and support of the publisher, Lippincott Williams & Wilkins. I want
to express thanks to my editor, Ms. Betty Sun (Acquisitions Editor), to
Mr. Dan Pepper (Associate Managing Editor), to Ms. Erica Lukenich
(Editorial Assistant), Ms. Jennifer Weir (Associate Production Manager), and to Mr. Joe Scott (Marketing Manager) for their encouragement, continuing interest, and confidence in this project. Their cooperation has given me the opportunity to make the improvements seen
herein.
Last, but certainly not least, I would like to express a special thanks
to my wife, Gretchen. She put up with me while these revisions were
in progress, carefully reviewed all changes in the text and all questions/answers, and was a tangible factor in getting everything done. I
dedicate this edition to Gretchen.
ix
CHAPTER
1
Introduction
and
Reader’s Guide
2
Introduction and Reader’s Guide
t a time when increasing numbers of atlases and textbooks are becoming available to students and instructors, it is appropriate to briefly outline the approach used in
this volume. Most books are the result of 1) the philosophic
approach of the author/instructor to the subject matter and
2) students’ needs as expressed through their suggestions
and opinions. The present atlas is no exception, and as a result, several factors have guided its further development.
These include an appreciation of what enhances learning in
the laboratory and classroom, the inherent value of correlating structure with function, the clinical value of understanding the blood supply to the central nervous system
(CNS), and the essential importance of integrating anatomy
with clinical information and examples. The goal is to make
it obvious to the user that structure and function in the CNS
are integrated elements and not separate entities.
Most neuroanatomic atlases approach the study of the
CNS from fundamentally similar viewpoints. These atlases
present brain anatomy followed by illustrations of stained
sections, in one or more planes. Although variations on this
theme exist, the basic approach is similar. In addition, most
atlases do not make a concerted effort to correlate vascular
patterns with external or internal brain structures. Also,
most atlases include little or no information on neurotransmitters and do not integrate clinical examples and information with the study of functional systems.
Understanding CNS structure is the basis for learning pathways, neural function, and for developing the skill to diagnose
the neurologically impaired patient. Following a brief period
devoted to the study of CNS morphology, a significant portion of many courses is spent learning functional systems. This
learning experience may take place in the laboratory because
it is here that the student deals with images of representative
levels of the entire neuraxis. However, few attempts have
been made to provide the student with a comprehensive and integrated guide—one that correlates, 1) external brain anatomy
with MRI and blood supply; 2) meninges and ventricles with
examples of meningeal, ventricular, and brain hemorrhage;
3) internal brain anatomy with MRI, blood supply, the organization of tracts and nuclei and selected clinical examples; 4)
summaries of clinically relevant pathways with neurotransmitters, numerous clinical correlations, and the essential concept of laterality; and 5) includes a large variety of images such
as angiogram, computed tomography (CT), magnetic resonance imaging (MRI), magnetic resonance angiography
(MRA), and magnetic resonance venography (MRV).
A
The present atlas addresses these points. The goal is not
only to show external and internal structure per se but also
to demonstrate that the relationship between brain anatomy
and MRI/CT, the blood supply to specific areas of the CNS
and the arrangement of pathways located therein, the neuroactive substances associated with pathways, and examples
of clinical deficits are inseparable components of the learning experience. An effort has been made to provide a format that is dynamic and flexible—one that makes the learning experience an interesting and rewarding exercise.
The relationship between blood vessels and specific brain
regions (external and/or internal) is extremely important
considering that approximately 50% of what goes wrong inside the skull, producing neurological deficits, is vascularrelated. To emphasize the value of this information, the distribution pattern of blood vessels is correlated with external
spinal cord and brain anatomy (Chapter 2) and with internal structures such as tracts and nuclei (Chapter 5), reviewed in each pathway drawing (Chapter 7), and shown in
angiograms, MRAs, and MRVs (Chapter 8). This approach
has several advantages: 1) the vascular pattern is immediately
related to the structures just learned, 2) vascular patterns
are shown in the sections of the atlas in which they belong,
3) the reader cannot proceed from one part of the atlas to
the next without being reminded of blood supply, and 4) the
conceptual importance of the distribution pattern of blood
vessels in the CNS is repeatedly reinforced.
The ability to diagnose a neurologically compromised patient is specifically related to a thorough understanding of
pathway structure, function, blood supply, and the relationships of this pathway to adjacent structures. To this end
Chapter 7 provides a series of semidiagrammatic illustrations
of various clinically relevant pathways. Each figure shows 1)
the trajectory of fibers that comprise the entire pathway; 2)
the laterality of fibers comprising the pathway, this being an
extremely important concept in diagnosis; 3) the positions
and somatotopy of fibers comprising each pathway at representative levels; 4) a review of the blood supply to the entire pathway; 5) important neurotransmitters associated
with fibers of the pathway; and 6) examples of deficits seen
following lesions of the pathway at various levels throughout the neuraxis. This chapter is designed to be used by itself
or integrated with other sections of the atlas; it is designed to
provide the reader with the structural and clinical essentials
of a given pathway in a single illustration.
Introduction and Reader’s Guide 3
The advent and common use of imaging methods (MRI,
MRA, and MRV) mandates that such images become an integral part of the educational process when teaching and/or
learning clinically applicable neuroscience. To this end, this
book contains about 175 MRI and CT images and 12 MRA and
MRV. All of these images are directly correlated with external
brain anatomy such as gyri and sulci, internal structures including pathways and nuclei, cranial nerves and adjacent structures, or they demonstrate examples of hemorrhages related
to the meninges and ventricles or the parenchyma of the brain.
Imaging the Brain (CT and MRI): Imaging the brain in
vivo is now commonplace for the patient with neurological
deficits that may indicate a compromise of the central nervous
system. Even most rural hospitals have, or have easy access to,
CT or MRI. With these facts in mind, it is appropriate to make
a few general comments on these imaging techniques and what
is routinely seen, or best seen, in each. For details of the methods and techniques of CT and MRI consult sources such as
Grossman (1996), Lee et al. (1999), or Buxton (2002).
Computed Tomography (CT): In CT, the patient is
passed between a source of x-rays and a series of detectors.
Tissue density is measured by the effects of x-rays on atoms
within the tissue as these x-rays pass through the tissue.
Atoms of higher number have a greater ability to attenuate
(stop) x-rays while those with lower numbers are less able to
attenuate x-rays. The various attenuation intensities are
computerized into numbers (Hounsfield units or CT numbers). Bone is given the value of +1,000 and is white, while
air is given a value of ⫺1,000 and is black. Extravascular
blood, an enhanced tumor, fat, the brain (grey and white
matter), and cerebrospinal fluid form an intervening continuum from white to black. A CT image of a patient with subarachnoid hemorrhage illustrates the various shades seen in a
CT (Fig. 1-1). In general, the following table summarizes the
white to black intensities seen for selected tissues in CT.
The Brain and Related Structures in CT
STRUCTURE/FLUID/SPACE
GREY SCALE
Bone, acute blood
Enhanced tumor
Subacute blood
Muscle
Grey matter
White matter
Cerebrospinal fluid
Air, Fat
Very white
Very white
Light grey
Light grey
Light grey
Medium grey
Medium grey to black
Very black
The advantages of CT are 1) it is rapidly done, which is
especially important in trauma; 2) it clearly shows acute and
subacute hemorrhages into the meningeal spaces and brain;
3) it shows bone (and skull fractures) to advantage; and 4)
it is less expensive than MRI. The disadvantages of CT are
1) it does not clearly show acute or subacute infarcts or ischemia, or brain edema; 2) it does not clearly differentiate
white from grey matter within the brain nearly as well as
MRI; and 3) it exposes the patient to ionizing radiation.
Magnetic Resonance Imaging (MRI): The tissues
of the body contain proportionately large amounts of protons (hydrogen). Protons have a positive nucleus, a shell of
negative electrons, and a north and south pole; they function like tiny spinning bar magnets. Normally, these atoms
are arranged randomly in relation to each other due to the
constantly changing magnetic field produced by the electrons. MRI uses this characteristic of protons to generate
images of the brain and body.
When radio waves are sent in short bursts into the magnet containing the patient, they are called a radiofrequency
pulse (RP). This pulse may vary in strength. When the frequency of the RP matches the frequency of the spinning proton, the proton will absorb energy from the radio wave (resonance). The effect is two-fold. First, the magnetic effects
of some protons are cancelled out and second, the magnetic
effects and energy levels in others are increased. When the
RP is turned off, the relaxed protons release energy (an
“echo”) that is received by a coil and computed into an image of that part of the body.
The two major types of MRI images (MRI/T1 and
MRI/T2) are related to the effect of RP on protons and the
reactions of these protons (relaxation) when the RP is
turned off. In general, those cancelled out protons return
slowly to their original magnetic strength. The image constructed from this time constant is called T1 (Fig. 1-2). On
the other hand, those protons that achieved a higher energy
level (were not cancelled-out) lose their energy more
rapidly as they return to their original state; the image constructed from this time constant is T2 (Fig. 1-3). The cre1-1 Computed Tomography (CT) in the axial plane of a patient
with subarachnoid hemorrhage. Bone is white, acute blood (white) ation of a T1-weighted image versus a T2-weighted image is
outlines the subarachnoid space, brain is grey, and cerebrospinal fluid based on a variation in the times used to receive the “echo”
from the relaxed protons.
in third and lateral ventricles is black.
4
Introduction and Reader’s Guide
The Brain and Related Structures in MRI
1-2 A sagittal T1 weighted Magnetic Resonance Image (MRI).
Brain is grey and cerebrospinal fluid is black.
NORMAL
T1
T2
Bone
Air
Muscle
White matter
Grey matter
Fat
CSF
Very black
Very black
Dark grey
Light grey
Dark grey
White
Very black
Very black
Very black
Dark grey
Dark grey
Light grey
Grey
Very white
ABNORMAL
T1
T2
Edema
Dark grey
Light grey to white
Tumor
Variable
Variable
Enhanced tumor
White
(Rarely done)
Dark grey
Light grey to white
The following table summarizes the white to black inten- Acute infarct
Subacute infarct
Dark grey
Light grey to white
sities seen in MRI images that are T1-weighted versus T2- Acute ischemia
Dark grey
Light grey to white
weighted. It should be emphasized that a number of varia- Subacute ischemia
Dark grey
Light grey to white
tions on these two general MRI themes are routinely seen in
the clinical environment.
The advantages of MRI are 1) it can be manipulated to visualize a wide variety of abnormalities or abnormal states
Chapter 2
within the brain; and 2) it can show great detail of the brain
This chapter presents 1) the gross anatomy of the spinal cord
in normal and abnormal states. The disadvantages of MRI
and its principal arteries; 2) the external morphology of the
are 1) it does not show acute or subacute subarachnoid hembrain, accompanied by MRIs and drawings of the vasculaorrhage or hemorrhage into the substance of the brain in any
ture patterns from the same perspective; 3) cranial nerves
detail; 2) it takes a much longer time to do and, therefore,
as seen in specimens and in MRI; and 4) the meninges and
is not useful in acute situations or in some types of trauma;
ventricular spaces. Emphasis is placed on correlating exter3) it is, comparatively, much more expensive than CT, and
nal brain and spinal cord anatomy with the respective vas4) the scan is extremely loud and may require sedation in
cular patterns and on correlating external brain structures
children.
and cranial nerves as seen in specimens with how the same
The ensuing discussion briefly outlines the salient features
structures appear in MRI. Information concerning the orgaof individual chapters. In some sections, considerable flexinization of the meninges includes clinical correlations, exbility has been designed into the format; at these points,
amples of extradural, so-called “subdural”, and subarachsome suggestions are made as to how the atlas can be used.
noid hemorrhages in CT and examples of cisterns in MRI.
In addition, new clinical correlations and examples have
The section showing the structure and relations of the venbeen included and a new chapter of USMLE-style review
tricular system now includes samples of hemorrhage into
questions has been added.
lateral, third, and fourth ventricles.
Chapter 3
The dissections in Chapter 3 offer views of some of those
brain structures introduced in Chapter 2. Certain structures
and/or structural relationships—for example, the orientation of the larger association bundles—are particularly
suited to such a presentation. This chapter uses a representative series of dissected views to provide a broader basis for
learning human neuroanatomy. Because it is not feasible to
illustrate every anatomic feature, the views and structures
selected are those that are usually emphasized in medical
1-3 A sagittal T2 weighted Magnetic Resonance Image (MRI). neurobiology courses. These views provide basic informaBrain is grey, blood vessels frequently appear black, and cerebrospinal tion necessary to make more detailed dissections, if appropriate, in a particular learning situation.
fluid is white.
Introduction and Reader’s Guide 5
Chapter 4
The study of general morphology of the hemisphere and
brainstem is continued in the two sections of Chapter 4. The
first section contains a representative series of unstained
coronal slices of brain, each of which is accompanied, on the
same page, by MRIs. The brain slice is labeled (by complete
names), and the MRIs are labeled with a corresponding abbreviation. The second section contains a series of unstained
brain slices cut in the axial plane, each of which is accompanied, again on the same page, by MRIs. Labeling of the axial
slices is as done for the coronal slices.
The similarities between the brain slices and the MRIs are
remarkable, and this style of presentation closely integrates
anatomy in the slice with that as seen in the corresponding
MRI. Because the brain, as sectioned at autopsy or in clinical pathologic conferences, is viewed as an unstained specimen, the preference here is to present the material in a format that will most closely parallel what is seen in these
clinical situations.
Chapter 5
This chapter has been revised with special emphasis on increasing the correlation between anatomical and clinical information. This new edition retains the quality and inherent
strengths of the line drawings and the stained sections being
located on facing pages in this chapter. However, an innovative approach (described below) is introduced that allows
the use of these images in their classic Anatomical Orientation and, at the same time, their conversion to the Clinical
Orientation so universally recognized and used in clinical
imaging techniques.
Chapter 5 consists of six sections covering, in sequence,
the spinal cord, medulla oblongata, cerebellar nuclei, pons,
midbrain, and diencephalon and basal nuclei, all with MRI.
In this format, the right-hand page contains a complete image of the stained section. The left-hand page contains a labeled line drawing of the stained section, accompanied by a
figure description, and a small orientation drawing. The section part of the line drawing is printed in a 60% screen of
black, and the leader lines and labels are printed at 100%
black. This gives the illustration a sense of depth and texture, reduces competition between lines, and makes the illustration easy to read at a glance.
Beginning with the first spinal cord level (coccygeal, Figure 5-1), the long tracts that are most essential to understanding how to diagnose the neurologically impaired
patient are colored. These tracts are the posterior column–
medial lemniscus system, the lateral corticospinal tract, and
the anterolateral system. In the brainstem, these tracts are
joined by the colorized spinal trigeminal tract, the ventral
trigeminothalamic tract, and all of the motor and sensory
nuclei of cranial nerves. This scheme continues rostrally
into the caudal nuclei of the dorsal thalamus and the posterior limb of the internal capsule. In addition to the coloring
of the artwork, each page has a key that specifies the structure and function of each colored structure. This approach
emphasizes anatomical–clinical integration.
Semidiagrammatic representations of the internal blood
supply to the spinal cord, medulla, pons, midbrain, and forebrain follow each set of line drawings and stained sections.
This allows the immediate, and convenient, correlation of
structure with its blood supply as one is studying the internal anatomy of the neuraxis. In addition, tables that summarize
the vascular syndromes of the spinal cord, medulla, pons, midbrain,
and forebrain are located on the pages facing each of these vascular drawings. While learning or reviewing the internal
blood supply to these parts of the neuraxis, one can also correlate the deficits seen when the same vessels are occluded.
It is essential to successful diagnosis to develop a good understanding of what structure is served by what vessel.
The diencephalon and basal nuclei section of this chapter
uses ten cross-sections to illustrate internal anatomy. It
should be emphasized that 8 of these 10 sections (those parallel to
each other) are all from the same brain.
The internal anatomy of the brainstem is commonly
taught in an anatomical orientation. That is, posterior structures, such as the vestibular nuclei and colliculi, are “up” in
the image, while anterior structures, such as the pyramid
and crus cerebri, are “down” in the image. However, when
the brainstem is viewed in the clinical setting, as in CT or
MRI, this orientation is reversed. In the clinical orientation,
posterior structures (4th ventricle, colliculi) are “down” in
the image while anterior structures (pyramid, basilar pons,
crus cerebri) are “up” in the image.
Recognizing that many users of this book are pursuing a
health care career (as a practitioner or teacher of future clinicians), it is essential to introduce MRI and CT of the brainstem into chapter 5. This accomplishes two important points.
First, it allows correlation of the size, shape, and configuration of brainstem sections (line drawings and stained slices)
with MRI and CT at comparable levels. Second, it offers the
user the opportunity to visualize how nuclei, tracts (and their
somatotopy) and vascular territories are represented in MRI
and CT. Understanding the brain in the Clinical Orientation
(as seen in MRI or CT) is extremely important in diagnosis.
To successfully introduce MRI and CT in the brainstem portion of chapter 5, a continuum from Anatomical Orientation
to Clinical Orientation to MRI needs to be clearly illustrated.
This is achieved by 1) placing a small version of the colorized
line drawing on the facing page (page with the stained section)
in Anatomical Orientation; 2) showing how this image is
flipped top to bottom into a Clinical Orientation; and 3) following this flipped image with (usually) T1 and T2 MRls at
levels comparable to the accompanying line drawing and
6
Introduction and Reader’s Guide
stained section (Fig. 1-4). This approach retains the anatomical strengths of the spinal cord and brainstem sections of
chapter 5 but allows the introduction of important concepts
regarding how anatomical information is arranged in images
utilized in the clinical environment.
Every effort has been made to use MRI and CT that match,
as closely as possible, the line drawings and stained sections in
the spinal cord and brainstem portions of chapter 5. Recognizing that this match is subject to the vicissitudes of angle and
individual variation, special sets of images were used in chapter 5. The first set consisted of T1- and T2-weighted MRI
generated from the same individual; these are identified, respectively, as “MRI, T1-weighted” and “MRI, T2-weighted”
in chapter 5. The second set consisted of CT images from a
patient who had an injection of the radiopaque contrast media Isovue-MR 200 (iopamidol injection 41 %) into the lumbar cistern. This contrast media diffused throughout the spinal
and cranial subarachnoid spaces, outlining the spinal cord and
brainstem (Fig. 1-5). Images at spinal levels show neural
structures as grey surrounded by a light subarachnoid space;
this is a “CT myelogram”. A comparable image at brainstem
levels (grey brain, light CSF) is a “CT cisternogram”. These
designations are used in chapter 5. While all matches are not
perfect, not all things in life or medicine are, the vast majority of matches between MRI, CT, and drawings/sections are
excellent and clearly demonstrate the intended points.
1-5 Computed Tomography (CT) of a patient following injection
of a radiopaque contrast media into the lumbar cistern. In this example, at the medullary level (a cisternogram), neural structures appear
grey and the subarachnoid space appears light.
The juxtaposition of MRI to stained section extends into
the forebrain portion of chapter 5. Many anatomic features
seen in the forebrain stained sections are easily identified in
the adjacent MRI. These particular MRI are not labeled so
as to allow the user to develop and practice his/her interpretive skills. The various subsections of chapter 5 can be
used in a variety of ways and will accommodate a wide range
of student and/or instructor preferences.
Chapter 6
The three-dimensional anatomy of internal structures in the
CNS can also be studied in stained sections that correlate similar structures in different planes. The photographs of stained
axial and sagittal sections and of MRIs in Chapter 6 are organized to provide four important levels of information. First,
the general internal anatomy of brain structures can be easily
Anatomical orientation
Clinical orientation
identified in each photograph. Second, axial photographs are
on left-hand pages and arranged from dorsal to ventral (Figures 6-1 to 6-9), whereas sagittal photographs are on righthand pages and arranged from medial to lateral (Figures 6-2
to 6-10). This setup, in essence, provides complete representation of the brain in both planes for use as independent
MRI, T1 weighted image
study sets (axial only, sagittal only) or as integrated/correlated sets (compare facing pages). Third, because axial and
sagittal sections are on facing pages and the plane of section of
each is indicated on its companion by a heavy line, the reader
can easily visualize the positions of internal structures in more
than one plane and develop a clear concept of three-dimenMRI, T2 weighted image
sional topography. In other words, one can identify structures
dorsal or ventral to the axial plane by comparing them with
the sagittal, and structures medial or lateral to the sagittal
plane by comparing them with the axial. Such comparisons facilitate a more full understanding of three-dimensional relaCT cisternogram
tionships in the brain. Fourth, the inclusion of MRIs with rep1-4 An example showing anatomical and clinical orientations of a resentative axial and sagittal stained sections provides
brainstem level and the corresponding T1 MRI, T2 MRI, and CT cister- excellent examples of the fact that structures seen in the
nogram. For additional examples and details see chapter 5, pages 84–133. teaching laboratory are easy to recognize in clinical images.
Introduction and Reader’s Guide 7
These MRIs are also not labeled so as to allow the user to de- general format as the preceding figures. Photocopies of
these blank master drawings can be used by the student for
velop his/her interpretive skills.
learning and/or review of any pathway and by the instructor to teach additional pathways not included in the atlas or
Chapter 7
as a substrate for examination questions. The flexibility of
This chapter provides summaries of a variety of clinically information as presented in Chapter 7 extends equally to
relevant CNS tracts and/or pathways and has four features student and instructor.
that enhance student understanding. First, the inclusion of
pathway information in atlas format broadens the basis one
Chapter 8
can use to teach functional neurobiology. This is especially
This chapter contains a series of angiograms (arterial and
the case when pathways are presented in a style that enhances the development of diagnostic skills. Second, each venous phases), magnetic resonance angiography (MRA)
drawing illustrates, in line color, a given pathway com- images, and magnetic resonance venography (MRV) impletely, showing its 1) origins, longitudinal extent, course ages. The angiograms are shown in lateral and anterior–
throughout the neuraxis and termination; 2) laterality—an posterior projections—some as standard views with correall-important issue in diagnosis; 3) point of decussation, if sponding digital subtraction images. MRA and MRV techapplicable; 4) position in representative cross sections of the nology are noninvasive methods that allow for the visualizabrainstem and spinal cord; and 5) the somatotopic organi- tion of arteries (MRA) and veins and venous sinuses (MRV).
zation of fibers within the pathway, if applicable. The blood There are, however, many situations when both arteries and
supply to each pathway is reviewed on the facing page. veins are seen with either method. Use of MRA and MRV
Third, a brief summary mentions the main neuroactive sub- is commonplace, and this technology is an important diagstances associated with cells and fibers composing particular nostic tool. A number of new vascular images have been insegments of the pathway under consideration. The action of cluded in this revised version of Chapter 8.
the substance, if widely agreed on, is indicated as excitatory
(⫹) or inhibitory (⫺). This allows the reader to closely correlate
Chapter 9
a particular neurotransmitter with a specific population of projecA primary goal in the study of functional human neurobition neurons and their terminals. The limitations of this approach, within the confines of an atlas, are self-evident. The ology is to become a competent health care professional.
transmitters associated with some pathways are not well Another, and equally significant, goal is to pass examinaknown; consequently, such information is not provided for tions. These may be course examinations, the National
some connections. Also, no attempt is made to identify sub- Board Subject Exam (some courses require these), or stanstances that may be colocalized, to discuss their synthesis or dardized tests, such as the USMLE Step 1 and Step 2, given
degradation, or to mention all neurotransmitters associated at key intervals and taken by all students.
The questions comprising chapter 9 were generated in
with a particular cell group. The goal here is to introduce
the reader to selected neurotransmitters and to integrate and the recognition that examinations are an essential part of the
correlate this information with a particular pathway, circuit, educational process. Whenever possible, and practical,
or connection. Fourth, the clinical correlations that accompany these questions are in the USMLE Step 1 style (single best
each pathway drawing provide examples of deficits resulting from answer). These questions emphasize 1) anatomical and clinlesions, at various levels in the neuraxis, of the fibers composing that ical concepts and correlations; 2) the application of basic huspecific pathway. Also, examples are given of syndromes or man neurobiology to medical practice; and 3) how neurodiseases in which these deficits are seen. The ways in which logical deficits and diseases relate to damage in specific parts
these clinical correlations can be used to enrich the learning of the nervous system. In general, the questions are grouped
by chapter. However, in some instances, questions draw on
process are described in Figure 7-3 on page 176.
The drawings in this section were designed to provide the information provided in more than one chapter. This is
maximum amount of information, to keep the extraneous sometimes essential in an effort to make appropriate
points to a minimum, and to do it all in a single, easy-to-fol- structural/functional/clinical correlations. At the end of
low illustration. A complete range of relevant information each group of questions the correct answers are provided
is contained in each drawing and in its description as ex- and explained. Included with the explanation is a reference
to the page (or pages) containing the answer, be that answer
plained in the second point above.
Because it is not possible to anticipate all pathways that in the text or in a figure. Although not exhaustive, this list
may be taught in a wide range of neurobiology courses, flex- of questions should provide the user of this atlas with an exibility has been designed into Chapter 7. The last figure in cellent opportunity for self-assessment covering a broad
each section is a blank master drawing that follows the same range of clinically relevant topics.
8
Introduction and Reader’s Guide
Rationale for Labels and Abbreviations
o universally accepted way to identify specific features
or structures in drawings or photographs exists. The
variety of methods seen in currently available atlases reflects
the personal preferences of the authors. Such is the case in
the present endeavor. The goal of this atlas is to present basic functional and clinical neuroanatomy in an understandable and useful format.
Among currently available atlases, most figures are labeled with either the complete names of structures or with
numbers or letters that are keyed to a list of the complete
names. The first method immediately imparts the greatest
amount of information; the second method is the most succinct. When using the complete names of structures, one
must exercise care to not compromise the quality or size of
the illustration, the number of structures labeled, or the size
of labels used. Although the use of single letters or numbers
results in minimal clutter on the figure, a major drawback is
the fact that the same number or letter may appear on several different figures and designate different structures in all
cases. Consequently, no consistency occurs between numbers and letters and their corresponding meanings as the
reader examines different figures. This atlas uses a combination of complete words and abbreviations that are clearly
recognized versions of the complete word.
In response to suggestions made by those using this book
over the years, the number of abbreviations in the sixth edition has been reduced, and the number of labels using the
complete name has been increased. Simultaneously, complete names and abbreviations have been used together in
some chapters to the full advantage of each method. For example, structures are labeled on a brain slice by the complete name, but the same structure in the accompanying
MRI is labeled with a corresponding abbreviation (see
N
Chapters 2 and 4). This uses the complete word(s) on the
larger image of a brain structure while using the shorter abbreviation on the smaller image of the MRI.
The abbreviations used in this atlas do not clutter the illustration; they permit labeling of all relevant structures and
are adequately informative while stimulating the thinking–learning process. The abbreviations are, in a very real
sense, mnemonics. When learning gyri and sulci of the occipital lobe, for example, one realizes that the abbreviation
“LinGy” in the atlas could only mean “lingual gyrus.” It could
not be confused with other structures in other parts of the
nervous system. Regarding the pathways, “RuSp” could
mean only “rubrospinal tract” and “LenFas,” the “lenticular
fasciculus.” As the reader learns more and more terminology from lectures and readings, he or she will be able to use
these abbreviations with minimal reference to the accompanying list. In addition, a subtle advantage of this method of
labeling is that, as the reader looks at the abbreviation and
momentarily pauses to ponder its meaning, he or she may
form a mental image of the structure and the complete
word. Because neuroanatomy requires one to conceptualize
and form mental images to more clearly understand CNS
relationships, this method seems especially useful.
References:
Bruxton, RB. Introduction to Functional Magnetic Resonance
Imaging, Principles and Techniques. Cambridge: Cambridge
University Press, 2002.
Grossman, CB. Magnetic Resonance Imaging and Computed Tomography of the Head and Spine. 2nd Ed. Baltimore:
Williams & Wilkins, 1996.
Lee, SH, Roa, KCVG, and Zimmerman, RA. Cranial MRI
and CT. 4th Ed. New York: McGraw-Hill Health Professions Division, 1999.
CHAPTER
2
External Morphology
of the
Central Nervous System
10 External Morphology of the Central Nervous System
Posterior View
C2 Posterior root (PR)
Posterior spinal
artery
Dura
Arachnoid
C3 PR
Denticulate
ligament
C4 PR
Posterior spinal
medullary artery
C5 PR
Anterior View
C2 Anterior root (AR)
Dura
Denticulate
ligament
C3 AR
Arachnoid
Anterior spinal
medullary artery
C4 AR
Anterior spinal
artery
C5 AR
2-1 Posterior (upper) and anterior (lower) views showing the general features of the spinal cord as seen at levels C2–C5. The dura and
arachnoid are reflected, and the pia is intimately adherent to the spinal
cord and rootlets. Posterior and anterior spinal medullary arteries (see
Figure 2-3 on facing page) follow their respective roots. The posterior
spinal artery is found medial to the entering posterior rootlets (and the
dorsolateral sulcus), while the anterior spinal artery is in the anterior
median fissure (see also Figure 2-2, facing page).
The Spinal Cord 11
Posterior View
Sulci:
Posterior median
Posterior intermediate
Posterolateral
C7 Posterior root
Spinal (posterior root) ganglion
Fasciculus gracilis
Fasciculus cuneatus
Anterior View
Anterior spinal artery
C7 Anterior root
Anterior radicular
artery
Anterior funiculus
Anterior median fissure
2-2 Posterior (upper) and anterior (lower) views showing details
of the spinal cord as seen in the C7 segment. The posterior (dorsal) root
ganglion is partially covered by dura and connective tissue.
Posterior spinal arteries
Arterial
vasocorona
Basilar artery
Posterior inferior
cerebellar arteries
Vertebral arteries
Anterior spinal artery
Posterior spinal
medullary artery
Posterior radicular
artery (on dorsal root)
Sulcal arteries
Anterior spinal
medullary artery
Anterior radicular artery
(on ventral root)
Segmental artery
2-3 Semidiagrammatic representation showing the origin and general location of principal arteries supplying the spinal cord. The anterior and posterior radicular arteries arise at every spinal level and serve
their respective roots and ganglion. The anterior and posterior spinal
medullary arteries (also called medullary feeder arteries or segmental
medullary arteries) arise at intermittent levels and serve to augment
the blood supply to the spinal cord. The artery of Adamkiewicz is an
unusually large spinal medullary artery arising usually on the left in low
thoracic or upper lumbar levels (T9–L1). The arterial vasocorona is a
diffuse anastomotic plexus covering the cord surface.
12 External Morphology of the Central Nervous System
A
B
C
Thoracic
cord
T9
Dura and
arachnoid
LuSaCd
Lumbar and
sacral cord
(LuSaCd)
L1
SaCoCd
Sacral and
coccygeal
cord
(SaCoCd)
Lumbar
cistern
FTInt
Conus
medullaris
CaEq
Filum
terminale
internum
(FTInt)
L5
Cauda
equina
(CaEq)
S1
Posterior
root
ganglion
Dura and
arachnoid
2-4 Overall posterior (A,B) and sagittal MRI (C, T2-weighted)
views of the lower thoracic, lumbar, sacral, and coccygeal spinal cord
segments and the cauda equina. The dura and arachnoid are retracted
in A and B. The cauda equina is shown in situ in A, and in B the nerve
roots of the cauda equina have been spread laterally to expose the conus
medullaris and filum terminale internum. This latter structure is also
called the pial part of the filum terminale. See Figures 5-1 and 5-2 on
pages 84–87 for cross-sectional views of the cauda equina.
In the sagittal MRI (C), the lower portions of the cord, the filum
terminale internum, and cauda equina are clearly seen. In addition, the
intervertebral discs and the bodies of the vertebrae are clear. The lumbar cistern is an enlarged part of the subarachnoid space caudal to the
end of the spinal cord. This space contains the anterior and posterior
roots from the lower part of the spinal cord that collectively form the
cauda equina. The filum terminale internum also descends from the
conus medullaris through the lumbar cistern to attach to the inner surface of the dural sac. The dural sac ends at about the level of the S2 vertebra and is attached to the coccyx by the filum terminale externum
(also see Fig. 2-47 on page 47). A lumbar puncture is made by inserting a large gauge needle (18-22 gauge) between the L3 and L4 vertebra or L4 and L5 vertebra and retrieving a sample of cerebrospinal fluid
from the lumbar cistern. This sample may be used for a number of diagnostic procedures.
The Brain: Lobes 13
Central sulcus
Precentral sulcus
Postcentral sulcus
Parietooccipital
sulcus
Lobes
Frontal
A
Parietal
Temporal
Occipital
Preoccipital
notch
Lateral sulcus
Limbic
Insular
Central sulcus
Paracentral sulcus
Marginal sulcus (marginal
ramus of the cingulate
sulcus)
Cingulate sulcus
Corpus callosum
Parietooccipital
sulcus
B
Fornix
Diencephalon
Preoccipital notch
Calcarine
sulcus
Collateral sulcus
2-5 Lateral (A) and medial (B) views of the cerebral hemisphere
showing the landmarks used to divide the cortex into its main lobes.
On the lateral aspect, the central sulcus (of Rolando) separates
frontal and parietal lobes. The lateral sulcus (of Sylvius) forms the border between frontal and temporal lobes. The occipital lobe is located
caudal to an arbitrary line drawn between the terminus of the parietooccipital sulcus and the preoccipital notch. A horizontal line drawn
from approximately the upper two-thirds of the lateral fissure to the
rostral edge of the occipital lobe represents the border between parietal and temporal lobes. The insular cortex (see also Figs. 2-46 on page
45 and 3-1 on page 56) is located internal to the lateral sulcus. This part
of the cortex is made up of long and short gyri that are separated from
each other by the central sulcus of the insula. The insula, as a whole, is
separated from the adjacent portions of the frontal, parietal, and temporal opercula by the circular sulcus.
On the medial aspect, the cingulate sulcus separates medial portions
of frontal and parietal lobes from the limbic lobe. An imaginary continuation of the central sulcus intersects with the cingulate sulcus and
forms the border between frontal and parietal lobes. The parietooccipital sulcus and an arbitrary continuation of this line to the preoccipital notch separate the parietal, limbic, and temporal lobes from the
occipital lobe.
14 External Morphology of the Central Nervous System
Postcentral gyrus
3,1,2
Precentral gyrus
5
Surpamarginal gyrus
Pars opercularis
6
7
4
8
40
Angular gyrus
39
19
Pars triangularis
18
22
45
44
17
41
A
42
Pars orbitalis
47
3,1,2
Anterior paracentral gyrus
5
8
6
Posterior paracentral gyrus
4
7
19
Cuneus
18
B
17
Calcarine sulcus
18
19
2-6 Lateral (A) and medial (B) views of the cerebral hemisphere
showing the more commonly described Brodmann areas. In general,
area 4 comprises the primary somatomotor cortex, areas 3,1, and 2 the
primary somatosensory cortex, and area 17 the primary visual cortex.
Area 41 is the primary auditory cortex, and the portion of area 6 in the
caudal part of the middle frontal gyrus is generally recognized as the
frontal eye field.
The inferior frontal gyrus has three portions: a pars opercularis, pars
Lingual gyrus
triangularis, and a pars orbitalis. A lesion that is located primarily in areas 44 and 45 (shaded) will give rise to what is called a Broca aphasia,
also called expressive or nonfluent aphasia.
The inferior parietal lobule consists of supramarginal (area 40) and
angular (area 39) gyri. Lesions in this general area of the cortex
(shaded), and sometimes extending into area 22, will give rise to what
is known as Wernicke aphasia, also sometimes called receptive or fluent aphasia.
The Brain: Lobes 15
Precentral gyrus (primary somatomotor cortex)
Posrcentral gyrus (primary somatosensory cortex)
A
Anterior paracentral gyrus (somatomotor)
Posterior paracentral gyrus (somatosensory)
B
Left
inferior
visual
quadrant
2-7 Lateral (A) and medial (B) views of the cerebral hemisphere
showing the somatotopic organization of the primary somatomotor
and somatosensory cortices. The lower extremity and foot areas are located on medial aspects of the hemisphere in the anterior paracentral
(motor) and the posterior paracentral (sensory) gyri. The remaining
portions of the body extend from the margin of the hemisphere over
the convexity to the lateral sulcus in the precentral and postcentral
gyri.
In general, the precentral gyrus can be divided into three regions:
the lateral third representing the face area, the middle third represent-
ing the hand and upper extremity areas, and the medial third representing the trunk and the hip. Lesions of the somatomotor cortex result in motor deficits on the contralateral side of the body while lesions
in the somatosensory cortex result in a loss of sensory perception from
the contralateral side of the body.
The medial surface of the right hemisphere (B) illustrates the position of the left portions of the visual field. The inferior visual quadrant
is located in the primary visual cortex above the calcarine sulcus while
the superior visual quadrant is found in the cortex below the calcarine
sulcus.
16 External Morphology of the Central Nervous System
Longitudinal fissure
Superior frontal
gyrus (SFGy)
Middle frontal
gyrus (MFGy)
Superior frontal
sulcus (SFSul)
Precentral
sulcus (PrCSul)
Precentral
gyrus (PrCGy)
Central
sulcus (CSul)
Precentral
gyrus (PrCGy)
Postcentral
gyrus (PoCGy)
Central
sulcus (CSul)
Supramarginal
gyrus
Postcentral sulcus
Superior parietal lobule
Occipital gyri
Anterior
cerebral
arteries
MFGy
ACA
territory
SFGy
SFSul
PrCSul
CSul
PoCGy
PrCGy
Falx
cerebri
2-8 Dorsal view of the cerebral hemispheres showing the main gyri
and sulci and an MRI (inverted inversion recovery—lower left) and a
CT (lower right) identifying structures from the same perspective.
Note the area of infarction representing the territory of the anterior
cerebral artery (ACA).
The Brain: Gross Views, Vasculature, and MRI 17
Frontopolar
branches of ACA
Branches of MCA (M4)
Orbitofrontal
Prerolandic
Rolandic
Callosomarginal branches
(from ACA)
Parietal and
temporal
Paracentral branches
(from ACA)
Internal parietal branches
(from ACA)
Branches of PCA
Temporal (P3)
Parieto-occipital sulcus
Parieto-occipital (P4)
Calcarine (P4)
2-9 Dorsal view of the cerebral hemispheres showing the location
and general branching patterns of the anterior (ACA), middle (MCA),
and posterior (PCA) cerebral arteries. Gyri and sulci can be identified
by a comparison with Figure 2-8 (facing page).
Superior cerebral veins
Superior
sagittal sinus
Rolandic vein
To superficial middle
cerebral vein and
inferior anastomotic
vein
Greater anastomotic
vein (Trolard)
Superior cerebral veins
To sinus confluens
2-10 Dorsal view of the cerebral hemispheres showing the location
of the superior sagittal sinus and the locations and general branching
patterns of veins. Gyri and sulci can be identified by a comparison with
Figure 2-8 (facing page). See Figures 8-4 and 8-5 (pp. 243–244) for
comparable angiograms (venous phase) of the superior sagittal sinus.
18 External Morphology of the Central Nervous System
Precentral gyrus (PrCGy)
Precentral sulcus (PrCSul)
Central sulcus (CSul)
Postcentral gyrus (PoCGy)
Superior frontal gyrus
Superior frontal sulcus
Postcentral sulcus (PoCSul)
Superior parietal lobule
Supramarginal gyrus
Interparietal sulcus
Middle frontal gyrus (MFGy)
Inferior frontal sulcus (IFSul)
Angular gyrus
Inferior frontal gyrus:
Pars opercularis (PoP)
Pars triangularis (PTr)
Pars orbitalis (POrb)
Occipital
gyri (OGy)
Lateral sulcus (LatSul)
Superior temporal gyrus (STGy)
Superior temporal sulcus (STSul)
Middle temporal gyrus (MTGy)
Preoccipital notch
PrCGy
PrCSul
CSul
PoCSul
PoCGy
MFGy
LatSul
IFSul
OGy
PoP
PTr
POrb
MTGy
STGy
STSul
2-11 Lateral view of the left cerebral hemisphere showing the
principal gyri and sulci and an MRI (inversion recovery) identifying
many of these structures from the same perspective.
The Brain: Gross Views, Vasculature, and MRI 19
Central sulcus
Anterior and posterior
parietal branches of MCA
Rolandic branches
of MCA
Angular branches
of MCA
Prerolandic branches
of MCA
Posterior temporal
branches of MCA
Orbitofrontal branches
of MCA
Orbital branches
of anterior cerebral artery
Middle temporal
branches of MCA
Middle cerebral artery (MCA)
in lateral sulcus
2-12 Lateral view of the right cerebral hemisphere showing the
branching pattern of the middle cerebral artery. Gyri and sulci can be
identified by comparison with Figure 2-11 (facing page). The middle
cerebral artery initially branches in the depths of the lateral sulcus (as
M2 and M3 segments); these branches seen on the surface of the hemi-
Rolandic vein
Anterior temporal
branches of MCA
sphere represent the M4 segment. Terminal branches of the posterior
and anterior cerebral arteries course over the edges of the temporal and
occipital lobes, and parietal and frontal lobes, respectively (see Figure
2-9 on page 17). See Figure 8-1 (p. 240) for a comparable angiogram
of the middle and anterior cerebral arteries.
Greater anastomotic vein (Trolard)
Superior sagittal sinus
Superior cerebral veins
Superior cerebral
veins
Inferior anastomotic
vein (Labbé)
Straight sinus
Superficial middle
cerebral vein
Sinus confluens
Transverse sinus (TS)
To sphenoparietal sinus
To cavernous sinus
Inferior cerebral veins
To sphenoparietal sinus
Occipital sinus
Inferior cerebral veins
to sphenoparietal sinus
Temporal cerebral veins
TS
To sigmoid sinus
2-13 Lateral view of the right cerebral hemisphere showing the locations of sinuses and the locations and general branching patterns of
veins. Gyri and sulci can be identified by comparison with Figure 2-11
(facing page). Communications between veins and sinuses or between
sinuses are also indicated. See Figures 8-2 (p. 241) and 8-11 (p. 250)
for comparable angiogram and MRV of the sinuses and superficial
veins.
20 External Morphology of the Central Nervous System
Frontal pole
Olfactory bulb
Olfactory sulcus (OlfSul)
Gyrus rectus (GyRec)
Orbital gyri (OrbGy)
Olfactory tract
Temporal pole
Optic nerve
Infundibulum
Optic chiasm
Uncus (Un)
Optic tract (OpTr)
Mammillary
body (MB)
Interpeduncular
fossa (IPF)
Parahippocampal
gyrus
Inferior temporal
gyrus
Collateral sulcus
Crus cerebri (CC)
Substantia nigra
Occipitotemporal
gyri
Cerebral
aqueduct (CA)
Colliculi (Col)
Lingual gyrus
Occipital gyri
Occipital pole
GyRec
OrbGy
OlfSul
Middle
cerebral
artery
OpTr
MB
Anterior
cerebral
artery
OpTr
Hypothalamus
Un
Un
IPF
CC
IPF
Col
Temporal lobe
CA
Col
Cerebellum
2-14 Ventral view of the cerebral hemispheres and diencephalon
with the brainstem caudal to midbrain removed and two MRIs (inver-
sion recovery—lower left; T2-weighted—lower right) showing many
structures from the same perspective.
The Brain: Gross Views, Vasculature, and MRI 21
Orbitofrontal branches
of MCA
Orbital branches of ACA
Middle cerebral
artery (MCA)
Anterior cerebral
artery (ACA)
MCA in lateral sulcus
Lenticulostriate
branches of MCA
Anterior temporal
branch of PCA
(P3 segment)
Posterior cerebral
artery (PCA)
Posterior temporal
branch of PCA
(P3 segment)
Parieto-occipital
branch of PCA
(P4 segment)
Calcarine branch of PCA
(P4 segment)
2-15 Ventral view of the cerebral hemisphere with the brainstem
removed, which shows the branching pattern of the posterior cerebral
artery (PCA) and some branches of the anterior and middle cerebral
arteries. The P1 and P2 segments of the PCA are shown on Figure 2-21
on page 25. Shown here are P3 (origin of temporal arteries) and P4 (origin of calcarine and parietooccipital arteries) segments. Gyri and sulci
can be identified by comparison with Figure 2-14 (facing page).
Sphenoparietal sinus
Intercavernous sinuses
Anterior
Posterior
Cavernous sinus
Superior petrosal sinus
Pineal
Great cerebral vein
Inferior petrosal sinus
Sigmoid sinus
Straight sinus
—inferior sagittal sinus
—superior cerebellar veins
Internal jugular vein
TS
Transverse sinus (TS)
Sinus confluens
2-16 Ventral view of the cerebral hemisphere, with brainstem removed, showing the locations and relationships of the main sinuses.
Gyri and sulci can be identified by comparison with Figure 2-14 (facing
page). The listings preceded by an en-dash (–) under principal sinuses
are the main tributaries of that sinus. See Figures 8-5 (p. 245), 8–9
(p. 248), and 8–11 (p. 250) for comparable MRV of the transverse
sinus.
22 External Morphology of the Central Nervous System
Frontal pole
Olfactory bulb
Longitundinal fissure
Olfactory sulcus (OlfSul)
Orbital sulci
Orbital gyri (OrbGy)
Gyrus rectus (GyRec)
Olfactory tract
Temporal pole (TPole)
Basilar pons (BP)
Uncus
Occipitotemporal
sulcus
Parahippocampal
gyrus
Occipitotemporal
gyri
Collateral
sulcus
Middle cerebellar
peduncle (MCP)
Glossopharyngeal
nerve
Flocculus
Facial nerve
Vagus nerve
Vestibulocochlear
nerve
Abducens nerve
Medulla
Olive (inferior);
olivary eminence
Decussation
of pyramids
Cerebellum (Cbl)
GyRec
OlfSul
OrbGy
Tpole
Trigeminal
nerve
BP
MCP
Fourth
ventricle
Cbl
2-17 Ventral view of the cerebral hemispheres, diencephalon,
brainstem, and cerebellum and two MRIs (both T1-weighted images)
that shows structures from the same perspective. A detailed view of the
ventral aspect of the brainstem is seen in Figure 2-20 on page 24.
BP
The Brain: Gross Views, Vasculature, and MRI 23
Anterior cerebral artery
Internal carotid artery
Middle cerebral artery (MCA)
Optic nerve,
chiasm, and tract
Lenticulostriate
branches of MCA
Posterior communicating artery
Oculomotor nerve
Superior cerebellar artery
Posterior cerebral artery
Trochlear nerve
Basilar artery
Trigeminal nerve
Abducens nerve
Facial and
vestibulocochlear nerves
AICA
Branches of AICA
PICA
PSA
Branches of PICA
Anterior inferior
cerebellar artery (AICA)
Posterior inferior
cerebellar artery (PICA)
Vertebral artery
Posterior spinal artery (PSA)
Anterior spinal artery
2-18 Ventral view of the cerebral hemispheres, diencephalon,
brainstem, and cerebellum, which shows the arterial patterns created
by the internal carotid and vertebrobasilar systems. Note the cerebral
arterial circle (of Willis). Gyri and sulci can be identified by compari-
son with Figure 2-17 (facing page). Details of the cerebral arterial circle and the vertebrobasilar arterial pattern are shown in Figure 2-21 on
page 25. See Figure 8-9 and 8-10 (pp. 248–249) for comparable MRA
of the cerebral arterial circle and its major branches.
Superior ophthalmic vein
–from area of ophthalmic artery
Sphenoparietal sinus
–middle cerebral vein
Cavernous sinus
–cerebral vein
Intercaverous sinuses
Basilar plexus
Superior petrosal sinus
–cerebellar veins
–inferior cerebral veins
–tympanic veins
Inferior petrosal sinus
–veins of pons and medulla
–auditory veins
Sigmoid sinus
Internal jugular vein
Anterior vertebral
venous plexus
Occipital sinus
–posterior internal vertebral
venous plexus
2-19 Ventral view of the cerebral hemispheres, diencephalon,
brainstem, and cerebellum showing the locations and relationships of
Transverse sinus
–emissary veins
–inferior cerebral veins
–inferior cerebellar veins
Sinus confluens
–straight sinus
–superior sagittal sinus
principal sinuses and veins. The listings preceded by a dash (–) under
principal sinuses are the main tributaries of that sinus.
24 External Morphology of the Central Nervous System
Gyrus rectus
Infundibulum
Olfactory tract
Optic nerve
(cranial nerve II)
Optic chiasm
Mammillary body
Optic tract
Interpeduncular
fossa
Basilar pons
Oculomotor nerve
(cranial nerve III)
Crus cerebri
Trochlear nerve
(cranial nerve IV)
Trigeminal nerve
(cranial nerve V)
Parahippocampal
gyrus
Abducens nerve
(cranial nerve VI)
Middle cerebellar peduncle
(brachium pontis)
Facial nerve
(cranial nerve VII)
Intermediate nerve
Flocculus
Olive (inferior);
olivary eminence
Retroolivary sulcus
(postolivary sulcus)
Choroid plexus
Vestibulocochlear nerve
(cranial nerve VIII)
Glossopharyngeal nerve
(cranial nerve IX)
Vagus nerve
(cranial nerve X)
Hypoglossal nerve
(cranial nerve XII)
Preolivary sulcus
(exit of XIIth Nr.)
Pyramid
Accessory nerve
(cranial nerve XI)
Anterior median fissure
Decussation
of pyramids
2-20 Detailed ventral view of the diencephalon and brainstem
with particular emphasis on cranial nerves and related structures. The
dots on the left side represent the approximate position of the roots of
Brs of posterior inferior
cerebellar artery
the hypoglossal nerve on that side; the general position of the (spinal)
accessory nerve is shown on the right by the dark line.
The Brain: Gross Views, Vasculature, and MRI 25
Structures
Vessels
Medial striate artery
Anterior communicating artery
Anterior cerebral artery
Anterior and polar
temporal arteries
A2
A1
Posterior communicating artery
Ophthalamic artery
Internal carotid artery
Olfactory tract
Optic chiasm
Optic nerve
Anterior perforated substance
Uncal artery
Middle cerebral
artery
M1
M2
Optic tract
Mammillary body
Infundibulum
Lenticulostriate arteries
Anterior choroidal artery
Posterior cerebral artery
P1
P2
Posterior choroidal arteries
Quadrigeminal artery
Crus cerebri
Oculomotor nerve (III)
Trochlear nerve (IV)
Basilar pons
Superior cerebellar artery
Trigeminal nerve (V)
Abducens nerve (VI)
Facial nerve (VII)
Middle cerebellar
peduncle
Pontine arteries
Basilar artery
Vestibulocochlear
nerve (VIII)
Anterior inferior
cerebellar artery
Labyrinthine artery
Posterior inferior
cerebellar artery
Posterior spinal artery
Choroid plexus
Glossopharyngeal nerve (IX)
Vagus nerve (X)
Accessory nerve (XI)
Hypoglossal nerve (XII)
Olive (inferior);
olivary eminence
Vertebral artery
Cerebellum
Pyramid
Anterior spinal artery
2-21 Ventral view of the brainstem showing the relationship of
brain structures and cranial nerves to the arteries forming the vertebrobasilar system and the cerebral arterial circle (of Willis). The posterior spinal artery usually originates from the posterior inferior
cerebellar artery (left), but it may arise from the vertebral (right).
Although the labyrinthine artery may occasionally branch from the
basilar (right), it most frequently originates from the anterior inferior cerebellar artery (left). Many vessels that arise ventrally course
around the brainstem to serve dorsal structures. The anterior cerebral artery consists of A1 (between the internal carotid bifurcation
and the anterior communicating artery) and segments A2–A5 which
are distal to the anterior communicating artery (see Figure 8-3 on
p. 242 for details). Lateral to the internal carotid bifurcation is the
M1 segment of the middle cerebral artery (MCA), which divides and
continues as the M2 segments (branches) on the insular cortex. The
M3 branches of the MCA are those located on the inner surface of the
opercula, and the M4 branches are located on the lateral aspect of the
hemisphere. Between the basilar bifurcation and the posterior communicating artery is the P1 segment of the posterior cerebral artery;
P2 is between the posterior communicator and the first temporal
branches. See Figure 8-9, 8-10, and 8-12 (pp. 248, 249, 251) for
comparable MRA of the cerebral arterial circle and vertebrobasilar
system. See Figure 8-12 on p. 251 for blood supply of the choroid
plexus.
26 External Morphology of the Central Nervous System
Lateral geniculate
body
Crus cerebri
Trochlear nerve
Optic tract
Optic chiasm
Middle cerebellar
peduncle
Optic nerve
Infundibulum
Trigeminal nerve
motor root
Vestibulocochlear
nerve
Trigeminal nerve
sensory root
Facial nerve
Olive (inferior),
olivary eminence
Basilar pons
Abducens nerve
Pyramid
Retroolivary sulcus
(postolivary sulcus)
Preolivary sulcus
2-22 Lateral view of the left side of the brainstem emphasizing
structures and cranial nerves on the ventral aspect of the thalamus and
Anterior cerebral
artery
brainstem. Compare with Figure 2-24 on the facing page. The cerebellum and portions of the temporal lobe have been removed.
Olfactory tract
Medial olfactory stria
Optic nerve
Optic chiasm
Optic tract
Posterior perforated
substance
Lateral olfactory stria
Anterior perforated
substance
Infundibulum
Mammillary body
Crus cerebri
Trochlear nerve
Basilar pons
Lateral geniculate body
Trigeminal nerve
Medial geniculate body
Abducens nerve
Middle cerebellar peduncle
Vestibulocochlear nerve
2-23 View of the ventral aspect of the diencephalon and part of the
brainstem with the medial portions of the temporal lobe removed.
Facial nerve
Pyramid
Note structures of the hypothalamus, cranial nerves, and optic structures, including the lateral geniculate body.
The Brain: Gross Views, Vasculature, and MRI 27
Fornix
Choroid plexus, third ventricle
Optic tract
Posterior choroidal arteries
Thalamogeniculate artery
Dorsal thalamus
Lateral geniculate body
Posterior cerebral artery
Medial geniculate body
Mammillary body
Quadrigeminal artery
Superior colliculus
Posterior communicating
artery
Crus cerebri
Brachium of inferior colliculus
Internal carotid artery
Inferior colliculus
Oculomotor nerve
Superior cerebellar artery
Trochlear nerve
Trigeminal nerve
Motor root
Sensory root
Superior cerebellar peduncle
Anterior medullary velum
Basilar artery
Middle cerebellar peduncle
Anterior inferior
cerebellar artery
Vestibulocochlear nerve
Facial nerve
Labyrinthine artery
Abducens nerve
Glossopharyngeal nerve
Vagus nerve
Hypoglossal nerve
Accessory nerve
Posterior inferior
cerebellar artery
Choroid plexus,
fourth ventricle
Restiform body
Cuneate tubercle
Gracile tubercle
Posterior inferior cerebellar artery
Posterior spinal artery
Anterior spinal artery
2-24 Lateral view of the brainstem and thalamus showing the relationship of structures and cranial nerves to arteries. Arteries that serve
dorsal structures originate from ventrally located parent vessels. The
approximate positions of the posterior spinal and labyrinthine arteries,
when they originate from the vertebral and basilar arteries, respec-
Anterior
communicating
artery
Vertebral artery
tively, are shown as dashed lines. Compare with Figure 2-22 on the facing page. See Figure 8-7 (p. 246) for comparable angiogram of the vertebrobasilar system. See Figure 8-12 on p. 251 for blood supply of the
choroid plexus.
Anterior cerebral artery
A2
A1
Middle cerebral artery (M1)
Hypothalamus
Crus cerebri
Red nucleus
Posterior communicating
artery
Posterior cerebral artery
P2
P1
Cerebral aqueduct
Cortical branches of
posterior cerebral artery
2-25 A proton density MRI through basal regions of the hemisphere and through the midbrain showing several major vessels that
form part of the cerebral arterial circle (of Willis). Compare to Figure
2-21 on page 25. See Figure 8-9 and 8-10 (pp. 248–249) for comparable MRA of the cerebral arterial circle.
28 External Morphology of the Central Nervous System
Anterior paracentral gyrus (APGy)
Paracentral sulcus (ParCSul)
Central sulcus (CSul)
Posterior paracentral gyrus (PPGy)
Precentral sulcus (PrCSul)
Marginal sulcus (MarSul)
Precuneus (PrCun)
Cingulate gyrus (CinGy)
Superior frontal
gyrus (SFGy)
Parieto-occipital
sulcus (POSul)
Cingulate sulcus
(CinSul)
Cuneus (Cun)
Calcarine sulcus
(CalSul)
Lingual gyrus
(LinGy)
Sulcus of corpus
callosum (SulCC)
Isthmus of cingulate gyrus
Paraterminal gyri
Parolfactory gyri (ParolfGy)
Temporal pole
Occipitotemporal gyri
Parahippocampal gyrus
Uncus
Rhinal sulcus
APGy
PrCSul
ParCSul
CSul
PPGy
MarSul
SulCC
CinGy
PrCun
CinSul
ParolfGy
POSul
Cun
CalSul
LinGy
SFGy
MarSul
Corpus callosum
POSul
Colloid cyst
CalSul
Internal cerebral
vein
2-26 Midsagittal view of the right cerebral hemisphere and diencephalon, with brainstem removed, showing the main gyri and sulci
and two MRI (both T1-weighted images) showing these structures
from the same perspective. The lower MRI is from a patient with a
small colloid cyst in the interventricular foramen. When compared to
the upper MRI, note the enlarged lateral ventricle with resultant thinning of the corpus callosum.
A colloid cyst (colloid tumor) is a congenital growth usually discovered in adult life once the flow of CSF through the interventricular
foramina is compromised (obstructive hydrocephalus). The patient
may have headache, unsteady gait, weakness of the lower extremities,
visual or somatosensory disorders, and/or personality changes or confusion. Treatment is usually by surgical removal.
The Brain: Gross Views, Vasculature, and MRI 29
Internal frontal branches
Paracentral branches
Callosomarginal branch
of ACA
Internal parietal branches
Parietooccipital
branches of PCA
Pericallosal branch
of ACA
Frontopolar branches
of ACA
Orbital branches of ACA
Anterior cerebral artery (ACA)
Calcarine branch of PCA
Posterior temporal branches of PCA
Posterior cerebral artery (PCA)
Anterior temporal branches of PCA
2-27 Midsagittal view of the cerebral hemisphere and diencephalon showing the locations and branching patterns of anterior and
posterior cerebral arteries. The positions of gyri and sulci can be extrapolated from Figure 2-26 (facing page). Terminal branches of the
anterior cerebral artery arch laterally over the edge of the hemisphere
to serve medial regions of the frontal and parietal lobes, and the same
relationship is maintained for the occipital and temporal lobes by
branches of the posterior cerebral artery. See Figures 8-1 (p. 240) and
8-7 (p. 246) for comparable angiogram of anterior and posterior cerebral arteries.
Inferior sagittal sinus
Posterior vein of corpus callosum
Superior sagittal sinus
Internal occipital veins
TV
Veins of the
caudate nucleus
Straight sinus
Septal veins
Sinus
confluens
Superior
cerebellar vein
Anterior cerebral vein
Basal vein
Internal cerebral vein
2-28 Midsagittal view of the cerebral hemisphere and diencephalon that shows the locations and relationships of sinuses
and the locations and general branching patterns of veins. The
position of gyri and sulci can be extrapolated from Figure 2-26
Great
cerebral vein
Transverse
sinus
Occipital
sinus
(facing page). TV = Terminal vein (superior thalamostriate vein). See
Figures 8-2 (p. 241) and 8-11 (p. 250) for comparable angiogram (venous phase) and MRV showing veins and sinuses.
30 External Morphology of the Central Nervous System
Anterior paracentral gyrus (APGy)
Central sulcus (CSul)
Posterior paracentral gyrus (PPGy)
Paracentral sulcus (PCSul)
Superior frontal gyrus (SFGy)
Marginal sulcus (MarSul)
Precuneus (PCun)
Body of corpus callosum (BCorC)
Sulcus of the
corpus callosum (SulCorC)
Splenium of corpus
callosum (SplCorC)
Cingulate gyrus (CinGy)
Parieto-occipital
sulcus (POSul)
Cingulate sulcus
(CinSul)
Cuneus (Cun)
Lingual gyrus
(LinGy)
Genu of corpus
callosum (GCorC)
Calcarine sulcus
(CalSul)
Septum
Rostrum of corpus
callosum (RCorC)
Fornix (For)
Cerebellum (Cbl)
Midbrain tegmentum (MidTeg)
Basilar pons (BP)
Tonsil of cerebellum (Ton)
Pontine tegmentum (PonTeg)
Medulla (Med)
PCSul
SFGy
CSul
APGy
PPGy
MarSul
PCun
BCorC
SulCorC
SplCorC
POSul
CinGy
Cun
CinSul
GCorC
CalSul
RCorC
LinGy
For
Cbl
MidTeg
BP
PonTeg
Med
Ton
2-29 A midsagittal view of the right cerebral hemisphere and diencephalon with the brainstem and cerebellum in situ. The MRI (T1weighted image) shows many brain structures from the same perspective.
The Brain: Gross Views, Vasculature, and MRI 31
Dorsal thalamus (DorTh)
Body of fornix (For)
Septum pellucidum (Sep)
Massa intermedia
Choroid plexus of third ventricle
Stria medullaris thalami
Interventricular foramen
Column of fornix
Habenula
Anterior
commissure (AC)
Suprapineal
recess
Lamina terminalis
Posterior
commissure
Pineal (P)
Supraoptic recess
Superior
colliculus (SC)
Optic chiasm
(OpCh)
Quadrigeminal
cistern (QCis)
Hyth
Inferior
colliculus (IC)
Optic nerve
Cerebral
aqueduct (CA)
Anterior medullary
velum (AMV)
Fourth ventricle
(ForVen)
Infundibulum (In)
Infundibular recess
Mammillary body (MB)
Hypothalamic sulcus
Posterior inferior
cerebellar artery
Oculomotor nerve
Interpeduncular fossa (IpedFos)
Medulla
Basilar pons (BP)
For
DorTh
Sep
Internal cerebral vein
P
AC
Tentorium cerebelli
Hypothalamus
QCis
OpCh
In
SC
IC
Pituitary gland
AMV
MB
ForVen
IpedFos
BP
CA
2-30 A midsagittal view of the right cerebral hemisphere and diencephalon with the brainstem in situ focusing on the details primarily
related to the diencephalon and third ventricle. The MRI (T1-weighted
image) shows these brain structures from the same perspective. Hyth
⫽ hypothalamus.
32 External Morphology of the Central Nervous System
A
D
Midbrain
Anterior
quadrangular
lobule
Anterior
lobe (AntLb)
Posterior
quadrangular
lobule
Primary
fissure
Posterior
superior
fissure
E
Superior semilunar
lobule
Hemisphere
Bpon
Vermis (Ver)
AntLb
SCP
B
Fourth ventricle
Basilar pons (Bpon)
Medulla (Med)
Flocculus (Fl)
Tonsil (Ton)
F
Biventer
lobule
Med
Gracile
lobule
Ton
Inferior
semilunar
lobule
PostLb
Hemisphere
Vermis (Ver)
Ver
C
Cerebellar peduncles:
Superior (SCP)
Middle (MCP)
Inferior
Anterior
lobe (AntLb)
Colliculi:
Superior
Inferior
G
Primary fissure
AntLb
Horizontal
fissure
MCP
Fl
Flocculus (Fl)
Tonsil (Ton)
Posterior
lobe (PostLb)
Nodulus
2-31 Rostral (A, superior surface), caudal (B, inferior surface),
and an inferior view (C, inferior aspect) of the cerebellum. The view
in C shows the aspect of the cerebellum that is continuous into the
brainstem via cerebellar peduncles. The view in C correlates with superior surface of the brainstem (and middle superior cerebellar peduncles) as shown in Figure 2-34 on page 34.
Note that the superior view of the cerebellum (A) correlates closely
Med
PostLb
with cerebellar structures seen in axial MRIs at comparable levels (D,
E). Structures seen on the inferior surface of the cerebellum, such as
the tonsil (F), correlate closely with an axial MRI at a comparable level.
In G, note the appearance of the margin of the cerebellum, the general
appearance and position of the lobes, and the obvious nature of the
middle cerebellar peduncle. All MRI images are T1-weighted.
The Brain: Gross Views, Vasculature, and MRI 33
A
I
II,III
B
IV
II,III
V
V
Midbrain (Mid)
Primary
fissure (PriFis)
Basilar
pons (Bpon)
PriFis
Mid
VI
VII
Fourth
ventricle
(ForVen)
ForVen
Medulla
(Med)
VIII
VII
Bpon
Med
VIII
X
X
IX
IX
Posterolateral
fissure (PostLatFis)
II,III
C
IV
V
PriFis
Mid
VI
Bpon
ForVen
VII
Med
X
2-32 A median sagittal view of the cerebellum (A) showing its relationships to the midbrain, pons, and medulla. This view of the cerebellum also illustrates the two main fissures and the vermis portions of
lobules I-X. Designation of these lobules follows the method developed by Larsell.
IX
VIII
Lobules I-V are the vermis parts of the anterior lobe; lobules VI-IX
are the vermis parts of the posterior lobe; and lobule X (the nodulus)
is the vermis part of the flocculonodular lobe. Note the striking similarities between the gross specimen (A) and a median sagittal view of
the cerebellum in a T1-weighted MRI (B) and a T2-weighted MRI (C).
Peduncles
Middle cerebellar
Superior cerebellar
Inferior colliculus
Trochlear nerve
Flocculus
Crus cerebri
Trigeminal nerve:
Sensory root
Motor root
2-33 Lateral and slightly rostral view of the cerebellum and brainstem with the middle and superior cerebellar peduncles exposed. Note
the relationship of the trochlear nerve to the inferior colliculus and the
Basilar pons
relative positions of, and distinction between, motor and sensory roots
of the trigeminal nerve. See page 40, Figure 2-41D for an MRI showing the trochlear nerve.
34 External Morphology of the Central Nervous System
Inferior colliculus (IC)
Internal cerebral vein
Frenulum
Pineal
Medial
geniculate
body (MGB)
Superior colliculus (SC)
Pulvinar nuclear
complex (PuNu)
Brachium of
superior
colliculus
PulNu
SC
MGB
LGB
Lateral
geniculate
body (LGB)
IC
Brachium of
inferior
colliculus
Crus cerebri
Crus cerebri
Trochlear nerve
(cranial nerve IV)
Trochlear nerve
(cranial nerve IV)
Superior cerebellar
peduncle
Anterior medullary velum
Facial colliculus
Middle cerebellar
peduncle
Sulcus limitans
Superior fovea
Inferior cerebellar peduncle
(juxtarestiform body and
restiform body)
Striae medullares of
fourth ventricle
Lateral recess of
fourth ventricle
Vestibular area
Restiform body
Tela choroidea (cut edge)
Inferior fovea
Vagal trigone
Level of obex
Hypoglossal trigone
Tuberculum cuneatum (cuneate tubercle)
Tuberculum gracile
(gracile tubercle)
Posterior intermediate sulcus
Posterolateral sulcus
Trigeminal tubercle (tuberculum cinerum)
Cuneate fasciculus
Gracile fasciculus
2-34 Detailed dorsal view of the brainstem, with cerebellum removed, providing a clear view of the rhomboid fossa (and floor of the
fourth ventricle) and contiguous parts of the caudal diencephalon. The
dashed line on the left represents the position of the sulcus limitans and
the area of the inferior cerebellar peduncle is shown on the right. The
Posterior median sulcus
tuberculum cinereum is also called the trigeminal tubercle (tuberculum trigeminale) because it is the surface representation of the spinal
trigeminal tract and its underlying nucleus. Figure 3-10 on page 61 also
shows a comparable view of the brainstem and the posterior portions
of the diencephalon.
The Brain: Gross Views, Vasculature, and MRI 35
Vessels
Structures
Choroid plexus, third ventricle
Pineal
Habenula
Thalamogeniculate arteries
Brachium of superior colliculus
Medial thalamus
Lateral thalamus
Pulvinar nucleus
Superior
colliculus
Internal capsule
Choroid plexus,
lateral ventricle
Medial
and lateral
posterior choroidal arteries
Quadrigeminal artery
Superior cerebellar artery:
Medial branch
Lateral branch
Lateral geniculate body
Medial geniculate body
Brachium of inferior colliculus
Crus cerebri
Trochlear nerve (IV)
Inferior colliculus
Superior cerebellar peduncle
Anterior medullary velum
Facial colliculus
Vestibular area
Inferior cerebellar peduncle
Middle cerebellar peduncle
Choroid plexus, fourth ventricle
Hypoglossal trigone
Anterior inferior
cerebellar artery
Glossopharyngeal nerve (IX)
Vagal nerve (X)
Posterior inferior
cerebellar artery
Accessory nerve (XI)
Restiform body
Vagal trigone
Trigeminal tubercle (tuberculum cinereum)
Cuneate tubercle
Posterior spinal artery
Gracile tubercle
Gracile fasciculus
Cuneate fasciculus
2-35 Dorsal view of the brainstem and caudal diencephalon showing the relationship of structures and some of the cranial nerves to arteries. The vessels shown in this view have originated ventrally and
wrapped around the brainstem to gain their dorsal positions. In addi-
tion to serving the medulla, branches of the posterior inferior cerebellar artery also supply the choroid plexus of the fourth ventricle. The
tuberculum cinereum is also called the trigeminal tubercle. See Figure
8-12 on p. 251 for blood supply of the choroid plexus.
36 External Morphology of the Central Nervous System
Medial
geniculate body
Lateral geniculate
body
Brachium of
inferior colliculus
Superior colliculus
Crus cerebri
Inferior colliculus
Trochlear nerve
Superior cerebellar
peduncle
Motor root of
trigeminal nerve
Middle cerebellar
peduncle
Sensory root of
trigeminal nerve
Vestibulocochlear
nerve
Lateral recess of
fourth ventricle
Restiform body
Basilar pons
Posterior inferior
cerebellar artery
Tuberculum
cinereum
(trigeminal tubercle)
2-36 Lateral view of the left side of the brainstem emphasizing
structures that are located dorsally. The cerebellum and portions of the
temporal lobe have been removed. Compare with Figure 2-38 on the
facing page.
Superior cerebellar peduncle
Medial eminence
of fourth ventricle
Facial colliculus
Middle
cerebellar peduncle
Superior fovea
Vestibular area
Striae medullares
Lateral recess
Foramen of Luschka
Hypoglossal trigone
Vagal trigone
Sulcus limitans
Restiform body
Inferior fovea
Cuneate tubercle
Gracile tubercle
Tela choroidea
(cut edge)
2-37 The floor of the fourth ventricle (rhomboid fossa) and immediately adjacent structures. Also compare with Figure 2-34 on page 34.
The Brain: Gross Views, Vasculature, and MRI 37
Fornix
Choroid plexus, third ventricle
Optic tract
Posterior choroidal arteries
Thalamogeniculate artery
Dorsal thalamus
Lateral geniculate body
Posterior cerebral artery
Medial geniculate body
Mammillary body
Quadrigeminal artery
Superior colliculus
Posterior communicating
artery
Crus cerebri
Brachium of inferior colliculus
Internal carotid artery
Inferior colliculus
Oculomotor nerve
Superior cerebellar artery
Trochlear nerve
Trigeminal nerve
Motor root
Sensory root
Superior cerebellar peduncle
Anterior medullary velum
Basilar artery
Middle cerebellar peduncle
Anterior inferior
cerebellar artery
Vestibulocochlear nerve
Facial nerve
Labyrinthine artery
Abducens nerve
Glossopharyngeal nerve
Vagus nerve
Hypoglossal nerve
Accessory nerve
Posterior inferior
cerebellar artery
Choroid plexus,
fourth ventricle
Restiform body
Cuneate tubercle
Gracile tubercle
Posterior inferior cerebellar artery
Posterior spinal artery
Anterior spinal artery
2-38 Lateral view of the brainstem and thalamus, which shows the
relationship of structures and cranial nerves to arteries. The approximate positions of the labyrinthine and posterior spinal arteries, when
they originate from the basilar and vertebral arteries, respectively, are
Vertebral artery
shown as dashed lines. Arteries that distribute to dorsal structures originate from large ventral vessels. Compare with Figure 2-36 on the facing page.
38 External Morphology of the Central Nervous System
A
Optic nerve
Optic chiasm
Infundibulum
Optic tract
Crus cerebri
Mammillary
body
Interpeduncular
fossa
B
Optic nerve
Bulb of eye
Optic chiasm
Temporal lobe
Mammillary
body
Optic tract
Interpeduncular
fossa
Uncus
Midbrain
tegmentum
Crus cerebri
C
Dorsal thalamus
Frontal lobe
Interpeduncular
fossa
Optic nerve
Basilar pons
Bulb of eye
D
Anterior communicating
artery
Optic chiasm
Optic tract
Infundibulum
Optic nerve
Anterior cerebral
artery, A1 segment
Interpeduncular
fossa
Midbrain
2-39 Inferior view of the hemisphere showing the optic nerve (II),
chiasm, tract, and related structures (A). The MRIs of cranial nerve II
are shown in axial (B, T1-weighted; D, T2-weighted) and in oblique
sagittal (C, T1-weighted) planes. Note the similarity between the axial planes, especially (B), and the gross anatomical specimen. In addition, note the relationship between the anterior cerebellar artery, anterior communicating artery, and the structures around the optic
chiasm (D).
The anterior communicating artery or its junction with the anterior
cerebral artery (D) is the most common site of supratentorial (carotid
system) aneurysms. Rupture of aneurysms at this location is one of the
more common causes of spontaneous subarachnoid hemorrhage. The
proximity of these vessels to optic structures and the hypothalamus (D)
explain the variety of visual and hypothalamic disorders experienced by
these patients. A lesion of the optic nerve results in blindness in that
eye and loss of the afferent limb of the pupillary light reflex. Lesions
in, or caudal to, the optic chiasm result in deficits in the visual fields of
both eyes.
The Cranial Nerves 39
artery
Optic chiasm
Middle cerebral
artery
Posterior communicating
artery
Posterior cerebral
artery
Oculomotor nerve
Basilar artery
Superior cerebellar
artery
Basilar pons
B
Optic tract
Posterior cerebral
artery
Bulb of the eye
Superior cerebellar
artery
Oculomotor nerve
C
Internal carotid
artery
Oculomotor
nerve
Oculomotor
nerve
Temporal lobe
Uncus
Basilar pons
(rostral portion)
Fourth ventricle
(rostral portion)
D
Corpus callosum
Dorsal thalamus
Frontal lobe
Interpeduncular
fossa
Optic chiasm
Oculomotor nerve
2-40 Inferior view of the hemisphere showing the exiting fibers of
the oculomotor nerve (III), and their relationship to the posterior cerebral and superior cerebellar arteries (A). The MRIs of cranial nerve III
are shown in sagittal (B, T2-weighted; D, T1-weighted) and in axial
(C, T1-weighted) planes. Note the relationship of the exiting fibers of
the oculomotor nerve to the posterior cerebral and superior cerebellar arteries (A, B) and the characteristic appearance of the III nerve as
it passes through the subarachnoid space toward the superior orbital fissure (C). The sagittal section (D) is just off the midline and shows the
Superior colliculus
Inferior colliculus
Cerebellum
Basilar pons
position of the oculomotor nerve in the interpeduncular fossa rostral
to the basilar pons and caudal to optic structures.
That portion of the posterior cerebral artery located between the
basilar artery and the posterior communicating artery (A) is the P1 segment. The most common site of aneurysms in the infratentorial area
(vertebrobasilar system) is at the bifurcation of the basilar artery, also
called the basilar tip. Patients with aneurysms at this location may present with eye movement disorders and pupillary dilation due to damage to the root of the third nerve (A,B).
40 External Morphology of the Central Nervous System
A
Mammillary body
Lamina terminalis
Interpeduncular
fossa
Supraoptic recess
Cerebral aqueduct
Optic chiasm
Oculomotor nerve
Basilar pons
Infundibular recess
B
Optic tract
Posterior cerebral
artery
Superior cerebellar
artery
Optic nerve
Oculomotor nerve
Basilar pons
C
Oculomotor nerve
Posterior cerebral
artery
Posterior cerebral
artery
Interpeduncular
fossa
Superior cerebellar
artery
Crus cerebri
Midbrain
Anterior lobe
of cerebellum
D
Anterior cerebral artery
Middle cerebral artery
Hypothalamus
Interpeduncular fossa
Crus cerebri
Midbrain tegmentum
Optic tract
Mammillary body
Posterior cerebral artery
Trochlear nerve
(in ambient cistern)
Fourth ventricle
(rostral portion)
2-41 A median sagittal view of the brainstem and diencephalon (A)
reveals the position of the oculomotor nerve (III) in relation to adjacent
structures. The MRI in B and C show the position of the oculomotor
nerve in sagittal (B, T1-weighted) and in axial (C, T2-weighted) planes.
Note the relationship of the oculomotor nerve to the adjacent posterior
cerebral and superior cerebellar arteries (B, C). Also compare these images with that of figure 2-40B on page 39. In D (T2-weighted), the
trochlear nerve is seen passing through the ambient cistern around the
lateral aspect of the midbrain (compare with Fig. 2-32 on page 32).
The oculomotor (III) and trochlear (IV) nerves are the cranial
nerves of the midbrain. The third nerve exits via the interpeduncular
fossa to innervate four major extraocular muscles (see Fig. 7-15 on
page 201) and, through the ciliary ganglion, the sphincter pupillae
muscles. Damage to the oculomotor nerve may result in paralysis of
most eye movement, a dilated pupil, and loss of the efferent limb of
the pupillary light reflex, all in the ipsilateral eye. The fourth nerve is
unique in that it is the only cranial nerve to exit the posterior (dorsal)
aspect of the brainstem and is the only cranial nerve motor nucleus to
innervate, exclusively, a muscle on the contralateral side of the midbrain.
The Cranial Nerves 41
A
Basilar pons
Trigeminal nerve
Abducens nerve
Facial nerve
Flocculus
Vestibulocochlear
nerve
Pyramid
B
Internal carotid artery
Temporal lobe
Trigeminal nerve
Basilar artery
Middle cerebellar
peduncle
Trigeminal nerve
Basilar pons
Fourth ventricle
Pontine tegmentum
Cerebellum
C
Temporal lobe
Trigeminal
ganglion
Trigeminal
ganglion
Basilar artery
Superior cerebellar
artery
Trigeminal nerve
Tegmentum
of pons
Basilar pons
Fourth ventricle
Anterior lobe
of cerebellum
D
Midbrain
tegmentum
Root of trigeminal
nerve
E
Third ventricle
Crus cerebri
Interpeduncular
fossa
Sensory root of the
trigeminal nerve
Basilar pons
Basilar pons
Pyramid
2-42 The trigeminal nerve (V) is the largest of the cranial nerve
roots exiting the brainstem (A). It exits at an intermediate position on
the lateral aspect of the pons roughly in line with cranial nerves VII, IX,
and X. The fifth nerve, and these latter three, are mixed nerves in that
they have motor and sensory components. The trigeminal nerve is
shown in axial MRI (B, T1-weighted; C, T2-weighted) and in coronal
planes (D, E, both T1-weighted images). Note the characteristic appearance of the root of the trigeminal nerve as it traverses the subarachnoid space (B and C), the origin of the trigeminal nerve, and the
position of the sensory root of the nerve at the lateral aspect of the pons
in the coronal plane (D, E). In addition, the MRI in C clearly illustrates
the position of the trigeminal ganglion in the middle cranial fossa.
Trigeminal neuralgia (tic douloureux) is a lancinating paroxysmal
pain within the V2–V3 territories frequently triggered by stimuli
around the corner of the mouth. The causes are probably multiple and
may include neurovascular compression by the superior cerebellar
artery (see the apposition of this vessel to the nerve root in C), multiple sclerosis, tumors, and ephaptic transmission within the nerve or
ganglion.
42 External Morphology of the Central Nervous System
A
Abducens nerve
Facial nerve
Vestibulocochlear
nerve
Facial nerve
Pyramid
Olivary eminence
B
Cochlea
Vestibulocochlear
nerve
Glossopharyngeal
nerve
Vagus nerve
Hypoglossal nerve
Basilar artery
Abducens nerve
Semicircular canals
Pons-medulla junction
Vestibulocochlear
nerve
Lateral recess of
fourth ventricle
Facial nerve
Fourth ventricle
Tonsil of cerebellum
C
Cochlea
Semicircular canals
Abducens nerve
Cochlea
Cochlear portion of
VIIIth nerve (CPVIII)
VPVIII
Semicircular canals
CPVIII
Vestibular portion of
VIIIth nerve (VPVIII)
Fourth ventricle
Basilar pons
Cerebellum
D
Anterior inferior
cerebellar artery
Cochlea
Pontine tegmentum
Cochlear portion of
VIIIth nerve
Cochlea
Semicircular canals
Semicircular canals
Cerebellar vermis
Cerebellar
hemisphere
2-43 The cranial nerves at the pons medulla junction are the abducens (VI), the facial (VII), and the vestibulocochlear (VIII) (A). The
facial and vestibulocochlear nerves both enter the internal acoustic
meatus, the facial nerve distributing eventually to the face through the
stylomastoid foramen, and the vestibulocochlear nerve to structures
of the inner ear. MRI in the axial plane, B, C, D, (all T2-weighted images) show the relationships of the vestibulocochlear root and the facial nerve to the internal acoustic meatus. Also notice the characteristic appearance of the cochlea (B, C) and the semicircular canals (C).
In addition to these two cranial nerves, the labyrinthine branch of the
Vestibular portion of
VIIIth nerve
Cerebellar
tonsil
anterior inferior cerebellar artery also enters the internal acoustic
meatus.
The so-called acoustic neuroma, a tumor associated with the eighth
nerve, is actually a vestibular schwannoma since it arises from the
neurilemma sheath of the vestibular root. Most patients with this tumor have hearing loss, tinnitus and equilibrium problems, or vertigo.
As the tumor enlarges (to more than about 2 cm) it may cause facial
weakness (seventh root), numbness (fifth root), or abnormal corneal
reflex (fifth or seventh). Treatment is usually by surgery, radiation
therapy, or a combination thereof.
The Cranial Nerves 43
A
Abducens nerve
Olivary eminence
Facial nerve
Vestibulocochlear
nerve
Glossopharyngeal
nerve
Vagus nerve
Postolivary sulcus
Preolivary sulcus
Hypoglossal nerve
B
Preolivary sulcus
Retroolivary sulcus
(postolivary sulcus)
Glossopharyngeal
nerve
Pyramid
Olive (inferior)
Glossopharyngeal
nerve
Flocculus
Fourth ventricle
Restiform body
Tonsil of cerebellum
Cerebellum
C
Pyramid
Olive (inferior)
Retroolivary sulcus
Vagus nerve
Fourth ventricle
Tonsil of cerebellum
Cerebellum
D
Pyramid
Olivary eminence
Vagus nerve
Postolivary sulcus
Restiform body
Vagus nerve
Fourth ventricle
Tonsil of cerebellum
2-44 The glossopharyngeal (IX) and vagus (X) nerves (A) exit the
lateral aspect of the medulla via the postolivary sulcus; the ninth nerve
exits rostral to the row of rootlets comprising the tenth nerve (A).
These nerves are generally in line with the exits of the facial and
trigeminal nerves; all of these are mixed nerves. The exit of the glossopharyngeal nerve (A, B) is close to the pons–medulla junction and
correlates with the corresponding shape (more rectangular) of the
medulla. The vagus nerve exits at a slightly more caudal position (A,
C, D); the shape of the medulla is more square and the fourth ventri-
cle is smaller. The ninth and tenth cranial nerves and the spinal portion
of the accessory nerve (XI) exit the skull via the jugular foramen.
Glossopharyngeal neuralgia is a lancinating pain originating from
the territories served by the ninth and tenth nerves at the base of the
tongue and throat. Trigger events may include chewing and swallowing. Lesions of nerves passing through the jugular foramen (IX, X, XI)
may result in loss of the gag reflex (motor limb via ninth nerve), and
drooping of the ipsilateral shoulder accompanied by an inability to turn
the head to the opposite side against resistance (eleventh nerve).
44 External Morphology of the Central Nervous System
A
Abducens nerve
Olivary eminence
Facial nerve
Vestibulocochlear
nerve
Glossopharyngeal
nerve
Vagus nerve
Postolivary sulcus
Preolivary sulcus
B
Hypoglossal nerve
Pyramid
Preolivary sulcus
Olivary eminence
Hypoglossal nerve
Postolivary sulcus
Restiform body
Vagus nerve
Tonsil of cerebellum
C
Hypoglossal nerve
Medulla
Tonsil of cerebellum
Cerebellum
2-45 The hypoglossal nerve (XII) (A) exits the inferolateral aspect
of the medulla via the preolivary sulcus. This cranial nerve exits in line
with the abducens nerve found at the pons–medulla junction and in line
with the exits of the third and fourth nerves of the midbrain. The
twelfth nerve exit is characteristically located laterally adjacent to the
pyramid, which contains corticospinal fibers.
In axial MRI (B, T2-weighted; C, T1-weighted), note the charac-
teristic position of the hypoglossal nerve in the subarachnoid space and
its relation to the overall shape of the medulla. This shape is indicative
of a cranial nerve exiting at more mid-to-caudal medullary levels. In B,
note its relationship to the preolivary sulcus and olivary eminence. The
hypoglossal exits the base of the skull by traversing the hypoglossal
canal. A lesion of the hypoglossal nerve results in a deviation of the
tongue to the ipsilateral side on attempted protrusion.
The Insula 45
Precentral gyrus (PrCGy)
Superior frontal
gyrus
Central sulcus (CSul)
Postcentral gyrus
(PoCGy)
Middle frontal
gyrus (MFGy)
Gyri longi (GyLon–long
gyri of the insula)
Gyri breves
(GyBr–short gyri
of the insula)
Transverse temporal
gyrus (TrTemGy)
Central sulcus
of the insula (CSulIn)
Limen insulae (LimIn)
Temporal lobe (TLob)
PoCGy
PrCGy
CSul
MFGy
TrTemGy
GyLon
GyBr
CSulIn
LimIn
TLob
CSul
PrCGy
PoCGy
MFGy
CSulIn
GyBr
GyLon
TLob
2-46 Lateral view of the left cerebral hemisphere with the cortex
overlying the insula removed. Structures characteristic of the insular
cortex, and immediately adjacent areas, are clearly seen in the two
MRIs in the sagittal plane through lateral portions of the hemisphere
(inversion recovery—upper; T1-weighted image—lower).
46 External Morphology of the Central Nervous System
Comparison of Cerebral versus Spinal Meninges
CEREBRAL
SPINAL
Dura
Dura
• adherent to inner table of skull (no epidural space)
• composed of two fused layers (periosteal and meningeal),
which split to form sinuses
• separated from vertebrae by epidural space
• composed of one layer (spinal dura only; vertebrae
have their own periosteum)
Arachnoid (outer part of leptomeninges)
Arachnoid (outer part of leptomeninges)
• attached to dura in living condition (no subdural space)
• arachnoid villi (in superior sagittal sinus)
• arachnoid trabeculae
• subarachnoid space with many cisterns
• attached to dura in living condition (no subdural
space)
• no arachnoid villi
• few or no arachnoid trabeculae but larger arachnoid septae
• subarachnoid space with one cistern
Pia (inner part of leptomeninges)
Pia (inner part of leptomeninges)
• intimately adherent to surface of brain
• no pial specializations
• intimately adherent to surface of cord
• specializations in the form of denticulate ligaments,
filum terminale, and linea splendens
• follows vessels as they pierce the cord
• follows vessels as they pierce the cerebral cortex
Meningitis, Meningeal, Hemorrhages, Meningioma
A wide variety of disease processes and lesions may involve the
meninges; only a few examples are mentioned here.
Bacterial infections of the meninges (bacterial meningitis) are
commonly called leptomeningitis because the causative organisms are
usually found in the subarachnoid space and involve the pia and arachnoid. The organism seen in about one-half of adult cases is Streptococcus
pneumoniae, while in neonates and children up to about 1 year it is Escherichia coli. The patient becomes acutely ill (i.e., confusion, fever, stiff
neck, stupor), may have generalized or focal signs/symptoms, and, if
not treated rapidly, will likely die. Treatment is with appropriate antibiotics. Patients with viral meningitis may become ill over a period
of several days, experience headache, confusion, and fever, but, with
supportive care, will recover after an acute phase of approximately 1–2
weeks. These patients usually recover with no permanent deficits.
The most common cause of an epidural (extradural) hematoma
is a skull fracture that results in a laceration of a major dural vessel, such
as the middle meningeal artery. In approximately 15% of cases, bleeding may come from a venous sinus. The extravasated blood dissects the
dura mater off the inner table of the skull; there is no preexisting (extradural) space for the blood to enter. These lesions are frequently
large, lens (lenticular) shaped, may appear loculated, and are “short
and thick” compared to subdural hematomas (see Fig. 2-48 on page
48). The patient may lapse into a coma and, if the lesion is left untreated, death may result. In some cases, the patient may initially be
unconscious followed by a lucid interval (the patient is wide awake),
then subsequently deteriorate rapidly and die; this is called “talk and
die.” Treatment of choice for large lesions is surgical removal of the
clot and coagulation of the damaged vessel.
Tearing of bridging veins (veins passing from the brain outward
through the arachnoid and dura), usually the result of trauma, is a common cause of subdural hematoma. This designation is somewhat a
misnomer because the extravasated blood actually dissects through a
specialized, yet structurally weak, cell layer at the dura-arachnoid interface; this is the dural border cell layer. There is no preexisting “subdural space” in the normal brain. Acute subdural hematomas, more
commonly seen in younger patients, are usually detected immediately
or within a few hours after the precipitating incident. Chronic subdural
hematomas, usually seen in the elderly, are frequently of unknown origin; may take days or weeks to become symptomatic; and cause a progressive change in mental status of the patient. This lesion appears
“long and thin,” compared to an epidural hematoma, follows the surface of the brain, and may extend for considerable distances (see Fig.
2-48 on p. 48 and Fig. 2-51 on p. 51). Treatment is surgical evacuation (for larger or acute lesions) or close monitoring for small, asymptomatic, or chronic lesions.
The most common cause of subarachnoid hemorrhage is trauma.
In approximately 80% of patients with spontaneous (nontraumatic) subarachnoid hemorrhage, the precipitating event is rupture
of an intracranial aneurysm. Symptomatic bleeding from an arteriovenous malformation occurs in approximately 5% of cases. Blood collects
in, and percolates through, the subarachnoid space and cisterns (see
Fig. 2-51 on page 51). Sometimes, the deficits seen (assuming the patient is not in coma) may be a clue as to location, especially if cranial
nerves are nearby. Onset is sudden; the patient complains of an excruciating headache and may remain conscious, become lethargic and disoriented, or may be comatose. Treatment of an aneurysm is to surgically occlude it (by clip or coil), if possible, and to protect against the
development of vasospasm. During surgery, some blood in the subarachnoid space and cisterns may be removed.
Tumors of the meninges (meningiomas) are classified in different
ways but they usually arise from arachnoid cap/stem cells (a small
number are dural in origin) around the villi or at places where vessels
or cranial nerves penetrate the dura-arachnoid. These tumors grow
slowly (symptoms may develop almost imperceptibly over years), are
histologically benign, may result in hyperostosis of the overlying skull,
and frequently contain calcifications. In decreasing order, meningiomas are found in the following locations: parasagittal area ⫹ falx
(together 29%), convexity 15%, sella 13%, sphenoid ridge 12%, and
olfactory groove 10%. Treatment is primarily by surgical removal, although some meningiomas are treated by radiotherapy.
The Meninges, Cisterns, and Meningeal and Cisternal Hemorrhages 47
Superior sagittal sinus
Arachnoid villus
Lateral lacunae
Skull
Cerebrum
Dura mater
Arachnoid mater
Arachnoid trabeculae
Pia mater
Transverse
sinus
Falx cerebri
Tentorium
cerebelli
Cerebellum
Cistern
Skull
Dura mater
Subarachnoid space
Arachnoid mater
Cerebral vessel
and branch
Pia mater
Arachnoid trabeculae
Vertebrae
Spinal nerves
Spinal vessel
Dura mater
Dura mater
Intervertebral
ligament
Epidural space
Conus medullaris
Vertebra
Cauda equina
Lumbar cistern
Arachnoid mater
Filum terminale (interum)
Denticulate ligament
Pia mater
Coccygeal ligament (filum terminale externum)
Coccyx
2-47 Semidiagrammatic representation of the central nervous system and its associated meninges. The details show the relationships of
the meninges in the area of the superior sagittal sinus, on the lateral aspect of the cerebral hemisphere, and around the spinal cord. Cerebrospinal fluid is produced by the choroid plexi of lateral, third, and
fourth ventricles. It circulates through the ventricular system (small arrows) and enters the subarachnoid space via the medial foramen of Magendie and the two lateral foramen of Luschka. In the living situation
the arachnoid is attached to the inner surface of the dura. There is no
actual or potential subdural space.
48 External Morphology of the Central Nervous System
A
C
B
D
Hemorrhage in brain
2-48 Examples of epidural (extradural) hemorrhages (A, B) and of
acute (C) and subacute (D) subdural hematoma. Note the lenticular
shape of the epidural lesions (A, B), their loculated appearance, and
their location external to the substance of the brain. In contrast, the
acute subdural lesion (C) is quite thin and extends over a longer distance on the cortex.
In D, the subdural hematoma has both chronic and subacute phases.
The chronic phase is indicated by the upper two and lower two arrows
where the blood is replaced by fluid, and the subacute phase by the
middle arrow where fresher blood has entered the lesion. Note the extent of this lesion on the surface of the cortex and its narrowness compared to epidural lesions. The patient in D also has small hemorrhages
into the substance of the brain, the larger of these in the region of the
genu of the internal capsule. Images A–D are CT. For additional comments on epidural and subdural hemorrhages see page 46.
The Meninges, Cisterns, and Meningeal and Cisternal Hemorrhages 49
A
B
C
Blood in frontal lobe
Temporal horn
Blood in third
ventricle
Blood in cerebral
aqueduct
2-49 Examples of hemorrhages into the substance of the brain that,
in some cases, have also resulted in blood in the ventricular system.
The large hemorrhages into the hemisphere (A, B) have resulted in enlargement of the ventricles, a midline shift, and, in the case of A, a
small amount of blood in the posterior horn of the lateral ventricle. In
these examples, the lesion is most likely a result of hemorrhage from
lenticulostriate branches of the M1 segment.
Blood in the substance of the brain and in the ventricular system may
also result from trauma (C). In this example (C), blood is seen in the
frontal lobe and in the third ventricle and cerebral aqueduct. The enlarged temporal horns (C) of the lateral ventricles are consistent with
the interruption of CSF flow through the cerebral aqueduct (noncommunicating hydrocephalus). Images A–C are CT.
50 External Morphology of the Central Nervous System
A
Paracallosal cistern
Quadrigeminal cistern
Lamina terminalis
cistern
B
Chiasmatic cistern
Fourth ventricle
Interpeduncular cistern
C
Prepontine cistern
Premedullary cistern
D
Cisterna magna
B
Lamina terminalis cistern
Optic tract
Sylvian cistern
Crural cistern
Midbrain
Interpeduncular cistern
Ambient cistern
Quadrigeminal cistern
Inferior colliculus
C
Prepontine cistern
Trigeminal nerve
Basilar artery
Superior
cerebellopontine
cistern
Basilar pons
Fourth ventricle
D
Premedullary cistern
Medulla
Cisterna magna
2-50 A median sagittal MRI (A, T2-weighted) of the brain showing the positions of the major cisterns associated with midline structures. Axial views of the midbrain (B, T1-weighted), pons (C,
T2-weighted), and medulla (D, T2-weighted) represent the corresponding planes indicated in the sagittal view (A).
Cisterns are the enlarged portions of the subarachnoid space that
Inferior
cerebellopontine
cistern
contain arteries and veins, roots of cranial nerves, and, of course, cerebrospinal fluid. Consequently, the subarachnoid space and cisterns are
continuous one with the other. In addition, the subarachnoid space
around the brain is continuous with that around the spinal cord. Compare these cisterns with blood-filled parts of the subarachnoid space and
cisterns in Figure 2-51 on the facing page.
The Meninges, Cisterns, and Meningeal and Cisternal Hemorrhages 51
A
B
Subdural
hemorrhage
Lamina terminalis
cistern
Supraoptic recess
Sylvian
cistern
Interpeduncular
cistern
Crural
cistern
Temporal
horn
Blood on
insular cortex
Midbrain
Ambient
cistern
Quadrigeminal
cistern
C
Lamina terminalis
cistern
Blood on
insula
Interpeduncular
cistern
Cerebellum
Blood on tentorium
cerebelli
2-51 Blood in the subarachnoid space and cisterns. In these CT examples, blood occupies the subarachnoid space and cisterns, outlining
these areas in white. Consequently, the shape of the cisterns is indicated by the configuration of the white area, the white area representing blood.
Around the base of the brain (A), it is easy to identify the cisterns
related to the midbrain, the supraoptic recess which is devoid of blood,
and blood extending laterally into the Sylvian cistern. In some cases
(B), subdural hemorrhage may penetrate the arachnoid membrane and
result in blood infiltrating between gyri, such as this example with
D
Third
ventricle
Sylvian
cistern
Crural
cistern
Ambient
cistern
Rostral part
of fourth
ventricle
blood on the cortex of the insula. In C, the blood is located around the
midbrain (crural and ambient cisterns), extends into the Sylvian cistern, and into the cistern of the lamina terminalis. The sharp interface
between the lamina terminalis cistern (containing blood) and the third
ventricle (devoid of blood) represents the position of the lamina terminalis. In D, blood is located in cisterns around the pons but avoids
the rostral part of the fourth ventricle. Compare these images with the
locations of some of the comparable cisterns as seen in Figure 2-50 on
the facing page. Images A–D are CT.
52 External Morphology of the Central Nervous System
Massa intermedia
Body of lateral
ventricle
Pineal recess
Suprapineal recess
Third ventricle
Anterior horn of
lateral ventricle
Posterior commissure
Pineal
Atrium of lateral ventricle
(and glomus choroideum)
Interventricular
foramen
Posterior horn of
lateral ventricle
Anterior commissure
Tectum
Lamina terminalis
Cerebral aqueduct
Infundibular recess
Supraoptic recess
Fourth ventricle
Optic chiasm
Infundibulum
Lateral recess of fourth ventricle
Mammillary body
Amygdaloid nuclear complex
Foramen of
Luschka
Dorsal cerebellomedullary
cistern (cisterna magna)
Inferior horn of lateral ventricle
Bordering Structures
Ventricular Space
Genu of corpus callosum
Head of caudate nucleus
Anterior horn of lateral ventricle
Septum pellucidum
Body of lateral ventricle
(ventral to body of corpus callosum)
Body of caudate nucleus
Third ventricle
Fornix
Suprapineal recess
Amygdaloid nuclear complex
Inferior horn of
lateral ventricle
Tail of caudate nucleus
Hippocampal formation
Splenium of
corpus callosum
Optic radiations
Tapetum
Cerebral aqueduct
Atrium of lateral ventricle
(contains glomus choroideum)
Lateral recess of
fourth ventricle
Fourth ventricle
Posterior horn of lateral ventricle
2-52 Lateral (above) and dorsal (below) views of the ventricles and
the choroid plexus. The dashed lines show the approximate positions
of some of the important structures that border on the ventricular
space. The choroid plexus is shown in red and structures bordering on
the various portions of the ventricular spaces are color-coded; these
colors are continued in Figure 2-53 on the facing page. Note the relationships between the choroid plexus and various parts of the ventricular system. The large expanded portion of the choroid plexus found
in the area of the atrium is the glomus (glomus choroideum). See Figure 8-12 on p. 251 for details of blood supply to the choroid plexus.
The Ventricles and Ventricular Hemorrhage 53
Corpus callosum
(body)
A
Caudate nucleus
Corpus callosum
(body)
B
Anterior horn of
lateral ventricle
Septum
pellucidum
Septum
pellucidum
C
Caudate nucleus (body)
Stria terminalis
Corpus callosum (body)
Body of lateral
ventricle
Fornix (F)
Caudate nucleus
(head)
Corpus callosum
(rostrum)
Body of lateral
ventricle
Interventricular
foramen
Anterior
commissure
F
Choroid plexus (CP)
Fornix
CP
Third ventricle
Third ventricle
Gyrus rectus
Dorsal thalamus
Hypothalamus
Massa intermedia
Optic chiasm
Mammillary body
A
B C
D
Hypothalamus
E
F
D
Caudate nucleus
G
Fornix
Dorsal thalamus
Third ventricle
Amygdaloid nuclear
complex
Optic tract
Inferior horn of
lateral ventricle
E
Body of lateral
ventricle
Corpus callosum
Pulvinar
G
Optic radiations
Tapetum
F
Optic radiations
Tapetum
Pineal
Cerebral aqueduct
Inferior horn of
lateral ventricle
Hippocampal formation
Corpus callosum
(splenium)
Atrium of lateral
ventricle
Calcarine
sulcus
Calcar avis
Caudate nucleus (tail)
Hippocampal formation
Posterior horn of
lateral ventricle
2-53 Lateral view of the ventricular system and corresponding
semidiagrammatic cross-sectional representations from rostral (A) to
caudal (G) identifying specific structures that border on the ventricular space. In the cross-sections, the ventricle is outlined by a heavy line,
and the majority of structures labeled have some direct relevance to the
ventricular space at that particular level. The color-coding corresponds
to that shown in Figure 2-52 on the facing page.
54 External Morphology of the Central Nervous System
A
B
Anterior horn of
lateral ventricle
Anterior horn
Third
ventricle
Atrium of
lateral ventricle
Posterior horn of
lateral ventricle
C
Temporal horn of
lateral ventricle
Basilar pons
Tegmentum of pons
Fourth ventricle
Cerebellum
D
Pons-medulla junction
Fourth ventricle
Lateral recess of
fourth ventricle
Cerebellum
2-54 Examples of hemorrhage occupying portions of the ventricular system (ventricular hemorrhage). In these CT images, blood appears white within the ventricles. Consequently, the shape of the ventricular system is outlined by the white area, and the specific portion
of the ventricular system is correspondingly labeled.
Note blood in the anterior horn, atrium, and posterior horn of the
lateral ventricles (A, B), and blood clearly outlining the shape of the
third ventricle (B). Blood also clearly outlines central portions of the
fourth ventricle (C) and caudal portions of the fourth ventricle (D), including an extension of blood into the left lateral recess of the fourth
ventricle. In addition to these images, Figure 2-49 on page 49 shows
blood in the cerebral aqueduct and in the most inferior portions of the
third ventricle. Images A–D are CT.
CHAPTER
3
Dissections of the
Central Nervous System
56 Dissections of the Central Nervous System
Central sulcus
Gyri:
Precentral
Postcentral
Superior parietal lobule
Supramarginal gyrus
Long insular gyri
Short insular gyri
Central sulcus of insula
Gyri:
Transverse temporal
Superior temporal
Superior temporal sulcus
3-1 Lateral view of the right cerebral hemisphere with the inferior
and parts of the middle frontal gyri and precentral and postcentral gyri
removed to show the insular cortex, transverse temporal gyri, and related structures.
Superior longitudinal
fasciculus
Uncinate fasciculus
External capsule
3-2 Dissection of the lateral aspect of the right cerebral hemisphere
showing the locations and relationships of some of the main bundles of
subcortical white matter. This dissection is deep to that shown in Figure 3-1 (above) and superficial to that shown in Figure 3-3 on page 57.
Lateral, Medial, and Ventral Aspects 57
Corona radiata
Superior
longitudinal
fasciculus
Lenticular
nucleus
Uncinate fasciculus
3-3 Dissection of the lateral aspect of the right cerebral hemisphere
showing the relationship between fibers radiating from the internal
capsule (corona radiata) and those of the superior longitudinal fascicu-
Occiptofrontal
fasciculus
lus. The lenticular nucleus is shown in situ, lateral to the internal capsule. This is a deeper dissection of the specimen shown in Figure 3-2
on page 56.
Internal capsule (IC):
Posterior limb
Genu
Anterior limb
Optic radiations
Retrolenticular limb of IC
3-4 Dissection of the lateral aspect of the right cerebral hemisphere
showing the internal capsule and the concavity left by removal of the
lenticular nucleus. Note the other bundles of subcortical white matter.
This is a deeper dissection of the specimen shown in Figure 3-3
(above).
58 Dissections of the Central Nervous System
Cingulum
Calcarine sulcus
Corpus callosum
Spiral fibers of hippocampus
3-5 Dissection of the medial aspect of the left cerebral hemisphere
showing the cingulum and spiral fibers of the hippocampus.
Optic:
Nerve
Chiasm
Infundibulum
Amygdaloid
complex
Tract
Inferior horn of
lateral ventricle
Crus cerebri
Hippocampus
Lateral geniculate
body
Calcar avis
Medial geniculate
body
3-6 Overview of a dissection showing the ventral aspect of the
cerebral hemispheres. Note the structures related to ventricular spaces
and the structures located at the mesencephalon–diencephalon interface. A number of structures in addition to those labeled can be identified.
Posterior horn of
lateral ventricle
Overall Views 59
Optic chiasm
Infundibulum
Optic nerve
Olfactory tract
Anterior perforated
substance
Tuber cinereum
Crus cerebri
Amygdaloid complex
Mammillary body
Temporal horn
Optic tract
Posterior perforated
substance
Medial geniculate body
Hippocampus
Substantia nigra
Red nucleus
Lateral geniculate body
Brachium of
superior colliculus
Periaqueductal gray
Pulvinar
Choroid plexus
Splenium of
corpus callosum
Superior colliculus
Great cerebral vein
3-7 Detailed view of a dissection showing the ventral aspects of the
cerebral hemispheres; this is of the same specimen shown in Figure
3-6 on page 58. Note the continuum of optic nerve, chiasm, and tract
to the lateral geniculate body; the relationship of the optic tract to the
crus cerebri; and the relationship of hypothalamic structures on the
ventral aspect of the brain. In addition to those labeled, other structures can be identified.
60 Dissections of the Central Nervous System
Corpus callosum
Head of caudate
Septum pellucidum
Transverse
temporal gyrus
Dorsal thalamus
Fornix
Choroid plexus
3-8 Dissected view of the brain from the dorsal aspect showing
structures associated with the lateral ventricles. Note the appearance
of insular and transverse temporal gyri, the fornix, and other structures
in addition to those labeled.
Column of fornix
Anterior horn of
lateral ventricle
Head of caudate
Interventricular
foramen
Anterior nucleus
of thalamus
Massa
intermedia
Third ventricle
Pineal
Habenula
Colliculi
3-9 Dissected view of the brain from the dorsal aspect showing lateral and third ventricles, the dorsal surface of the diencephalon, the insula and transverse temporal gyri, and the colliculi. The majority of the
fornix and the roof of the third ventricle have been removed. The small
tufts of choroid plexus identify the locations of the interventricular
foramina. Note the massa intermedia traversing the third ventricle and
other structures in addition to those labeled.
Overall Views 61
Transverse
cerebral fissure
Fornix
Suprapineal recess
Brachium of superior colliculus
Pineal
Brachium of inferior colliculus
Caudate nucleus
Optic radiations
Choroid
plexus
Tapetum
Pulvinar
Glomus
Brachium
of SC
Lateral
geniculate
body
SC
Temporal
inferior horn
IC
CC
CC
*
*
Hippocampus
Medial
geniculate
body
Frenulum
Anterior medullary
velum
Cerebellar peduncles:
Middle
Flocculus
Superior
Inferior
Lateral recess,
fourth ventricle
Posterior column:
Tubercles
Fasciculi
3-10 A dissection showing caudal diencephalic structures, several
telencephalic structures, and the interface of the mesencephalon with
caudal parts of the thalamus. On the right side, note the continuation
between the fornix and hippocampus; on the left, these structures have
been removed to expose the underlying pulvinar. The superior colli-
culi (SC), the inferior colliculi (IC), and the crus cerebri (CC), as seen
from the dorsal aspect, are identified. The asterisks represent the exit
points of the trochlear nerves. For further details of the dorsal brainstem, see Figure 2-34 on page 34. Note structures in addition to those
labeled.
CHAPTER
4
Internal Morphology
of the Brain in
Slices and MRI
Brain Slices in the Coronal Plane with MRI
Orientation to Coronal MRIs:
When looking at a coronal
MRI image, you are viewing the image as if you are looking at
the face of the patient. Consequently, the observer’s right is the
left side of the brain in the MRI and the left side of the patient’s
brain. Obviously, the concept of what is the left side versus what
is the right side of the patient’s brain is enormously important
when using MRI (or CT) to diagnose a neurologically impaired
individual.
To reinforce this concept, the rostral surface of each coronal
brain slice was photographed. So, when looking at the slice, the
observer’s right field of view is the left side of the brain slice. This
view of the slice correlates exactly with the orientation of the
brain as seen in the accompanying coronal MRIs.
64 Internal Morphology of the Brain in Slices and MRI
Cingulate gyrus (CinGy)
Cingulum (Cin)
Body of corpus
callosum (BCorCl)
Anterior horn of lateral
ventricle (AH of Lven)
Head of caudate nucleus
(HCaNu)
Septum pellucidum
Anterior limb of
internal capsule
(ALIntCap)
Rostrum of corpus
callosum (RCorCl)
Putamen (Put)
External capsule
(ExtCap)
Subcallosal gyrus
Claustrum (Cl)
Insula (In)
Extreme capsule
(ExtrmCap)
Middle cerebral
artery
Temporal lobe
(TemLb)
Nucleus accumbens
(NuAcc)
Optic nerve (OpNr)
CinGy
BCorCl
AH of
LVen
Cin
HCaNu
BCorCl
ALIntCap
In
ExtrmCap
NuAcc
AH of
LVen
Put
ExtCap
Cl
RCorCl
ExtrmCap
TemLb
OpNr
Olfactory tract
4-1 The rostral surface of a coronal section of brain through the anterior limb of the internal capsule and the head of the caudate nucleus. The two
MRI images (both are inversion recovery) are at the same plane and
show many of the structures identified in the brain slice.
Coronal Brain Slice—MRI Correlation 65
Head of caudate nucleus
(HCaNu)
Body of corpus
callosum (BCorCl)
Septum (Sep)
Anterior limb of
internal capsule (ALintCap)
Corona radiata (CorRad)
Column of fornix
(ColFor)
Putamen (Put)
External Capsule
(ExtCap)
Insula (In)
Anterior
commissure (AC)
Claustrum (Cl)
Extreme capsule (ExtCap)
Claustrum
Globus pallidus (GP)
Ventral striatum
Ventral pallidum
Middle cerebral
artery
Supraoptic recess
Optic chiasm (OpCh)
Infundibulum (Inf)
Sep
CorRad
ColFor
BCorCl
HCaNu
ALIntCap
In
GP
AC
Optic
tract
Uncus
Sep
CorRad
Put
AC
Cl
Third
ventricle
Inf
GP
AC
OpCh
ExtrmCap
4-2 The rostral surface of a coronal section of brain through the level
of the anterior commissure and the column of the fornix. The two MRI im-
ages (both are inversion recovery) are at the same plane and show many
of the structures identified in the brain slice.
66 Internal Morphology of the Brain in Slices and MRI
Body of corpus callosum (BCorCl)
Anterior tubercle of
thalamus (AntTub)
Head of caudate nucleus
(HCaNu)
Body of lateral
ventricle (BLatVen)
Internal capsule (IntCap)
Corona radiata (CorRad)
Stria terminalis and
terminal vein
Putamen (Put)
Insula (In)
External capsule
(ExtCap)
Globus pallidus (GP)
Insula (In)
Claustrum (Cl)
Column of fornix
(ColFor)
Extreme capsule
(ExtrmCap)
Column of fornix
(ColFor)
Third ventricle (ThrVen)
Amygdaloid nuclear
complex (AmyNu)
Amygdaloid nuclear
complex (AmyNu)
Hypothalamus
BLatVen
Optic Tract (OpTr)
BCorCl
HCaNu
Body of
fornix
CorRad
IntCap
AntTub
Anterior
nucleus
Put
ExtCap
In
ThrVen
Ventral
anterior
nucleus
OpTr
Hippocampus
GP
OpTr
Hyth
AmyNu
ThrVen
4-3 The rostral surface of a coronal section of brain through the level
of the anterior tubercle of the thalamus and the column of the fornix just caudal to the anterior commissure. Portions of the columns of the fornix
and the septum (drawn in as black lines) were removed to more ade-
quately expose the anterior tubercles of the thalamus. The terminal vein
is also called the superior thalamostriate vein. The two MRI images
(both are inversion recovery) are at the same plane and show many of
the structures identified in the brain slice.
Coronal Brain Slice—MRI Correlation 67
Body of corpus callosum (BCorCl)
Body of lateral ventricle (BLatVen)
Septum
Body of caudate nucleus
(BCaNu)
Body of fornix (BFor)
Anterior nucleus
of thalamus (AntNu)
Ventral anterior nucleus
of thalamus (VA)
Stria terminalis and
terminal vein
Posterior limb of
internal capsule (PLIntCap)
Corona radiata (CorRad)
Ventral anterior nucleus
of thalamus (VA)
Putamen (Put)
Mammillothalamic
tract
Insula (In)
External capsule
(ExtCap)
Globus pallidus (GP)
Claustrum
Extreme capsule
(ExtrmCap)
Third ventricle (ThrVen)
Amygdaloid nuclear
complex (AmyNu)
Dorsomedial nucleus
of thalamus
Mammillary body (MB)
Optic Tract
Hippocampal formation
Inferior horn of lateral
ventricle (IHLatVen)
Oculomotor nerve
Hippocampal formation (Hip)
Interpeduncular fossa
Posterior cerebral artery
BCorCl
BLatVen
BFor
Thalamus
Put
GP
Basilar pons (BP)
BLatVen
BCaNu
BCorCl
AntNu
VA
PLintCap
BFor
AntNu
VA
PLIntCap
Put
In
IHLatVen
Hip
Hip
IHLatVen
MB
BP
4-4 The rostral surface of a section of brain through the anterior nucleus of the thalamus, mammillothalamic tract, and mammillary bodies. The
two MRI images (both are inversion recovery) are at the same plane and
show many of the structures identified in the brain slice.The globus
pallidus is clearly divided into its lateral and medial segments in the brain
slice. Additionally, the terminal vein is also called the superior thalamostriate vein.
68 Internal Morphology of the Brain in Slices and MRI
Body of corpus callosum (BCorCl)
Body of fornix (BFor)
Body of lateral ventricle (BLatVen)
Stria terminalis and
terminal vein
Body of caudate nucleus
(BCaNu)
Corona radiata (CorRad)
Dorsomedial nucleus
of thalamus (DMNu)
Third ventricle and
massa intermedia
Ventral lateral nucleus
of thalamus (VL)
Posterior limb of internal
capsule (PLIntCap)
External capsule
Putamen (Put)
Claustrum
Insula (In)
Internal medullary
lamina (IML)
Extreme capsule
Globus pallidus (GP)
Subthalamic nucleus
Third ventricle (ThrVen)
Red nucleus
Tail of caudate nucleus
Substantia nigra (SN)
Inferior horn of
lateral ventricle
Hippocampal formation (Hip)
Interpeduncular fossa (IPF)
Optic tract
Basilar pons (BP)
Crus cerebri (CC)
Corticospinal fibers
BFor
BCorCl
BFor
BCaNu
DMNu
ThrVen
PLIntCap
Put
IML
VL
ThrVen
Put
SN
In
GP
CC
Hip
IPF
SN
IPF
BP
4-5 The rostral surface of a coronal section of brain through caudal
parts of the ventral lateral nucleus, the massa intermedia, the subthalamic nucleus, and basilar pons. The two MRI images (both are inversion recovery)
are at the same plane and show many of the structures identified in the
brain slice. The terminal vein is also called the superior thalamostriate
vein.
Coronal Brain Slice—MRI Correlation 69
Body of corpus callosum (BCorCl)
Body of fornix (BFor)
Body of lateral ventricle (BLatVen)
Body of caudate nucleus
(BCaNu)
Stria terminalis and
terminal vein
Corona radiata
Dorsomedial nucleus
of thalamus (DMNu)
Lateral dorsal nucleus
of thalamus
Internal medullary
lamina (IML)
Ventral posterolateral
nucleus of thalamus (VPL)
External capsule
Centromedian nucleus
of thalamus
Claustrum
Insula
Putamen (Put)
Ventral posteromedial
nucleus of thalamus
Posterior limb
of internal
capsule (PLIntCap)
Tail caudate nucleus
Inferior horn of lateral
ventricle (IHLatVen)
Optic tract
Hippocampal formation
(Hip)
Third ventricle
Interpeduncular fossa (IPF)
Crus Cerebri (CC)
Red nucleus (RNu)
Trigeminal nerve (TriNr)
Substantia nigra
Basilar pons
BFor
BCorCl
BLatVen
IML
PLIntCap
RNu
BCaNu
DMNu
VPL
Put
Hip
CC
TriNr
IPF
4-6 The rostral surface of a coronal section of brain through the lateral dorsal and centromedian nuclei, rostral midbrain (red nucleus), and corticospinal fibers in the basilar pons. The MRI image (inversion recovery)
TriNr
BP
is at the same plane and shows many of the structures identified in the
brain slice. The terminal vein is also called the superior thalamostriate
vein.
70 Internal Morphology of the Brain in Slices and MRI
Body of corpus callosum (BCorCl)
Body of lateral ventricle (BLatVen)
Body of fornix (BFor)
Fimbria of fornix (FFor)
Body of caudate nucleus
(BCaNu)
Stria terminalis and
terminal vein
Pulvinar (Pul)
Pulvinar (Pul)
Retrolenticular limb of
internal capsule
Medial geniculate
nucleus (MGNu)
Posterior commissure
Lateral geniculate
nucleus (LGNu)
Tail of caudate
nucleus
Inferior horn of lateral
ventricle (HLatVen)
Lateral geniculate
nucleus (LGNu)
Hippocampal
formation (Hip)
Pretectal area (PrTecAr)
Cerebral aqueduct (CA)
Periaqueductal gray
Decussation of superior
cerebellar peduncle
Pyramid
Middle cerebellar peduncle (MCP)
BCorCl
BLatVen
BCaNu
BFor
FFor
Pul
Pul
Pul
MGNu
PrTecAr
LGNu
LGNu
IHLatVen
Hip
MGNu
Basilar
pons
Basilar
pons
CA
Trigeminal nerve
4-7 The rostral surface of a coronal section of brain through the pulvinar, medial, and lateral geniculate nuclei, the basilar pons, and middle cerebellar peduncle. The two MRI images (both inversion recovery) are at the
same plane and show many of the structures in the brain slices. The
terminal vein is also called the superior thalamostriate vein. For details
of the cerebellum see Figures 2-31 to 2-33 on pp. 32 and 33.
Coronal Brain Slice—MRI Correlation 71
Body of corpus callosum (BCorCl)
Crus of fornix (CrF)
Fimbria of fornix (FFor)
Body of lateral ventricle (BLatVen)
Pulvinar (Pul)
Body of caudate
nucleus (BCaNu)
Retrolenticular limb
of internal capsule
Stria terminalis and
terminal vein
Fimbria of hippocampus
Pineal
Hippocampal
formation (Hip)
Tail caudate nucleus
Inferior horn of lateral
ventricle (IHLatVen)
Superior colliculus (SC)
Cerebral aqueduct (CA)
Periaqueductal gray (Pag)
Flocculus
Medulla (Med)
Middle cerebellar peduncle (MCP)
BCorCl
BLatVen
FFor
Splenium
of corpus
callosum
Pul
BLatVen
SC
SC
Hip
Hip
CA
Pag
Med
IHLatVen
Pag
MCP
MCP
Pyramid
4-8 The rostral surface of a coronal section of brain through the pulvinar nucleus, the superior colliculus, the middle cerebellar peduncle, and the
rostral portion of the medulla oblongata. The two MRI images (both are
inversion recovery) are at the same plane and show many of the struc-
tures identified in the brain slice. The terminal vein is also called the
superior thalamostriate vein. For details of the cerebellum see Figures
2-31 to 2-33 on pp. 32 and 33.
72 Internal Morphology of the Brain in Slices and MRI
Splenium of corpus
callosum (SpCorCl)
Towards posterior horn of
lateral ventricle (PHLatVen)
Hippocampal
commissure
(HipCom)
Crus of fornix (CrFor)
Hippocampal
formation
Optic Radiations
(OpRad)
Towards inferior horn
of lateral ventricle
(IHLatVen)
Tapetum (Tap)
Inferior colliculus (IC)
Trochlear nerve
Superior cerebellar
peduncle (SCP)
Middle cerebellar peduncle (MCP)
Fourth ventricle (ForVen)
Restiform body
Medulla (Med)
Pyramid (Py)
CrFor
SpCorCl
HipCom
Tap
IHLatVen
OpRad
IC
PHLat
Ven
Hip
IC
SCP
MCP
ForVen
4-9 The rostral surface of a coronal section of brain through the splenium of corpus callosum, the inferior colliculus, the middle cerebellar peduncle in the base of the cerebellum, and the rostral portion of the
medulla oblongata. The plane of the section is also through the atrium
Med
Py
of the lateral ventricles. The two MRI images (both are inversion recovery) are at the same plane and show many of the structures identified in the brain slice. For details of the cerebellum see Figures 2-31 to
2-33 on pp. 32 and 33.
Internal Morphology
of the Brain in
Slices and MRI
Brain Slices in the Axial Plane with MRI
Orientation to Axial MRIs:
When looking at an axial MRI
image, you are viewing the image as if standing at the patient’s
feet and looking toward his or her head while the patient is lying
on his or her back. Consequently, and as is the case in coronal images, the observer’s right is the left side of the brain in the MRI
and the left side of the patient’s brain. It is absolutely essential to
have a clear understanding of this right-versus-left concept when
using MRI (or CT) in the diagnosis of the neurologically impaired
patient.
To reinforce this concept, the ventral surface of each axial slice
was photographed. So, when looking at the slice, the observer’s
right is the left side of the brain slice. This view of the slice correlates exactly with the orientation of the brain as seen in the accompanying axial MRIs.
74 Internal Morphology of the Brain in Slices and MRI
Body of corpus callosum
(toward the genu)
Corona radiata (CorRad)
Caudate nucleus
(CaNu)
Stria terminalis and
terminal vein
Corpus callsoum (CorCl)
Body of lateral
ventricle (BLatVen)
Body of corpus callosum
(toward the splenium)
Anterior
forceps
CorCl
CaNu
CorCl
CorRad
BLatVen
Septum
pellucidum
Posterior
forceps
4-10 Ventral surface of an axial section of brain through dorsal portions of corpus callosum. The plane of the section just touches the upper
portion of the body of caudate nucleus. The two MRI images (both inver-
sion recovery) are at a similar plane and show some of the structures
identified in the brain slice. The terminal vein is also called the superior
thalamostriate vein.
Axial Brain Slice—MRI Correlation 75
Cingulate gyrus
Anterior cerebral arteries
Genu of corpus
calllosum
Head of caudate
nucleus (HCaNu)
Anterior horn of lateral
ventricle (AHLatVen)
Stria terminalis and
terminal vein
Body of fornix
Anterior nucleus of
thalamus
Anterior tubercle
Corona radiata (CorRad)
Ventral anterior
nucleus of thalamus
Lateral thalamic nuclei
Tail of caudate
nucleus
Dorsomedial nucleus
of thalamus
Lateral ventricle (LatVen)
Tail of caudate nucleus
Crus of fornix
Splenium of corpus callosum
Caudate
nucleus
LatVen
Putamen
CorRad
AHLatVen
HCaNu
Internal
capsule
Septum
pellucidum
Dorsal
thalamus
Atrium of
lateral
ventricle
4-11 Ventral surface of an axial section of brain through the splenium
of corpus callosum and the head of the caudate nucleus. This plane includes
only a small portion of the dorsal thalamus. The two MRI images (inver-
sion recovery—left; T2-weighted—right) are at a comparable plane
and show some of the structures identified in the brain slice. The terminal vein is also called the superior thalamostriate vein.
76 Internal Morphology of the Brain in Slices and MRI
Anterior horn of lateral
ventricle (AHLatVen)
Genu of corpus callosum
Septum pellucidum
Stria terminalis and
terminal vein
Head of caudate nucleus (HCaNu)
Genu of internal
capsule (GIntCap)
Column of fornix (ColFor)
Anterior limb of
internal capsule (ALIntCap)
Anterior nucleus of thalamus
Ventral anterior nucleus
of thalamus
Putamen (Put)
Choroid plexus in third ventricle
Globus pallidus (GP)
Insula (In)
Third ventricle (ThrVen)
Claustrum
Posterior limb of internal
capsule (PLIntCap)
Internal medullary lamina
Dorsomedial nucleus
of thalamus (DMNu)
Ventral lateral nucleus
of thalamus
Habenular nucleus
(Hab)
Centromedian nucleus
of thalamus
Ventral posterolateral
nucleus of thalamus
Retrolenticular limb of
internal capsule (RLIntCap)
Tail of caudate nucleus
Fimbria of hippocampus
Pulvinar (Pul)
Hippocampal formation (Hip)
Atrium of lateral
ventricle (ALatVen)
Crus of fornix
Optic radiations
Tapetum
Splenium of corpus callosum (SpCorCl)
Posterior horn of lateral ventricle (PHLatVen)
AHLatVen
ALIntCap
HCaNu
ALIntCap
Put
GIntCap
ColFor
PLIntCap
GP
Lateral
thalamic
nuclei
RLIntCap
GIntCap
PLIntCapr
ThrVen
Lateral
thalamic
nuclei
RLIntCap
DMNu
ALatVen
Pul
Hab
Hip
4-12 Ventral surface of an axial section of brain through the genu of
the corpus callosum, head of caudate nucleus, centromedian nucleus, and dorsal portions of the pulvinar. The two MRI images (inversion recovery—
left; T2-weighted—right) are at the same plane and show many of the
PHLatVen
ThrVen
SpCorCl
structures identified in the brain slice. The arrowheads in the brain slice
and in the MRIs are pointing to the mammillothalamic tract. The terminal vein is also called the superior thalamostriate vein.
Axial Brain Slice—MRI Correlation 77
Genu of corpus callosum
Anterior commissure (AC)
Subcallosal gyri
Head of caudate nucleus (HCaNu)
Anterior limb of internal
capsule (ALIntCap)
Column of fornix (ColFor)
Third ventricle
Putamen (Put)
Mammillothalamic tract (MtTr)
Globus pallidus (GP)
External capsule
Insula (In)
Claustrum
Extreme capsule
Posterior limb of internal
capsule (PLIntCap)
Ventral posterior
thalamic nuclei
Subthalamic nucleus
Brachium of superior
colliculus
Red nucleus (RNu)
Pulvinar
Lateral geniculate
nucleus (LGNu)
Tail of caudate nucleus
Retrolenticular limb of
internal capsule
Tapetum (Tap)
Fimbria of hippocampus
Optic radiation (OpRad)
Hippocampal formation (Hip)
Atrium of lateral ventricle
(ALatVen)
Medial geniculate
nucleus (MGNu)
Posterior horn of lateral ventricle
(PHLatVen)
Pineal
Superior colliculus (SC)
Splenium of corpus callosum (SpCorCl)
AC
ColFor
HCaNu
ALIntCap
Put
GP
In
ColFor
AC
MtTr
MtTr
PLIntCap
Lateral
thalamic
nuclei
Dorsomedial
nucleus
RNu
LGNu
Massa
intermed.
Pul
MGNu
Hip
ALatVen
Crus of
fornix
Tap
SC
PHLatVen
OpRad
SpCorCl
4-13 Ventral surface of an axial section of brain through the anterior
commissure, column of fornix, medial and lateral geniculate nuclei, and superior colliculus. The medial and lateral segments of the globus pallidus are visible on the slice. The lateral and medial segments of the globus pallidus
can be discerned on the right side of the brain. The MRI images (both
T2-weighted) are at approximately the same plane and show many of
the structures identified in the brain slice.
78 Internal Morphology of the Brain in Slices and MRI
Hypothalamus (HyTh)
Anterior cerebral arteries (ACA)
Head of
caudate nucleus
Lamina terminalis
Third ventricle (ThrVen)
Nucleus accumbens
Optic tract (OpTr)
Anterior perforated
substance
Uncus
Crus cerebri (CC)
Amygdaloid
nuclear complex
Inferior horn of
lateral ventricle
(IHLatVen)
Mammillary body (MB)
Interpeduncular
fossa (IPF)
Hippocampal
formation
Lateral geniculate
nucleus
Substantia
nigra (SN)
Tail of caudate nucleus
Hippocampal formation (Hip)
Decussation of superior
cerebellar peduncle
Choroid plexus in
inferior horn
Inferior colliculus (IC)
Periaqueductal gray
Cerebellum (Cbl)
Cerebral Aqueduct (CA)
ACA
OpTr
ThrVen
HyTh
Un
SN
ThrVen
MB
CC
IHLatVen
IPF
Hip
CA
IC
Posterior
cerebral
artery
Cbl
4-14 Ventral surface of an axial section of brain through the hypothalamus, mammillary body, crus cerebri, and inferior colliculus. The two MRI
images (inversion recovery—left; T2-weighted—right) are at similar
Posterior horn
lateral ventricle
planes and show many of the structures identified in the brain slice. For
details of the cerebellum see Figures 2-31 to 2-33 on pp. 32 and 33.
Axial Brain Slice—MRI Correlation 79
Basilar artery (BA)
Corticospinal fibers
Medial lemniscus (ML)
Basilar pons (BP)
Tegmentum of pons (TegP)
Fourth ventricle (ForVen)
Nucleus coeruleus
Superior cerebellar
peduncle (SCP)
Vermis of anterior lobe
of cerebellum (VCbl)
Hemisphere of anterior lobe
of cerebellum (HCbl)
Hemisphere of posterior lobe
of cerebellum
BA
Temporal
lobe
BP
ML
HCbl
TegP
HCbl
VCbl
SCP
VCbl
ForVen
Occipital
lobe
4-15 Ventral surface of an axial section of brain through rostral
parts of the basilar pons and the anterior lobe of the cerebellum. The two MRI
images (T2-weighted—left; inversion recovery—right) are at the same
Occipital
lobe
plane and show many of the structures identified in the brain slice. For
details of the cerebellum see Figures 2-31 to 2-33 on pp. 32 and 33.
80 Internal Morphology of the Brain in Slices and MRI
Basilar artery (BA)
Basilar pons (BP)
Trigeminal nerve (TriNr)
Medial lemniscus
Tegmentum of pons (TegP)
Middle cerebellar
peduncle (MCP)
Fourth ventricle (ForVen)
Dentate nucleus (DNu)
Vermis of posterior lobe
of cerebellum (VCbl)
Hemisphere of posterior lobe
of cerebellum (HCbl)
Temporal
lobe
BP
TriNr
Trigeminal
ganglion
MCP
TriNr
BA
Temporal lobe
Lesion
in BP
ForVen
BP
TegP
TegP
HCbl
ForVen
BA
BP
MCP
TegP
ForVen
DNu
VCbl
4-16 Ventral surface of an axial section of brain through the middle
regions of the basilar pons, the exit of the trigeminal nerve, the fourth ventricle, and the cerebellar nuclei. The three MRI images (inverted inversion
recovery—upper left; T2-weighted—upper right; T1-weighted—
lower) are at the same planes and show many of the structures identified in the brain slice. Note the lesion in the basilar pons (upper right).
For details of the cerebellum see Figures 2-31 to 2-33 on pp. 32
and 33.
Axial Brain Slice—MRI Correlation 81
Basilar artery
Basilar pons
Anterior median fissure (AMF)
Pyramid (Py)
Preolivary sulcus (PreOIS)
Olivary eminence (OlEm)
Vestibulocochlear nerve
Vagus and glossopharyngeal
nerves
Retroolivary sulcus
(Postolivary sulcus) (PoOIS)
Restiform body (RB)
Medial lemniscus
Tonsil of cerebellum (TCbl)
Hemisphere of posterior lobe
of cerebellum (HCbl)
Fourth ventricle (ForVen)
Vermis of posterior lobe
of cerebellum (VCbl)
AMF
Py
PreOlS
OlEm
PoOlS
RB
TCbl
TCbl
ForVen
HCbl
VCbl
OlEm
Lesion-Lateral
medullary syndrome
RB
ForVen
4-17 Ventral surface of an axial section of brain through portions of
the medulla oblongata, just caudal to the pons–medulla junction and the
posterior lobe of the cerebellum. The three MRI images (T1-weighted—
upper left and right; T2-weighted—lower) are at the same plane and
show many of the structures identified in the brain slice. Note the lateral
medullary lesion (lower), also known as the posterior inferior artery
syndrome or the lateral medullary syndrome (of Wallenberg). For details of the cerebellum see Figures 2-31 to 2-33 on pp. 32 and 33.
CHAPTER
5
Internal Morphology of the Spinal Cord
and Brain in Stained Sections
Basic concepts that are essential when one is initially learning how
to diagnose the neurologically impaired patient include 1) an understanding of cranial nerve nuclei and 2) how these structures relate to long tracts. The importance of these relationships is clearly
seen in the combinations of deficits that generally characterize lesions at different levels of the neuraxis. First, deficits of only the
body that may present as motor or sensory losses (long tracts) on
the same, or opposite, sides are indicative of spinal cord lesions
(e.g., Brown-Sequard syndrome). Spinal cord injuries characteristically have motor and sensory levels; these are the lowest functional
levels remaining in the compromised patient. Second, cranial
nerve deficits (on one side of the head) in combination with long
tract signs (on the opposite side of the body) characterize lesions
in the brainstem (e.g., lateral medullary or Weber syndromes).
These patterns of loss are frequently called alternating or crossed
deficits. In these examples cranial nerve signs are better localizing
signs than are long tract signs. A localizing sign can be defined as an
objective neurologic abnormality that correlates with a lesion (or
lesions) at a specific neuroanatomical location (or locations).
Third, motor and sensory deficits on the same side of the head and
body are usually indicative of a lesion in the forebrain.
Color Coded Cranial Nerve Nuclei and Long Tracts:
Cranial nerve nuclei are coded by their function: pink, sensory;
red, motor. These structures are colored bilaterally to make it easy
to correlate cranial nerve and long tract function on both sides of
the midline. For example, one can easily correlate damage to the
hypoglossal nerve root and the adjacent corticospinal fibers on one
side while comparing this pattern with the clinical picture of a lateral medullary syndrome on the other side.
Long tracts are color-coded beginning at the most caudal spinal
cord levels (e.g., see Figures 5-1 and 5-2), with these colors extending into the dorsal thalamus (see Figure 5-30) and the posterior
limb of the internal capsule (see Figures 5-31 and 5-32). The colorized spinal tracts are the fasciculus gracilis (dark blue), the fasciculus cuneatus (light blue)*, the anterolateral system (dark green),
and the lateral corticospinal tract (grey). In the brainstem, these
spinal tracts are joined by the spinal trigeminal tract and ventral
trigeminothalamic fibers (both are light green). The long tracts are
color-coded on one side only, to emphasize 1) laterality of function
and dysfunction, 2) points at which fibers in these tracts may decussate, and 3) the relationship of these tracts to cranial nerves.
A color key appears on each page. This key identifies the various tracts and nuclei by their color and specifies the function of
each structure on each page. This approach not only emphasizes
anatomical and clinical concepts, but also lends itself to a variety
of instructional settings.
Correlation of MRI and CT with Spinal Cord and
Brainstem: As one is learning basic anatomical concepts it is
essential to consider how this information may be used in the
clinical environment. To this end, MRI (T1- and T2-weighted)
and CT (myelogram/cisternogram) images are introduced into
the spinal cord and brainstem sections of this chapter (see also
Chapter 1). To show the relationship between basic anatomy and
how MRI and CT are viewed, a series of self-explanatory illustrations are provided on each set of facing pages in these sections.
This continuum of visual information consists of (1) a small version of the colorized line drawing in an Anatomical Orientation,
(2) a top-to-bottom flip of this illustration that brings it into a
Clinical Orientation, and (3) a CT (spinal cord) or MRI and CT
(brainstem) that follows this clinically oriented image. Every effort is made to identify and use MRI and CT that correlate, as
closely as possible, with their corresponding line drawing and
stained section. This approach recognizes and retains the
strength of the anatomical approach, introduces essential clinical
concepts while at the same time allowing the user to customize
the material to suit a range of educational applications.
*The dark and light blue colors represent information originating from lower
and upper portions of the body, respectively.
Post. column/med. lemniscus sys.
(proprioception/vibratory sense,
discriminative touch)
Medial motor nuclei (lamina IX)
Lateral motor nuclei (lamina IX)
Intermediate zone (lamina VII)
Nucleus proprius (laminae III,IV)
Substanita gelatinosa (lamina II)
Corticospinal fibers
(somatomotor)
ProSp
Anterolateral system
(pain/thermal sense,
touch from body)
Reticulospinal tract
Anterolateral system
Propriospinal fibers (ProSp)
Lateral corticospinal tract
Posteromarginal nucleus (lamina I)
Dorsolateral tract
Gracile fasciculus
5-1 Transverse section of the spinal cord showing the characteristics of a sacral level. The gray matter occupies most of the crosssection; its H-shaped appearance is not especially obvious at
sacral–coccygeal levels. The white matter is a comparatively thin
mantle. The sacral cord, although small, appears round in the CT
myelogram. Note the appearance of the sacral spinal cord surrounded by the upper portion of the cauda equina (left) and the cauda
equina as it appears caudal to the conus medullaris in the lumbar cistern (right). Compare with Figure 2-4 on page 12.
84 Internal Morphology of the Spinal Cord and Brain in Stained Sections
Clinical orientation
Clinical orientation
Anatomical orientation
Anatomical orientation
CT myelogram
CT myelogram
Sacral spinal
cord
Cauda equina
Lumbar cistern
CT myelogram
CT myelogram
The Spinal Cord With CT 85
Post. column/med. lemniscus sys.
(proprioception/vibratory sense,
discriminative touch)
Medial motor nuclei (lamina IX)
Area of lamina VIII
Lateral motor nuclei (lamina IX)
Intermediate zone (lamina VII)
Area of lamina VI
Area of lamina V
Nucleus proprius (laminae III,IV)
Substanita gelatinosa (lamina II)
Posteromarginal nucleus (lamina I)
Medial division fibers
ProSp
ProSp
Corticospinal fibers
(somatomotor)
Medullary (lateral)
reticulospinal fibers
Anterolateral system
Propriospinal fibers (ProSp)
Rubrospinal tract
Lateral corticospinal tract
Dorsolateral tract
Anterior corticospinal tract
Medial longitudinal fasciculus
Anterior root fibers
Lateral vestibulospinal tract and
pontoreticulospinal (Medial
reticulospinal) tract
Gracile fasciculus
Anterolateral system
(pain/thermal sense,
touch from body)
Anterior median fissure
Posterior median sulcus
5-2 Transverse section of the spinal cord showing its characteristic
appearance at lumbar levels (L4). Posterior and anterior horns are large
in relation to a modest amount of white matter, and the general shape
of the cord is round. Fibers of the medial division of the posterior root
directly enter the gracile fasciculus. The lumbar spinal cord appears
round in the CT myelogram. The roots of upper portions of the cauda
equina surround the lower levels of the lumbar spinal cord (right).
86 Internal Morphology of the Spinal Cord and Brain in Stained Sections
Anatomical orientation
Clinical orientation
CT myelogram
Cauda equina
Posterior root
Lumbar spinal
cord
Anterior root
CT myelogram
The Spinal Cord With CT 87
ProSp
Post. column/med. lemniscus sys.
(proprioception/vibratory sense,
discriminative touch)
Corticospinal fibers
(somatomotor)
Anterior median fissure
Anterolateral system
(pain/thermal sense,
touch from body)
Anterior corticospinal tract
Medial longitudinal fasciculus
Anterolateral system
Anterior root fibers
Medullary (lateral) reticulospinal tract
Anterior spinocerebellar tract
Propriospinal fibers (ProSp)
Rubrospinal tract
Lateral corticospinal tract
Posterior spinocerebellar tract
Medial division fibers
of posterior root
Cuneate fasciculus
Posterior intermediate sulcus
Gracile fasciculus
Posterior median sulcus
Anterior white commissure
Pontoreticulospinal tract
Lateral vestibulospinal tract
Spino-olivary fibers
Medial motor nuclei (lamina IX)
Intermediolateral cell column
(lamina VII)
Dorsal thoracic nucleus
of Clarke (lamina VII)
Nucleus proprius
(laminae III,IV)
Posteromarginal nucleus and
substantia gelatinosa (laminae I,II)
Dorsolateral tract
Posterolateral sulcus
ProSp
5-3 Transverse section of the spinal cord showing its characteristic
appearance at thoracic levels (T4). The white matter appears large in
relation to the rather diminutive amount of gray matter. Posterior and
anterior horns are small, especially when compared to low cervical levels and to lumbar levels. The overall shape of the cord is round. The
thoracic spinal cord appears round in CT myelogram.
88 Internal Morphology of the Spinal Cord and Brain in Stained Sections
Anatomical orientation
Clinical orientation
CT myelogram
Posterior root
Anterior root
CT myelogram
The Spinal Cord With CT 89
Post. column/med. lemniscus sys.
(proprioception/vibratory sense,
discriminative touch)
Anterolateral system
(pain/thermal sense,
touch from body)
Medial longitudinal fasciculus
and tectospinal tract
Pontoreticulospinal (medial reticulospinal) tract
Lateral vestibulospinal tract
Anterolateral sulcus
Spino-olivary fibers
Anterolateral system
Medullary (lateral)
reticulospinal tract
Anterior spinocerebellar tract
Rubrospinal tract
Propriospinal
fibers (ProSp)
Lateral corticospinal tract
Posterior spinocerebellar tract
Posterolateral sulcus
Cuneate fasciculus
Posterior intermediate sulcus
Gracile fasciculus
Anterior median fissure
Corticospinal fibers
(somatomotor)
Anterior corticospinal tract
Anterior white commissure
ProSp
Medial motor nuclei (lamina IX)
Area of lamina VIII
Lateral motor nuclei
(lamina IX)
Intermediate Zone (lamina VII)
Area of lamina VI
Area of lamina V
Reticular nucleus of
cervical cord
Nucleus proprius (laminae III, IV)
Substantia gelatinosa (lamina II)
Posteromarginal nucleus (lamina I)
Dorsolateral tract
Interfascicular fasciculus
Posterior median sulcus
5-4 Transverse section of the spinal cord showing its characteristic
appearance at lower cervical levels (C7). The anterior horn is large,
and there is—proportionally and absolutely—a large amount of white
matter. The overall shape of the cord is oval. The lower portions of the
cervical spinal cord appears oval in MRI (left) and in CT myelogram
(center and right).
90 Internal Morphology of the Spinal Cord and Brain in Stained Sections
Anatomical orientation
Clinical orientation
MRI, T2 weighted image
CT myelogram
CT myelogram
The Spinal Cord With CT 91
Post. column/med. lemniscus sys.
(proprioception/vibratory sense,
discriminative touch)
ProSp
Corticospinal fibers
(somatomotor)
Pyramidal decussation
Anterior corticospinal tract
Medial motor nuclei
Spino-olivary fibers
Spinal accessory nucleus
Magnocellular portion of
Spinal trigeminal nucleus
Gelatinosa portion of
Spinal trigeminal nucleus
Spinal trigeminal tract and
Dorsolateral tract junction
ProSp
Anterolateral system
(pain/thermal sense,
touch from body)
Tectospinal tract
Spinal trigeminal and/or ventral
trigeminothalamic fibers (pain/
thermal sense, touch from head)
Medial longitudinal fasciculus
Pontoreticulospinal (Medial reticulospinal) tract
Lateral vestibulospinal tract
Anterolateral system
Medullary (lateral)
reticulospinal tract
Anterior spinocerebellar tract
Propriospinal
fibers (ProSp)
Rubrospinal
tract
Motor
Sensory
Posterior spinocerebellar
tract
Lateral corticospinal tract
Cuneate fasciculus
Posterior intermediate sulcus
Gracile fasciculus
Posterior median sulcus
Cranial
nerve
nuclei
5-5 Transverse section of the spinal cord at the C1 level. Lateral
corticospinal fibers are now located medially toward the decussation of
the corticospinal fibers, also called the motor decussation or pyramidal
decussation (see also Figure 5-8, page 98). At this level, fibers of the
spinal trigeminal tract are interdigitated with those of the dorsolateral
tract. The spinal cord at C1 and C2 levels appear round in CT myelogram when compared to low cervical levels (see Figure 5-4).
92 Internal Morphology of the Spinal Cord and Brain in Stained Sections
Anatomical orientation
Clinical orientation
CT myelogram
Posterior root
CT myelogram
The Spinal Cord With CT 93
Structure Damage
• Medial portions of both
lateral corticospinal
tracts; ventral grey
horns at cervical levels
• Irregular loss of pain
• Anterolateral system
and temperature sensa- fibers (partial
tions bilaterally over
involvement
body below lesion
bilaterally)
Comment: Hyperextension of the neck may
cause damage to the vertebral arteries (which give
rise to the anterior spinal artery), or it may directly
damage the anterior spinal artery, causing a spasm.
This vascular damage leads to a temporary or permanent interruption of blood supply. Deficits may resolve within a few hours or may be permanent, depending on the extent of vascular complication.
Sparing of the dorsal columns (proprioception, vibratory sense) is a hallmark; approximately the anterior
two-thirds of the spinal cord is ischemic.
Thrombosis of Anterior Spinal Artery: This
may occur in a hypotensive crisis, as a result of trauma
resulting from a dissecting aortic aneurysm, or in patients with atherosclerosis. It may occur at all spinal
levels but is more frequently seen in thoracic and lumbosacral levels unless trauma is the primary cause. Results are bilateral flaccid paraplegia (if lesion is below cervical levels) or quadriplegia (if lesion is in cervical
levels), urinary retention, and loss of pain and temperature
sensation. Flaccid muscles may become spastic over a
period of a day to weeks, with hyperactive deep tendon reflexes and extensor plantar (Babinski) reflexes. In addition,
lesions at high cervical levels may also result in paralysis of respiratory muscles. The artery of Adamkiewicz
(an especially large spinal medullary artery) is usually
located at spinal levels T12–L1 and more frequently
arises on the left side. Occlusion of this vessel may infarct lumbosacral levels of the spinal cord.
Hemorrhage in the spinal cord: This is rarely
seen but may result from trauma or bleeding from
• Bilateral paresis or
flaccid paralysis of
upper extremities
Deficit
spinal artery.
Acute Central Cervical Spinal Cord Syndrome: Results from occlusion of the anterior
More frequently found in lower cord levels. Symptoms
(micturition problems are seen early, motor deficits, lower
back pain) may appear over time and may seem to resolve then recur (get better then worse). These lesions
are usually found external to the cord (extramedullary)
and can be surgically treated, especially when the major feeding vessels are few in number and easily identified. Foix-Alajouanine syndrome is an inflammation of
spinal veins with subsequent occlusion that results in
infarct of the spinal cord and a necrotic myelitis. The
symptoms are ascending pain and a flaccid paralysis.
Brown-Sequard syndrome: This syndrome is
a hemisection of the spinal cord that may result from
trauma, compression of the spinal cord by tumors or
hematomas, or significant protrusion of an intervertebral disc. The deficits depend on the level of the
causative lesion. The classic signs are (1) a loss of pain
and thermal sensation on the contralateral side of the
body beginning about 1–2 segments below the level
of the lesion (damage to anterolateral system fibers), (2)
a loss of discriminative touch and proprioception on
the ipsilateral side of the body below the lesion (interruption of posterior column fibers), and (3) a paralysis
on the ipsilateral side of the body below the lesion
(damage to lateral corticospinal fibers). This syndrome is
classified as an incomplete spinal cord injury (see below)
and the majority of patients with this lesion will regain some type of motor and sensory function. Compression of the spinal cord may result in some, but not
all, of the signs and symptoms of the syndrome.
Syringomyelia: This condition is cavitation of
central portion of the spinal cord. A cavitation of the
central canal with an ependymal cell lining is hydromyelia. A syrinx may originate in central portions of
the spinal cord, may communicate with the central
canal, and is most commonly seen in cervical levels of
the spinal cord. The most common deficits are a bilateral loss of pain and thermal sensation due to damage to
the anterior white commissure: the loss reflects the levels
of the spinal cord damaged (e.g., a cape distribution
Arteriovenous malformation in spinal cord:
congenital vascular lesions. Symptoms may develop
rapidly or gradually in stepwise fashion, and blood is
usually present in the cerebrospinal fluid.
Vascular Syndromes or Lesions of the Spinal Cord
A
AH
AWCom
CenC
IZ
L
N
PH
S
T
Representation of arm fibers
Anterior (ventral) horn
Anterior white commissure
Central canal
Intermediate zone
Representation of leg fibers
Representation of neck fibers
Posterior (dorsal) horn
Representation of sacral fibers
Representation of truck fibers
Abbreviations
5-6 Semidiagrammatic representation of the internal blood supply
to the spinal cord. This is a tracing of a C4 level, with the positions of
principal tracts superimposed on the left and the general pattern of
blood vessels superimposed on the right.
over the shoulder and upper extremities). The other commonly seen
deficit results from extension of the cavity into the anterior horn(s).
The result is unilateral or bilateral paralysis of the upper extremities (cervical levels) or lower extremities (lumbosacral levels) due to damage to the
anterior motor neuron cells. This paralysis is characteristically a lower
motor neuron deficit. A syrinx in the spinal cord, particularly in cervical levels, may be associated with a variety of other developmental defects in the nervous system.
Spinal Cord Lesions: A complete spinal cord lesion is characterized
by a bilateral and complete loss of motor and sensory function below
the level of lesion persisting for more than 24 hours. The vast majority
of the patients with complete lesions (95%+) will suffer some permanent deficits. Incomplete spinal cord lesions are those with preservation of
sacral cord function at presentation. The above described cases are examples of incomplete spinal cord lesions.
94 Internal Morphology of the Spinal Cord and Brain in Stained Sections
Medial longitudinal fasciculus and
anterior cortocospinal tract
Reticulospinal and
vestibulospinal tracts
Anterolateral system
Anterior spincerebellar tract
Rubrospinal tract
Posterior spinocerebellar tract
Propriospinal fibers
Lateral corticospinal tract
Dorsolateral tract
Fasciculus cuneatus
AH
IZ
Sulcal artery
PH
Fasciculus gracilis
AWCom
CenC
N
A
A
Anterior spinal artery
AVC
T
T
L
L
Anterior spinal medullary artery
S
Arterial
vasocorona
(AVC)
Posterior spinal medullary artery
Posterior spinal artery
Anterior radicular artery
to anterior root
Segmental
artery
Posterior radicular
artery to posterior root
Arterial Patterns Within The Spinal Cord With Vascular Syndromes 95
5-7 All of the brainstem sections used in Figures 5-9 through 5-13
(medulla), 5-17 through 5-20 (pons), and 5-22 through 5-25 (midbrain) are from an individual who had an infarct (green in drawing) in
the posterior limb of the internal capsule. This lesion damaged corticospinal fibers (grey in drawing), resulting in a contralateral hemiplegia of the arm and leg, and damaged sensory radiations that travel from
thalamic nuclei to the somatosensory cortex through the posterior limb
of the internal capsule. Although the patient survived the initial
episode, corticospinal fibers (grey) distal to the lesion (green) underwent degenerative changes and largely disappeared. This Wallerian
(anterograde) degeneration takes place because the capsular infarct effectively separates the descending corticospinal fibers from their cell
bodies in the cerebral cortex. Consequently, the location of corticospinal fibers in the middle one-third of the crus cerebri of the midbrain, in the basilar pons, and in the pyramid of the medulla is characterized by the obvious lack of myelinated axons in these structures
when compared to the opposite side. In the brainstem, these degenerated fibers are ipsilateral to their cells of origin but are contralateral to
their destination in the spinal cord—hence, the contralateral motor
deficit. These photographs give the user the unique opportunity of seeing where corticospinal fibers are located at all levels of the human
brainstem. Also, one is constantly reminded of 1) the relationship of
corticospinal fibers to other structures, 2) the deficits one can expect
to see at representative levels due to this lesion, and 3) the general appearance of degenerated fibers in the human central nervous system.
These images can be adapted to a wide range of instructional formats.
96 Internal Morphology of the Spinal Cord and Brain in Stained Sections
Degenerated
corticospinal
fibers
Degenerated
corticospinal
fibers
Spinal cord
Medulla
Pons
Midbrain
Infarct in
internal capsule
Internal capsule,
posterior limb
Motor cortex
(precentral gyrus)
Degenerated Corticospinal Tract 97
Post. column/med. lemniscus sys.
(proprioception/vibratory sense,
discriminative touch)
Corticospinal fibers
(somatomotor)
Spino-olivary fibers
Tectospinal tract
Medial motor nuclei
Medial longitudinal
fasciculus
Accessory nucleus
Spinal trigeminal nucleus (pars caudalis)
Gelantinosa
Magnocellular
Spinal trigeminal tract
Central gray
Pyramid
Anterolateral system
(pain/thermal sense,
touch from body)
Pyramidal decussation
Anterior spinocerebellar tract
Anterolateral system
Spinal trigeminal and/or ventral
trigeminothalamic fibers (pain/
thermal sense, touch from head)
Anterior corticospinal tract
Motor
Sensory
Posterior spinocerebellar tract
Rubrospinal tract
Reticulospinal fibers
Spinal trigeminal tract
Cuneate nucleus
Cuneate fasciculus
Vestibulospinal tract and
reticulospinal tract
Gracile nucleus
Gracile fasciculus
Cranial
nerve
nuclei
5-8 Transverse section of the medulla through the decussation of the
pyramids (motor decussation, pyramidal decussation, crossing of corticospinal fibers). This is the level of the spinal cord–medulla transition.
The corticospinal fibers have moved from their location in the lateral funiculus to the motor decussation and will cross to form the pyramid on
the opposite side.
98 Internal Morphology of the Spinal Cord and Brain in Stained Sections
Anatomical orientation
Clinical orientation
MRI, T1-weighted image
MRI, T2-weighted image
CT cisternogram
The Medulla Oblongata With MRI and CT 99
Arcuate nucleus
Pyramid
Post. column/med. lemniscus sys.
(proprioception/vibratory sense,
discriminative touch)
Corticospinal fibers
(somatomotor)
Medial accessory olivary nucleus
Principal olivary nucleus
Hypoglossal nerve
Vestibulospinal fibers
and reticulospinal fibers
Lateral reticular nucleus
Ventral trigeminothalamic tract
Fascicles of hypoglossal nerve
Nucleus ambiguus
Spinal trigeminal nucleus
(pars caudalis)
Spinal trigeminal tract
Posterior spinocerebellar
tract
Cuneate nucleus
Cuneate fasciculus
Gracile fasciculus
Gracile nucleus
Central gray
Anterolateral system
(pain/thermal sense,
touch from body)
Spinal trigeminal and/or ventral
trigeminothalamic fibers (pain/
thermal sense, touch from head)
Medial longitudinal fasciculus
Tectospinal tract
Medial lemniscus
Degenerated corticospinal fibers
Preolivary sulcus
Principal olivary nucleus
Anterolateral system
Retroolivary sulcus
(postolivary sulcus)
Motor
Sensory
Anterior spinocerebellar tract
Rubrospinal tract
Internal arcuate fibers
Restiform body
Accessory cuneate nucleus
Hypoglossal nucleus
Solitary nuclei and tract
Posterior longitudinal fasciculus
Dorsal motor nucleus of vagus
Cranial
nerve
nuclei
5-9 Transverse section of the medulla through the dorsal column nuclei (nucleus gracilis and nucleus cuneatus), caudal portions of the hypoglossal nucleus, caudal end of the principal olivary nucleus, and middle
portions of the sensory decussation (crossing of internal arcuate fibers).
100 Internal Morphology of the Spinal Cord and Brain in Stained Sections
Anatomical orientation
Clinical orientation
MRI, T1-weighted image
MRI, T2-weighted image
CT cisternogram
The Medulla Oblongata With MRI and CT 101
Post. column/med. lemniscus sys.
(proprioception/vibratory sense,
discriminative touch)
Pyramid
Corticospinal fibers
(somatomotor)
Nucleus raphe, obscurus
Arcuate nucleus
Medial accessory olivary nucleus
Preolivary sulcus
Ventral trigeminothalamic tract
Principal olivary nucleus
Retroolivary sulcus
(postolivary sulcus)
Hypolossal fibers
Lateral reticular nucleus
Spinal trigeminal nucleus
(pars interpolaris)
Spinal trigeminal tract
Nucleus ambiguus
Accessory cuneate nucleus
Cuneate nucleus
Cuneate fasciculus
Gracile nucleus
Level of obex
Anterolateral system
(pain/thermal sense,
touch from body)
Rubrospinal tract
Olivocerebellar fibers
Motor
Sensory
Central tegmental tract
and amiculum of olive
Cranial
nerve
nuclei
Posterior accessory olivary nucleus
Anterolateral system
Spinal trigeminal and/or ventral
trigeminothalamic fibers (pain/
thermal sense, touch from head)
Medial longitudinal fasciculus
Tectospinal tract
Medial lemniscus
Degenerated corticospinal fibers
Reticular formation
Anterior spinocerebellar tract
Internal arcuate fibers
Restiform body
(+ juxtarestiform body=
inferior cerebellar peduncle)
Solitary nuclei and tract
Dorsal motor nucleus of vagus
Hypoglossal nucleus
Posterior longitudinal fasciculus
Area postrema
5-10 Transverse section of the medulla through rostral portions of
the sensory decussation (crossing of internal arcuate fibers), obex, and the
caudal one-third of the hypoglossal and principal olivary nuclei.
102 Internal Morphology of the Spinal Cord and Brain in Stained Sections
Anatomical orientation
Clinical orientation
MRI, T1-weighted image
MRI, T2-weighted image
CT cisternogram
The Medulla Oblongata With MRI and CT 103
Post. column/med. lemniscus sys.
(proprioception/vibratory sense,
discriminative touch)
Pyramid
Corticospinal fibers
(somatomotor)
Nucleus raphe, pallidus
Arcuate nucleus
Medial accessory olivary nucleus
Ventral trigeminothalamic tract
Hypoglossal nerve
Principal olivary nucleus
Posterior accessory
olivary nucleus
Lateral reticular nucleus
Vagus nerve
Nucleus ambiguus
Restiform body
( + Juxtarestiform body =
Inferior cerebellar peduncle)
Accessory
cuneate nucleus
Inferior (spinal)
vestibular nucleus
Medial
vestibular nucleus
Dorsal motor nucleus
of the vagus
Nucleus raphe, obscurus
Medial longitudinal fasciculus
Tectospinal tract
Medial lemniscus
Anterolateral system
(pain/thermal sense,
touch from body)
Olivocerebellar fibers
Spinal trigeminal and/or ventral
trigeminothalamic fibers (pain/
thermal sense, touch from head)
Motor
Sensory
Central tegmental tract
and amiculum of olive
Anterolateral system
Cranial
nerve
nuclei
Anterior spinocerebellar tract
Rubrospinal tract
Spinal trigeminal tract
Spinal trigeminal nucleus
(pars interpolaris)
Solitary nuclei and tract
Degenerated corticospinal fibers
Reticular formation
Sulcus limitans
Hypoglossal nucleus
Posterior longitudinal fasciculus
5-11 Transverse section of the medulla through rostral portions of
the hypoglossal nucleus and the middle portions of the principal olivary nucleus. The fourth ventricle has flared open at this level, and the restiform body is enlarging to become a prominent structure on the dorsolateral aspect of the medulla.
104 Internal Morphology of the Spinal Cord and Brain in Stained Sections
Anatomical orientation
Clinical orientation
MRI, T1-weighted image
MRI, T2-weighted image
CT cisternogram
The Medulla Oblongata With MRI and CT 105
Solitary tract
Restiform body
Post. column/med. lemniscus sys.
(proprioception/vibratory sense,
discriminative touch)
Corticospinal fibers
(somatomotor)
Arcuate nucleus
Medial accessory olivary nucleus
Ventral trigeminothalamic tract
Principal olivary nucleus
Posterior accessory olivary nucleus
Reticulospinal fibers
Anterior spinocerebellar tract
Glossopharyngeal
nerve
Cochlear nerve
Anterior (ventral)
cochlear nucleus
Posterior (dorsal)
cochlear nucleus
Cerebellum
Solitary nuclei
Inferior salivatory nucleus
Spinal trigeminal and/or ventral
trigeminothalamic fibers (pain/
thermal sense, touch from head)
Nucleus raphe, pallidus
Degenerated corticospinal fibers
Olivocerebellar fibers
Motor
Sensory
Central tegmental tract
and amiculum of olive
Anterolateral system
Nucleus ambiguus
Rubrospinal tract
Cranial
nerve
nuclei
Spinal trigeminal nucleus (pars oralis)
Pontobulbar nucleus
Spinal trigeminal tract
Posterior (dorsal)
cochlear nucleus
Anterior (ventral)
cochlear nucleus
Inferior (or spinal) vestibular nucleus
Stria medullares of fourth ventricle
Medial vestibular nucleus
Nucleus prepositus
Reticular formation
Anterolateral system
(pain/thermal sense,
touch from body)
Medial longitudinal fasciculus
Tectospinal tract
Medial lemniscus
Pyramid
Nucleus raphe, obscurus
Posterior longitudinal fasciculus
5-12 Transverse section of the medulla through the posterior (dorsal) and anterior (ventral) cochlear nuclei and root of the glossopharyngeal
nerve. This corresponds to approximately the rostral third to fourth of
the principal olivary nucleus.
106 Internal Morphology of the Spinal Cord and Brain in Stained Sections
Anatomical orientation
Clinical orientation
MRI, T1-weighted image
MRI, T2-weighted image
CT cisternogram
The Medulla Oblongata With MRI and CT 107
Post. column/med. lemniscus sys.
(proprioception/vibratory sense,
discriminative touch)
Anterolateral system
(pain/thermal sense,
touch from body)
Medial
lemniscus
Nucleus raphe, obscurus
Medial longitudinal fasciculus
Superior medullary velum
Pontine nuclei
Pyramid
Reticular formation
Corticospinal fibers
(somatomotor)
Nucleus raphe, pallidus
Nucleus raphe, magnus
Principal olivary nucleus
Ventral trigeminothalamic tract
Tectospinal tract
Superior olive
Secondary cochlear fibers
Spinal trigeminal tract
Pontobulbar nucleus
(or body)
Vestibular root of
VIIIth nerve
Anterior (ventral)
cochlear nucleus
Spinal trigeminal nucleus
(pars oralis)
Solitary nuclei and tract
Lateral vestibular nucleus
Medial vestibular nucleus
Superior vestibular nucleus
Posterior longitudinal fasciculus
Spinal trigeminal and/or ventral
trigeminothalamic fibers (pain/
thermal sense, touch from head)
Motor
Sensory
Cranial
nerve
nuclei
Anterolateral system
Anterior spinocerebellar
tract
Rubrospinal tract
Facial nucleus
Facial nerve
Solitary tract
Secondary cochlear fibers
Central tegmental tract
Inferior
cerebellar
peduncle
Abducens nucleus
Restiform body
Juxtarestiform body
Superior olive
Degenerated corticospinal fibers
Superior
cerebellar
peduncle
5-13 Transverse section of the medulla–pons junction through the
rostral pole of the principal olivary nucleus and through the facial motor
nucleus. This plane is just caudal to the main portions of the abducens
nucleus. Pontine nuclei at this level may also be called arcuate nuclei.
108 Internal Morphology of the Spinal Cord and Brain in Stained Sections
Anatomical orientation
Clinical orientation
MRI, T1-weighted image
MRI, T2-weighted image
CT cisternogram
The Medulla Oblongata With MRI and CT 109
• Hypoglossal nerve in
medulla or hypoglossal nucleus
• Pyramid (corticospinal fibers)
• Medial lemniscus
Structure Damaged
Structure Damaged
• Contralateral loss of
• Anterolateral system
pain and thermal
fibers
sense on body
• Ipsilateral loss of pain
• Spinal trigeminal
and thermal sense on face tract and nucleus
• Dysphagia, soft palate
• Nucleus ambiguus,
paralysis, hoarseness,
roots of 9th and 10th
diminished gag
nerves
reflex
Deficits
Comment: The medial medullary syndrome is rare
compared to the more common occurence of the lateral
medullary syndrome. Nystagmus may result if the lesion
involves the medial longitudinal fasciculus or the nucleus prepositus hypoglossi. The lesion may involve
ventral trigeminothalamic fibers, but diminished pain
and thermal sense from the contralateral side of the face
is rarely seen. The combination of a contralateral hemiplegia and ipsilateral deviation of the tongue is called an
inferior alternating hemiplegia when the lesion is at this
level.
Lateral Medullary Syndrome: Results from occulsion of posterior inferior cerebellar artery or
branches of PICA to dorsolateral medulla (PICA syndrome, Wallenberg syndrome). In many cases the lateral medullary syndrome frequently results from occlusion of the vertebral artery with consequent loss of flow
into PICA.
• Contralateral hemiplegia
of arm and leg
• Contralateral loss of
position sense, vibratory
sense and discriminative
touch
• Deviation of tongue to
ipsilateral side when
protruded; muscle
atrophy and fasciculations
Deficits
Medial Medullary Syndrome: Results from occlusion of branches of anterior spinal artery.
• Restiform body and
spinocerebellar fibers
• Vestibular nuclei
(mainly inferior and
medial)
• Descending
hypothalamospinal
fibers
In addition to the above, involvement of
the solitary tract and nucleus may (rarely) cause dysageusia. Dyspnea and tachycardia may be seen in patients with
damage to the dorsal motor nucleus of the vagus. It is also
possible that damage to respiratory centers in the reticular formation or to the vagal motor nucleus may result
in hiccup (singultus). Bilateral medullary damage may
cause the syndrome of “Ondine’s curse,” an inability to
breathe without willing it or “thinking about it.”
Tonsillar Herniation: Although the cerebellar tonsil is not part of the medulla, the herniation of this structure (tonsillar herniation) down through the foramen
magnum has serious consequences for function of the
medulla. The coneing of the cerebellar tonsils into, and
through, the foramen magnum may compress the
medulla resulting in cardiac and respiratory arrest. This is
due to a combination of pressure on the medulla and the
occlusion of small vessels serving cardiac and respiratory
centers in the lateral area of the medulla. Patients experiencing a sudden herniation of the cerebellar tonsils
may lose consciousness rapidly and die.
Syringobulbia: A cavitation within the brainstem
(syringobulbia) may exist with syringomyelia, be independent of syringomyelia, or in some cases both may exist and communicate with each other. The cavity in syringobulbia is usually on one side of the midline of the
medulla. Signs and symptoms of syringobulbia may include weakness of tongue muscles (hypoglossal nucleus or
nerve), weakness of pharyngeal, palatal, and vocal musculature (ambiguus nucleus), nystagmus (vestibular nuclei),
and loss of pain and thermal sensation on the ipsilateral side
of the face (spinal trigeminal tract and nucleus or crossing of trigeminothalamic fibers).
Comment:
• Ipsilateral Horner
syndrome (miosis,
ptosis, anhidrosis,
flushing of face)
• Nausea, diplopia,
tendency to fall to
ipsilateral side,
nystagmus, vertigo
• Ataxia to the
ipsilateral side
Vascular Syndromes or Lesions of the Medulla Oblongata
RetF
FCu
FGr
ML
NuCu
NuGr
Py
RB
Cuneate fasciculus
Gracile fasciculus
Medial lemniscus
Cuneate nucleus
Gracile nucleus
Pyramid
Restiform body (⫹ juxtarestiform body ⫽
inferior cerebellar peduncle)
Reticular formation
Abbreviations
5-14 Semidiagrammatic representation of the internal distribution
of arteries in the medulla oblongata. Selected main structures are labeled primarily on the left side of each section and the general pattern
of arterial distribution overlies these structures on the right side. The
general distribution patterns of arteries in the medulla as illustrated
here may vary from patient to patient. For example, the territories
served by adjacent vessels may overlap to differing degrees at their
margins or the territory of a particular vessel may be smaller or larger
than seen in the typical pattern.
110 Internal Morphology of the Spinal Cord and Brain in Stained Sections
Rubrospinal
tract
Anterolateral
system
Posterior spinocerebellar tract
Lateral corticospinal tract
Posterior spinocerebellar tract
Spinal trigeminal tract
and nucleus
Hypoglossal nucleus
Solitary nuclei and tract
FCu
NuCu
Py
NuGr
RB
Medial decussation
Anterolateral system
Hypoglossal nerve
Spinal trigeminal tract
and nucleus
Nucleus ambiguus
Solitary nuclei and tract
Pyramidal decussation
Anterolateral system
FGr
Nucleus ambiguus
Internal arcuate fibers
Caudal
RetF
Hypoglossal nucleus
Inferior olivary complex
Anterolateral system
Dorsal motor nucleus of vagus
Rostral
Vestibular nuclei
ML
RetF
Vertebral artery plus paramedian branches of caudal portions
of basilar artery
Anterior inferior cerebellar artery
Posterior inferior cerebellar artery
Vertebral artery
Posterior spinal artery (and arterial vasocorona in spinal cord)
Inferior olivary complex
(principal nucleus)
Corticospinal fibers
Fourth ventricle
Medial longitudinal fasciculus
Nucleus prepositus
Solitary nuclei and tract
Anterior spinal artery
Medial lemniscus (ML)
Pyramid (Py)
Spinal trigeminal tract
and nucleus
Posterior (dorsal) cochlear nucleus
Arterial Patterns Within The Medulla Oblongata With Vascular Syndromes 111
Spinal trigeminal tract
DNu
Post. column/med. lemniscus sys.
(proprioception/vibratory sense,
discriminative touch)
GNu
Corticospinal fibers
(somatomotor)
Nucleus prepositus
Spinal trigeminal nucleus (pars oralis)
Anterior (ventral)
cochlear nucleus
Choroid plexus
Tela choroidea
Hilum of dentate nucleus
Inferior medullary velum
Emboliform nucleus
(anterior interposed
cerebellar nucleus)
DNu
FNu
Solitary nuclei and tract
Medial vestibular nucleus
Spinal trigeminal and/or ventral
trigeminothalamic fibers (pain/
thermal sense, touch from head)
Motor
Sensory
Cranial
nerve
nuclei
Inferior (spinal)
vestibular nucleus
Restiform body
Lateral recess of
fourth ventricle
Posterior (dorsal)
cochlear nucleus
Tonsil of cerebellum
Dentate nucleus, DNu
(lateral cerebellar nucleus)
Globose nucleus, GNu
(posterior interposed cerebellar nucleus)
Fastigial nucleus, FNu
(medial cerebellar nucleus)
and caudal portions of the globosus and emboliform nuclei. For additional details of the medulla at this level see figure 5-12 on page
106.
Medial longitudinal fasciculus
Tectospinal tract
Anterolateral system
(pain/thermal sense,
touch from body)
Nodulus
Uvula
5-15 Transverse section through the dorsal aspects of
medulla at the level of the cochlear nuclei and the cerebellar nuclei.
The plane corresponds to about the middle of the dentate nucleus
112 Internal Morphology of the Spinal Cord and Brain in Stained Sections
The Cerebellar Nuclei 113
Corticospinal fibers
(somatomotor)
Facial nerve
Central tegmental tract
Post. column/med. lemniscus sys.
(proprioception/vibratory sense,
discriminative touch)
Spinal trigeminal nucleus
(pars oralis)
Spinal trigeminal tract
Medial vestibular nucleus
Superior vestibular nucleus
Superior cerebellar peduncle
(brachium conjunctivum)
ENu
GNu
Anterolateral system
(pain/thermal sense,
touch from body)
Spinal trigeminal and/or ventral
trigeminothalamic fibers (pain/
thermal sense, touch from head)
Facial nerve
Motor
Sensory
Cranial
nerve
nuclei
Inferior
cerebellar
peduncle
Lateral vestibular nucleus
Restiform body
Juxtarestiform body
Dentate nucleus
(lateral cerebellar nucleus)
Emboliform nucleus, ENu
(anterior interposed cerebellar
nucleus)
Globose nucleus, GNu
(posterior interposed
cerebellar nucleus)
Facial motor nucleus
Abducens nerve
Medial longitudinal fasciculus
Tectospinal tract
Abducens nucleus
FNu
Fastigial nucleus, FNu
(medial cerebellar nucleus)
5-16 Transverse section through dorsal portions of pons at the
level of the abducens nucleus (and facial colliculus) and through rostral
portions of the cerebellar nuclei. For additional details of the pons at this
level see Figure 5-17 on page 116.
114 Internal Morphology of the Spinal Cord and Brain in Stained Sections
The Cerebellar Nuclei 115
SSNu
Corticospinal fibers
(somatomotor)
Ventral trigeminothalamic tract
Corticospinal fibers
Abducens nerve
Pontine nuclei
Superior olive
Lateral lemniscus
Post. column/med. lemniscus sys.
(proprioception/vibratory sense,
discriminative touch)
Facial motor nucleus
Facial nerve
Spinal trigeminal nucleus
(pars oralis)
Spinal trigeminal tract
Solitary nuclei and tract
Lateral vestibular nucleus
Juxtarestiform body
Medial vestibular nucleus
Superior
vestibular
nucleus
Abducens nucleus
Posterior longitudinal fasciculus
Anterolateral system
(pain/thermal sense,
touch from body)
Pontine nuclei
Spinal trigeminal and/or ventral
trigeminothalamic fibers (pain/
thermal sense, touch from head)
Motor
Sensory
Pontocerebellar fibers
Cranial
nerve
nuclei
Trapezoid body
and nuclei
Central tegmental tract
Anterolateral system
Rubrospinal tract
Anterior spinocerebellar
tract
Trigeminal nerve
Trigeminal motor nucleus
Principal sensory nucleus
Superior salivatory
nucleus, SSNu
Mesencephalic tract
and nucleus
Facial nerve,
internal genu
Restiform body
Superior vestibular nucleus
Degenerated corticospinal fibers
Nucleus raphe, magnus
Medial
lemniscus
Reticular formation
Superior
cerebellar
peduncle
Medial longitudinal fasciculus
Tectospinal tract
Superior medullary velum
5-17 Transverse section of the caudal
pons through the facial motor nucleus, abducens
nucleus (and facial colliculus), and the intramedullary course of fibers of facial and abducens nerves.
116 Internal Morphology of the Spinal Cord and Brain in Stained Sections
Anatomical orientation
Clinical orientation
MRI, T1-weighted image
MRI, T2-weighted image
CT cisternogram
The Pons With MRI and CT 117
Post. column/med. lemniscus sys.
(proprioception/vibratory sense,
discriminative touch)
Pontine nuclei
Pontine nuclei
Medial lemniscus
Fac,G
Corticospinal fibers
(somatomotor)
Corticospinal fibers
Ventral trigeminothalamic tract
Abducens nerve
Superior olive
Lateral lemniscus
Anterolateral system
Facial motor nucleus
Facial nerve
Spinal trigeminal nucleus
and tract (rostral end)
Abducens nucleus
Mesencephalic nucleus
and tract
Superior salivatory nucleus
Superior vestibular
nucleus
Posterior longitudinal fasciculus
Medial longitudinal fasciculus
Tectospinal tract
Anterolateral system
(pain/thermal sense,
touch from body)
Degenerated corticospinal fibers
Motor
Sensory
Cranial
nerve
nuclei
Central tegmental tract
Rubrospinal tract
Anterolateral system
Trigeminal nerve
Middle cerebellar peduncle
Trigeminal motor nucleus
(caudal part)
Principal sensory nucleus
(caudal part)
Anterior spinocerebellar tract
Pontocerebellar fibers
Spinal trigeminal and/or ventral
trigeminothalamic fibers (pain/
thermal sense, touch from head)
Nucleus raphe, magnus
Trapezoid body
Reticular
formation
Superior
cerebellar
peduncle
Mesencephalic nucleus
and tract
Facial nerve, Internal genu
(Fac, G)
Superior medullary velum
5-18 Transverse section of the pons through the rostral pole of the
facial nucleus and the internal genu of the facial nerve and rostral portions
of the abducens nucleus.
118 Internal Morphology of the Spinal Cord and Brain in Stained Sections
Anatomical orientation
Clinical orientation
MRI, T1-weighted image
MRI, T2-weighted image
CT cisternogram
The Pons With MRI and CT 119
Post. column/med. lemniscus sys.
(proprioception/vibratory sense,
discriminative touch)
Nucleus raphe, pontis
Reticular formation
Corticospinal fibers
(somatomotor)
Corticospinal fibers
Reticulotegmental nucleus
Medial lemniscus
Lateral lemniscus, nucleus
Superior olive
Lateral lemniscus
Trigeminal nerve
Trigeminal motor nucleus
Principal sensory nucleus
Tectospinal tract
Posterior longitudinal fasciculus
Medial longitudinal fasciculus
Anterolateral system
(pain/thermal sense,
touch from body)
Pontine nuclei
Pontine nuclei
Superior medullary velum
Spinal trigeminal and/or ventral
trigeminothalamic fibers (pain/
thermal sense, touch from head)
Degenerated corticospinal fibers
Motor
Sensory
Cranial
nerve
nuclei
Ventral trigeminothalamic tract
Central tegmental tract
Rubrospinal tract
Anterolateral system
ASCT
Middle cerebellar peduncle
(brachium pontis)
Lateral lemniscus
Trigeminal motor nucleus
Nucleus ceruleus
Anterior spinocerebellar tract
(ASCT)
Superior cerebellar peduncle
(brachium conjunctivum)
Mesencephalic tract
Pontocerebellar fibers
Mesencephalic nucleus
Central grey (periventricular grey)
5-19 Transverse section of the pons through the principal sensory
nucleus and motor nucleus of the trigeminal nerve.
120 Internal Morphology of the Spinal Cord and Brain in Stained Sections
Anatomical orientation
Clinical orientation
MRI, T1-weighted image
MRI, T2-weighted image
CT cisternogram
The Pons With MRI and CT 121
Corticospinal fibers
Post. column/med. lemniscus sys.
(proprioception/vibratory sense,
discriminative touch)
Basilar pons
Trigeminal nerve
Middle Cerebellar peduncle
(brachium pontis)
Ventral
trigeminothalamic
tract
Medial lemniscus
Anterolateral system
Nucleus centralis, superior
Central tegmental tract
Superior cerebellar peduncle
(brachium conjunctivum)
Cerebral aqueduct
Corticospinal fibers
(somatomotor)
Dorsal trigeminothalamic tract
Trochlear nerve, exit
Central gray (Periaqueductal gray)
Motor
Sensory
Degenerated corticospinal fibers
Spinal trigeminal and/or ventral
trigeminothalamic fibers (pain/
thermal sense, touch from head)
Pontine nuclei
Rubrospinal tract
Cranial
nerve
nuclei
Pontocerebellar
fibers
Tectospinal tract
Lateral lemniscus and
nuclei of lateral lemniscus
Reticular formation
Medial longitudinal fasciculus
Mesencephalic nucleus and tract
Locus ceruleus
Nucleus raphe, dorsalis
Anterolateral system
(pain/thermal sense,
touch from body)
Frenulum
5-20 Transverse section of the rostral pons through the exit of the
trochlear nerve and rostral portions of the exit of the trigeminal nerve.
122 Internal Morphology of the Spinal Cord and Brain in Stained Sections
Anatomical orientation
Clinical orientation
MRI, T1-weighted image
MRI, T2-weighted image
CT cisternogram
The Pons With MRI and CT 123
• Abducens nerve
fibers or
nucleus
• Paramedian
pontine reticular
formation
(pontine gaze
center)
• Corticospinal
fibers in basilar
pons
• Medial
lemniscus
Structure Damaged
Comment: The combination of corticospinal deficits
on one side of the body coupled with a cranial nerve motor deficit on the opposite is called a middle alternating
hemiplegia when the lesion is at this level. Diplopia will result (abducens nerve lesion) on gaze toward the side of the
lesion. Involvement of the abducens nucleus may also result in an inability to adduct the contralateral medial rectus muscle (damage to abducens internuclear neurons).
At caudal levels the lesion may extend lateral to involve
the lateral lemniscus (hypacusis), parts of the middle cerebellar peduncle (some ataxia), the facial motor nucleus (ipsilateral facial paralysis), the spinal trigeminal tract and nucleus (ipsilateral loss of pain and thermal sensation from the
face), and the anterolateral system (contralateral loss of pain
and thermal sensation from the body). At rostral pontine levels the lesion may extend into the medial lemniscus or may
involve only the arm fibers within this structure (cortralateral loss of vibratory sense, proprioception, and discriminative
touch), the motor nucleus of the trigeminal nerve (ipsilateral paralysis of masticatory muscles), or may damage the anterolateral system and rostral portions of the spinal
trigeminal tract and nucleus (loss of pain and thermal sensation from the body [contralateral] and from the face [ipsilateral]).
• Paralysis of conjugate gaze
toward side of lesion
• Contralateral loss or
decrease of position
and vibratory sense
and discriminative
touch (arm and leg)
• Ipsilateral lateral rectus
muscle paralysis
• Contralateral hemiplegia
of arm and leg
Deficits
Medial Pontine Syndrome: Results from occlusion of paramedian branches of basilar artery.
• Paramedian
pontine reticular
formation (at mid
to caudal levels)
• Anterolateral
system
• Facial motor
nucleus (caudal
levels)
• Trigeminal motor
nucleus
(midpontine levels)
• Descending
hypothalamospinal
fibers
• Spinal trigeminal
tract and nucleus
• Middle and
superior cerebellar
peduncles (caudal
and rostral levels)
• Vestibular and
cochlear nerves
and nuclei
Structure Damaged
Comment: The various combinations of these
deficits may vary depending on whether the lesion is located in lateral pontine areas at caudal levels versus lateral
pontine areas at rostral levels. As noted above lesions located in lateral portions of the pontine tegmentine may
also extend medial at either caudal or rostral levels and
give rise to some of the deficits discussed above in the section on medial pontine syndrome.
• Ipsilateral loss of pain
and thermal sense
from face
• Contralateral loss of pain
and thermal sense
from body
• Paralysis of conjugate
horizontal gaze
• Ipsilateral Horner
syndrome
• Ipsilateral paralysis of
masticatory muscles
• Vertigo, nausea,
nystagmus, deafness,
tinnitus, vomiting
(at caudal levels)
• Ipsilateral paralysis of
facial muscles
• Ataxia, unsteady gait,
fall toward side of lesion
Deficit
Lateral Pontine Syndrome: Results from occlusion of long circumferential branches of basilar artery.
Vascular Syndromes or Lesions of the Pons
RetF
SCP
ML
MLF
RB
BP
CSp
CTT
MCP
Basilar pons
Corticospinal fibers
Central tegmental tract
Middle cerebellar peduncle
(brachium pontis)
Medial lemniscus
Medial longitudinal fasciculus
Restiform body (⫹juxtarestiform body ⫽
inferior cerebellar)
Reticular formation
Superior cerebellar peduncle
(brachium conjunctivum)
Abbreviations
5-21 Semidiagrammatic representation of the internal distribution of arteries in the pons. Selected main structures are labeled
on the left side of each section; the general pattern of arterial distribution overlies these structures on the right side. Some patients
may have variations of the general distribution patterns of arteries
to the pons as shown here. For example, the adjacent territories
served by vessels may overlap to differing degrees at their margins
or the territory of a particular vessel may be smaller or larger than
seen in the general pattern.
124 Internal Morphology of the Spinal Cord and Brain in Stained Sections
RB
MCP
Basilar pons (BP)
Abducens nerve
Anterolateral system
Facial motor nucleus
Facial nerve
Spinal trigeminal
tract
Spinal trigeminal
nucleus
Vestibular nuclei
Caudal
CSp
ML
MLF
Anterolateral system
Abducens nucleus
Trigeminal nerve
Mesencephalic
Motor
Principal sensory
Trigeminal nuclei:
BP
MCP
Fourth ventricle
Superior medullary velum
Rostral
ML
CTT
RetF
SCP
SCP
Ventral trigeminothalamic fibers
Anterolateral system
Lateral lemniscus
Medial longitudinal fasciculus (MLF)
Mesencephalic nucleus and tract
CSp
ML
Long circumferential branches of basilar
artery and branches of superior
cerebellar artery
Short circumferential branches of basilar
artery
Long circumferential branches of basilar
artery and branches of anterior inferior
cerebellar artery
Paramedian branches of basilar artery
Trochlear nerve
Arterial Patterns Within The Pons With Vascular Syndrome 125
Inferior colliculus, central nucleus
Medial lemniscus
Ventral trigeminothalamic tract
Anterolateral system
Dorsal trigeminothalamic tract
Trochlear nerve
Reticular formation
Inferior colliculus, external nucleus
Lateral lemniscus
Post. column/med. lemniscus sys.
(proprioception/vibratory sense,
discriminative touch)
Corticospinal fibers
Crus cerebri
Spinal trigeminal and/or ventral
trigeminothalamic fibers (pain/
thermal sense, touch from head)
Degenerated corticospinal fibers
Pontine nuclei
Motor
Sensory
Pontocerebellar fibers
Parietopontine fibers
Occipitopontine fibers
Temporopontine fibers
Nucleus centralis, superior
Tectospinal tract
Central tegmental tract
Medial longitudinal fasciculus
Nucleus ceruleus
Mesencephalic nucleus and tract
Nucleus raphe, dorsalis
Cerebral aqueduct
Central gray (periaqueductal gray)
Anterolateral system
(pain/thermal sense,
touch from body)
Superior cerebellar
peduncle, decussation
Corticospinal fibers
(somatomotor)
Posterior longitudinal fasciculus
Rubrospinal tract
Inferior colliculus, commissure
Inferior colliculus, pericentral nucleus
Cranial
nerve
nuclei
5-22 Transverse section of the brainstem at the pons–midbrain
junction through the inferior colliculus, caudal portions of the decussation of the superior cerebellar peduncle, and rostral parts of the basilar pons.
The plane of section is just caudal to the trochlear nucleus.
126 Internal Morphology of the Spinal Cord and Brain in Stained Sections
Anatomical orientation
Clinical orientation
MRI, T1-weighted image
MRI, T2-weighted image
CT cisternogram
The Midbrain With MRI and CT 127
FPon
Corticospinal fibers
(somatomotor)
Pontine nuclei
Rubrospinal tract
Corticospinal
fibers
Post. column/med. lemniscus sys.
(proprioception/vibratory sense,
discriminative touch)
TPon
OPon
PPon
Central tegmental tract
Medial longitudinal fasciculus
Dorsal trigeminothalamic tract
Reticular formation
Mesencephalic nucleus and tract
Ventral trigeminothalamic tract
Crus cerebri
Superior colliculus
Inferior colliculus, brachium
Cerebral aqueduct
Anterolateral system
(pain/thermal sense,
touch from body)
Substantia nigra,
pars compacta
Spinal trigeminal and/or ventral
trigeminothalamic fibers (pain/
thermal sense, touch from head)
Motor
Sensory
Cranial
nerve
nuclei
Degenerated corticospinal fibers
Parietopontine fibers (PPon)
Occipitopontine fibers (OPon)
Temporopontine fibers (TPon)
Frontopontine fibers (FPon)
Interpeduncular nucleus
Anterolateral
system
Medial lemniscus
Tectospinal tract
Spinothalamic tract
Spinotectal tract
Trochlear nucleus
Nucleus raphe, dorsalis
Posterior longitudinal fasciculus
Interpeduncular fossa
Superior cerebellar
peduncle, decussation
Central gray (periaqueductal gray)
C
( c o o r tic
rtic on
u
o
bu cle
lba ar
f
r
fib iber
er s
s)
5-23 Transverse section of the midbrain through the trochlear nucleus and decussation of the superior cerebellar peduncle. The section also includes caudal parts of the superior colliculus and the rostral tip of the basilar pons.
128 Internal Morphology of the Spinal Cord and Brain in Stained Sections
Anatomical orientation
Clinical orientation
MRI, T1-weighted image
g
g
MRI, T2-weighted image
g
g
CT cisternogram
g
The Midbrain With MRI and CT 129
Crus cerebri
Pallidonigral fibers
Nigrostriatal fibers
Corticonigral fibers
Post. column/med. lemniscus sys.
(proprioception/vibratory sense,
discriminative touch)
Arm
Leg
Trunk
Corticospinal
fibers
PPon
OPon
TPon
SNpr
SNpc
Corticospinal fibers
(somatomotor)
FPon
Medial
lemniscus
C
(c orti
or co
tic nu
o
bu cle
lba ar
r fi
f
ibe ber
rs s
)
Medial geniculate nucleus
Ventral trigeminothalamic tract
Inferior colliculus, brachium
Reticular formation
Dorsal trigeminothalamic tract
Superior colliculus
Central gray (periaqueductal gray)
Oculomotor nerve
Anterolateral system
(pain/thermal sense,
touch from body)
Cerebral aqueduct
Anterior (ventral) tegmental decussation
Rubrospinal tract
Frontopontine fibers (FPon)
Spinal trigeminal and/or ventral
trigeminothalamic fibers (pain/
thermal sense, touch from head)
Motor
Sensory
Cranial
nerve
nuclei
Superior cerebellar peduncle, decussation
Degenerated corticospinal fibers
Parietopontine fibers (PPon)
Occipitopontine fibers (OPon)
Temporopontine fibers (TPon)
Substantia nigra
pars reticulata (SNpr)
Substantia nigra
pars compacta (SNpc)
Red nucleus
Posterior (dorsal)
tegmental decussation
Central tegmental tract
Spinothalamic tract
Medial longitudinal fasciculus
Spinotectal tract
Mesencephalic nucleus and tract
Oculomotor nucleus
Posterior longitudinal fasciculus
includes rostral portions of the decussation of the superior cerebellar peduncle, which, at this level, are intermingled with the caudal part of the
red nucleus. Leg ⫽ lower extremity; Arm ⫽ upper extremity.
Interpeduncular nucleus
5-24 Transverse section of the midbrain through the superior colliculus, caudal parts of the oculomotor nucleus, and caudal parts of the red
nucleus. The plane of section is caudal to the Edinger-Westphal nucleus but
130 Internal Morphology of the Spinal Cord and Brain in Stained Sections
Anatomical orientation
Clinical orientation
MRI, T1-weighted image
MRI, T2-weighted image
CT cisternogram
The Midbrain With MRI and CT 131
TPon
OPon
FPon
Post. column/med. lemniscus sys.
(proprioception/vibratory sense,
discriminative touch)
Corticospinal fibers
(somatomotor)
Substantia nigra,
pars compacta (SNpc)
Substantia nigra,
pars reticulata (SNpr)
Corticonuclear fibers
(corticobulbar fibers)
Leg
Trunk
Arm
PPon
Medial geniculate
nucleus
Pulvinar nuclear
complex
Corticospinal fibers
Corticonigral fibers
Pallidonigral fibers
Nigrostriatal fibers
Optic tract
Medial
lemniscus
Lateral
geniculate
nucleus
Spinothalamic tract
Edinger-Westphal nucleus
Red nucleus
Central grey (periaqueductal grey)
Mesencephalic tract and nucleus
Superior colliculus, brachium
Cerebral aqueduct
Medial longitudinal fasciculus
Central tegmental tract
Habenulopeduncular tract
Spinal trigeminal and/or ventral
trigeminothalamic fibers (pain/
thermal sense, touch from head)
Motor
Sensory
Cranial
nerve
nuclei
Cerebellorubral fibers and
cerebellothalamic fibers
Degenerated corticospinal fibers
Parietopontine fibers (PPon)
Occipitopontine fibers (OPon)
Temporopontine fibers (TPon)
Peripeduncular nucleus
Brachium, inferior colliculus
Ventral trigeminothalamic tract
Dorsal trigeminothalamic tract
Frontopontine fibers (FPon)
pr
SN
Spinotectal tract
Superior colliculus
Posterior longitudinal fasciculus
Oculomotor nuclei
Oculomotor nerve
Anterolateral system
(pain/thermal sense,
touch from body)
Superior colliculus, commissure
plane of this section is also through caudal portions of the diencephalon including the pulvinar nuclear complex and the medial and lateral geniculate nuclei. Leg ⫽ lower extremity; Arm ⫽ upper extremity.
SN
pc
5-25 Transverse section of the midbrain through the superior colliculus, rostral portions of the oculomotor nucleus, including the EdingerWestphal nucleus, and the exiting fibers of the oculomotor nerve. The
132 Internal Morphology of the Spinal Cord and Brain in Stained Sections
Anatomical orientation
Anatomical orientation
Clinical orientation
Clinical orientation
MRI, T1 weighted image
MRI, T1-weighted image
MRI, T2 weighted image
MRI, T2-weighted image
MRI, T2-
MRI, T2-weighted image
MRI, T1
MRI, T1-weighted image
The Midbrain With MRI and CT 133
Parietopontine fibers
Occipitopontine fibers
Temporopontine fibers
Supraoptic nucleus
Fornix (F)
Optic
tract
(OpTr)
Subthalamic nucleus
Medial
geniculate
nucleus (MGNu)
Medial longitudinal fasciculus
Nucleus of Cajal
Nucleus of Darkschewitsch
Central grey (periaqueductal grey)
Cerebral aqueduct
Spinal trigeminal and/or ventral
trigeminothalamic fibers (pain/
thermal sense, touch from head)
Motor
Sensory
Cranial
nerve
nuclei
Transition from
crus cerebri (CC) to
internal capsule
Cerebellorubral fibers and
Cerebellothalamic fibers
Medial lemniscus
Lateral geniculate
nucleus (LGNu)
Spinothalamic tract
Brachium of
superior colliculus
contains some mammillotegmental fibers. Structures at the midbrainthalamus junction are best seen in an MRI angled to accommodate that
specific plane. To make the transition from drawing to stained section
to MRI easy, selected structures in the MRI are labeled.
Mammillothalamic tract (MtTr)
Red nucleus
(RNu)
Anterolateral system
(pain/thermal sense,
touch from body)
Posterior
commissure
Pineal
Third ventricle
Hypothalamus
Pretectal
nuclei
Superior
colliculus
Corticospinal fibers
(somatomotor)
Habenulopeduncular tract
Frontopontine fibers
Corticonuclear fibers
(corticobulbar fibers)
Corticospinal
fibers
Post. column/med. lemniscus sys.
(proprioception/vibratory sense,
discriminative touch)
Peripeduncular nucleus
Ventral trigeminothalamic tract
Central tegmental tract
Pulvinar nuclear complex (Pul)
Dorsal
trigeminothalamic tract
5-26 Slightly oblique section through the midbrain–diencephalon
junction. The section passes through the posterior commissure, the rostral end of the red nucleus, and ends just dorsal to the mammillary body.
At this level, the structure labeled mammillothalamic tract probably also
134 Internal Morphology of the Spinal Cord and Brain in Stained Sections
Anatomical orientation
Clinical orientation
MRI, T2-weighted image
Pul
Pul
MRI, inversion recovery
LGNu
MGNu
OpTr
RNu
CC
F, MTTr
OpTr
The Midbrain With MRI and CT 135
Structure Damaged
• Red nucleus and
cerebellothalamic
fibers
• Oculomotor nerve
Structure Damaged
Comment: The lesion in this syndrome may extend laterally into the medial lemniscus and the dorsally
adjacent ventral trigeminothalamic fibers. If this was the
case, there could conceivably be a loss or diminution of
• Ipsilateral paralysis of
eye movement: eye
oriented down and out
and pupil dilated
and fixed
• Contralateral ataxia
and tremor of
cerebellar origin
Deficit
Central Midbrain Lesion (Claude syndrome)
Comment: This combination of motor deficits at
this level of the brainstem is called a superior alternating
hemiplegia. This pattern consists of ipsilateral paralysis of
eye movement and contralateral hemiplegia of the upper and
lower extremities. Damage to the corticonuclear (corticobulbar) fibers in the crus cerebri may result in a partial deficit in tongue and facial movement on the contralateral side. These cranial nerve deficits are seen as a
deviation of the tongue to the side opposite the lesion on
attempted protrusion and a paralysis of the lower half of the
facial muscles on the contralateral side. Although parts of
the substantia nigra are frequently involved, akinesia or
dyskinesia are not frequently seen.
• Contralateral hemiplegia • Corticospinal fibers
of arm and leg
in crus cerebri
• Ipsilateral paralysis of
• Oculomotor nerve
eye movement: eye
oriented down and out
and pupil dilated and
fixed
Deficit
Medial Midbrain (Weber) Syndrome: May result from occlusion of paramedian branches of P1 segment of posterior cerebral artery.
position and vibratory sense and of discriminative touch
from the contralateral arm and partial loss of pain and
thermal sensation from the contralateral face.
Benedikt syndrome: This results from a larger
lesion of the midbrain that essentially involves both of
the separate areas of Weber and Claude. The main
deficits are contralateral hemiplegia of arm and leg (corticospinal fibers), ipsilateral paralysis of eye movement
with dilated pupil (oculomotor nerve), and cerebellar
tremor and ataxia (red nucleus and cerebellothalamic
fibers). Slight variations may be present based on the extent of the lesion.
Parinaud syndrome: This syndrome is usually
caused by a tumor in the pineal region, such as germinoma, astrocytoma, pineocytoma/pineoblastoma, or
any of a variety of other tumors that impinge on the superior colliculi. The potential for occlusion at the cerebral aqueduct in these cases also indicates that hydrocephalus may be a component of this syndrome. The
deficits in these patients consist of a paralysis of upward
gaze (superior colliculi), hydrocephalus (occlusion of the
cerebral aqueduct), and eventually a failure of eye movement due to pressure on the oculomotor and trochlear
nuclei. These patients may also exhibit nystagmus due to
involvement of the medial longitudinal fasciculus.
Uncal Herniation: Herniation of the uncus occurs
in response to large and rapidly expanding lesions in the
cerebral hemisphere, this being a supratentorial location.
Uncal herniation is an extrusion of the uncus through the
tentorial notch (tentorial incisura) with resultant pressure
on the oculomotor nerve and the crus cerebri of the midbrain. Initially the pupils, unilaterally or bilaterally, may
dilate or respond slowly to light, followed by weakness of
oculomotor movement. As herniation progresses the
pupils will be fully dilated, eye movements regulated by
the oculomotor nerve may be slow or absent, and the eyes
will deviate slightly laterally due to the unopposed actions
of the abducens nerves. There is usually weakness on the
contralateral side of the body due to compression of corticospinal fibers in the crus cerebri. However, if pressure
is sufficient the entire midbrain may shift so that there can
be contralateral as well as ipsilateral weakness due to
pressure on the same side and pressure on the opposite
side of the crus cerebri. This hemiplegia ipsilateral to the
Vascular Syndromes or Lesions of the Midbrain
IC
LGNu
MGNu
ML
RNu
SC
SCP
SN
BP
CC
DecSCP
Basilar pons
Crus cerebri
Decussation of the superior cerebellar
peduncle
Inferior colliculus
Lateral geniculate nucleus
Medial geniculate nucleus
Medial lemniscus
Red nucleus
Superior colliculus
Superior cerebellar peduncle
Substantia nigra
Abbreviations
5-27 Semidiagrammatic representation of the internal distribution
of arteries in the midbrain. Selected main structures are labeled on the
left side of each section; the typical pattern of arterial distribution overlies these structures on the right side. The general distribution patterns
of the vessels to the midbrain as shown here may vary somewhat from
patient to patient. For example, the adjacent territories served by
neighboring vessels may overlap to differing degrees at their margins
or the territory of a particular vessel may be larger or smaller than seen
in the general pattern.
herniation and ipsilateral to the oculomotor deficits is called the Kernohan
phenomenon. As damage from the pressure on the midbrain extends down
and into the upper pons the pupils are dilated and fixed, eye movement
is largely absent, respiration is decreased, and the patient will become
decerebrate (upper and lower extremities extended, toes pointed inward, fingers flexed, forearm pronated, head and neck extended).
136 Internal Morphology of the Spinal Cord and Brain in Stained Sections
CC
ML
Interpeduncular fossa
Medial longitudinal fasciculus
Anterolateral
system
Lateral lemniscus
Periaqueductal gray
Cerebral aqueduct
Caudal
BP
SCP
Anterolateral system
Ventral trigeminothalamic
fibers
Medial longitudinal fasciculus
Mesencephalic nucleus
Trochlear nucleus
Rostral
SN
CC
ML
DecSCP
IC
Oculomotor nerve
Ventral trigeminothalmic
fibers
LGNu
MGNu
Anterolateral system
Oculomotor nucleus
CC
SN
ML
Edinger-Westphal nucleus
Thalamogeniculate artery
Quadrigeminal and superior cerebellar arteries (level of
inferior colliculus), quadrigeminal and posterior medial
choroidal arteries (level of superior colliculus)
Lateral branches of quadrigeminal (level of inferior colliculus),
quadrigeminal and posterior medial choroidal arteries (level
of superior colliculus)
Anterolateral (short circumferential) branches of the
quadrigeminal and medial posterior choroidal arteries
Anteromedial (paramedian) branches of basilar bifurcation
and P1 segment
RNu
SC
Arterial Patterns Within The Midbrain With Vascular Syndromes 137
Lateral ventricle,
inferior horn
Fimbria of
hippocampus
Caudate
nucleus, tail
Fornix, crus
Stria terminalis
Optic radiations
Caudate
nucleus, body
Hippocampal commissures
Trochlear nerve
Pulvinar
Medial longitudinal stria
of indusium griseum
Cingulum
Superior cistern
Pineal
Corpus callosum,
splenium
Cingulate gyrus
Cerebellum
Superior cerebellar peduncle
Inferior colliculus
Hippocampal
formation
Caudate
nucleus
Choroid plexus
Tapetum
Atrium of lateral ventricle
Lateral longitudinal stria
5-28 Coronal section of forebrain through the splenium of the corpus
callosum and crus of fornix, and extending into the inferior colliculus and exit
of the trochlear nerve. Many of the structures labeled in this figure can be
easily identified in the T1-weighted MRI adjacent to the photograph.
138 Internal Morphology of the Spinal Cord and Brain in Stained Sections
The Diencephalon and Basal Nuclei With MRI 139
Lateral ventricle,
inferior horn
Alveus of
hippocampus
Caudate
nucleus, tail
StTer and bed
nucleus
Lateral
geniculate
nucleus
Optic radiations
Inferior pulvinar
nucleus
Medial
geniculate
nucleus
Corticospinal fibers
(somatomotor)
Inferior colliculus, brachium
Superior colliculus, brachium
Post. column/med. lemniscus sys.
(proprioception/vibratory sense,
discriminative touch)
Stria terminalis (StTer)
Caudate nucleus, body
Lateral ventricle, body
Fornix, body
Medial longitudinal stria
Anterolateral system
(pain/thermal sense,
touch from body)
Superior cistern
Corpus callosum,
body
Lateral
nucleus
Spinal trigeminal and/or ventral
trigeminothalamic fibers (pain/
thermal sense, touch from head)
Motor
Sensory
Cranial
nerve
nuclei
Hippocampal
formation
Retrolenticular limb
Sublenticular limb
Internal capsule
Central grey (periaqueductal grey)
Hippocampus,
fimbria of
Pulvinar
nuclear
complex
Insula
External medullary lamina
Choroid plexus
Lateral longitudinal stria
of indusium griseum
Trochlear nucleus
Medial
nucleus
Cingulum
5-29 Coronal section of the forebrain through the pulvinar and beled in this figure can be easily identified in the T1-weighted MRI
the medial and lateral geniculate nuclei. The section extends into up- adjacent to the photograph.
per portions of the midbrain tegmentum. Many of the structures
laCingulate gyrus
140 Internal Morphology of the Spinal Cord and Brain in Stained Sections
The Diencephalon and Basal Nuclei With MRI 141
Subthalamic
nucleus
Globus pallidus:
Lateral segment
Medial segment
Ventral posteromedial
nucleus of thalamus
Ventral posterolateral
nucleus of thalamus
Post. column/med. lemniscus sys.
(proprioception/vibratory sense,
discriminative touch)
Medial
nucleus
Corticospinal fibers
(somatomotor)
Third
ventricle
Pineal
Anterolateral system
(pain/thermal sense,
touch from body)
Hypothalamus
Habenular nucleus
Mammillothalamic tract
Thalamic fasciculus
Lenticular fasciculus
Zona incerta
Lateral
nucleus
Pulvinar
nuclear
complex
Habenular commissure
Column of fornix
Ansa lenticularis
Internal capsule,
posterior limb
Motor
Sensory
Anterior commissure
Spinal trigeminal and/or ventral
trigeminothalamic fibers (pain/
thermal sense, touch from head)
Dorsomedial
nucleus of
thalamus
Centromedian nucleus of thalamus
Habenulopeduncular tract
Cranial
nerve
nuclei
5-30 Slightly oblique section of the forebrain through the pulvinar, ventral posteromedial, and ventral posterolateral nuclei. The section extends rostrally through the
subthalamic nucleus and ends in the caudal hypothalamus just dorsal to the mammillary
bodies as seen by the position of the (postcommissural) fornix.
142 Internal Morphology of the Spinal Cord and Brain in Stained Sections
The Diencephalon and Basal Nuclei With MRI 143
Lateral ventricle,
inferior horn
Caudate
nucleus, tail
Optic tract
Globus pallidus:
Lateral segment
Medial segment
External capsule
Claustrum
Extreme capsule
Internal capsule,
posterior limb
Corticospinal fibers
(somatomotor)
Putamen
Crus cerebri
Hippocampal formation
StTer
Choroid plexus
Stria terminalis (StTer)
Caudate nucleus, body
Fornix, body
Lateral longitudinal stria
Medial longitudinal stria
of indusium griseum
Crus
cerebri
Ventral lateral
nucleus
Substantia nigra
Cerebellothalamic fibers
Basilar pons
Red
nucleus
Dorsomedial
nucleus
Posterior cerebral artery
Alveus of hippocampus
Subthalamic nucleus
Lenticular fasciculus
Zona incerta
Thalamic fasciculus
Insula
External medullary lamina and
thalamic reticular nucleus
Internal medullary lamina
Lateral dorsal nucleus of thalamus
Stria medullaris thalami
Lateral ventricle, body
Corpus callosum, body
of the structures labeled in this figure can be easily identified
in the T1-weighted MRI adjacent to the photograph.
Cingulum
Cingulate gyrus
5-31 Coronal section of the forebrain through the lateral
dorsal nucleus, massa intermedia, and subthalamic nucleus. Many
144 Internal Morphology of the Spinal Cord and Brain in Stained Sections
The Diencephalon and Basal Nuclei With MRI 145
Lateral ventricle,
inferior horn
Amygdaloid
nuclear
complex
Optic tract
External capsule
Claustrum
Extreme capsule
Internal capsule,
posterior limb
Corticospinal fibers
(somatomotor)
Insula
Putamen
lat.
Globus
pallidus
Anterior nucleus
Stria terminalis
Caudate nucleus, body
Third ventricle
Cingulate gyrus
Posterior hypothalamus
Mammillary body
med.
Fornix, body
Lateral longitudinal stria
of indusium griseum
Medial longitudinal stria
Hippocampal formation
Mammillothalamic tract
Ventral lateral
nucleus
Alveus of hippocampus
Subthalamic nucleus
Lenticular fasciculus
Zona incerta
Thalamic fasciculus
Internal medullary lamina
External medullary lamina and
thalamic reticular nucleus
Dorsomedial nucleus of
thalamus
Stria medullaris thalami
Choroid plexus
Lateral ventricle, body
Corpus callosum, body
structures labeled in this figure can be easily identified in the
T1-weighted MRI.
Cingulum
5-32 Coronal section of the forebrain through the anterior nucleus of the thalamus and mammillary body. Many of the
146 Internal Morphology of the Spinal Cord and Brain in Stained Sections
The Diencephalon and Basal Nuclei With MRI 147
Lateral ventricle,
inferior horn
Caudate
nucleus, tail
Optic tract
Subthalamic
nucleus
Lenticular
fasciculus
Thalamic
fasciculus
Extreme capsule
Globus pallidus,
lateral segment
Claustrum
External capsule
Internal capsule,
posterior limb
Mammillothalamic
tract
Hippocampus
Putamen
Stria terminalis
Caudate
nucleus, body
Substantia nigra
Crus cerebri
VL to VA
transition
Anterior nucleus
Zona
incerta
Stria medullaris thalami
Dorsomedial
nucleus
Oculomotor nerve
Third
ventricle
Corpus callosum,
body
Red nucleus
Ventral lateral
nucleus
Choroid plexus
Fornix, body
Corticonigral fibers
Pallidonigral fibers
Nigrostriatal fibers
Lateral geniculate
nucleus
Cerebellorubral fibers and
cerebellothalamic fibers
External medullary
lamina and
thalamic reticular
nucleus
Internal medullary lamina
Lateral ventricle, body
5-33 Slightly oblique section of the forebrain through the anterior
nucleus of the thalamus and the subthalamic nucleus. The section also includes the rostral portion of the midbrain tegmentum. Many of the structures labeled in this figure can be easily identified in the T1-weighted
MRI adjacent to the photograph.
148 Internal Morphology of the Spinal Cord and Brain in Stained Sections
The Diencephalon and Basal Nuclei With MRI 149
Claustrum
Supraoptic
decussation
Ventral amygdalofugal
fibers
Anterior commissure
Fornix, column
Globus pallidus:
Lateral segment
Medial segment
External capsule
Extreme capsule
Insula
Putamen
Caudate nucleus, head
Amygdaloid nucleus (complex)
Optic tract
Stria terminalis
Internal capsule,
genu
Lateral longitudinal stria
of indusium griseum
Medial longitudinal stria
Arcuate
Dorsomedial
Ventromedial
Supraoptic
Third
ventricle
Lateral ventricle
Corpus callosum,
body
Cingulate gyrus
Hypothalamic
nuclei
Lateral hypothalamic
area
Anterior
nucleus
Ansa lenticularis
Basal nucleus
of Meynert
Lenticular fasciculus
Ventral anterior nucleus
Interventricular foramen
Choroid plexus
Fornix, column
thalamus. Many of the structures labeled in this figure can be
easily identified in the T1-weighted MRI adjacent to the photograph.
Septum pellucidum
5-34 Coronal section of the forebrain through the interventricular foramen, genu of the internal capsule, rostral tip of the
dorsal thalamus, and about the middle one-third of the hypo-
150 Internal Morphology of the Spinal Cord and Brain in Stained Sections
The Diencephalon and Basal Nuclei With MRI 151
Lateral olfactory
stria
Diagonal band
(of Broca)
External capsule
Extreme capsule
Claustrum
Internal capsule,
anterior limb
Insula
Stria terminalis
Uncus
Supraoptic decussation
Anterior perforated
substance
Basal nucleus of Meynert
Putamen
Anterior commissure
Fornix, column
Preoptic area
of hypothalamus
Globus pallidus,
lateral segment
Septum pellucidum
Infundibulum
Lateral ventricle,
anterior horn
Cingulum
Cingulate gyrus
Corpus callosum,
body
Third ventricle
Optic tract
Supraoptic nucleus
Amygdaloid nucleus
Caudate nucleus, head
Septal nuclei
Lateral longitudinal stria
Medial longitudinal stria
of indusium griseum
5-35 Coronal section of the forebrain through the anterior commissure and rostral aspects of the hypothalamus. Many of the structures labeled in this figure can be easily identified in the T1-weighted MRI.
152 Internal Morphology of the Spinal Cord and Brain in Stained Sections
The Diencephalon and Basal Nuclei With MRI 153
Claustrum
External capsule
Extreme capsule
Anterior cerebral artery
Diagonal band (of Broca)
Lateral olfactory stria
Globus pallidus,
lateral segment
Caudate nucleus,
head
Medial longitudinal stria
of indusium griseum
Cingulate gyrus
Optic chiasm
Septum
pellucidum
Corpus callosum, body
Lateral ventricle,
anterior horn
Anterior cerebral
arteries
Paraterminal gyrus
Nucleus accumbens
Putamen
Lateral longitudinal stria
Cingulum
Middle cerebral artery
Insula
Internal capsule,
anterior limb
accumbens. Many of the structures labeled in this figure can be easily identified in the T1-weighted MRI adjacent to the photograph.
Medial
olfactory
stria
5-36 Coronal section of the forebrain through the head of the
caudate nucleus, rostral portions of the optic chiasm, and the nucleus
154 Internal Morphology of the Spinal Cord and Brain in Stained Sections
The Diencephalon and Basal Nuclei With MRI 155
Subcallosal gyrus
Claustrum
Internal capsule,
anterior limb
Olfactory sulcus
Corpus callosum, rostrum
Olfactory tract
Lateral longitudinal stria
Septum
pellucidum
Lateral ventricle,
anterior horn
Gyrus rectus (straight gyrus)
Orbital gyri
Putamen
Extreme capsule
External capsule
Medial longitudinal stria
of indusium griseum
Caudate nucleus,
head
Cingulum
Anterior cerebral arteries
Corpus callosum, body
Anterior cerebral arteries
Cingulate gyrus
5-37 Coronal section of forebrain through the head of the caudate
nucleus and the anterior horn of the lateral ventricle. Many of the structures
labeled in this figure can be easily identified in the T1-weighted MRI
adjacent to the photograph.
156 Internal Morphology of the Spinal Cord and Brain in Stained Sections
The Diencephalon and Basal Nuclei With MRI 157
clusion of distal branches of the ACA results in motor
Occlusion of Distal Branches of the Anterior
(ACA) or Middle (MCA) Cerebral Arteries: Oc-
result in contralateral hemiplegia (corticospinal fibers) and
a loss, or diminution, of sensory perception (pain, thermal sense, proprioception) caused by damage to thalamocortical fibers traversing the posterior limb to the overlying sensory cortex. If the lesion extends into the genu of
the capsule, a partial paralysis of facial muscles and tongue
movement may also occur contralaterally.
Infarction of Posterior Thalamic Nuclei: Occlusion of vessels to posterior thalamic regions results in
either a complete sensory loss (pain/thermal sense, touch,
vibratory and position sense) on the contralateral side of
the body, or a dissociated sensory loss. In the latter case the
patient may experience pain/thermal sensory losses but
not position/vibratory losses, or vice versa. As the lesion resolves the patient may experience intense persistent pain, thalamic pain, or anesthesia dolorosa.
Occlusion of Lenticulostriate Branches to Internal Capsule: Damage to the internal capsule may
Forebrain vascular lesions result in a wide range of
deficits that include motor and sensory losses and a variety of cognitive disorders. Forebrain vessels may be
occluded by a thrombus. This is a structure (usually a
clot) formed by blood products and frequently attached
to the vessel wall. Deficits may appear slowly, or wax
and wane, as the blood flow is progressively restricted.
Vessels may also be occluded by embolization. A foreign body, or embolus (fat, air, piece of thrombus, piece
of sclerotic plaque, clump of bacteria, etc.), is delivered
from some distant site into the cerebral circulation where
it lodges in a vessel. Since this is a sudden event deficits
usually appear quickly and progress rapidly. Interruption
of blood supply to a part of the forebrain will result in an
infarct of the area served by the occluded vessel.
Lesion in the Subthalamic Nucleus: Small vascular lesions occur in the subthalamic nucleus, resulting
in rapid and unpredictable flailing movements of the
contralateral extremities (hemiballismus). Movements
are more obvious in the arm than in the leg. The clinical
expression of this lesion is through corticospinal fibers,
therefore it is on the contralateral side of the body.
and sensory losses in the contralateral foot, leg, and thigh
owing to damage to the anterior and posterior paracentral gyri (primary motor and sensory cortices for lower
extremity). Occlusion of distal branches of MCA results
in contralateral motor and sensory losses of the upper
extremity, trunk, and face with sparing of the leg and
foot, and a consensual deviation of the eyes to the ipsilateral side. This represents damage to the precentral
and postcentral gyri and to the frontal eye fields.
Watershed Infarct: Sudden systemic hypotension, hypoperfusion, or embolic showers may result in
infarcts at border zones between the territories served
by the ACA, MCA, and posterior cerebral artery
(PCA). Anterior watershed infarcts (at the ACA–MCA
junction) result in a contralateral hemiparesis (mainly
leg) and expressive language or behavioral changes. Posterior watershed infarcts (MCA–PCA interface) result in
visual deficits and language problems.
Anterior Choroidal Artery Syndrome: Occlusion of this vessel may result from small emboli or small
vessel disease. This syndrome may also occur as a complication of temporal lobectomy (removal of the temporal lobe to treat intractable epilepsy). The infarcted
area usually includes the optic tract, lower portions of
the basal nuclei, and lower aspects of the internal capsule. The patient experiences a contralateral hemiplegia, hemihypethesia, and homonymous hemianopsia. These
deficits are due to, respectively, involvement of corticospinal fibers in the posterior limb of the internal capsule or possibly in the crus cerebri, involvement of
thalamocortical fibers in the posterior limb of the internal capsule, and involvement of the fibers of the optic tract.
Parkinson Disease: Parkinson disease (paralysis
agitans) results from a loss of the dopamine-containing
cells in the substantia nigra. Although this part of the
brain is located in the midbrain, the terminals of these
nigrostriatal fibers are in the putamen and the caudate
nucleus. The classic signs and symptoms of this disease
are a stooped posture, resting tremor, rigidity, shuffling or festinating gait, and difficulty initiating or maintaining
movement (akinesia, hypokinesia, or bradykinesia). Initially, the tremor and walking difficulty may appear on
Vascular Syndromes or Lesions of the Forebrain
APS
BCorCl
CC
CM
DMNu
GP
HyTh
PulNu
Put
SplCorCl
VA
VL
Anterior perforated substance
Body of corpus callosum
Crus cerebri
Centromedian nucleus of thalamus
Dorsomedial nucleus of thalamus
Globus pallidus
Hypothalamus
Pulvinar nuclear complex
Putamen
Splenium of the corpus callosum
Ventral anterior nucleus of thalamus
Ventral lateral nucleus of thalamus
Abbreviations
5-38 Semidiagrammatic representation of the internal distribution of arteries to the diencephalon, basal ganglia, and internal capsule. Selected structures are labeled on the left side of
each section; the general pattern of arterial distribution overlies
these structures on the right side. The general distribution patterns of arteries in the forebrain as shown here may vary from
patient to patient. For example, the adjacent territories served
by neighboring vessels may overlap to varying degrees at their
margins or the territory of a particular vessel may be larger or
smaller than seen in the general pattern.
one side of the body, but these signs usually spread to both sides with
time. This is a neurodegenerative disease that has, in its later stages,
a dementia component.
Transient Ischemic Attack: A transient ischemic attack, commonly called TIA, is a temporary (and frequently focal) neurologic
deficit that usually resolves within 10 to 30 minutes from the onset
of symptoms. The cause is temporary occlusion of a vessel or inadequate perfusion of a restricted vascular territory. TIA that last 60+
minutes may result in some permanent deficits. This vascular event
may take place anywhere in the central nervous system but is more
common in the cerebral hemisphere.
158 Internal Morphology of the Spinal Cord and Brain in Stained Sections
Medial geniculate
nucleus
Lateral geniculate
nucleus
Retrolenticular
limb of
internal capsule
Crus of fornix
PulNu
Red nucleus
External capsule
Anterior nucleus of thalalmus
Pineal
Put
CC
CM
VL
DM
Nu
Optic tract
Put
P
Hippocampal formation
Tail of caudate nucleus
G
DMNu
VA-VL
Put
APS
GP
Optic tract
BCorCl
Anteromedial branches of anterior cerebral artery and anterior
communicating artery
Medial striate branch of anterior cerebral artery (branch of A2)
Anterolateral branches of middle and anterior cerebral artery
Posteromedial branches of posterior cerebral artery (P1 segment) and
branches of posterior communicating artery
Thalamoperforating branches of posterior cerebral artery (branch of P1)
Lateral striate branches (lenticulostriate arteries) of the middle
cerebral artery
Anterior choroidal artery
Thalamogeniculate branches of posterior cerebral artery (branch of P2)
Amygdaloid
nuclear
complex
Septum pellucidum
Hippocampal formation
HyTh
Medial posterior choroidal artery
Subthalamic nucleus
Hypothalamus
Mammillary body
Insula
Claustrum
Anterior limb of internal capsule
Substantia nigra
Posterior limb of internal capsule
SplCorCl
Lateral dorsal nucleus
Stria terminalis
Caudal
Body of fornix
Body of caudate nucleus
Rostral
Head of caudate nucleus
Column of fornix
Arterial Patterns Within the Forebrain With Vascular Syndromes 159
CHAPTER
6
Internal Morphology of the Brain in Stained
Sections: Axial–Sagittal Correlations with MRI
Although the general organization of Chapter 6 has been described in Chapter 1 (the reader may wish to refer back to this
section), it is appropriate to reiterate its unique features at this
point. Each set of facing pages has photographs of an axial stained
section (left-hand page) and a sagittal stained section (right-hand
page). In addition to individually labeled structures, a heavy line
appears on each photograph. This prominent line on the axial
section represents the approximate plane of the sagittal section
located on the facing page. On the sagittal section this line signifies the approximate plane of the corresponding axial section.
The reader can identify features in each photograph and then, using this line as a reference point, visualize structures that are located either above or below that plane (axial to sagittal comparison) or medial or lateral to that plane (sagittal to axial
comparison). This method of presentation provides a format for
reconstructing and understanding three-dimensional relationships within the central nervous system.
The magnetic resonance image (MRI) placed on every page
in this chapter gives the reader an opportunity to compare internal brain anatomy, as seen in stained sections, with those
structures as visualized in clinical images generated in the same
plane. Even a general comparison reveals that many features, as
seen in the stained section, can be readily identified in the adjacent MRI.
This chapter is also organized so that one can view structures in
either the axial or the sagittal plane only. Axial photographs appear
on left-hand pages and are sequenced from dorsal to ventral (oddnumbered Figures 6-1 through 6-9), while sagittal photographs are
on the right-hand pages and progress from medial to lateral (evennumbered Figures 6-2 through 6-10). Consequently, the user can
identify and follow structures through an axial series by simply flipping through the left-hand pages or through a sagittal series by flipping through the right-hand pages. The inherent flexibility in this
chapter should prove useful in a wide variety of instructional/
learning situations. The drawings shown in the following illustrate
the axial and sagittal planes of the photographs in this chapter.
Fig. 6-6
Fig. 6-4
Fig. 6-8
Fig. 6-10
Fig. 6-2
Axial Planes
Fig. 6-1
Fig. 6-3
Fig. 6-5
Fig. 6-7
Fig. 6-9
Sagittal
Planes
162 Internal Morphology of the Brain in Stained Sections
CorCl
Anterior horn of
lateral ventricle
Sep
CaNu,H
Put
For
VA
AntNu
IntCap:
AL
G
Pl
VL
DMNu
CM
Cl
VPL
Hab
PulNu
StTer
CaNu,T
Hip
Com
CP
Hip,F
Hip
OpRad
Atrium of
lateral ventricle
6-1 Axial section through the head of the caudate nucleus and several
key thalamic nuclei (anterior, centromedian, pulvinar, habenular). The
heavy line represents the approximate plane of the sagittal section
shown in Figure 6-2 (facing page). Many of the structures labeled in
this photograph can be clearly identified in the adjacent T1-weighted
MRI.
Abbreviations
AntNu
CaNu,H
CaNu,T
CI
CM
CorCI
CP
DMNu
For
Hab
Hip
Hip,F
Anterior nucleus of thalamus
Caudate nucleus, head
Caudate nucleus, tail
Claustrum
Centromedial nucleus of thalamus
Corpus callosum
Choroid plexus
Dorsomedial nucleus of thalamus
Fornix, column
Habenular nucleus
Hippocampal formation
Hippocampus, fimbria
HipCom
IntCap,AL
IntCap,G
IntCap,PL
OpRad
PulNu
Put
Sep
StTer
VA
VL
VPL
Hippocampal commissure
Internal capsule, anterior limb
Internal capsule, genu
Internal capsule, posterior limb
Optic radiations
Pulvinar nuclear complex
Putamen
Septum pellucidum
Stria terminalis
Ventral anterior nucleus of thalamus
Ventral lateral nucleus of thalamus
Ventral posterolateral nucleus
Axial–Sagittal Correlations 163
For,B
AntNu
LDNu
CorCl,Spl
CorCl,G
DMNu
SMT
PrTecNu
Hab
SC
AC
PoCom
IC
RNu
For,Col
HyTh
TroNr
MtTr
MLF
MB
OcNr
FNu
OpNr
SCP,Dec
AbdNu
BP
ML
Py
NuGr
PO
HyNu
LCSp
6-2 Sagittal section through the column of the fornix, anterior thalamic
nucleus, red nucleus, and medial portions of the pons (abducens nucleus),
cerebellum (fastigial nucleus), and medulla (nucleus gracilis). The heavy line
represents the approximate plane of the axial section shown in Figure
6-1 (facing page). Many of the structures labeled in this photograph can
be clearly identified in the adjacent T1-weighted MRI.
Abbreviations
AbdNu
AC
AntNu
BP
CorCI,G
CorCI,Spl
DMNu
FNu
For,B
For,Col
Hab
HyNu
HyTh
IC
LCsp
LDNu
Abducens nucleus
Anterior commissure
Anterior nucleus of thalamus
Basilar pons
Corpus callosum, genu
Corpus callosum, splenium
Dorsomedial nucleus of thalamus
Fastigial nucleus (medial cerebellar nucleus)
Fornix, body
Fornix, column
Habenular nuclei
Hypoglossal nucleus
Hypothalamus
Inferior colliculus
Lateral corticospinal tract
Lateral dorsal nucleus
MB
ML
MLF
MtTr
NuGr
OcNr
OpNr
PO
PoCom
PrTecNu
Py
RNu
SC
SCP,Dec
SMT
TroNr
Mammillary body
Medial lemniscus
Medial longitudinal fasciculus
Mammillothalamic tract
Nucleus gracilis
Oculomotor nerve
Optic nerve
Principal olivary nucleus
Posterior commissure
Pretectal nuclei
Pyramid
Red nucleus
Superior colliculus
Superior cerebellar peduncle, decussation
Stria medullaris thalami
Trochlear nerve
164 Internal Morphology of the Brain in Stained Sections
Anterior horn of
lateral ventricle
Sep
CaNu,H
For,Col
GPL
Ins
Put
VA
MtTr
IntCap,AL
Cl
DMNu
ExtCap
VL
InTCap,PL
CM
VPM
VPL
HabCom
MGNu
SC,Br
SC
StTer
PulNu
CaNu, T
OpRad
Hip
Tap
6-3 Axial section through the head of the caudate nucleus, centromedian nucleus, medial geniculate body, and superior colliculus. The heavy line
represents the approximate plane of the sagittal section shown in Fig-
ure 6-4 (facing page). Many of the structures labeled in this photograph
can be clearly identified in the adjacent T2-weighted MRI.
Abbreviations
CaNu,H Caudate nucleus, head
CaNu,T Caudate nucleus, tail
Cl Claustrum
CM Centromedian nucleus of thalamus
DMNu Dorsomedial nucleus of thalamus
ExtCap External capsule
For,Col Fornix, column
Sep Septum pellucidum
GPL Globus pallidus, lateral segment
Hab,Com Habenular commissure
Hip Hippocampal formation
Ins Insula
IntCap,AL Internal capsule, anterior limb
IntCap,PL Internal capsule, posterior limb
MGNu
MtTr
OpRad
PulNu
Put
SC
SC,Br
StTer
VA
VL
VPL
VPM
Tap
Medial geniculate nucleus
Mammillothalamic tract
Optic radiations
Pulvinar nuclear complex
Putamen
Superior colliculus
Superior colliculus, brachium
Stria terminalis
Ventral anterior nucleus of thalamus
Ventral lateral nucleus of thalamus
Ventral posterolateral nucleus of thalamus
Ventral posteromedial nucleus of thalamus
Tapetum
Axial–Sagittal Correlations 165
LatVen,AH
MtTr
CorCl,B
LDNu
For,B
CorCl, Spl
AntNu
DMNu CM
VA
AC
PulNu
SC
H
ThFas
RNu
Hyth
LenFas
AnLen
IC
SN
OpTr
SCP
CC
OlfTr
ML
ForVen
BP
NuGr
FacNu
PO
SolNu & Tr
NuCu
6-4 Sagittal section through anterior and ventral anterior thalamic nuclei, red nucleus and central areas of the pons, cerebellum (and superior peduncle), and medulla (solitary nuclei and tract). Note the position of the
facial motor nucleus at the pons-medulla junction. The heavy line repre-
sents the approximate plane of the axial section shown in Figure 6-3
(facing page). Many of the structures labeled in this photograph can be
clearly identified in the adjacent T1-weighted MRI.
Abbreviations
AC
AnLen
AntNu
BP
CC
CM
CorCl,B
CorCl, Spl
DMNu
FacNu
For,B
ForVen
H
HyTh
IC
LatVen,AH
LenFas
Anterior commissure
Ansa lenticularis
Anterior nucleus of thalamus
Basilar pons
Crus cerebri
Centromedian nucleus
Corpus callosum, body
Corpus callosum, splenium
Dorsomedial nucleus of thalamus
Facial nucleus
Fornix, body
Fourth ventricle
Prerubral field
Hypothalamus
Inferior colliculus
Lateral ventricle, anterior horn
Lenticular fasciculus
LDNu
ML
MtTr
NuCu
NuGr
OlfTr
OpTr
PO
PulNu
RNu
SC
SCP
SN
SolNu&Tr
ThFas
VA
Lateral dorsal nucleus
Medial lemniscus
Mammillothalamic tract
Nucleus cuneatus
Nucleus gracilis
Olfactory tract
Optic tract
Principal olivary nucleus
Pulvinar nuclear complex
Red nucleus
Superior colliculus
Superior cerebellar peduncle (brachium
conjunctivum)
Substantia nigra
Solitary nuclei and tract
Thalamic fasciculus
Ventral anterior nucleus of thalamus
166 Internal Morphology of the Brain in Stained Sections
AC
CaNu,H
LT
GPL
Put
GPM
For,Col
Hyth
MtTr
IntCap,AL
Cl
VL
VPM
SC
CM
IntCap:
VPL
PL
RL
MGNu
CeGy
SC
PulNu
Hip,F
OpRad
Hip
CP
ALV
6-5 Axial section through the head of the caudate nucleus, ventral posteromedial nucleus, medial geniculate body, and ventral parts of the pulvinar.
The heavy line represents the approximate plane of the sagittal section
shown in Figure 6-6 (facing page). Many of the structures labeled in
this photograph can be clearly identified in the adjacent T1-weighted
MRI.
Abbreviations
AC
ALV
CaNu,H
CeGy
CI
CM
CP
For,Col
GPL
GPM
Hip
Hip,F
HyTh
Anterior commissure
Atrium of lateral ventricle
Caudate nucleus, head
Central gray (periaqueductal gray)
Claustrum
Centromedian nucleus of thalamus
Choroid plexus
Fornix, column
Globus pallidus, lateral segment
Globus pallidus, medial segment
Hippocampal formation
Hippocampus, fimbria
Hypothalamus
IntCap,AL
IntCap,Pl
IntCap,RL
LT
MGNu
MtTr
OpRad
PulNu
Put
SC
VL
VPL
VPM
Internal capsule, anterior limb
Internal capsule, posterior limb
Internal capsule, retrolenticular limb
Lamina terminalis
Medial geniculate nucleus
Mammillothalamic tract
Optic radiations
Pulvinar nuclear complex
Putamen
Superior colliculus
Ventral lateral nucleus of thalamus
Ventral posterolateral nucleus of thalamus
Ventral posteromedial nucleus of thalamus
Axial–Sagittal Correlations 167
CorCl,G
LDNu
CorCl,Spl
DMNu
PulNu
VL
VA
CaNu,H
CM
SC
H
RNu
AC
IC
LL
SN
SCP
ENu
OpTr
AnLen
CC
SOpNu
LenFas
CSNu
ML
TriMoNu
FacNr
OCblF
NuCu
6-6 Sagittal section through central regions of the diencephalon (centromedian nucleus) and midbrain (red nucleus), and through lateral areas of
the pons (trigeminal motor nucleus) and medulla (nucleus cuneatus). The
heavy line represents the approximate plane of the axial section shown
in Figure 6-5 (facing page). Many of the structures labeled in this photograph can be clearly identified in the adjacent T1-weighted MRI.
Abbreviations
AC
AnLen
CaNu,H
CC
CM
CorCl,G
CorCl,Spl
CSNu
DMNU
ENu
FacNr
H
IC
LenFas
LDNu
Anterior commissure
Ansa lenticularis
Caudate nucleus, head
Crus cerebri
Centromedian nucleus of thalamus
Corpus callosum, genu
Corpus callosum, splenium
Chief (prinicipal) sensory nucleus of trigeminal nerve
Dorsomedial nucleus of thalamus
Emboliform nucleus (anterior interposed cerebellar nucleus)
Facial nerve
Field of Forel (prerubral field)
Inferior colliculus
Lenticular fasciculus
Lateral dorsal nucleus of thalamus
LL
ML
NuCu
OCblF
OpTr
PulNu
RNu
SC
SCP
SN
SOpNu
TriMoNu
VA
VL
Lateral lemniscus
Medial lemniscus
Nucleus cuneatus
Olivocerebellar fibers
Optic tract
Pulvinar nuclear complex
Red nucleus
Superior colliculus
Superior cerebellar peduncle (brachium conjunctivum)
Substantia nigra
Supraoptic nucleus
Trigeminal motor nucleus
Ventral anterior nucleus of thalamus
Ventral lateral nucleus of thalamus
168 Internal Morphology of the Brain in Stained Sections
CaNu
Ins
LT
HyTh
For
MtTr
AC
RNu
OpTr
CC
LGNu
ML
StTer
IC,Br
CaNu,T
MGNu
Hip
IC
OpRad
6-7 Axial section through the hypothalamus, red nucleus, inferior colliculus, and lateral geniculate body. The heavy line represents the approximate plane of the sagittal section shown in Figure 6-8 (facing
page). The axial plane through the hemisphere, when continued into
the midbrain, represents a slightly oblique section through the mesencephalon. The position of the lamina terminalis is indicated by the double-dashed lines. Many of the structures labeled in this photograph can
be clearly identified in the adjacent T1-weighted MRI.
Abbreviations
AC
CaNu
CaNu,T
CC
For
Hip
HyTh
IC
IC,Br
Ins
Anterior commissure
Caudate nucleus
Caudate nucleus, tail
Crus cerebri
Fornix
Hippocampal formation
Hypothalamus
Inferior colliculus
Inferior colliculus, brachium
Insula
LGNu
LT
MGNu
ML
MtTr
OpRad
OpTr
RNu
StTer
Lateral geniculate nucleus
Lamina terminalis
Medial geniculate nucleus
Medial lemniscus
Mammillothalamic tract
Optic radiation (geniculocalcarine fibers)
Optic tract
Red nucleus
Stria terminalis
Axial–Sagittal Correlations 169
AC
Zl
ThFas
Lenfas
VPM
CaNu
BrSC
VL
PulNu
GPL
SThNu
GPM
MGNu
Put
SN
OpTr
Hip
AmyNu
CC
DNu
MCP
PCNu
6-8 Sagittal section through the caudate nucleus, central parts of the
diencephalon (ventral posteromedial nucleus), and lateral portions of the
pons and cerebellum (dentate nucleus). The heavy line represents the ap-
proximate plane of the axial section shown in Figure 6-7 (facing page).
Many of the structures labeled in this photograph can be clearly identified in the adjacent T1-weighted MRI.
Abbreviations
AC
AmyNu
BrSC
CaNu
CC
DNu
GPL
GPM
Hip
LenFas
MCP
Anterior commissure
Amygdaloid nucleus (complex)
Brachium of superior colliculus
Caudate nucleus
Crus cerebri
Dentate nucleus (lateral cerebellar nucleus)
Globus pallidus, lateral segment
Globus pallidus, medial segment
Hippocampal formation
Lenticular fasciculus
Middle cerebellar peduncle (brachium pontis)
MGNu
OpTr
PCNu
PulNu
Put
SN
SThNu
ThFas
VL
VPM
ZI
Medial geniculate nucleus
Optic tract
Posterior cochlear nucleus
Pulvinar nuclear complex
Putamen
Substantia nigra
Subthalamic nucleus
Thalamic fasciculus
Ventral lateral nucleus of thalamus
Ventral posteromedial nucleus of thalamus
Zona incerta
170 Internal Morphology of the Brain in Stained Sections
LT
SOR
OpTr
IR
HyTh
AmyNu
MB
CC
CP
SN
Hip
ML
CaNu,T
SCP,Dec
FHip
LatVen,IH
MLF
DenGy
LL
SCP
OpRad
6-9 Axial section through ventral portions of the hypothalamus
(supraoptic recess and mammillary body) and forebrain (amygdaloid nucleus), and through the superior cerebellar peduncle decussation in the midbrain. The heavy line represents the approximate plane of the sagittal
section shown in Figure 6-10 (facing page). The axial plane through the
hemisphere, when continued into the midbrain, represents a slightly
oblique section through the mesencephalon. Many of the structures labeled in this photograph can be clearly identified in the adjacent T1weighted MRI.
Abbreviations
AmyNu
CaNu,T
CC
CP
DenGy
FHip
Hip
HyTh
IR
LatVen,lH
LL
Amygdaloid nucleus (complex)
Caudate nucleus, tail
Crus cerebri
Choroid plexus
Dentate gyrus
Fimbria of hippocampus
Hippocampal formation
Hypothalamus
Infundibular recess of third ventricle
Lateral ventricle, inferior (temporal) horn
Lateral lemniscus
LT
MB
ML
MLF
OpRad
OpTr
SCP
SCP,Dec
SN
SOR
Lamina terminalis
Mammillary body
Medial lemniscus
Medial longitudinal fasciculus
Optic radiations
Optic tract
Superior cerebellar peduncle (brachium
conjunctivum)
Superior cerebellar peduncle, decussation
Sustantia nigra
Supraoptic recess of third ventricle
Axial–Sagittal Correlations 171
VL + VPL
CaNu,B
EML + ThRetNu
OpTr
AC
OpRad
PulNu
CalSul
ALV
Hip
GPL
LGNu
Put
GPM
FHip
CP
DenGy
Hip
DNu
AmyNu
LatVen,IH
6-10 Sagittal section through the putamen, amygdaloid nucleus, and
hippocampus and through the most lateral portions of the diencephalon
(external medullary lamina and ventral posterolateral nucleus). The heavy
line represents the approximate plane of the axial section shown in Figure 6-9 (facing page). Many of the structures labeled in this photograph
can be clearly identified in the adjacent T1-weighted MRI.
Abbreviations
AC
ALV
AmyNu
CalSul
CaNu,B
CP
DenGy
DNu
EML
FHip
GPL
Anterior commissure
Atrium of lateral ventricle
Amygdaloid nucleus (complex)
Calcarine sulcus
Caudate nucleus, body
Choroid plexus
Dentate gyrus
Dentate nucleus
External medullary lamina
Fimbria of hippocampus
Globus pallidus, lateral segment
GPM
Hip
LatVen,lH
LGNu
OpRad
OpTr
PulNu
Put
ThRetNu
VL
VPL
Globus pallidus, medial segment
Hippocampal formation
Lateral ventricle, inferior (temporal) horn
Lateral geniculate nucleus
Optic radiations
Optic tract
Pulvinar nuclear complex
Putamen
Thalamic reticular nuclei
Ventral lateral nucleus of thalamus
Ventral posterolateral nucleus of thalamus
CHAPTER
7
Synopsis of Functional Components,
Tracts, Pathways, and Systems
The study of regional neurobiology (brain structures in gross specimens, in brain slices, in stained sections, and in MRI and CT) is
the basis for the study of systems neurobiology (tracts, pathways,
cranial nerves and their functions), which, in turn, is the basis for
understanding and diagnosing the neurologically impaired patient. Building on the concepts learned in earlier chapters on external and internal brain anatomy in specimens and in MRI and
CT, on brain vascular patterns, and on the relationships of cranial nerves with long tracts, this chapter explores systems neurobiology with a particular emphasis on clinical correlations.
The format of each set of facing pages is designed to summarize, accurately and consisely, the relationships of a given tract
or pathway. This includes, but is not limited to, 1) the location
of the cells of origin for a given tract/pathway, 2) its entire
course throughout the neuraxis and cerebrum, 3) the location of
the decussation of these fibers, if applicable, 4) the neurotransmitters associated with the neurons comprising the tract/pathway,
5) a brief review of its blood supply, and 6) a summary of a number of deficits seen as a result of lesions at various points in the
tract/pathway. The structure of an atlas does not allow a detailed
definition of each clinical term on the printed page. However,
the full definition of each clinical term or phrase is available on
the CD that comes with this atlas; these are taken from the current edition of Stedman’s Medical Dictionary. In this respect, the
full definitions are actually available in this book. Researching the
full definition of a clinical term or phrase is a powerful and effective learning tool. Also, each clinical term or phrase is available in any standard medical dictionary or comprehensive neurology text.
The layout of the drawings in this chapter clearly shows the
laterality of the tract/pathway. That is, the relationship between
the location of the cell of origin and the termination of the fibers
making up a tract/pathway or the projections of cranial nerve
nuclei. This information is absolutely essential to understanding the position of a lesion and correlating this fact with the deficits seen in the
neurologically compromised patient. For example, is the deficit on
the same side as the lesion (ipsilateral), on the opposite side (contralateral), or on both sides (bilateral)? The concept of laterality
is usually expressed as “right,” “left,” or “bilateral” in reference
to the side of the deficit(s) when written on the patient’s chart.
This chapter is designed to maximize the correlation between
structure and function, to provide a range of clinical examples
for each tract/pathway, and to help the user develop a knowledge base that can be easily integrated into the clinical setting.
174 Synopsis of Functional Components, Tracts, Pathways, and Systems
SSA
SL
G
S
E
SL
SVA
GVA
G
V
E
G
S
A
S
V
E
G
S
E
G
V
E
S
V
E
SG
VV
AA
S
S
A
G
S
A
posterior
GSA
SL
GVA
GVE
medial
lateral
GSE
anterior
GSA
G
S
E
GVA
GVE
G
V
E
G
V
A
G
S
A
GSE
SL
7–1 A semidiagrammatic summary of the positions of functional
components as seen in the developing neural tube (left) and in the
spinal cord and brainstem of the adult (right). In the neural tube, the
alar plate and its associated GSA and GVA components are posterior
(dorsal) to the sulcus limitans (SL) while the basal plate and its related
GVE and GSE components are anterior (ventral) to the SL. In the adult
spinal cord, this general posterior/anterior relationship is maintained,
although the neural canal (as central canal) is reduced and/or absent.
Two major changes occur in the transition from spinal cord to brainstem in the adult. First, as the central canal of the cervical cord enlarges
into the fourth ventricle and the cerebellum develops, the posterior
portion of the neural tube is rotated laterally. Consequently, in the
adult, the sulcus limitans is present in the brainstem with motor components (adult derivatives of the basal plate) medial to it, and sensory
components (adult derivatives of the alar plate) are located laterally.
Second, in the brainstem, special functional components (SVE to muscles of pharyngeal arch origin; SVA taste and olfaction; SSA vestibular,
auditory, and visual systems) are intermingled with the rostral continuation of the general functional components as found in the spinal cord.
In the brainstem, however, there is a slight transposition of the SVE
and GSA functional components. Embryologically, SVE cell groups appear between those associated with GSE and GVE components. As development progresses, however, SVE cell groups migrate (open arrow) to anterolateral areas of the tegmentum. Cell groups associated
with the GSA functional component are displaced from their posterolateral position in the developing brainstem by the newly acquired cell
groups having SSA components (as well as other structures). Consequently, structures associated with the GSA component are located
(open arrow) in more anterolateral and lateral areas of the brainstem.
The approximate border between motor and sensory regions of the
brainstem is represented by an oblique line drawn through the brainstem beginning at the SL. The medial (from midline) to lateral positions of the various functional components, as shown on the far right
of this figure, are taken from their representative diagrams of brainstem and cord and are directly translatable to Figure 7–2 (facing page).
The color-coding of the components on this figure correlate with that
in Figure 7–2 on the facing page.
Abbreviations
GSA
GSE
GVA
GVE
General somatic afferent
General somatic efferent
General visceral afferent
General visceral efferent
SSA
SVA
SVE
SL
Special somatic afferent
Special visceral afferent
Special visceral efferent
Sulcus limitans
Components of Cranial and Spinal Nerves 175
Cranial nerves
G
S
E
G
V
E
S
V
E
S G
V V
A A
S
S
A
G
S
A
Midbrain
2
1. Oculomotor nuc. (GSE)
2. Edinger-Westphal nuc. (GVE)
3. Trochlear nuc. (GSE)
4. Mesencephalic nuc. & tr.
of V (GSA)
1
4
3
Pons
SL
7
5. Abducens nuc. (GSE)
6. Sup. salivatory nuc. (GVE)
7. Motor trigeminal nuc. (SVE)
8. Motor facial nuc. (SVE)
9. Principle sensory nuc of V (GSA)
10. Spinal trigeminal nuc. (GSA)
(pars oralis)
9
10
5
S
8
6
L
a
13
17
M
SP
14
15
11
18
12
b
SL
16
Medulla oblongata
11. Hypoglossal nuc. (GSE)
12. Dorsal motor nuc. of vagus (GVE)
13. Inf. salivatory nuc. (GVE)
14. Nuc. ambiguus (SVE)
15. Solitary nuc. and tr.
15a: gustatory nuc. (SVA)
15b: cardiorespiratory nuc (GVA)
16. Vestibular nuclei (SSA)
S = Sup; L = Lat; M = Med; Sp. = Spinal
17. Cochlear nuc. (SSA)
18. Spinal trigeminal nuc. (GSA)
(pars interpolaris, pars caudalis)
20
SL
Cervical
cord
24
21
19
Thoracic
cord
22
23
Spinal cord
19. Medial motor cell column (GSE)
20. Accessory nuc. (GSE)
21. Lateral motor cell columns (GSE)
22. Intermediolateral cell column (GVE)
23. Visceral afferent receptive areas (GVA)
24. Substantia gelatinosa, nucleus proprious
and associated GSA receptive areas
25. Sacral parasympathetics (GVE)
21
Lumbosacral
cord
25
G
S
E
G
V
E
G
V
A
G
S
A
Spinal nerves
7–2 The medial to lateral positions of brainstem cranial nerve
and spinal cord nuclei as shown here are the same as in Figure 7–1.
This diagrammatic posterior (dorsal) view shows 1) the relative positions and names of specific cell groups and their associated functional components, 2) the approximate location of particular nuclei
in their specific division of brainstem and/or spinal cord, and 3) the
rostrocaudal continuity of cell columns (either as continuous or dis-
continuous cell groups) from one division of the brainstem to the
next or from brainstem to spinal cord. The nucleus ambiguus is a
column of cells composed of distinct cell clusters interspersed with
more diffusely arranged cells, much like a string of beads. Nuclei associated with cranial nerves I (olfaction, SVA) and II (optic, SSA) are
not shown. The color-coding used on this figure correlates with that
on Figure 7–1 (facing page).
176 Synopsis of Functional Components, Tracts, Pathways, and Systems
7–3 Orientation drawing for pathways. The trajectory of most
pathways illustrated in Chapter 7 appears on individualized versions of
this representation of the central nervous system (CNS). Although
slight changes are made in each drawing, so as to more clearly diagram
a specific pathway, the basic configuration of the CNS is as represented
here. This allows the user to move from pathway to pathway without
being required to learn a different representation or drawing for each
pathway; also, laterality of the pathway, a feature essential to diagnosis (see introduction), is inherently evident in each illustration.
The forebrain (telencephalon and diencephalon) is shown in the
coronal plane, and the midbrain, pons, medulla, and spinal cord are
represented through their longitudinal axes. The internal capsule is
represented in the axial plane in an effort to show the rostrocaudal distribution of fibers located therein.
The reader should become familiar with the structures and regions
as shown here because their locations and relationships are easily transferable to subsequent illustrations. It may also be helpful to refer back
to this illustration when using subsequent sections of this chapter.
Neurotransmitters: Three important facts are self-evident in the
descriptions of neurotransmitters that accompany each pathway drawing. These are illustrated by noting, as an example, that glutamate is
found in corticospinal fibers (see Figure 7–10). First, the location of neuronal cell bodies containing a specific transmitter is indicated (glutamatecontaining cell bodies are found in cortical areas that project to the spinal
cord). Second, the trajectory of fibers containing a particular neurotransmitter is obvious from the route taken by the tract (glutaminergic corti-
cospinal fibers are found in the internal capsule, crus cerebri, basilar
pons, pyramid, and lateral corticospinal tract). Third, the location of terminals containing specific neurotransmitters is indicated by the site(s) of
termination of each tract (glutaminergic terminals of corticospinal fibers
are located in the spinal cord gray matter). In addition, the action of most
neuroactive substances is indicated as excitatory (⫹) or inhibitory (⫺).
This level of neurotransmitter information, as explained here for glutaminergic corticospinal fibers, is repeated for each pathway drawing.
Clinical Correlations: The clinical correlations are designed to
give the user an overview of specific deficits (i.e., hemiplegia, athetosis)
seen in lesions of each pathway and to provide examples of some syndromes or diseases (i.e., Brown-Sequard syndrome, Wilson disease) in
which these deficits are seen. Although purposefully brief, these correlations highlight examples of deficits for each pathway and provide a
built-in mechanism for expanded study. For example, the words in
italics in each correlation are clinical terms and phrases that are defined
on the CD (from Stedman’s) included with this atlas or can be found
in standard medical dictionaries and clinical neuroscience textbooks.
Consulting these sources, especially the CD available in this atlas, will
significantly enhance understanding of the deficits seen in the neurologically compromised patient. Expanded information, based on the
deficits mentioned in this chapter, is integrated into some of the questions for chapter 7. Referring to such sources will allow the user to
glean important clinical points that correlate with the pathway under
consideration, and enlarge his or her knowledge and understanding by
researching the italicized words and phrases.
Abbreviations
CE
Cer
CinSul
CaNu
CM
CorCI
Dien
DMNu
For
GP
GPl
GPm
HyTh
IC
IntCap,AL
IntCap,G
Cervical enlargement of spinal cord
Cervical levels of spinal cord
Cingulate sulcus
Caudate nucleus (⫹ Put ⫽ neostriatum)
Centromedian (and intralaminar) nuclei
Corpus callosum
Diencephalon
Dorsomedial nucleus of thalamus
Fornix
Globus pallidus (paleostriatum)
Globus pallidus, lateral segment
Globus pallidus, medial segment
Hypothalamic area
Internal capsule
Internal capsule, anterior limb
Internal capsule, genu
IntCap,PL
LatSul
LatVen
LSE
LumSac
L-VTh
Mes
Met
Myelen
Put
SThNu
Telen
Thor
VPL
VPM
Internal capsule, posterior limb
Lateral sulcus (Sylvian sulcus)
Lateral ventricle
Lumbosacral enlargement of spinal cord
Lumbosacral level of spinal cord
Lateral and ventral thalamic nuclei
excluding VPM and VPL
Mesencephalon
Metencephalon
Myelencephalon
Putamen (⫹ CaNu ⫽ neostriatum)
Subthalamic nucleus
Telencephalon
Thoracic levels of spinal cord
Ventral posterolateral nucleus of thalamus
Ventral posteromedial nucleus of thalamus
Orientation 177
Midline
Cerebral cortex
CinSul
LatVen
Basal ganglia
CorCl
CaNu
For
Telen
&
Dien
DMNu
L-VTh
LatSul
CM
IC
IC
GPl
VPL
Put
GPm
VPM
Internal Capsule
IntCap, AL
CaNu
HyTh
SThNu
Midbrain
Rostral
Mes
Put
GP
IntCap, PL
IntCap, G
Dien
Met
Pons & Cerebellum
Caudal
Myelen
Medulla
Cer
CE
Spinal Cord
Thor
LSE
LumSac
Midline
178 Synopsis of Functional Components, Tracts, Pathways, and Systems
Posterior (Dorsal) Column-Medial Lemniscus System
7–4 The origin, course, and distribution of fibers composing the
posterior (dorsal) column (PC)-medial lemniscus (ML) system. This illustration shows the longitudinal extent, the positions in representative cross-sections of brainstem and spinal cord, and the somatotopy of
fibers in the PC and ML. The ML undergoes positional changes as it
courses from the myelencephalon (medulla) rostrally toward the mesencephalic-diencephalic junction. In the medulla, ML and ALS fibers
are widely separated and receive different blood supplies, whereas in
the midbrain, they are served by a common arterial source. As the ML
makes positional changes, the somatotopy therein follows accordingly.
Fibers of the postsynaptic posterior column system (shown in green)
are considered in detail in Figure 7–6 on page 182.
Neurotransmitters: Acetylcholine and the excitatory amino
acids glutamate and aspartate are associated with some of the largediameter, heavily myelinated fibers of the posterior horn and posterior
columns.
Clinical Correlations: Damage to posterior column fibers on one
side of the spinal cord (as in the Brown-Sequard syndrome) results in an ipsilateral loss of vibratory sensation, position sense, and discriminative
touch (astereognosis, stereoagnosis) below the level of the lesion. The term
stereoanesthesia is frequently used to specify a lesion of peripheral nerves
that results in an inability to perceive proprioceptive and tactile sensations. The term tactile agnosia is sometimes considered to be synonymous with these preceding three terms. However, tactile agnosia is also
used to describe deficits seen in lesions of the parietal cortex. Bilateral
damage (as in tabes dorsalis or subacute combined degeneration of the spinal
cord) produces bilateral losses. Although ataxia is the most common feature in patients with tabes dorsalis, they also have a loss of deep tendon
reflexes, severe lancinating pain over the body below the head (more common in the lower extremity), and bladder dysfunction. The ataxia that
may be seen in patients with posterior column lesions (sensory ataxia) is
due to a lack of proprioceptive input and position sense. These individuals tend to forcibly place their feet to the floor in an attempt to stimulate such sensory input. A patient with mild ataxia due to posterior column disease may compensate for the motor deficit by using visual cues.
Patients with subacute combined degeneration (SCD) of the spinal cord
first have signs and symptoms of posterior column involvement, followed later by signs of corticospinal tract damage (spastic weakness of legs,
increased deep tendon reflexes, Babinski sign).
Rostral to the sensory decussation, medial lemniscus lesions result
in contralateral losses that include the entire body excluding the head.
Brainstem lesions involving medial lemniscus fibers usually include adjacent structures, result in motor and additional sensory losses, and
may reflect the distribution patterns of vessels (as in medial medullary or
medial pontine syndromes). Large lesions in the forebrain may result in a
complete contralateral loss of modalities carried in the posterior
columns and anterolateral systems, or may produce pain (as in the thalamic syndrome).
Abbreviations
ALS
BP
CC
CTT
FCu
FGr
IAF
IC
ML
MLF
NuCu
Anterolateral system
Basilar pons
Crus cerebri
Central tegmental tract
Cuneate fasciculus
Gracile fasciculus
Internal arcuate fibers
Internal capsule
Medial lemniscus
Medial longitudinal fasciculus
Cuneate nucleus
NuGr
PC
PO
PoCGy
PPGy
PRG
Py
RB
RNu
SN
VPL
Gracile nucleus
Posterior column
Principal olivary nucleus
Postcentral gyrus
Posterior paracentral gyrus
Posterior (dorsal) root ganglia
Pyramid
Restiform body
Red nucleus
Substantia nigra
Ventral posterolateral nucleus of thalamus
Somatotopy of Body Areas
A
L
N
T
C2
S5
Fibers conveying input from upper extremity
Fibers conveying input from lower extremity
Fibers conveying input from neck
Fibers conveying input from trunk
Fibers from approximately the second
cervical level
T5
Fibers from approximately the fifth sacral
level
Fibers from approximately the fifth thoracic
level
Review of Blood Supply to DC-ML System
STRUCTURES
PC in Spinal Cord
ML in Medulla
ML in Pons
ML in Midbrain
VPL
Posterior Limb of IC
ARTERIES
penetrating branches of arterial vasocorona (see Figure 5–6)
anterior spinal (see Figure 5–14)
overlap of paramedian and long circumferential branches of basilar
(see Figure 5–21)
short circumferential branches of posterior cerebral and superior
cerebellar (see Figure 5–27)
thalamogeniculate branches of posterior cerebral (see Figure 5–38)
lateral striate branches of middle cerebral (see Figure 5–38)
Sensory Pathways 179
Posterior Column-Medial Lemniscus System
Gy
PoC
Trunk
r
pe ity
Up rem
t
ex
Thigh
Leg
PPGy
Somatosensory cortex
Somatotopy in PC and ML
Foot
Face
Post. limb, IC
A
T
L
Position of ML
L
A
ALS
ML
L
VPL
T
RNu
CC
ML
A
SN
ALS
MLF
CTT
L
ML
T
A
ML
BP
RB
MLF
A
ALS
ML
NuCu
T
ML
PO
L
Py
IAF
NuGr
NuGr
NuCu
FCu
ML
S5
IAF
T5
C2
Py
PRG, T6
L
FGr
FGr
T
A
FCu
PRG, T6
Laminae III-V
N
180 Synopsis of Functional Components, Tracts, Pathways, and Systems
Anterolateral System
7–5 The longitudinal extent and somatotopy of fibers composing the
anterolateral system (ALS). The ALS is a composite bundle containing
ascending fibers that terminate in the reticular formation (spinoreticular fibers), the mesencephalon (spinotectal fibers to deep layers of the
superior colliculus, spinoperiaqueductal fibers to the periaqueductal
grey), the hypothalamus (spinohypothalamic fibers), and the sensory relay nuclei of the dorsal thalamus (spinothalamic fibers). Other fibers in
the ALS include spinoolivary projections to the accessory olivary nuclei.
Spinothalamic fibers terminate primarily in the VPL and reticulothalamic fibers terminate in some intralaminar nuclei, and in medial areas of
the posterior thalamic complex.
Fibers from the PAG and nucleus raphe dorsalis enter the nucleus
raphe magnus and adjacent reticular area. These latter sites, in turn,
project to laminae I, II, and V of the spinal cord via raphespinal and
reticulospinal fibers that participate in the modulation of pain transmission in the spinal cord.
Neurotransmitters: Glutamate (⫹), calcitonin gene-related
peptide, and substance P(⫹)-containing posterior (dorsal) root ganglion cells project into laminae I, II (heavy), V (moderate), and III, IV
(sparse). Some spinoreticular and spinothalamic fibers contain
enkephalin (⫺), somatostatin (⫺), and cholecystokinin (⫹). In addition to enkephalin and somatostatin, some spinomesencephalic fibers
contain vasoactive intestinal polypeptide (⫹). Neurons in the PAG and
nucleus raphe dorsalis containing serotonin and neurotensin project
into the nuclei raphe magnus and adjacent reticular formation. Cells in
these latter centers that contain serotonin and enkephalin send
processes to spinal cord laminae I, II, and V. Serotonergic raphespinal
or enkephalinergic reticulospinal fibers may inhibit primary sensory
fibers or projection neurons, conveying nociceptive (pain) information.
Clinical Correlations: Spinal lesions involving the anterolateral
system (as in the Brown-Sequard syndrome) result in a loss of pain and
temperature sensations on the contralateral side of the body beginning
one to two levels caudal to the lesion. Syringomyelia produces bilateral
sensory losses restricted to adjacent dermatomes because of damage to
the anterior (ventral) white commissure. Vascular lesions in the spinal
cord (such as acute central cervical cord syndrome) may result in a bilateral
and splotchy loss of pain and thermal sense below the lesion because
the ALS has a dual vascular supply.
Vascular lesions in the lateral medulla (posterior inferior cerebellar artery
syndrome) or lateral pons (anterior inferior cerebellar artery occlusion)
result in a loss of pain and thermal sensations over the entire contralateral side of the body (ALS) as well as on the ipsilateral face (spinal
trigeminal tract and nucleus), coupled with other motor and/or sensory
deficits based on damage to structures these vessels serve. Note that the
ALS and PC-ML systems are separated in the medulla (in different vascular territories) but are adjacent to each other in the midbrain (basically in the same vascular territory). Consequently, medullary lesions
will not result in deficits related to both pathways, while a lesion in the
midbrain may result in a contralateral loss of pain, thermal, vibratory,
and discriminative touch sensations on the body, excluding the head.
Profound loss of posterior column and anterolateral system modalities, or intractable pain and/or paresthesias (as in the thalamic syndrome),
may result from vascular lesions in the posterolateral thalamus. Socalled thalamic pain may also be experienced by patients who have
brainstem lesions.
Abbreviations
A
ALS
AWCom
CC
IC
L
MCP
ML
MLF
Nu
NuDark
NuRa,d
NuRa,m
PAG
PoCGy
PPGy
Input from upper extremity regions
Anterolateral system
Anterior (ventral) white commissure
Crus cerebri
Internal capsule
Input from lower extremity regions
Middle cerebellar peduncle
Medial lemniscus
Medial longitudinal fasciculus
Nuclei
Nucleus of Darkschewitsch
Nucleus raphe, dorsalis
Nucleus raphe, magnus
Periaqueductal gray
Postcentral gyrus
Posterior paracentral gyrus
PRG
Py
RaSp
RB
RetF
RetTh
RNu
S
SC
SpRet
SpTec
SpTh
T
VPL
I-VIII
Posterior (dorsal) root ganglion
Pyramid
Raphespinal fibers
Restiform body
Reticular formation (of midbrain)
Reticulothalamic fibers
Red nucleus
Input from sacral regions
Superior colliculus
Spinoreticular fibers
Spinotectal fibers
Spinothalamic fibers
Input from thoracic regions
Ventral posterolateral nucleus of thalamus
Laminae I-VIII of Rexed
Review of Blood Supply to ALS
STRUCTURES
ALS in Spinal Cord
ALS in Medulla
ALS in Pons
ALS in Midbrain
VPL
Posterior Limb of IC
ARTERIES
penetrating branches of arterial vasocorona and branches of central
(see Figures 5–6 and 5–14)
caudal third, vertebral; rostral two-thirds, posterior inferior cerebellar
(see Figure 5–14)
long circumferential branches of basilar (see Figure 5–21)
short circumferential branches of posterior cerebral, superior
cerebellar (see Figure 5–27)
thalamogeniculate branches of posterior cerebral (see Figure 5–38)
lateral striate branches of middle cerebral (see Figure 5–38)
Sensory Pathways 181
Anterolateral System
Gy
PoC
Trunk
Thigh
Leg
r
pe y
Up emit
r
t
ex
PPGy
Somatosensory cortex
Foot
Post. limb, IC
Fac
e
Somatotopy of ALS fibers
Intralaminar Nu
A
Position of ALS fibers
T
L
SpTec
SC
SC, RetF, PAG
NuDark
VPL
PAG
SpTh
ML
L TA
RNu
PAG, NuRa,d
CC
RetTh
ALS
MLF
L TA
ALS
ML
NuRa,m
BP
SpRet
RaSp
RB
MLF
S
A
ALS
ALS
L
T
ML
Py
RaSp
ALS
PRG
Laminae I-VIII
PRG
AWCom
S
L
T
ALS
A
AWCom
RaSp to
Laminae
I, II, V
182 Synopsis of Functional Components, Tracts, Pathways, and Systems
Postsynaptic-Posterior (Dorsal) Column System and the
Spinocervicothalamic Pathway
7–6 The origin, course, and distribution of fibers composing the
postsynaptic-posterior column system (upper) and the spinocervicothalamic pathway (lower). Postsynaptic-posterior column fibers
originate primarily from cells in lamina IV (some cells in laminae III and
V-II also contribute), ascend in the ipsilateral dorsal fasciculi, and end
in their respective nuclei in the caudal medulla. Moderate-to-sparse
collaterals project to a few other medullary targets.
Fibers of the spinocervical part of the spinocervicothalamic pathway
also originate from cells in lamina IV (less so from III and V). The axons
of these cells ascend in the posterior part of the lateral funiculus (this is
sometimes called the dorsolateral funiculus) and end in a topographic
fashion in the lateral cervical nucleus: lumbosacral projections terminate posterolaterally and cervical projections anteromedially. Cells of
the posterior column nuclei and the lateral cervical nucleus convey information to the contralateral thalamus via the medial lemniscus.
Neurotransmitters: Glutamate (⫹) and possibly substance P
(⫹) are present in some spinocervical projections. Because some cells
in laminae III-V have axons that collateralize to both the lateral cervi-
cal nucleus and the dorsal column nuclei, glutamate (and substance P)
may also be present in some postsynaptic dorsal column fibers.
Clinical Correlations: The postsynaptic-posterior column and
spinocervicothalamic pathways are not known to be major circuits in
the human nervous system. However, the occurrence of these fibers
may explain a well known clinical observation. Patients that have received an anterolateral cordotomy (this lesion is placed just ventral to the
denticulate ligament) for intractable pain may experience complete or
partial relief, or there may be a recurrence of pain perception within
days or weeks. Although the cordotomy transects fibers of the anterolateral system (the main pain pathway), this lesion spares the posterior
horn, posterior columns, and spinocervical fibers. Consequently, the
recurrence of pain perception (or even the partial relief of pain) in
these patients may be explained by these postsynaptic-dorsal column
and spinocervicothalamic projections. Through these connections,
some nociceptive (pain) information may be transmitted to the ventral
posterolateral nucleus and on to the sensory cortex, via circuits that bypass the anterolateral system and are spared in a cordotomy.
Abbreviations
ALS
AWCom
FCu
FGr
IAF
LCerNu
ML
NuCu
NuGr
PRG
Anterolateral system
Anterior (ventral) white commissure
Cuneate fasciculus
Gracile fasciculus
Internal arcuate fibers
Lateral cervical nucleus
Medial lemniscus
Cuneate nucleus
Gracile nucleus
Posterior (dorsal) root ganglion
Review of Blood Supply to Dorsal Horn, FGr, FCu, LCerNu
STRUCTURES
FGr, FCu in Spinal Cord
LCerNu
NuGr NuCu
ARTERIES
penetrating branches of arterial vasocorona and some branches
from central (sulcal) (see Figure 5–6)
penetrating branches of arterial vasocorona and branches from
central (see Figure 5–6)
posterior spinal (see Figure 5–14)
Sensory Pathways 183
Postsynaptic-Posterior (Dorsal) Column System and the
Spinocervicothalamic Pathway
FGr
ML
FCu
Other
brainstem
targets
IAF
NuCu
PRG
NuGr
FGr
Laminae IV
(III-VII)
FCu
ALS
PRG
FGr
FGr
Laminae IV
(III-VII)
ALS
ML
LCerNu
FGr
AWCom
FCu
Laminae IV (III-VII)
PRG
PRG
Dorsolateral region
of lateral funiculus
ALS
184 Synopsis of Functional Components, Tracts, Pathways, and Systems
Trigeminal Pathways
7–7 The distribution of general sensory (GSA) information originating on cranial nerves V (trigeminal), VII (facial), IX (glossopharyngeal), and X (vagus). Some of these primary sensory fibers end in the
chief sensory nucleus, but most form the spinal trigeminal tract and
end in the spinal trigeminal nucleus.
Neurons in the spinal trigeminal nucleus and in ventral parts of the
chief sensory nucleus give rise to crossed anterior (ventral) trigeminothalamic fibers. Collaterals of these ascending fibers influence the hypoglossal, facial (corneal reflex, supraorbital, or trigeminofacial reflex), and
trigeminal motor nuclei; mesencephalic collaterals are involved in the
jaw reflex, also called the jaw-jerk reflex. Collaterals also enter the dorsal
motor vagal nucleus (vomiting reflex), the superior salivatory nucleus
(tearing/lacrimal reflex), and the nucleus ambiguus and adjacent reticular
formation (sneezing reflex). Uncrossed posterior (dorsal) trigeminothalamic fibers arise from posterior regions of the chief sensory nucleus.
Neurotransmitters: Substance P (⫹)-containing and cholecystokinin (⫹)-containing trigeminal ganglion cells project to the spinal
trigeminal nucleus, especially its caudal part (pars caudalis). Glutamate
(⫹) is found in many trigeminothalamic fibers arising from the chief sensory nucleus and the pars interpolaris of the spinal nucleus. It is present
in fewer trigeminothalamic fibers from the pars caudalis and in almost
none from the pars oralis. The locus ceruleus (noradrenergic fibers) and
the raphe nuclei (serotonergic fibers) also project to the spinal nucleus.
Enkephalin (⫺)-containing cells are present in caudal regions of the
spinal nucleus, and enkephalinergic fibers are found in the nucleus ambiguus and in the hypoglossal, facial, and trigeminal motor nuclei.
Clinical Correlations: Lesions of the trigeminal ganglion or nerve
proximal to the ganglion result in 1) a loss of pain, temperature, and tac-
tile sensation from the ipsilateral face, oral cavity, and teeth; 2) ipsilateral paralysis of masticatory muscles; and 3) ipsilateral loss of the
corneal reflex. Damage to peripheral portions of the trigeminal nerve
may be traumatic (skull fracture, especially of supraorbital and infraorbital branch), inflammatory (as in herpes zoster), or result from tumor
growth. The deficit would reflect the peripheral portion of the trigeminal nerve damaged.
Trigeminal neuralgia (tic douloureux) is a severe burning pain restricted
to the peripheral distribution of the trigeminal nerve, usually its V2
(maxillary) division. This pain may be initiated by any contact to areas
of the face such as the corner of the mouth, nose, lips, or cheek (e.g.,
shaving, putting make-up on, chewing, or even smiling). The attacks
frequently occur without warning, may happen only a few times a
month to many times in a single day, and are usually seen in patients
40 years of age or older. One probable cause of trigeminal neuralgia is
compression of the trigeminal root by aberrant vessels, most commonly a loop of the superior cerebellar artery (see page 41). Other
causes may include tumor, multiple sclerosis, and ephaptic transmission
(ephapse) in the trigeminal ganglion. This is the most common type of
neuralgia.
In the medulla, fibers of the spinal trigeminal tract and ALS are
served by the posterior inferior cerebellar artery (PICA). Consequently, an alternating hemianesthesia is one characteristic feature of the
PICA syndrome. This is a loss of pain and thermal sensations on one side
of the body and the opposite side of the face. Pontine gliomas may produce a paralysis of masticatory muscles (motor trigeminal damage) and
some loss of tactile input (chief sensory nucleus damage), as well as
other deficits based on what adjacent structures may be involved.
Abbreviations
ALS
CC
CSNu
DTTr
FacNu
GSA
HyNu
IC
ManV
MaxV
Anterolateral system
Crus cerebri
Chief (principal) sensory
nucleus
Dorsal trigeminothalamic tract
Facial nucleus
General somatic afferent
Hypoglossal nucleus
Internal capsule
Mandibular division of
trigeminal nerve
Maxillary division of trigeminal
nerve
MesNu
ML
OpthV
RB
RetF
RNu
SpTNu
SpTTr
TriMoNu
TMJ
VPL
Mesencephalic nucleus
Medial lemniscus
Ophthalmic division of
trigeminal nerve
Restiform body
Reticular formation
Red nucleus
Spinal trigeminal nucleus
Spinal trigeminal tract
Trigeminal motor nucleus
Temporomandibular joint
Ventral posterolateral nucleus
of thalamus
VPM
VTTr
Ventral posteromedial nucleus
of thalamus
Ventral trigeminothalamic
tract
Ganglia
1
2
3
4
Trigeminal ganglion
Geniculate ganglion
Superior of glossopharyngeal
Superior of vagus
Review of Blood Supply to SpTT, SpTNu,
and Trigeminothalamic Tracts
STRUCTURES
SpTTr and SpTNu
in Medulla
SpTTr and SpTNu in Pons
Trigeminothalamic Fibers
in Midbrain
VPM
Posterior Limb of IC
ARTERIES
caudal third, vertebral; rostral two-thirds, posterior
inferior cerebellar (see Figure 5–14)
long circumferential branches of basilar (see Figure 5–21)
short circumferential branches of posterior cerebral and
superior cerebellar (see Figure 5–27)
thalamogeniculate branches of posterior cerebral
(see Figure 5–38)
lateral striate branches of middle cerebral (see Figure 5–38)
Sensory Pathways 185
Trigeminal Pathways
Trunk
Thigh
r
pe ity
Up rem
t
ex
Leg
Foot
Face
Somatosensory
cortex
Posterior
limb, IC
VPM
DTTr
VTTr
MesNu
1
MesNu
ALS
DTTr
TriMoNu
TriMoNu
VTTr
GSA, skin of face, forehead
and part of scalp; membranes of nose and of
nasal, maxillary and frontal
sinuses; oral cavity, teeth;
ant. 2/3 of tongue; muscles
of mastication, TMJ; cornea
and conjunctiva; dura of
mid. and ant. cranial fossae
SpTTr
RNu
ML
Origin of SA Data
CSNu
Position of Trigeminal tracts
FacNu
CC
SpTNu
RetF
VTTr
2
3
GSA, external auditory meatus,
med. and lat. surfaces
of ear (conchae)
GSA, small area on ear
Somatotopy in SpTTr and SpTNu
4
Input from 7,9,10
SpTTr
RB
Man. V
Max. V
Opth. V
SpTTr
SpTNu
SpTTr
SpTNu
ALS
HyNu
SpTNu
GSA, med. and lat. surfaces
of ear (conchae); post.
wall and floor of external
auditory meatus; tympanic
membrane; dura of post.
cranial fossa
186 Synopsis of Functional Components, Tracts, Pathways, and Systems
Solitary Pathways
7–8 Visceral afferent input (SVA-taste; GVA general visceral sensation) on cranial nerves VII (facial), IX (glossopharyngeal), and X (vagus) enters the solitary nuclei via the solitary tract. What we commonly call the solitary “nucleus” is actually a series of small nuclei that
collectively form this rostrocaudal-oriented cell column.
Solitary cells project to the salivatory, hypoglossal, and dorsal motor vagal nuclei and the nucleus ambiguus. Solitary projections to the
nucleus ambiguus are largely bilateral and are the intermediate neurons
in the pathway for the gag reflex. The afferent limb of the gag-reflex is
carried on the glossopharyngeal nerve, and the efferent limb originates
from the nucleus ambiguus. In this respect, the efferent limb travels on
both the glossopharyngeal and vagus nerves. Although not routinely
tested, the gag-reflex should be evaluated in patients with dysarthria, dysphagia, or hoarnessness. Solitariospinal fibers are bilateral with a contralateral preponderance and project to the phrenic nucleus, the intermediolateral cell column, and the ventral horn. The VPM is the
thalamic center through which visceral afferent information is relayed
onto the cerebral cortex.
Neurotransmitters: Substance P (⫹)-containing and cholecystokinin (⫹)-containing cells in the geniculate ganglion (facial nerve)
and in the inferior ganglia of the glossopharyngeal and vagus nerves
project to the solitary nucleus. Enkephalin (⫺), neurotensin, and
GABA (⫹) are present in some solitary neurons that project into the
adjacent dorsal motor vagal nucleus. Cholecystokinin (⫹), somatostatin (⫺), and enkephalin (⫺) are present in solitary neurons, in cells
of the parabrachial nuclei, and in some thalamic neurons that project
to taste, and other visceral areas, of the cortex.
Clinical Correlations: Lesions of the geniculate ganglion, or facial nerve proximal to the ganglion, result in 1) ipsilateral loss of taste
(ageusia) from the anterior two-thirds of the tongue and 2) an ipsilateral
facial (Bell) palsy. Although a glossopharyngeal nerve lesion will result in
ageusia from the posterior third of the tongue on the ipsilateral side, this
loss is difficult to test. On the other hand, glossopharyngeal neuralgia is an
idiopathic pain localized to the peripheral sensory branches of the IXth
nerve in the posterior pharynx, posterior tongue, and tonsillar area. Although comparatively rare, glossopharyngeal neuralgia may be aggravated by talking or even swallowing. Occlusion of the posterior inferior
cerebellar artery (as in the posterior inferior cerebellar artery or lateral
medullary syndrome), in addition to producing an alternate hemianesthesia,
will also result in ageusia from the ipsilateral side of the tongue because
the posterior inferior cerebellar artery serves the solitary tract and nuclei in the medulla.
Interestingly, lesions of the olfactory nerves or tract (anosmia, loss
of olfactory sensation; dysosmia, distorted olfactory sense) may affect
how the patient perceives taste. Witness the fact that the nasal congestion accompanying a severe cold will markedly affect the sense of taste.
Abbreviations
AmyNu
CardResp
GustNu
GVA
HyNu
HyTh
Inf VNu
MVNu
NuAm
PBNu
RB
Amygdaloid nucleus (complex)
Cardiorespiratory portion (caudal) of
solitary nucleus
Gustatory nucleus (rostral portion of
solitary nucleus)
General visceral afferent
Hypoglossal nucleus
Hypothalamus
Inferior (or spinal) vestibular nucleus
Medial vestibular nucleus
Nucleus ambiguus
Parabrachial nuclei
Restiform body
SalNu
SolTr & Nu
SVA
Tr
VA
VPM
Salivatory nuclei
Solitary tract and nuclei
Special visceral afferent
Tract
Visceral afferent
Ventral posteromedial nucleus of
thalamus
Number Key
1
2
3
4
Geniculate ganglion of facial
Inferior ganglion of glossopharyngeal
Inferior ganglion of vagus
Dorsal motor vagal nucleus
Review of Blood Supply to SolNu and SolTr
STRUCTURES
SolNu and Tr in
inferior cerebellar
Ascending Fibers
in Pons
VPM
Posterior Limb of IC
ARTERIES
caudal medulla, anterior spinal; rostral medulla, posterior inferior
cerebellar (See Figure 5–14)
long circumferential branches of basilar and branches of superior
cerebellar (see Figure 5–21)
thalamogeniculate branches of posterior cerebral (see Figure 5–38)
lateral striate branches of middle cerebral (see Figure 5–38)
Sensory Pathways 187
Solitary Pathways
Thigh
Trunk
U
ex ppe
tr e r
mi
ty
Leg
Foot
ce
Fa
HyTh
AmyNu
VPM
PBNu
Origin of VA data
to HyNu,
SalNu
SVA, taste, ant. 2/3 of tongue
1
SVA
(GustNu)
2
4
SolTr and Nu
3
GVA
(CardResp)
NuAm
GVA, submand., subling., lac.
glds.
SVA, taste, post. 1/3 of tongue
GVA, parotid gld.; mucosa of
pharynx; tonsillar sinus;
post. 1/3 of tongue;
carotid body
SVA, taste buds at root of
tongue and on epiglottis
GVA, pharynx; larynx; aortic
bodies; thoracic and
abdominal viscera
Solitariospinal Tr
Position of SolTr & Nu
MVNu
InfVNu
RB
SolTr and Nu
188 Synopsis of Functional Components, Tracts, Pathways, and Systems
7–9 Blank master drawing for sensory pathways. This illustration is
provided for self-evaluation of sensory pathway understanding, for the
instructor to expand on sensory pathways not covered in the atlas, or
both.
Sensory Pathways 189
190 Synopsis of Functional Components, Tracts, Pathways, and Systems
Corticospinal Tracts
7–10 The longitudinal extent of corticospinal fibers and their position and somatotopy at representative levels within the neuraxis. The
somatotopy of corticospinal fibers in the basilar pons is less obvious than
in the internal capsule, crus cerebri, pyramid, or spinal cord. In the decussation of the pyramids, fibers originating from upper extremity areas of the cerebral cortex cross rostral to those that arise from lower extremity areas. In addition to fibers arising from the somatomotor area
of the cerebral cortex (area 4), a significant contingent also originate
from the postcentral gyrus (areas 3, 1, 2); the former terminate primarily in laminae VI-IX, while the latter end mainly in laminae IV and
V. Prefrontal regions, especially area 6, and parietal areas 5 and 7 also
contribute to the corticospinal tract.
Neurotransmitters: Acetylcholine, gamma-aminobutyric acid
(⫺), and substance P (⫹, plus other peptides) are found in small cortical neurons presumed to function as local circuit cells or in corticocortical connections. Glutamate (⫹) is present in cortical efferent
fibers that project to the spinal cord. Glutaminergic corticospinal fibers
and terminals are found in all spinal levels but are especially concentrated in cervical and lumbosacral enlargements. This correlates with
the fact that approximately 55% of all corticospinal fibers terminate in
cervical levels of the spinal cord, approximately 20% in thoracic levels, and approximately 25% in lumbosacral levels. Some corticospinal
fibers may branch and terminate at multiple spinal levels. Lower motor neurons are influenced by corticospinal fibers either directly or indirectly via interneurons. Acetylcholine and calcitonin gene-related
peptides are present in these large motor cells and in their endings in
skeletal muscle.
Clinical Correlations: Myasthenia gravis, a disease characterized
by moderate to profound weakness of skeletal muscles, is caused by
circulating antibodies that react with postsynaptic nicotinic acetylcholine receptors. Progressive muscle fatigability throughout the day
is a hallmark of this disease. Ocular muscles are usually affected first
(diplopia, ptosis), and in approximately 50% of patients, facial and
oropharyngeal muscles are commonly affected ( facial weakness, dysphagia, dysarthria). Weakness may also be seen in limb muscles but almost always in combination with facial/oral weaknesses.
Injury to corticospinal fibers on one side of the cervical spinal cord
(as in the Brown-Sequard syndrome) results in weakness (hemiparesis) or
paralysis (hemiplegia) of the ipsilateral upper and lower extremities. In
addition, and with time, these patients may exhibit features of an upper
motor neuron lesion (hyperreflexia, spasticity, loss of superficial abdominal
reflexes, and the Babinski sign). Bilateral cervical spinal cord damage
above C4–C5 may result in paralysis of all four extremities (quadriplegia). Unilateral spinal cord lesions in thoracic levels may result in paralysis of the ipsilateral lower extremity (monoplegia). If the thoracic spinal
cord damage is bilateral both lower extremities may be paralyzed (paraplegia). Small lesions within the decussation of the pyramids may result
in a bilateral paresis of the upper extremities (lesion in rostral portions)
or a bilateral paresis of the lower extremities (lesion in caudal portions)
based on the crossing patterns of fibers within the decussation.
Rostral to the pyramidal decussation, vascular lesions in the medulla
(the medial medullary syndrome), pons (the Millard-Gubler or Foville syndromes), or midbrain (the Weber syndrome) all produce alternating
(crossed) hemiplegias. These present as a contralateral hemiplegia of the
upper and lower extremities, coupled with an ipsilateral paralysis of
the tongue (medulla), facial muscles or lateral rectus muscle (pons),
and most eye movements (midbrain). Sensory deficits are frequently
seen as part of these syndromes. Lesions in the internal capsule (lacunar strokes) produce contralateral hemiparesis sometimes coupled with
various cranial nerve signs due to corticonuclear (corticobulbar) fiber
involvement. Bilateral weakness, indicative of corticospinal involvement, is also present in amyotrophic lateral sclerosis.
Abbreviations
ACSp
ALS
APGy
BP
CC
CNu
CSp
IC
Anterior corticospinal tract
Anterolateral system
Anterior paracentral gyrus
Basilar pons
Crus cerebri
Corticonuclear (corticobulbar)
fibers
Corticospinal fibers
Internal capsule
LCSp
ML
MLF
PO
PrCGy
Py
RB
RNu
SN
Lateral corticospinal tract
Medial lemniscus
Medial longitudinal fasciculus
Principal olivary nucleus
Precentral gyrus
Pyramid
Restiform body
Red nucleus
Substantia nigra
Somatotopy of CSp Fibers
A Position of fibers coursing to
upper extremity regions of
spinal cord
L Position of fibers coursing to
lower extremity regions of
spinal cord
T Position of fibers coursing to
thoracic regions of spinal cord
Review of Blood Supply to Corticospinal Fibers
STRUCTURES
Posterior Limb of IC
Crus Cerebri in
Midbrain
CSp in BP
Py in Medulla
LCSp in Spinal Cord
ARTERIES
lateral striate branches of middle cerebral (see
Figure 5–38)
paramedian and short circumferential
branches of basilar and posterior
communicating (see Figure 5–27)
paramedian branches of basilar (see Figure 5–21)
anterior spinal (see Figure 5–14)
penetrating branches of arterial vasocorona (leg
fibers), branches of central artery (arm fibers)
(See Figure 5–6)
Motor Pathways 191
Corticospinal Tracts
Gy
PrC per ty
Up emi
tr
ex
Trunk
Thigh
Leg
APGy
Somatomotor cortex
Foot
Fa c
e
Somatotopy of CSp
Post. limb, IC
A
T
L
Position of CSp
ALS
ML
RNu
SN
CC
L
T
CSp fibers in CC
VesNu
ALS
A
Face
(CNu Fibers)
MLF
CSp fibers in BP
ML
L
A
T
BP
CSp
RB
CSp fibers in Py
MLF
ALS
ML
PO
Pyramidal (motor) decussation
L
T
A
Py
LCSp
L
LCSp
T
ACSp
A
Laminae
IV-IX
ALS
ACSp
192 Synopsis of Functional Components, Tracts, Pathways, and Systems
Corticonuclear (Corticobulbar) Fibers
7–11 The origin, course, and distribution of corticonuclear (corticobulbar) fibers to brainstem motor nuclei. These fibers influence—either directly or through neurons in the immediately adjacent reticular
formation—the motor nuclei of oculomotor, trochlear, trigeminal, abducens, facial, glossopharyngeal and vagus (both via nucleus ambiguus),
spinal accessory, and hypoglossal nerves.
Corticonuclear (corticobulbar) fibers arise in the frontal eye fields (areas 6 and 8 in caudal portions of the middle frontal gyrus), the precentral gyrus (somatomotor cortex, area 4), and some originate from the
postcentral gyrus (areas 3,1, 2). Fibers from area 4 occupy the genu of
the internal capsule, but those from the frontal eye fields (areas 8,6) may
traverse caudal portions of the anterior limb, and some (from areas
3,1,2), may occupy the most rostral portions of the posterior limb.
Fibers that arise in areas 8 and 6 terminate in the rostral interstitial nucleus
of the medial longitudinal fasciculus (vertical gaze center) and in the paramedian pontine reticular formation (horizontal gaze center); these areas, in turn,
project respectively to the IIIrd and IVth, and to the VIth nuclei. Fibers
from area 4 terminate in, or adjacent to, cranial nerve motor nuclei excluding those of III, IV, and VI.
Although not illustrated here, the superior colliculus receives cortical input from area 8 and from the parietal eye field (area 7) and also
projects to the riMLF and PPRF. In addition, it is important to note
that descending cortical fibers (many arising in areas 3, 1, 2) project to
sensory relay nuclei of some cranial nerves and to other sensory relay
nuclei in the brainstem, such as those of the posterior column system.
Neurotransmitters: Glutamate (⫹) is found in many corticofugal axons that directly innervate cranial nerve motor nuclei and in
those fibers that terminate near (indirect), but not in, the various motor nuclei.
Clinical Correlations: Lesions involving the motor cortex (as in
cerebral artery occlusion) or the internal capsule (as in lacunar strokes
or occlusion of lenticulostriate branches of M1) give rise to a contralateral hemiplegia of the arm and leg (corticospinal fiber involvement) coupled with certain cranial nerve signs. Strictly cortical lesions
may produce a transient gaze palsy in which the eyes deviate toward the
lesioned side and away from the side of the hemiplegia. In addition to
a contralateral hemiplegia, common cranial nerve findings in capsular lesions may include 1) deviation of the tongue toward the side of the
weakness and away from the side of the lesion when protruded and 2)
paralysis of facial muscles on the contralateral lower half of the face
(central facial palsy). This reflects the fact that corticonuclear (corticobulbar) fibers to genioglossus motor neurons and to facial motor neurons serving the lower face are primarily crossed. Interruption of corticonuclear fibers to the nucleus ambiguus may result in weakness of
palatal muscles contralateral to the lesion; the uvula will deviate towards the ipsilateral (lesioned) side on attempted phonation. In addition, a lesion involving corticonuclear fibers to the accessory nucleus
may result in drooping of the ipsilateral shoulder (or an inability to elevate the shoulder against resistance) due to trapezius weakness, and
difficulty in turning the head (against resistance) to the contralateral
side due to weakness of the sternocleidomastoid muscle. In contrast to
the alternating hemiplegia seen in some brainstem lesions, hemisphere
lesions result in spinal and cranial nerve deficits that are generally, but
not exclusively, contralateral to the cerebral injury.
Brainstem lesions, especially in the midbrain or pons, may result in
the following: 1) vertical gaze palsies (midbrain), 2) the Parinaud syndrome—paralysis of upward gaze (tumors in area of pineal), 3) internuclear ophthalmoplegia (lesion in MLF between motor nuclei of III and
VI), 4) horizontal gaze palsies (lesion in PPRF), or 5) the one-and-a-half
syndrome. In the latter case, the lesion is adjacent to the midline and involves the abducens nucleus and adjacent PPRF, internuclear fibers
from the ipsilateral abducens that are crossing to enter the contralateral MLF, and internuclear fibers from the contralateral abducens nucleus that cross to enter the MLF on the ipsilateral (lesioned) side. The
result is a loss of ipsilateral abduction (lateral rectus) and adduction
(medial rectus, the “one”) and a contralateral loss of adduction (medial
rectus, the “half ”); the only remaining horizontal movement is contralateral abduction via the intact abducens motor neurons.
Abbreviations
AbdNu
AccNu
FacNu
HyNu
IC
NuAm
Abducens nucleus
Accessory nucleus (spinal accessory nu.)
Facial nucleus
Hypoglossal nucleus
Internal capsule
Nucleus ambiguus
OcNu
PPRF
riMLF
Oculomotor nucleus
Paramedian pontine reticular formation
Rostral interstitial nucleus of the medial
longitudinal fasciculus
TriMoNu Trigeminal motor nucleus
TroNu Trochlear nucleus
Review of Blood Supply to Cranial Nerve Motor Nuclei
STRUCTURES
OcNu and EWNu
TriMoNu
AbdNu and FacNu
NuAm
HyNu
ARTERIES
paramedian branches of basilar bifurcation and medial branches
of posterior cerebral and posterior communicating (see Figure
5–27)
long circumferential branches of basilar (see Figure 5–21)
long circumferential branches of basilar (see Figure 5–21)
posterior inferior cerebellar (see Figure 5–14)
anterior spinal (see Figure 5–14)
Motor Pathways 193
Corticonuclear (Corticobulbar) Fibers
Motor cortex,
precentral gyrus
Frontal eye fields
riMLF
OcNu
TroNu
Genu of IC
TriMoNu
Bilateral for upper face
PPRF
AbdNu
FacNu
Crossed for lower face
NuAm
= Direct to motor
neurons of nucleus
Crossed for uvula
(soft palate)
= Indirect to motor
neurons via adjacent
reticular formation
Crossed for
genioglossus muscle
HyNu
= Bilateral projections
AccNu
= Primarily crossed
projections
194 Synopsis of Functional Components, Tracts, Pathways, and Systems
Tectospinal and Reticulospinal Tracts
7–12 The origin, course, position in representative cross-sections
of brainstem and spinal cord, and the general distribution of tectospinal
and reticulospinal tracts. Tectospinal fibers originate from deeper layers of the superior colliculus, cross in the posterior (dorsal) tegmental
decussation, and distribute to cervical cord levels. Several regions of
cerebral cortex (e.g., frontal, parietal, temporal) project to the tectum, but the most highly organized corticotectal projections arise from
the visual cortex. Pontoreticulospinal fibers (medial reticulospinal)
tend to be uncrossed, while those from the medulla (bulboreticulospinal or lateral reticulospinal) are bilateral but with a pronounced
ipsilateral preponderance. Corticoreticular fibers are bilateral with a
slight contralateral preponderance and originate from several cortical
areas.
Neurotransmitters: Corticotectal projections, especially those
from the visual cortex, utilize glutamate (⫹). This substance is also
present in most corticoreticular fibers. Some neurons of the gigantocellular reticular nucleus that send their axons to the spinal cord, as
reticulospinal projections, contain enkephalin (⫺) and substance P
(⫹). Enkephalinergic reticulospinal fibers may be part of the descend-
ing system that modulates pain transmission at the spinal level. Many
reticulospinal fibers influence the activity of lower motor neurons.
Clinical Correlations: Isolated lesions of only tectospinal and
reticulospinal fibers are essentially never seen. Tectospinal fibers project to upper cervical levels where they influence reflex movement of
the head and neck. Such movements may be diminished or slowed in
patients with damage to these fibers. Pontoreticulospinal (medial reticulospinal) fibers are excitatory to extensor motor neurons and to neurons innervating axial musculature; some of these fibers may also inhibit flexor motor neurons. In contrast, some bulboreticulospinal
(lateral reticulospinal) fibers are primarily inhibitory to extensor motor neurons and to neurons innervating muscles of the neck and back;
these fibers may also excite flexor motor neurons via interneurons.
Reticulospinal (and vestibulospinal) fibers contribute to the spasticity
that develops in patients having lesions of corticospinal fibers. These
reticulospinal and vestibulospinal fibers (see Figure 7-13 on page 196)
also contribute to the tonic extension of the arms and legs seen in decerebrate rigidity when spinal motor neurons are released from descending cortical control.
Abbreviations
ALS
ATegDec
Anterolateral system
Anterior tegmental decussation
(rubrospinal fibers)
BP Basilar pons
CC Crus cerebri
CRet Corticoreticular fibers
CTec Corticotectal fibers
GigRetNu Gigantocellular reticular nucleus
LCSp Lateral corticospinal tract
ML Medial lemniscus
MLF Medial longitudinal fasciculus
MVNu Medial vestibular nucleus
OcNu Oculomotor nucleus
PO Principal olivary nucleus
PTegDec Posterior tegmental decussation
(tectospinal fibers)
Py Pyramid
RB Restiform body
RetNu Reticular nuclei
RetSp Reticulospinal tract(s)
RNu Red nucleus
RuSp Rubrospinal tract
SC Superior colliculus
SN Substantia nigra
SpVNu Spinal (or inferior) vestibular nucleus
TecSp Tectospinal tract
Review of Blood Supply to SC, Reticular Formation of Pons
and Medulla, and TecSp and RetSp Tracts in Cord
STRUCTURES
SC
Pontine Reticular
Formation
Medullary Recticular
Formation
TecSp and RetSp
ARTERIES
long circumferential branches (quadrigeminal branch) of posterior
cerebral plus some from superior cerebellar and posterior
choroidal (see Figure 5–27)
long circumferential branches of basilar plus branches of superior
cerebellar in rostral pons (see Figure 5–21)
branches of vertebral plus paramedian branches of basilar at
medulla-pons junction (see Figure 5–14)
branches of central artery (TecSp and Medullary RetSp); Tracts
penetrating branches of arterial vasocorona (Pontine RetSp) (see
Figures 5–14 and 5–6)
Motor Pathways 195
Tectospinal and Reticulospinal Tracts
CRet
CTec
Postition of TecSp and RetSp
SC
CTec
PTegDec
SC
TecSp
ML
RNu
SN
CRet
CC
CRet
PTegDec (TecSp)
ATegDec (RuSp)
MLF
Pontine RetNu:
oralis
TecSp
RetNu of Pons
caudalis
ML
ALS
BP
InfVNu
Pontine RetSp
MVNu
RB
ALS
GigRetNu
PO
MLF
TecSp
GigRetNu
ML
Py
Pontine RetSp
TecSp
Medullary RetSp
LCSp
Medullary RetSp
to Laminae VII
(VI,VII, IX)
Pontine RetSp
to Laminae VIII
(VII,IX)
ALS
TecSp
to Laminae VI, VII (VIII)
of cervical levels
196 Synopsis of Functional Components, Tracts, Pathways, and Systems
Rubrospinal and Vestibulospinal Tracts
7–13 The origin, course, and position in representative cross-sections of brainstem and spinal cord, and the general distribution of
rubrospinal and vestibulospinal tracts. Rubrospinal fibers cross in the anterior (ventral) tegmental decussation and distribute to all spinal levels
although projections to cervical levels clearly predominate. Cells in dorsomedial regions of the red nucleus receive input from upper extremity
areas of the motor cortex and project to cervical cord, but those in ventrolateral areas of the nucleus receive some fibers from lower extremity
areas of the motor cortex and may project in sparse numbers to lumbosacral levels. The red nucleus also projects, via the central tegmental
tract, to the ipsilateral inferior olivary complex (rubroolivary fibers).
Medial and lateral vestibular nuclei give rise to the medial and lateral
vestibulospinal tracts, respectively. The former tract is primarily ipsilateral, projects to upper spinal levels, and is considered a component
of the medial longitudinal fasciculus in the spinal cord. The latter tract
is ipsilateral and somatotopically organized; fibers to lumbosacral levels
originate from dorsal and caudal regions of the lateral nucleus, while
those to cervical levels arise from its rostral and more ventral areas.
Neurotransmitters: Glutamate (⫹) is present in corticorubral
fibers. Some lateral vestibulospinal fibers contain aspartate (⫹),
whereas glycine (⫺) is present in a portion of the medial vestibulospinal projection. There are numerous gamma-aminobutyric acid
(⫺)-containing fibers in the vestibular complex; these represent the
endings of cerebellar corticovestibular fibers.
Clinical Correlations: Isolated injury to rubrospinal and vestibulospinal fibers is really not seen in humans. Deficits in fine distal limb
movements seen in monkeys following experimental rubrospinal lesions may be present in humans. However, these deficits are overshadowed by the hemiplegia associated with injury to the adjacent corticospinal fibers. The contralateral tremor seen in patients with the
Claude syndrome (a lesion of the medial midbrain) is partially related to
damage to the red nucleus as well as to the adjacent cerebellothalamic
fibers. These patients may also have a paucity of most eye movement
on the ipsilateral side and a dilated pupil (mydriasis) due to concurrent
damage to exiting rootlets of the oculomotor nerve.
Medial vestibulospinal fibers primarily inhibit motor neurons innervating extensors and neurons serving muscles of the back and neck. Lateral vestibulospinal fibers may inhibit some flexor motor neurons, but
they mainly facilitate spinal reflexes via their excitatory influence on spinal
motor neurons innervating extensors. Vestibulospinal and reticulospinal
(see Figure 7-12 on page 194) fibers contribute to the spasticity seen in patients with damage to corticospinal fibers or to the tonic extension of the
extremities in patients with decerebrate rigidity. In the case of decerebrate
rigidity, the descending influences on spinal flexor motor neurons (corticospinal, rubrospinal) is removed; the descending brainstem influence on
spinal extensor motor neurons predominates; this is augmented by excitatory spinoreticular input (via ALS) to some of the centers giving rise to
reticulospinal fibers (see also Figure 7-12 on page 194).
Abbreviations
ATegDec Anterior tegmental decussation
(rubrospinal fibers)
CC Crus cerebri
CorRu Corticorubral fibers
FacNu Facial nucleus
InfVNu Inferior (or spinal) vestibular nucleus
LCSp Lateral corticospinal tract
LRNu Lateral reticular nucleus
LVNu Lateral vestibular nucleus
LVesSp Lateral vestibulospinal tract
ML Medial lemniscus
MLF Medial longitudinal fasciculus
MVessp
MVNu
OcNu
PTegDec
Py
RNu
RuSp
SC
SVNu
TecSp
VesSp
Medial vestibulospinal tract
Medial vestibular nucleus
Oculomotor nucleus
Posterior tegmental decussation (tectospinal
fibers)
Pyramid
Red nucleus
Rubrospinal tract
Superior colliculus
Superior vestibular nucleus
Tectospinal tract
Vestibulospinal tracts
Review of Blood Supply to RNu, Vestibular Nuclei, MFL
and RuSp, and Vestibulospinal Tracts in Cords
STRUCTURES
RNu
Vestibular Nuclei
MLF
MVesSp
LVesSp and RuSp
ARTERIES
medial branches of posterior cerebral and posterior communicating
plus some from short circumferential branches of posterior cerebral
(see Figure 5–27)
posterior inferior cerebellar in medulla (see Figure 5–14) and long
circumferential branches in pons (see Figure 5–21)
long circumferential branches of basilar in pons (see Figure 5–21)
and anterior spinal in medulla (see Figure 5–14)
branches of central artery (see Figures 5–6 and 5–14)
penetrating branches of arterial vasocorona plus terminal branches
of central artery (see Figure 5–6)
Motor Pathways 197
Rubrospinal and Vestibulospinal Tracts
Thigh
Leg
Face
Foot
CorRu
Position of RuSp and VesSp
PTegDec
SC
OCNu
RNu
ML
RNu
PTegDec (TecSp)
RuSp
CC
SVNu
ATegDec (RuSp)
LVNu
MVNu
MVNu
FacNu
InfVNu
MLF
RuSp
SpVNu
LRNu
LVesSp
Py
ML
MVesSp
in MLF
LCSp
LVesSp
RuSp
to Laminae
V-VIII
RuSp
MVesSp
LVesSp
MVesSp
LCSp
RuSp
LVesSp
to Laminae
VII and VIII
198 Synopsis of Functional Components, Tracts, Pathways, and Systems
7–14 Blank master drawing for motor pathways. This illustration
is provided for self-evaluation of motor pathways understanding, for
the instructor to expand on motor pathways not covered in this atlas,
or both.
Motor Pathways 199
200 Synopsis of Functional Components, Tracts, Pathways, and Systems
Cranial Nerve Efferents (III, IV, VI, XI-AccNu, XII)
7–15 The origin and peripheral distribution of GSE fibers from the
oculomotor, trochlear, abducens, spinal accessory, and hypoglossal
nuclei. Also shown are GVE fibers arising from the Edinger-Westphal
nucleus and the distribution of postganglionic fibers from the ciliary
ganglion. Internuclear abducens neurons project, via the MLF, to contralateral oculomotor neurons that innervate the medial rectus muscle
(internuclear ophthalmoplegia pathway).
Some authors specify the functional component of neurons in the
accessory nucleus as special visceral efferent, some specify it as somatic
efferent, and some are noncommittal. Because, in humans, the trapezius and sternocleidomastoid muscles originate from cervical somites
located caudal to the last pharyngeal arch, the functional component is
designated here as GSE. In addition, experiments in animals reveal that
motor neurons innervating the trapezius and sternocleidomastoid
muscles are found in cervical cord levels C1 to approximately C6.
Neurotransmitters: Acetylcholine (and probably calcitonin
gene-related peptide, CGRP) is found in the motor neurons of cranial
nerve nuclei and in their peripheral endings. This substance is also
found in cells of the Edinger-Westphal nucleus and the ciliary ganglion.
Clinical Correlations: Myasthenia gravis (MG) is a disease caused
by autoantibodies that may directly block nicotinic acetylcholine
receptors or damage the postsynaptic membrane (via complement mediated lysis) thereby reducing the number of viable receptor sites. Ocular movement disorders (diplopia, ptosis) are seen first in approximately 50% of patients and are present in approximately 85% of all
MG patients. Movements of the neck and tongue may also be impaired,
with the latter contributing to dysphagia and dysarthria.
Lesions of the IIIrd nerve (as in the Weber syndrome or in carotid cavernous aneurysms) may result in 1) ptosis, 2) lateral and downward deviation of the eye, and 3) diplopia (except on ipsilateral lateral gaze). In
addition, the pupil may be unaffected (pupillary sparing) or dilated and
fixed. Lesions in the midbrain that involve the root of the IIIrd nerve
and the crus cerebri give rise to a superior alternating (crossed) hemiplegia.
This is a paralysis of most eye movement and possibly a dilated pupil
on the ipsilateral side and a contralateral hemiplegia of the extremities.
Damage to the MLF (as in multiple sclerosis or small vessel occlusion) between the VIth and IIIrd nuclei results in internuclear ophthalmoplegia; on
attempted lateral gaze, the opposite medial rectus muscle will not adduct.
A lesion of the IVth nerve (frequently caused by trauma) produces diplopia
on downward and inward gaze (tilting the head may give some relief ), and
the eye is slightly elevated when the patient looks straight ahead.
Diabetes mellitus, trauma, or pontine gliomas are some causes of VIth
nerve dysfunction. In these patients, the affected eye is slightly adducted, and diplopia is pronounced on attempted gaze to the lesioned
side. Damage in the caudal and medial pons may involve the fibers of
the VIth nerve and the adjacent corticospinal fibers in the basilar pons,
giving rise to a middle alternating (crossed) hemiplegia. The deficits are an
ipsilateral paralysis of the lateral rectus muscle and a contralateral
hemiplegia of the extremities. The XIth nerve may be damaged centrally (as in syringobulbia or amyotrophic lateral sclerosis) or at the jugular
foramen with resultant paralysis of the ipsilateral sternocleidomastoid
and upper parts of the trapezius muscle.
Central injury to the XIIth nucleus or fibers (as in the medial
medullary syndrome or in syringobulbia) or to its peripheral parts (as in
polyneuropathy or tumors) results in deviation of the tongue toward the
lesioned side on attempted protrusion. A lesion in the medial aspects
of the medulla will give rise to an inferior alternating (crossed) hemiplegia.
This is characterized by a paralysis of the ipsilateral side of the tongue
(XIIth root damage) and contralateral hemiplegia of the extremities
(damage to corticospinal fibers in the pyramid).
Abbreviations
AbdNr
AbdNu
AccNr
AccNu
BP
CC
EWNu
FacCol
HyNr
HyNu
ML
MLF
OcNr
Abducens nerve
Abducens nucleus
Accessory nerve
Accessory nucleus (spinal accessory nu.)
Basilar pons
Crus cerebri
Edinger-Westphal nucleus
Facial colliculus
Hypoglossal nerve
Hypoglossal nucleus
Medial lemniscus
Medial longitudinal fasciculus
Oculomotor nerve
OcNu
PO
Py
RNu
SC
SCP,Dec
TroDec
TroNr
TroNu
Oculomotor nucleus
Principal olivary nucleus
Pyramid
Red nucleus
Superior colliculus
Superior cerebellar peduncle, decussation
Trochlear decussation
Trochlear nerve
Trochlear nucleus
Ganglion
1
Ciliary
Review of Blood Supply to OcNu, TroNu, AbdNu and HyNu,
and the Internal Course of their Fibers
STRUCTURES
OcNu and Fibers
TroNu
AbdNu
Abducens Fibers in BP
HyNu and Fibers
ARTERIES
medial branches of posterior cerebral and posterior
communicating (see Figure 5–27)
paramedian branches of basilar bifurcation (see Figure 5–27)
long circumferential branches of basilar (see Figure 5–21)
paramedian branches of basilar (see Figure 5–21)
anterior spinal (see Figure 5–14)
Motor Pathways 201
Cranial Nerve Efferents (III, IV, VI, XI-AccNu, and XII)
Position of Nucleus
and Internal
Route of Fibers
SC
ML
SN
OcNu and
EWNu
OcNu
RNu
EWNu
OcNr
CC
1
OcNr
TroNu
Muscles Innervated
Ciliary; Sphincter of iris
TroNr
Med. Rectus
Inf. Oblique; Inf. and
Med. recti
TroDec
Exit of TroNr
TroNu
MLF
Sup. rectus
TroDec
Levator palpebrae
TroNr
MLF
CC
Sup. Oblique
AbdNu
SCP, Dec
Lat. rectus
AbdNr
FacCol
AbdNu
MLF
HyNu
ML
BP
HyNr
AbdNr
Intrinsic tongue muscles,
and styloglossus,
hyoglossus,
genioglossus
AccNr
AccNu
HyNu
Sternocleidomastoid
Trapezius
MLF
PO
ML
Py
HyNr
202 Synopsis of Functional Components, Tracts, Pathways, and Systems
Cranial Nerve Efferents (V, VII, IX, and X)
7–16 The origin and peripheral distribution of fibers arising from
the SVE motor nuclei of the trigeminal, facial, and glossopharyngeal and
vagus (via the nucleus ambiguus) nerves. Also shown are the origin of
GVE preganglionic parasympathetic fibers from the superior (to facial
nerve) and inferior (to glossopharyngeal nerve) salivatory nuclei and
from the dorsal motor vagal nucleus. Their respective ganglia are indicated as well as the structures innervated by postganglionic fibers arising from each. The SVE functional component specifies cranial nerve
motor nuclei that innervate head muscles that arose, embryologically,
from pharyngeal arches. Muscles innervated by the trigeminal nerve (V)
come from the 1st arch, those served by the facial nerve (VII) from the
2nd arch; the stylopharyngeal muscle originates from the 3rd arch and
is innervated by the glossopharyngeal nerve (IX), and the muscles derived from the 4th arch are served by the vagus nerve (X).
Neurotransmitters: The transmitter found in the cells of cranial nerve motor nuclei, and in their peripheral endings, is acetylcholine; CGRP is also colocalized in these motor neurons. This
substance is also present in preganglionic and postganglionic parasympathetic neurons.
Clinical Correlations: Patients with myasthenia gravis frequently
have oropharyngeal symptoms and complications that result in
dysarthria, and dysphagia. These individuals have difficulty chewing and
swallowing, their jaw may hang open, and the mobility of facial muscles is decreased. Imparied hearing (weakness of tensor tympani) and
hyperacusis (weakness of stapedius) may also be present.
Lesions of the Vth nerve (as in meningiomas or trauma) result in 1) loss
of pain, temperature, and touch on the ipsilateral face and in the oral
and nasal cavities; 2) paralysis of ipsilateral masticatory muscles ( jaw de-
viates to the lesioned side when closed);, and 3) loss of the afferent limb
of the corneal reflex. If especially large, a vestibular schwannoma may
compress the trigeminal nerve root and result in a hemifacial sensory
loss that may include the oral cavity. Trigeminal neuralgia (tic douloureux)
is an intense, sudden, intermittent pain emanating from the area of the
cheek, oral cavity, or adjacent parts of the nose (distribution of V2 or
V3, see also Figure 7-7 on page 184).
Tumors (such as chordoma or vestibular schwannoma), trauma, or
meningitis may damage the VIIth nerve, resulting in 1) an ipsilateral facial palsy (or Bell palsy); 2) loss of taste from the ipsilateral two-thirds
of the tongue; and (3) decreased secretion from the ipsilateral lacrimal,
nasal, and sublingual and submandibular glands. Injury distal to the
chorda tympani produces only an ipsilateral facial palsy. A paralysis of
the muscles on one side of the face with no paralysis of the extremities
is a facial hemiplegia. On the other hand, intermittent and involuntary
contraction of the facial muscles is called hemifacial spasm. One cause is
compression of the facial root by an artery, most commonly a loop of
the anterior inferior cerebellar artery or its branches. These patients
may also have deficits (vertigo, tinnitus, hearing loss) suggesting involvement of the adjacent vestibulocochlear nerve.
Because of their common origin from NuAm, adjacent exit from the
medulla, and passage through the jugular foramen, the IXth and Xth
nerves may be damaged together (as in amyotrophic lateral sclerosis or in
syringobulbia). The results are dysarthria, dysphagia, dyspnea, loss of taste
from the ipsilateral caudal tongue, and loss of the gag reflex, but no significant autonomic deficits. Bilateral lesions of the Xth nerve are lifethreatening because of the resultant total paralysis (and closure) of the
muscles in the vocal folds (vocalis muscle).
Abbreviations
AbdNu
ALS
BP
DVagNu
FacNr
FacNu
GINr
HyNu
ISNu
MesNu
ML
MLF
NuAm
PSNu
Abducens nucleus
Anterolateral system
Basilar pons
Dorsal motor nucleus of vagus
Facial nerve
Facial nucleus
Glossopharyngeal nerve
Hypoglossal nucleus
Inferior salivatory nucleus
Mesencephalic nucleus
Medial lemniscus
Medial longitudinal fasciculus
Nucleus ambiguus
Principal (chief ) sensory nucleus
SpTNu
SpTTr
SSNu
TecSp
TriMoNu
TriNr
VagNr
Spinal trigeminal nucleus
Spinal trigeminal tract
Superior salivatory nucleus
Tectospinal tract
Trigeminal motor nucleus
Trigeminal nerve
Vagus nerve
Ganglia
1
2
3
4
Pterygopalatine
Submandibular
Otic
Terminal and/or intramural
Review of Blood Supply to TriMoNu, FacNu, DMNu and NuAm,
and the Internal Course of Their Fibers
STRUCTURES
TriMoNu and Trigeminal Root
FacNu and Internal Genu
DMNu and NuAm
ARTERIES
long circumferential branches of basilar (see Figure 5–21)
long circumferential branches of basilar (see Figure 5–21)
branches of vertebral and posterior inferior cerebellar
(see Figure 5–14)
Motor Pathways 203
Cranial Nerve Efferents (V, VII, IX, and X)
Position of Nucleus
and Internal
Route of Fibers
TriMotNu
MesNu
MLF
PSNu
TecSp
Motor root
of TriNr
ALS
ML
BP
Structures Innervated
Masticatory muscles and
tensor tympani,
tensor veli palatini,
mylohyoid,
digastric (ant. belly)
TriMotNu
Motor root
of TriNr
AbdNu
SpTTr &
SpTNu
FacNu
Muscles of facial expression,
and stapedius, buccinator,
stylohyoid, platysma
digastric (post. belly)
FacNr
SSNu
MLF
AbdNu
GINr
ISNu
SSNu
1
NuAm
ML
Lacrimal gld.; mucous
membranes of nose
and mouth
FacNr
FacNu
2
Submandibular and
sublingual glds.
VagNr
3
DVagNu
DVagNu
HyNu
Parotid gld.
MLF
TecSp
NuAm
ML
Stylopharyngeus
Striated mus. of pharynx,
larynx, esophagus
VagNr
SpTTr and
SpTNu
4
Thoracic and abdomnal
viscera; smooth and
cardiac muscle; glandular
epithelium
204 Synopsis of Functional Components, Tracts, Pathways, and Systems
Spinocerebellar Tracts
7–17 The origin, course, and distribution pattern of fibers to the
cerebellar cortex and nuclei from the spinal cord (posterior [dorsal]
and anterior [ventral] spinocerebellar tracts, rostral spinocerebellar
fibers) and from the external cuneate nucleus (cuneocerebellar
fibers). Also illustrated is the somatotopy of those fibers originating
from the spinal cord. These fibers enter the cerebellum via the restiform body, the larger portion of the inferior cerebellar peduncle, or
in relationship to the superior cerebellar peduncle. After these fibers
enter the cerebellum, collaterals are given off to the cerebellar nuclei
while the parent axons of spinocerebellar and cuneocerebellar fibers
pass on to the cortex, where they end as mossy fibers in the graunular
layer. Although not shown here, there are important ascending spinal
projections to the medial and dorsal accessory nuclei of the inferior
olivary complex (spino-olivary fibers). The accessory olivary nuclei
(as well as the principal olivary nucleus) project to the cerebellar cortex and send collaterals into the nuclei (see Figure 7-18 on page 206).
Neurotransmitters: Glutamate (⫹) is found in some spinocere-
bellar fibers, in their mossy fiber terminals in the cerebellar cortex, and
in their collateral branches that innervate the cerebellar nuclei.
Clinical Correlations: Lesions, or tumors, that selectively damage only spinocerebellar fibers are rarely, if ever, seen in humans. The
ataxia one might expect to see in patients with a spinal cord hemisection
(as in the Brown-Sequard syndrome) is masked by the hemiplegia resulting
from the concomitant damage to lateral corticospinal (and other) fibers.
Friedreich ataxia (hereditary spinal ataxia) is an autosomal recessive disorder the symptoms of which usually appear between 8 and 15 years
of age. There is degeneration of anterior and posterior spinocerebellar
tracts plus the posterior columns and corticospinal tracts. Degenerative changes are also seen in Purkinje cells in the cerebellum, in posterior root ganglion cells, in neurons of the Clarke column, and in some
nuclei of the pons and medulla. The axial and appendicular ataxia seen
in these patients correlates partially with the spinocerebellar degeneration and also partially with proprioceptive losses via the degeneration
of posterior column fibers.
Abbreviations
ACNu
ALS
AMV
ASCT
Cbl
CblNu
CCblF
DNuC
FNL
IZ
L
MesNu
ML
PRG
Accessory (external or lateral) cuneate
nucleus
Anterolateral system
Anterior medullary velum
Anterior (ventral) spinocerebellar tract
Cerebellum
Cerebellar nuclei
Cuneocerebellar fibers
Dorsal nucleus of Clarke
Flocculonodular lobe
Intermediate zone
Lumbar representation
Mesencephalic nucleus
Medial lemniscus
Posterior (dorsal) root ganglion
PSCT
PSNu
Py
RB
RSCF
RuSp
S
SBC
SCP
SpTNu
SpTTr
T
TriMoNu
VesNu
Posterior (dorsal) spinocerebellar tract
Principal (chief ) sensory nucleus of
trigeminal nerve
Pyramid
Restiform body
Rostral spinocerebellar fibers
Rubrospinal tract
Sacral representation
Spinal border cells
Superior cerebellar peduncle
Spinal trigeminal nucleus
Spinal trigeminal tract
Thoracic representation
Trigeminal motor nucleus
Vestibular nuclei
Review of Blood Supply to Spinal Cord Grey Matter,
Spinocerebellar Tracts, RB, and SCP
STRUCTURES
Spinal Cord Grey
PSCT and ASCT in Cord
RB
SCP
Cerebellum
ARTERIES
branches of central artery (see Figure 5–6)
penetrating branches of arterial vasocorona (see Figure 5–6)
posterior inferior cerebellar (See Figure 5–14)
long circumferential branches of basilar and superior
cerebellar (see Figure 5–21)
posterior and anterior inferior cerebellar and superior
cerebellar
Cerebellum and Basal Nuclei (Ganglia) 205
Spinocerebellar Tracts
Position of SCP
AMV
SCP
ASCT
MesNu
TriMoNu
ML
PSNu
Lobules II-IV
ASCT
on SCP
Lobules II-IV
Lobule V
Ant.
Lobe
Lobule
V
Recrossing ASCT
fibers in Cbl
CblNu
CblNu
RB
RB
FNL
Post. Lobe
CCblF
Lobule VIII
Lobule VIII
ACNu
RSCF
Somatotopy Position
PRG
Lamina VII
at C4-C8
VesNu
PSCT
RB
DNuC
SpTTr & Nu
ASCT
ALS + RuSp
ASCT
Intermediate zone (IZ)
and "spinal border"
cells (SBC)
Py
PRG
DNuC
PSCT
PSCT
T
L
ASCT
S
L
IZ
T
ASCT
SBC
206 Synopsis of Functional Components, Tracts, Pathways, and Systems
Pontocerebellar, Reticulocerebellar, Olivocerebellar, Ceruleocerebellar,
Hypothalamocerebellar, and Raphecerebellar Fibers
7–18 Afferent fibers to the cerebellum from selected brainstem areas and the organization of corticopontine fibers in the internal capsule
and crus cerebri as shown here. The cerebellar peduncles are also indicated. Pontocerebellar axons are mainly crossed, reticulocerebellar
fibers may be bilateral (from RetTegNu) or mainly uncrossed (from
LRNu and PRNu), and olivocerebellar fibers (OCblF) are exclusively
crossed. Raphecerebellar, hypothalamocerebellar, and ceruleocerebellar fibers are, to varying degrees, bilateral projections. Although all afferent fibers to the cerebellum give rise to collaterals to the cerebellar
nuclei, those from pontocerebellar axons are relatively small, having
comparatively small diameters. Olivocerebellar axons end as climbing
fibers, reticulocerebellar and pontocerebellar fibers as mossy fibers, and
hypothalamocerebellar and ceruleocerebellar axons end in all cortical
layers. These latter fibers have been called multilayered fibers in the literature because they branch in all layers of the cerebellar cortex.
Neurotransmitters: Glutamate (⫹) is found in corticopontine
projections and in most pontocerebellar fibers. Aspartate (⫹) and cor-
ticotropin (⫹)-releasing factor are present in many olivocerebellar
fibers. Ceruleocerebellar fibers contain noradrenalin, histamine is
found in hypothalamocerebellar fibers, and some reticulocerebellar
fibers contain enkephalin. Serotonergic fibers to the cerebellum arise
from neurons found in medial areas of the reticular formation (open
cell in Figure 7–18) and, most likely, from some cells in the adjacent
raphe nuclei.
Clinical Correlations: Common symptoms seen in patients with
lesions involving nuclei and tracts that project to the cerebellum are
ataxia (of trunk or limbs), an ataxic gait, dysarthria, dysphagia, and disorders of eye movement such as nystagmus. These deficits are seen in
some hereditary diseases (such as olivopontocerebellar degeneration, ataxia
telangiectasia, or hereditary cerebellar ataxia), in tumors (brainstem
gliomas), in vascular diseases (lateral pontine syndrome), or in other conditions such as alcoholic cerebellar degeneration or pontine hemorrhages
(see Figure 7-19 on page 208 for more information on cerebellar lesions).
Abbreviations
AntLb
CblNu
CerCblF
CPonF
CSp
DAO
FPon
Hyth
HythCblF
IC
LoCer
LRNu
MAO
MCP
ML
NuRa
OCblF
OPon
PCbIF
PostLb
Anterior limb of internal capsule
Cerebellar nuclei
Ceruleocerebellar fibers
Cerebropontine fibers
Corticospinal fibers
Dorsal accessory olivary nucleus
Frontopontine fibers
Hypothalamus
Hypothalamocerebellar fibers
Internal capsule
Nucleus (locus) ceruleus
Lateral reticular nucleus
Medial accessory olivary nucleus
Middle cerebellar peduncle
Medial lemniscus
Raphe nuclei
Olivocerebellar fibers
Occipitopontine fibers
Pontocerebellar fibers
Posterior limb of internal capsule
PonNu
PO
PPon
PRNu
Py
RB
RCblF
RetLenLb
RNu
RetTegNu
SCP
SubLenLb
SN
TPon
Pontine nuclei
Principal olivary nucleus
Parietopontine fibers
Paramedian reticular nuclei
Pyramid
Restiform body
Reticulocerebellar fibers
Retrolenticular limb of internal capsule
Red nucleus
Reticulotegmental nucleus
Superior cerebellar peduncle
Sublenticular limb of internal capsule
Substantia nigra
Temporopontine fibers
Number Key
1
2
3
Nucleus raphe, pontis
Nucleus raphe, magnus
Raphecerebellar fibers
Review of Blood Supply to Precerebellar Relay Nuclei in Pons
and Medulla, MCP, and RB
STRUCTURES
Pontine Tegmemtum
Basilar Pons
Medulla RetF and IO
MCP
RB
ARTERIES
long circumferential branches of basilar plus some from superior
cerebellar (see Figure 5–21)
paramedian and short circumferential branches of basilar (See
Figure 5–21)
branches of vertebral and posterior inferior cerebellar (see
Figure 5–14)
long circumferential branches of basilar and branches of anterior
inferior and superior cerebellar (see Figure 5–21)
posterior inferior cerebellar (see Figure 5–14)
Cerebellum and Basal Nuclei (Ganglia) 207
Pontocerebellar, Reticulocerebellar, Olivocerebellar, Ceruleocerebellar,
Hypothalamocerebellar, and Raphecerebellar Fibers
Position of Associated
Tracts and Nuclei
AntLb
(FPon)
PostLb
(PPon)
IC
Hyth
SubLenLb
(TPon)
RetLenLb
(OPon)
CPonF
HythCblF
LoCer
RetTegNu
ML
CerCblF
RNu
SCP
SN
MCP
PPon
OPon
TPon
1
PonNu
FPon
PCblF
NuRa
2
3
CblNu
RetTegNu
OCblF
MCP
RB
DAO
ML
CPonF
RCblF
PCblF
CSp
PO
PonNu
LRNu
PRNu
MAO
PRNu
RB
OCblF
LRNu
PO
Py
OCblF
PCblF
CerCblF
208 Synopsis of Functional Components, Tracts, Pathways, and Systems
Cerebellar Cortioconuclear, Nucleocortical, and Corticovestibular Fibers
7–19 Cerebellar corticonuclear fibers arise from all regions of the
cortex and terminate in an orderly (mediolateral and rostrocaudal) sequence in the ipsilateral cerebellar nuclei. For example, corticonuclear
fibers from the vermal cortex terminate in the fastigial nucleus, those
from the intermediate cortex terminate in the emboliform and globosus nuclei, and those from the lateral cortex terminate in the dentate nucleus. Also, cerebellar corticonuclear fibers from the anterior lobe typically terminate in more rostral regions of these nuclei while those from
the posterior lobe terminate more caudally. Cerebellar corticovestibular fibers originate primarily from the vermis and flocculonodular lobe,
exit the cerebellum via the juxtarestiform body, and end in the ipsilateral vestibular nuclei. These projections arise from Purkinje cells.
Nucleocortical processes originate from cerebellar nuclear neurons
and pass to the overlying cortex in a pattern that basically reciprocates
that of the corticonuclear projection; they end as mossy fibers. Some
nucleocortical fibers are collaterals of cerebellar efferent axons. The
cerebellar cortex may influence the activity of lower motor neurons
through, for example, the cerebellovestibular-vestibulospinal route.
Neurotransmitters: Gamma-aminobutyric acid (GABA) (⫺) is
found in Purkinje cells and is the principal transmitter substance present in cerebellar corticonuclear and corticovestibular projections.
However, taurine (⫺) and motilin (⫺) are also found in some Purkinje cells. GABA-ergic terminals are numerous in the cerebellar nuclei
and vestibular complex. Some of the glutamate-containing mossy
fibers in the cerebellar cortex represent the endings of nucleocortical
fibers that originate from cells in the cerebellar nuclei.
Clinical Correlations: Numerous disease entities can result in
cerebellar dysfunction including viral infections (echovirus), hereditary
diseases (see Figure 7–18), trauma, tumors (glioma, medulloblastoma),
occlusion of cerebellar arteries (cerebellar stroke), arteriovenous malformation of cerebellar vessels, developmental errors (such as the DandyWalker syndrome or the Arnold-Chiari deformity), or the intake of toxins.
Usually, damage to only the cortex results in little or no dysfunction
unless the lesion is quite large or causes an increase in intracranial pressure. However, lesions involving both the cortex and nuclei, or only
the nuclei, will produce obvious cerebellar signs.
Lesions involving midline structures (vermal cortex, fastigial nuclei) and/or the flocculonodular lobe result in truncal ataxia (titubation
or tremor), nystagmus, and head tilting. These patients may also have a
wide-based (cerebellar) gait, are unable to walk in tandem (heel to toe),
and may be unable to walk on their heels or on their toes. Generally,
midline lesions result in bilateral motor deficits affecting axial and
proximal limb musculature.
Damage to the intermediate and lateral cortices and the globose,
emboliform, and dentate nuclei results in various combinations of the
following deficits: dysarthria, dysmetria (hypometria, hypermetria), dysdiadochokinesia, tremor (static, kinetic, intention), rebound phenomenon, unsteady and wide-based (cerebellar) gait, and nystagmus. One of the more
commonly observed deficits in patients with cerebellar lesions is an intention tremor, which is best seen in the finger-nose test. The finger-to-finger test is also used to demonstrate an intention tremor and to assess
cerebellar function. The heel-to-shin test will show dysmetria in the lower
extremity. If the heel-to-shin test is normal in a patient with his/her
eyes open, the cerebellum is intact. If this test is repeated in the same
patient with eyes closed and is abnormal, this would suggest a lesion in
the posterior column-medial lemniscus system.
Cerebellar damage in intermittent and lateral areas (nuclei or cortex plus nuclei) causes movement disorders on the side of the lesion
with ataxia and gait problems on that side; the patient may tend to fall
toward the side of the lesion. This is because the cerebellar nuclei project to the contralateral thalamus, which projects to the motor cortex
on the same side, which projects to the contralateral side of the spinal
cord via the corticospinal tract. Other circuits (cerebellorubalrubospinal) and feedback loops (cerebelloolivary-olivocerebellar) follow similar routes. Consequently, the motor expression of unilateral
cerebellar damage is toward the lesioned side because of these doubly
crossed pathways.
Lesions of cerebellar efferent fibers, after they cross the midline in
the decussation of the superior cerebellar peduncle, will give rise to
motor deficits on the side of the body (excluding the head) contralateral to the lesion. This is seen in midbrain lesions such as the Claude syndrome.
Abbreviations
CorNu
CorVes
Flo
IC
InfVesNu
JRB
LC
LVesSp
LVNu
MLF
Corticonuclear fibers
Corticovestibular fibers
Flocculus
Intermediate cortex
Inferior (spinal) vestibular nucleus
Juxtarestiform body
Lateral cortex
Lateral vestibulospinal tract
Lateral vestibular nucleus
Medial longitudinal fasciculus
MVesSp
MVNU
NL, par
NM, par
NuCor
SVNu
VC
Medial vestibulospinal tract
Medial vestibular nucleus
Lateral cerebellar nucleus, parvocellular
region
Medial cerebellar nucleus,
parvocellular region
Nucleocortical fibers
Superior vestibular nucleus
Vermal cortex
Review of Blood Supply to Cerebellum and Vestibular Nuclei
STRUCTURES
Cerebellar Cortex
Cerebellar Nuclei
Vestibular Nuclei
ARTERIES
branches of posterior and anterior inferior cerebellar and superior
cerebellar
anterior inferior cerebellar and superior cerebellar
posterior inferior cerebellar in medulla, long circumferential
branches of basilar in pons
Cerebellum and Basal Nuclei (Ganglia) 209
Cerebellar Corticonuclear, Nucleocortical, and Corticovestibular Fibers
NuCor
VC
IC
CorNu
CorVes
NuCor
4
3
LC
2
CorNu
1
NM, par
Nodulus
NL, par
Flo
JRB
SVNu
LVNu
MLF
InfVNu
MVNu
LVesSp
MVesSp
Cerebellar Nuclei:
1= Medial (Fastigial)
2= Posterior Interposed (Globose)
3= Anterior Interposed (Emboliform)
4= Lateral (Dentate)
210 Synopsis of Functional Components, Tracts, Pathways, and Systems
Cerebellar Efferent Fibers
7–20 The origin, course, topography, and general distribution of
fibers arising in the cerebellar nuclei. Cerebellofugal fibers project to
several thalamic areas (VL and VA), to intralaminar relay nuclei in addition to the centromedian, and to a number of midbrain, pontine, and
medullary targets. Most of the latter nuclei project back to the cerebellum (e.g., reticulocerebellar, pontocerebellar), some in a highly organized manner. For example, cerebello-olivary fibers from the dentate nucleus (DNu) project to the principal olivary nucleus (PO), and
neurons of the PO send their axons back to the lateral cerebellar cortex, with collaterals going to the DNu.
The cerebellar nuclei can influence motor activity through, as examples, the following routes: 1) cerebellorubral-rubrospinal, 2) cere-
belloreticular-reticulospinal, 3) cerebellothalamic-thalamocorticalcorticospinal, and others. In addition, some direct cerebellospinal
fibers arise in the fastigial nucleus as well as in the interposed nuclei.
Neurotransmitters: Many cells in the cerebellar nuclei contain
glutamate (⫹), aspartate (⫹), or gamma-aminobutyric acid (⫺). Glutamate and aspartate are found in cerebellorubral and cerebellothalamic fibers, whereas some GABA-containing cells give rise to cerebellopontine and cerebello-olivary fibers. Some cerebelloreticular
projections may also contain GABA.
Clinical Correlations: Lesions of the cerebellar nuclei result in a
range of motor deficits depending on the location of the injury. Many
of these are described in Figure 7–19 on page 208.
Abbreviations
ALS
AMV
BP
CblOl
CblTh
CblRu
CC
CeGy
CM
CSp
DAO
DNu
ENu
EWNu
FNu
GNu
IC
InfVNu
INu
LRNu
LVNu
MAO
ML
MLF
MVNu
NuDark
Anterolateral system
Anterior medullary velum
Basilar pons
Cerebello-olivary fibers
Cerebellothalamic fibers
Cerebellorubral fibers
Crus cerebri
Central grey (periaqueductal grey)
Centromedian nucleus of thalamus
Corticospinal fibers
Dorsal accessory olivary nucleus
Dentate nucleus (lateral cerebellar nucleus)
Emboliform nucleus (anterior interposed
cerebellar nucleus)
Edinger-Westphal nucleus
Fastigial nucleus (medial cerebellar nucleus)
Globose nucleus (posterior interposed
cerebellar nucleus)
Inferior colliculus
Inferior (spinal) vestibular nucleus
Interstitial nucleus
Lateral reticular nucleus
Lateral vestibular nucleus
Medial accessory olivary nucleus
Medial lemniscus
Medial longitudinal fasciculus
Medial vestibular nucleus
Nucleus of Darkschewitsch
OcNu
PO
PonNu
RetForm
RNu
RuSp
SC
SCP
SCP, Dec
SN
SVNu
ThCor
ThFas
TriMoNu
VL
VPL
VSCT
ZI
Oculomotor nucleus
Principal olivary nucleus
Pontine nuclei
Reticular formation
Red nucleus
Rubrospinal tract
Superior colliculus
Superior cerebellar peduncle
Superior cerebellar peduncle, decussation
Substantia nigra
Superior vestibular nucleus
Thalamocortical fibers
Thalamic fasciculus
Trigeminal motor nucleus
Ventral lateral nucleus of thalamus
Ventral posterolateral nucleus of thalamus
Ventral spinocerebellar tract
Zona incerta
Number Key
1
2
3
4
5
Ascending projections to superior
colliculus, and possibly ventral lateral and
ventromedial thalamic nuclei
Descending crossed fibers from superior
cerebellar peduncle
Uncinate fasciculus (of Russell)
Juxtarestiform body to vestibular nuclei
Reticular formation
Review of Blood Supply to Cerebellar Nuclei and
Their Principal Efferent Pathways
STRUCTURES
Cerebellar Nuclei
SCP
Midbrain Tegmemtum
(RNu, CblTh,
CblRu, OcNu)
VPL, CM, VL, VA
IC
ARTERIES
anterior inferior cerebellar and superior cerebellar
long circumferential branches of basilar and superior cerebellar
(see Figure 5–21)
paramedian branches of basilar bifurcation, short circumferential
branches of posterior cerebral, branches of superior cerebellar
(see Figure 5–27)
thalamogeniculate branches of posterior cerebral, thalamoperforating branches of the posteromedial group of posterior
cerebral (see Figure 5–38)
lateral striate branches of middle cerebral (see Figure 5–38)
Cerebellum and Basal Nuclei (Ganglia) 211
Cerebellar Efferent Fibers
CSp
VL
ThCor
CM
VPL
ThFas
Zl
NuDark, INu,
OcNu, EWNu
Position of SCP,
CblTh, and CblRu
RNu
SC
CeGy
CeGy
1
SCP
ML
2
3
4
RetForm
CblTh &
CblRu
RNu
SN
CC
PonNu
DNu
IC
SVNu
FNu
MLF
5
ENu
GNu
LVNu
ML
SN
CblOl
5
SCP, Dec
InfVNu
LRNu
5
MVNu
VSCT
DAO
5
PO
MAO
AMV
SCP
TriMoNu
ALS & RuSp
ML
BP
Cerebellospinal
fibers
212 Synopsis of Functional Components, Tracts, Pathways, and Systems
7–21 Blank master drawing for pathways projecting to the cerebellar cortex, and for efferent projections of cerebellar nuclei. This illustration is provided for self-evaluation of understanding of pathways
to the cerebellar cortex and from the cerebellar nuclei, for the instructor to expand on cerebellar afferent/efferent pathways not covered in the atlas, or both.
Cerebellum and Basal Nuclei (Ganglia) 213
214 Synopsis of Functional Components, Tracts, Pathways, and Systems
Striatal Connections
7–22 The origin, course, and distribution of afferent fibers to, and
efferent projections from, the neostriatum. These projections are extensive, complex, and in large part, topographically organized; only
their general patterns are summarized here. Afferents to the caudate
and putamen originate from the cerebral cortex (corticostriate fibers),
from several of the intralaminar thalamic nuclei (thalamostriate), from
the substantia nigra-pars compacta (nigrostriate), and from some of the
raphe nuclei. Neostriatal cells send axons into the globus pallidus (paleostriatum) as striopallidal fibers and into the substantia nigra pars
reticulata as a strionigral projection.
Neurotransmitters: Glutamate (⫹) is found in corticostriate
fibers, and serotonin is found in raphestriatal fibers from the nucleus
raphe dorsalis. Four neuroactive substances are associated with striatal
efferent fibers, these being gamma-aminobutyric acid (GABA)(⫺),
dynorphin, enkephalin(⫺), and substance P(⫹). Enkephalinergic and
GABA-ergic striopallidal projections are numerous to the lateral pallidum (origin of pallidosubthalamic fibers), while dynorphin-containing terminals are more concentrated in its medial segment (source of
pallidothalamic fibers). Enkephalin and GABA are also present in strionigral projections to the pars reticulata. Because substance P and
GABA are found in striopallidal and strionigral fibers, some of the former may be collaterals of the latter. Dopamine is present in nigrostriatal projection neurons and in their terminals in the neostriatum.
Clinical Correlations: Degenerative changes and neuron loss in
the caudate nucleus and putamen result in movement disorders. Examples are seen in Sydenham chorea (rheumatic chorea), Huntington disease
(a dominantly inherited disease), and Wilson disease (a genetic error in
the patient’s ability to metabolize copper). In persons with Parkinson
disease, a loss of the dopamine-containing cells in the pars compacta of
the substantia nigra and of their nigrostriatal terminals in the caudate
nucleus and putamen occurs.
Sydenham chorea is a disease usually seen in children between 5 and
15 years of age, resulting from infection with hemolytic streptococcus.
The choreiform movements are brisk and flowing, irregular, and may involve muscles of the limbs, face, oral cavity, and trunk. Dystonia may
be seen; muscle weakness is common. In most patients, the disease resolves following successful treatment of the infection.
Huntington disease is a progressive genetic disorder the symptoms of
which appear at 35 to 45 years of age. There is loss of GABA-ergic and
enkephalinergic cells in the neostriatum (primarily the caudate) and
cell loss in the cerebral cortex. Loss of neostriatal cell terminals in the
lateral and medial segments of the globus pallidus correlate, respectively, with the development of choreiform movements and later with
rigidity and dystonia. Loss of cortical neurons correlate, respectively,
with personality changes and eventual dementia. Huntington chorea is
rapid, unpredictable, and may affect muscles of the extremities, face,
and trunk; abnormal movements seem to flow through the body. Patients commonly attempt to mask the abnormal movement by trying
to make it appear to be part of an intended movement (parakinesia).
Symptoms in Wilson disease (hepatolenticular degeneration) appear in
persons between 10 to 20 years of age. Copper accumulates in the basal
nuclei (ganglia) and the frontal cortex, with resultant spongy degeneration in the putamen and cortex. These patients may show athetoid
movements, rigidity and spasticity, dysarthria, dysphagia, contractures, and
tremor. A unique movement of the hand and/or upper extremity in
these patients is called a flapping tremor (asterixis) sometimes described
as a wing-beating tremor. Copper can also be seen in the cornea
(Kayser-Fleischer ring) in these patients.
In Parkinson disease (onset at 50 to 60 years of age), there is a progressive loss of dopaminergic cells in the substantia nigra-pars compacta, of their terminals in the caudate and putamen, and of their dendrites that extend into the substantia nigra-pars reticulata. Patients
with Parkinson disease characteristically show a resting tremor (pillrolling), rigidity (cog wheel or lead pipe), and bradykinesia or hypokinesia.
The slowness of movement may also be expressed in speech (dysarthria,
hypophonia, tachyphonia) and in writing (micrographia). These patients
have a distinct stooped flexed posture and a festinating gait. Behavioral
changes are also seen. Parkinson disease and Huntington disease are
progressive neurodegenerative disorders.
Dystonia, a movement disorder seen in some patients with basal nuclei disease, is characterized by increased/sustained muscle contractions that cause twisting of the trunk or extremities resulting in abnormal posture. These patients may also have unusual and repetitive
movements of the extremities or of the neck (cervical dystonia or spasmodic torticollis). Dystonia may be an inherited progressive disease or
have other causes and may be seen in children or young adults. The
symptoms may initially appear during movements or when talking but
in later stages may be present at rest.
Abbreviations
CaNu
CorSt
GPL
GPM
NigSt
Put
RaNu
RaSt
Caudate nucleus
Corticostriate fibers
Globus pallidus, lateral segment
Globus pallidus, medial segment
Nigrostriatal fibers
Putamen
Raphe nuclei
Raphestriatal fibers
SNpc
SNpr
StNig
StPal
SThNu
ThSt
ZI
Substantia nigra, pars compacta
Substantia nigra, pars reticulata
Striatonigral fibers
Striatopallidal fibers
Subthalamic nucleus
Thalamostriatal fibers
Zona incerta
Review of Blood Supply to Caudate, Putamen, SN, CC, and IC
STRUCTURES
Caudate, Putamen and IC
SN and CC
ARTERIES
medial striate a. for head of caudate and lateral striate
branches of middle cerebral for Put and IC (see Figure
5–38)
paramedian branches of basilar bifurcation, short
circumferential branches of posterior cerebral and some
from superior cerebellar (see Figure 5–27)
Cerebellum and Basal Nuclei (Ganglia) 215
Striatal Connections
Cerebral cortex
CorSt
CorSt
Ca,Nu
StPal
ThSt
Intralaminar
nuclei
Put
StPal
NigSt
CorSt
Zl
SThNu
SNpc
NigSt
StNig
SNpr
RaSt
RaNu
GPL
GPM
216 Synopsis of Functional Components, Tracts, Pathways, and Systems
Pallidal Efferents and Nigral Connections
7–23 The origin, course, and distribution of efferent projections
of the globus pallidus (upper illustration), and connections of the substantia nigra (lower drawing) that were not shown in relation to the
pallidum or in Figure 7–22 on page 215. The ansa lenticularis (dashed
line) arches around the internal capsule and passes caudally to join in
the formation of the thalamic fasciculus. Pallidosubthalamic fibers originate primarily from the lateral pallidal segment, but pallidothalamic
projections, via the ansa lenticularis and lenticular fasciculus, arise
mainly from its medial segment. The substantia nigra has extensive
connections, the clinically most important being the dopaminergic nigrostriatal fibers. The globus pallidus influences motor activity by way
of pallidothalamic-thalamocortical-corticospinal (and corticonuclear
[corticobulbar]) pathways.
Neurotransmitters: Gamma-aminobutyric acid (⫺)-containing
cells in the globus pallidus give rise to pallidonigral projections, which
end primarily in the substantia nigra-pars reticulata. Although GABA
is also found in some subthalamopallidal axons, this latter projection
contains many glutaminergic (⫹) fibers. Dopamine-containing, GABA
(⫺)-containing, and glycine (⫺)-containing cells are present in the
substantia nigra. Of these, dopamine is found in pars compacta neu-
rons, which give rise to nigrostriatal, nigroamygdaloid, and several
other projections; GABA in pars reticulata cells, which give rise to nigrocollicular and nigrothalamic fibers; and glycine in some local circuit
nigral neurons. Glutamate (⫹) is found in corticonigral fibers, and
serotonin (⫺) is associated with raphenigral fibers; these latter fibers
originate primarily from the nucleus raphe dorsalis.
The dopaminergic projections to the frontal cortex, shown here as
arising only from SNpc, originates from this cell group as well as from
the immediately adjacent ventral tegmental area. Excessive activity in
neurons comprising this projection may play a partial role in schizophrenia.
Clinical Correlation: Movement disorders associated with lesions in the neostriatum and substantia nigra are reviewed in Figure
7–22 on page 214. Hemorrhage into, the occlusion of vessels serving
or a tumor within, the subthalamic nucleus will result in violent flailing movements of the extremities, a condition called hemiballismus.
Hemiballistic movements are seen contralateral to the lesion because
the motor expression of this lesion is through the corticospinal tract.
Lesions confined to the globus pallidus, as in hemorrhage of lenticulostriate arteries, may result in hypokinesia and rigidity without tremor.
Abbreviations
AmyNig
AmyNu
AnLent
CaNu
CM
CorNig
CSp
GPL
GPM
LenFas
NigAmy
NigCol
NigTec
NigSTh
NigTh
PalNig
Amygdalonigral fibers
Amygdaloid nucleus (complex)
Ansa lenticularis
Caudate nucleus
Centromedian nucleus of thalamus
Corticonigral fibers
Corticospinal fibers
Globus pallidus, lateral segment
Globus pallidus, medial segment
Lenticular fasciculus (H2)
Nigroamygdaloid fibers
Nigrocollicular fibers
Nigrotectal fibers
Nigrosubthalamic fibers
Nigrothalamic fibers
Pallidonigral fibers
PedPonNu
Put
RaNu
SC
SNpc
SNpr
SThFas
SThNig
SthNu
ThCor
ThFas
VA
VL
VM
ZI
Pedunculopontine nucleus
Putamen
Raphe nuclei
Superior colliculus
Substantia nigra, pars compacta
Substantia nigra, pars reticulata
Subthalamic fasciculus
Subthalamonigral fibers
Subthalamic nucleus
Thalamocortical fibers
Thalamic fasciculus (H1)
Ventral anterior nucleus of thalamus
Ventral lateral nucleus of thalamus
Ventromedial nucleus of thalamus
Zona incerta
Review of Blood Supply to Pallidum, Subthalamic Area, and SN
STRUCTURES
ARTERIES
GPM/GPL lateral striate branches of middle cerebral and branches of anterior choroidal
(see Figure 5–38)
SThNu
posteromedial branches of posterior cerebral and posterior communicating
(see Figure 5–38)
SN
branches of basilar bifurcation, medial branches of posterior cerebral and
posterior communicating, short circumferential branches of posterior
cerebral (see Figure 5–27)
Cerebellum and Basal Nuclei (Ganglia) 217
Pallidal Efferents and Nigral Connections
Motor cortex
ThCor
CaNu
Intralaminar
nuclei
VA
VL
Put
ThFas
CSp
LenFas
GPL
Zl
Forel's Field H
SThNu
GPM
SThNig and
Nig Sth
SThFas
AnLent
PalNig
SNpr
SNpc
PedPonNu
CorNig
Nigral efferents
to frontal cortex
Ca,Nu
VM
VL
Put
NigTh
GPL
Zl
GPM
SthNu
Nigral efferents to
olfactory tubercle
and bed nucleus of
the stria terminalis
SC
SNpc
AmyNu
NigCol
NigAmy and AmyNig
SNpr
RaNu
218 Synopsis of Functional Components, Tracts, Pathways, and Systems
7–24 Blank master drawing for connections of the basal ganglia.
This illustration is provided for self-evaluation of understanding of
basal ganglia connections, for the instructor to expand on basal nuclei
pathways not covered in this atlas, or both.
Cerebellum and Basal Nuclei (Ganglia) 219
220 Synopsis of Functional Components, Tracts, Pathways, and Systems
Pupillary Pathways
7–25 The origin, course, and distribution of fibers involved in the
pathway for the pupillary light reflex. In addition, the pathway for sympathetic innervation of the dilator muscle of the iris is also depicted. The
pathway from the midbrain reticular formation to the intermediolateral
cell column may also have a multisynaptic component, with relay stations in the pontine and medullary reticular formations. Postganglionic
sympathetic fibers to the head originate from the superior cervical ganglion. Although not shown, descending projections to the intermediolateral cell column also originate from various hypothalamic areas and
nuclei (hypothalamospinal fibers), some of which receive retinal input.
Neurotransmitters: Acetylcholine is the transmitter found in
the preganglionic and postganglionic autonomic fibers shown in this illustration. In addition, N-acetylaspartylglutamate is present in some
retinal ganglion cells (retinogeniculate projections).
Clinical Correlations: Total or partial blindness in one or both
eyes may result from a variety of causes (such as gliomas, meningiomas,
strokes, aneurysms, infections, and demyelinating diseases); lesions may
occur at any locus along the visual pathway. A complete lesion (for example, a transection) of the optic nerve will result in blindness and loss
of the pupillary light reflex (direct response) in the eye on the injured
side and a loss of the pupillary light reflex (consensual response) in the
opposite eye when shining a light in the blind eye. On the other hand, shining a light in the normal eye will result in a pupillary light reflex (direct
response) in that eye and a consensual response in the blind eye. A pituitary adenoma may damage the crossing fibers in the optic chiasm (pro-
ducing a bitemporal hemianopsia) or the uncrossed fibers in the right (or
left) side of the optic chiasm. These lateral lesions produce a right (or
left) nasal hemianopsia.
Optic (geniculocalcarine) radiations (see Figures 7–26 and 7–27 on
pages 222 and 223) may pass directly caudal to the upper lip (cuneus)
of the calcarine sulcus or follow an arching route (the Meyer, or MeyerArchambault loop) through the temporal lobe to the lower bank (lingual
gyrus) of the calcarine sulcus. Temporal lobe lesions involving the
Meyer-Archambault loop, or involving fibers entering the lingual
gyrus, can produce a homonymous superior quadrantanopia. A homonymous
inferior quandrantanopia is seen in patients with damage to upper (parietal) parts of the geniculocalcarine radiations or to these fibers as they
enter the cuneus.
Damage to the visual cortex adjacent to the calcarine sulcus (distal
posterior cerebral artery occlusion) results in a right (or left) homonymous hemianopsia. With the exception of macular sparing, this deficit is
the same as that seen in optic tract lesions.
Vascular lesions (as in the lateral medullary syndrome), tumors (such
as brainstem gliomas), or syringobulbia may interrupt the descending projections from hypothalamus (hypothalamospinal fibers) and midbrain
to the intermediolateral cell column at upper thoracic levels. This may
result in a Horner syndrome (ptosis, miosis, and anhidrosis) on the ipsilateral side. The enophthalmos (a slight sinking of the eyeball into the orbit) frequently mentioned in relation to Horner syndrome is not really
very apparent in afflicted patients.
Abbreviations
CC
CilGang
EWNu
ILCC
LGNu
MGNu
ML
OcNr
OpCh
OpNr
OpTr
Crus cerebri
Ciliary ganglion
Edinger-Westphal nucleus
Intermediolateral cell column
Lateral geniculate nucleus
Medial geniculate nucleus
Medial lemniscus
Oculomotor nerve
Optic chiasm
Optic nerve
Optic tract
PoCom
PrTecNu
PulNu
RetF
RNu
SC
SC,Br
SCerGang
SN
WRCom
Posterior commissure
Pretectal nucleus
Pulvinar nuclear complex
Reticular formation
Red nucleus
Superior colliculus
Superior colliculus, brachium
Superior cervical ganglion
Substantia nigra
White ramus communicans
Review of Blood Supply to OpTr, MGB, LGB, SC,
and Midbrain Tegementum, Including PrTecNu
STRUCTURES
OpTr
MGNu, LGNu
SC and PrTecNu
Midbrain
Tegmentum
ARTERIES
anterior choroidal (see Figure 5–38)
thalamogeniculate branches of posterior cerebral (see Figure 5–38)
long circumferential branches (quadrigeminal) of posterior cerebral,
posterior choroidal, and some from superior cerebellar (to SC) (see
Figures 5–27 and 5–38)
paramedian branches of basilar bifurcation, medial branches
of posterior cerebral and posterior communicating, short
circumferential branches of posterior cerebral (see Figure 5–27)
Optic, Auditory, and Vestibular Systems 221
Pupillary Pathways
Dilator muscles of iris
Sphincter mus. of iris
Sphincter mus. of ciliary body
Ganglion cells
of retina
CilGang
OpNr
OcNr
OpCh
OpTr
OcNr
Midbrain
RetF
Via blood
vessels
CC
RNu
ML
LGNu
SN
LGNu
MGNu
SC,Br
MGNu
SC
SC,Br
PulNu
PoCom
PrTecNu
EWNu
PulNu
SCerGang
HySpF
Spinal nerve
ILCC
Thoracic cord
T1-T3
WRCom
Anterior root
222 Synopsis of Functional Components, Tracts, Pathways, and Systems
Visual Pathways
Visual fields
Ganglion cells
of retina
Retinae
LGNu Laminae
Optic nerve
mc
pc
1
2
3
4
5
6
Optic chiasm
Optic tract
Red nucleus
Substantia nigra
Crus cerebri
Medial lemniscus
Meyer's Loop
LGNu
MGNu
MGNu
SC,Br
SC,Br
PulNu
PulNu
Oculomotor Nu.
Edinger-Westphal Nu.
Sup. colliculus
Pretectal Nu.
Optic radiations
(in retrolenticular
limb of internal
capsule)
Cuneus
Lingual gyrus
CalSul
7–26 The origin, course, and distribution of the visual pathway are
shown. Uncrossed retinogeniculate fibers terminate in laminae 2, 3,
and 5, while crossed fibers end in laminae 1, 4, and 6. Geniculocalcarine fibers arise from laminae 3 through 6. Retinogeniculate and
geniculocalcarine pathways are retinotopically organized (see facing
page).
Neurotransmitters: Cholecystokinin (⫹) is present in some
geniculocalcarine fibers. N-acetylaspartylglutamate is found in some
retinogeniculate fibers, and in some lateral geniculate and visual cortex neurons.
Clinical Correlations: Deficits seen following lesions of various
parts of visual pathways are described in Figure 7-25 on p. 220.
Abbreviations
CalSul
LGNu
mc
pc
Calcarine sulcus
Lateral geniculate nucleus
Magnocellular
Parvocellular
MGNu
PulNu
SC,Br
Medial geniculate nucleus
Pulvinar nuclear complex
Superior colliculus, brachium
Optic, Auditory, and Vestibular Systems 223
Visual Pathways
LEFT
RIGHT
B'
A'
B
Visual fields
overlapped for
both eyes
A
Orientation fo all Levels
Except Visual Cortex
M
C'
D'
D
C
Dor
Dor
Lat
B
B
A
Visual fields for
individualoeyes
M
Lat
Ven
Ven
A'
B'
Med
A
M
C
C
D
D
D'
C'
D'
C'
D
D
C
Retinae
M
A
M
A
B
Optic nerves
C
D'
C'
D
M
A
B
A'
B'
D
C
C
M
B
A
B'
B
A'
Optic chiasm
B
A
Optic
tracts
M
A'
M
C
D
D'
M
A
D
Lateral
geniculate
nuclei
C
D'
M
B
C'
A'
D'
B'
C'
B'
C'
C
D
M
M
Optic radiations
A
B
A'
B'
Primary visual cortex
D'
M
D
M
A
C'
Cuneus
Calcarine sulcus
A'
Lingual gyrus
7–27 Semidiagrammatic representation of the retinographic
arrangement of visual and retinal fields, and the subsequent topography of these projections throughout the visual system. Upper-case letters identify the binocular visual fields (A, B, C, D), the macula (M),
and the monocular visual fields (A⬘, B⬘, C⬘, D⬘).
C
M
B
M
B'
Clinical Correlations: Deficits seen following lesions of various
parts of the visual pathway are described in Figure 7-25 on p. 220.
224 Synopsis of Functional Components, Tracts, Pathways, and Systems
7–28 Blank master drawing of visual pathways. This illustration is
provided for self-evaluation of visual pathway understanding, for the
instructor to expand on aspects of the visual pathways not covered in
the atlas, or both.
Optic, Auditory, and Vestibular Systems 225
226 Synopsis of Functional Components, Tracts, Pathways, and Systems
Auditory Pathways
7–29 The origin, course, and distribution of the fibers collectively
composing the auditory pathway. Central to the cochlear nerve and
dorsal and ventral cochlear nuclei this system is, in a general sense, bilateral and multisynaptic, as input is relayed through brainstem nuclei
en route to the auditory cortex. Synapse and crossing (or re-crossing)
of information can occur at several levels in the neuraxis. Consequently, central lesions rarely result in a total unilateral hearing loss.
The medial geniculate body is the thalamic station for the relay of auditory information to the temporal cortex.
Neurotransmitters: Glutamate (⫹) and aspartate (⫹) are found in
some spiral ganglion cells and in their central terminations in the cochlear
nuclei. Dynorphin-containing and histamine-containing fibers are also
present in the cochlear nuclei; the latter arises from the hypothalamus. A
noradrenergic projection to the cochlear nuclei and to the inferior colliculus originates from the nucleus locus ceruleus. Cells in the superior
olive that contain cholecystokinin and cells in the nuclei of the lateral lemniscus that contain dynorphin project to the inferior colliculus. Although
the olivocochlear bundle is not shown, it is noteworthy that enkephalin is
found in some of the cells that contribute to this projection.
Clinical Correlations: There are three categories of deafness.
Conductive deafness is due to problems of the external ear (obstruction
of the canal, wax build-up) or disorders of the middle ear (otitis media,
otosclerosis). Nerve deafness (sensorineural hearing loss) results from diseases
involving the cochlea or the cochlear portion of the vestibulocochlear
nerve. Central deafness results from damage to the cochlear nuclei or
possibly their central connections.
Hearing loss may result from trauma (such as fracture of the petrous
bone), demyelinating diseases, tumors, certain medications (streptomycin), or occlusion of the labyrinthine artery. Damage to the cochlear
part of the VIIIth nerve (as in vestibular schwannoma) results in tinnitus
and/or deafness (partial or total) in the ipsilateral ear. High-frequency
hearing losses are most common.
The Weber test and Rinne test are used to differentiate between neural
hearing loss and conduction hearing loss, and to lateralize the deficit. In
the Weber test, a tuning fork (512 Hz) is applied to the midline of the forehead or apex of the skull. In the normal patient, the sound (conducted
through the bones of the skull) is heard the same in each year. In the case
of nerve deafness (lesions of the cochlea or cochlear nerve), the sound is best
heard in the normal ear, while in conductive deafness, the sound is best heard
in the abnormal ear. In the Rinne test, a tuning fork (512 Hz) is placed
against the mastoid process. When the sound is no longer perceived, the
prongs are moved close to the external acoustic meatus, where the sound
is again heard; this is the situation in a normal individual (positive Rinne
test). In middle ear disease, the sound is not heard at the external meatus
after it has disappeared from touching the mastoid bone (abnormal or negative Rinne test). Therefore, a negative Rinne test signifies conductive
hearing loss in the ear tested. In mild nerve deafness (cochlea or cochlear
nerve lesions), the sound is heard by application of the tuning fork to the
mastoid and movement to the ear (the Rinne test is positive). In severe
nerve deafness, the sound may not be heard at either position.
In addition to hearing loss and tinnitus, large vestibular schwannomas
may result in other signs and symptoms. These include nausea, vomiting
and ataxia/unsteady gait (vestibular root involvement), weakness of facial muscles (facial root involvement), and altered sensation from the
face and a diminished corneal reflex (trigeminal root involvement).
There may also be general signs associated with increased intracranial
pressure (lethargy, headache, and vomiting).
Central lesions (as in gliomas or vascular occlusions) rarely produce
unilateral or bilateral hearing losses that can be detected, the possible
exception being pontine lesions that damage the trapezoid body and
nuclei. Injury to central auditory pathways and/or primary auditory
cortex may diminish auditory acuity, decrease the ability to hear certain tones, or make it difficult to precisely localize sounds in space. Patients with damage to secondary auditory cortex in the temporal lobe
experience difficulty in understanding and/or interpreting sounds (auditory agnosia).
Abbreviations
AbdNu
ACNu
ALS
CC
FacNu
IC
IC,Br
IC,Com
IC,SL
LGNu
LL
LL,Nu
MGNu
ML
Abducens nucleus
Anterior (ventral) cochlear nucleus
Anterolateral system
Crus cerebri
Facial nucleus
Inferior colliculus
Inferior colliculus, brachium
Inferior colliculus, commissure
Internal capsule, sublenticular limb
Lateral geniculate nucleus
Lateral lemniscus
Lateral lemniscus, nucleus
Medial geniculate nucleus
Medial lemniscus
MLF
PCNu
PulNu
RB
RetF
SC
SCP,Dec
SO
SpGang
SpTTr
TrapB
TrapNu
TTGy
Medial longitudinal fasciculus
Posterior (dorsal) cochlear nucleus
Pulvinar nuclear complex
Restiform body
Reticular formation
Superior colliculus
Superior cerebellar peduncle, decussation
Superior olive
Spiral ganglion
Spinal trigeminal tract
Trapezoid body
Trapezoid nucleus
Transverse temporal gyrus
Review of Blood Supply to Cochlear Nuclei, LL (and associated
structures), Pontine Tegmentum, IC, and MGB
STRUCTURES
Cochlear Nuclei
LL, SO in Pons
IC
MGB
ARTERIES
anterior inferior cerebellar (see Figure 5–14)
long circumferential branches of basilar (see Figure 5–21)
long circumferential branches (quadrigeminal branches) of basilar,
superior cerebellar (see Figure 5–27)
thalamogeniculate branches of posterior cerebral (see Figure 5–38)
Optic, Auditory, and Vestibular Systems 227
Auditory Pathways
PulNu
LGNu
MGNu
TTGY
Positions of LL and
Related Structures
IC,SL
IC,Br
IC,Com
SC
IC
IC,Com
LL
ALS
IC
LL,Nu
CC
ML
SCP,Dec
FacNu
SpTTr
LL
LL
FacNu
LL
SO
RetF
ALS
PCNu
SO
ML
TrapNu
TrapB
RetF
ACNu
SpGang
PCNu
ACNu
Hair cells in
organ of corti
LL
RB
SO
LL
ML
TrapB
228 Synopsis of Functional Components, Tracts, Pathways, and Systems
Vestibular Pathways
7–30 The origin, course, and distribution of the main afferent and
efferent connections of the vestibular nuclei (see also Figures 7–13,
7–19, and 7–20). Primary vestibular afferent fibers may end in the
vestibular nuclei or pass to cerebellar structures via the juxtarestiform
body. Secondary vestibulocerebellar axons originate from the vestibular nuclei and follow a similar path to the cerebellum. Efferent projections from the vestibular nuclei also course to the spinal cord through
vestibulospinal tracts (see Figure 7–13), as well as to the motor nuclei
of the oculomotor, trochlear, and abducens nerves via the MLF. Cerebellar structures most extensively interconnected with the vestibular
nuclei include the lateral regions of the vermal cortex of anterior and
posterior lobes, the flocculonodular lobe, and the fastigial (medial)
cerebellar nucleus.
Neurotransmitters: Gamma-aminobutyric (⫺) is the transmitter associated with many cerebellar corticovestibular fibers and their
terminals in the vestibular complex; this substance is also seen in cerebellar corticonuclear axons. The medial vestibular nucleus also has
fibers that are dynorphin-positive and histamine-positive; the latter
arise from cells in the hypothalamus.
Clinical Correlations: The vestibular part of the VIIIth nerve can
be damaged by many of the same insults that affect the cochlear nerve
(see Figure 7–29). Damage to vestibular receptors of the vestibular nerve
commonly results in vertigo. The patient may feel that his or her body is
moving (subjective vertigo) or that objects in the environment are moving
(objective vertigo). They have equilibrium problems, an unsteady (ataxic)
gait, and a tendency to fall to the lesioned side. Deficits seen in nerve lesions—or in brainstem lesions involving the vestibular nuclei, include
nystagmus, nausea, and vomiting, along with vertigo and gait problems. A
facial palsy may also appear in concert with VIIIth nerve damage in patients who have a vestibular schwannoma. These vestibular deficits, along
with partial or complete deafness, are seen in Ménière disease.
Lesions of those parts of the cerebellum with which the vestibular
nerve and nuclei are most intimately connected (flocculonodular lobe and
fastigial nucleus) result in nystagmus, truncal ataxia, ataxic gait, and a
propensity to fall to the injured side. The nystagmus seen in patients with
vestibular lesions and the internuclear ophthalmoplegia seen in some patients
with multiple sclerosis are signs that correlate with the interruption of
vestibular projections to the motor nuclei of III, IV, and VI via the MLF.
Abbreviations
AbdNu
ALS
Cbl
Cbl-CoVes
CblNu
HyNu
IC
InfVNu
JRB
LVesSp
LVNu
MesNu
ML
MLF
MVesSp
MVNu
OcNu
Abducens nucleus
Anterolateral system
Cerebellar
Cerebellar corticovestibular fibers
Cerebellar nuclei
Hypoglossal nucleus
Inferior colliculus
Inferior (spinal) vestibular nucleus
Juxtarestiform body
Lateral vestibulospinal tract
Lateral vestibular nucleus
Mesencephalic nucleus
Medial lemniscus
Medial longitudinal fasciculus
Medial vestibulospinal tract
Medial vestibular nucleus
Oculomotor nucleus
PAG
Py
RB
RNu
SC
SCP,Dec
SN
SolNu
SolTr
SpTTr
SVNu
TroNu
VesGang
VesCbl,Prim
VesCbl,Sec
Periaqueductal gray
Pyramid
Restiform body
Red nucleus
Superior colliculus
Superior cerebellar peduncle,
decussation
Substantia nigra
Solitary nucleus
Solitary tract
Spinal trigeminal tract
Superior vestibular nucleus
Trochlear nucleus
Vestibular ganglion
Vestibulocerebellar fibers, primary
Vestibulocerebellar fibers, secondary
Review of Blood Supply to Vestibular Nuclei, TroNu, and OcNu
STRUCTURES
Vestibular Nuclei
TroNu and OcNu
ARTERIES
posterior inferior cerebellar in medulla (see Figure 5–14), long
circumferential branches of basilar in pons (see Figure 5–21)
paramedian branches of basilar bifurcation, medial branches of
posterior cerebral and posterior communicating, short
circumferential branches of posterior cerebral (see Figure 5–27)
Optic, Auditory, and Vestibular Systems 229
Vestibular Pathways
Position of Vestibular Nuclei,
MLF, and Related Structures
SC
OcNu
PAG
MesNu
ML
RNu
SN
IC
OcNu
ALS
TroNu
TroNu
MLF
ML
SCP,Dec
MLF
CblNu
AbdNu
Cbl-CoVes
SVNu
JRB
JRB
SVNu
LVNu
AbdNu
MLF
VesCbl, Sec
SpTTr
LVNu
Cbl cortex
ALS
VesCbl, Prim
ML
VesGang
Crista ampullaris
Macula utriculi
Macula sacculi
MVNu
HyNu
InfVNu
MVNu and
InfVNu
RB
MLF
SpTTr
MVesSp in MLF
ML
Py
LVesSp
SolTr and Nu
230 Synopsis of Functional Components, Tracts, Pathways, and Systems
7–31 Blank master drawing for auditory or vestibular pathway.
This illustration is provided for self-evaluation of auditory or vestibular pathway understanding, for the instructor to expand on aspects of
these pathways not covered in the atlas, or both.
Optic, Auditory, and Vestibular Systems 231
232 Synopsis of Functional Components, Tracts, Pathways, and Systems
Hippocampal Connections
7–32 Selected afferent and efferent connections of the hippocampus (upper) and the mammillary body (lower) with emphasis on the
circuit of Papez. The hippocampus receives input from, and projects
to, diencephalic nuclei (especially the mammillary body via the postcommissural fornix), the septal region, and amygdala. The hippocampus receives cortical input from the superior and middle frontal gyri,
superior temporal and cingulate gyri, precuneus, lateral occipital cortex, occipitotemporal gyri, and subcallosal cortical areas. The mammillary body is connected with the dorsal and ventral tegmental nuclei,
anterior thalamic nucleus (via the mammillothalamic tract), septal nuclei, and through the mammillotegmental tract, to the tegmental pontine and reticulotegmental nuclei.
Neurotransmitters: Glutamate (⫹)-containing cells in the
subiculum and Ammon’s horn project to the mammillary body, other
hypothalamic centers, and the lateral septal nucleus through the fornix.
Cholecystokinin (⫹) and somatostatin (⫺) are also found in hippocampal cells that project to septal nuclei and hypothalamic structures. The septal nuclei and the nucleus of the diagonal band give rise
to cholinergic afferents to the hippocampus that travel in the fornix. In
addition, a gamma-aminobutyric acid (⫺) septohippocampal projection originates from the medial septal nucleus. Enkephalin and glutamate containing hippocampal afferent fibers arise from the adjacent entorhinal cortex; the locus ceruleus gives origin to noradrenergic fibers
to the dentate gyrus, Ammon’s horn, and subiculum; and serotoninergic fibers arise from the rostral raphe nuclei.
Clinical Correlations: Dysfunction associated with damage to
the hippocampus is seen in patients with trauma to the temporal lobe,
as a sequel to alcoholism, and as a result of neurodegenerative changes
seen in the dementing diseases (such as Alzheimer disease and Pick disease).
Bilateral injury to the hippocampus results in loss of recent memory
(remote memory is unaffected), impaired ability to remember recent
(new) events, and difficulty in turning a new experience (something
just done or experienced) into a longer-term memory that can be retrieved at a later time. Also, memory that depends on visual, tactile,
or auditory discrimination is noticeably affected. These represent visual
agnosia, tactile agnosia, and auditory agnosia, respectively.
In the Korsakoff syndrome (amnestic confabulatory syndrome) there is memory loss, dementia, amnesia, and a tendency to give confabulated responses. This type of response is fluent but consists of a string of unrelated,
or even made up, “memories” that never actually occurred or make no
sense. This may lead to an incorrect conclusion that the patient is suffering from dementia. In addition to lesions in the hippocampus in these patients, the mammillary bodies and dorsomedial nucleus of the thalamus
are noticeably affected. The Korsakoff syndrome (see also the WernickeKorsakoff syndrome) as seen in chronic alcoholics is largely owing to thiamine deficiency and can be treated with therapeutic doses of this vitamin.
Abbreviations
AC
AmHrn
Amy
AntNu
CC, G
CC,Spl
Cing
CingGy
CorHip
DenGy
EnCtx
For
GyRec
Hip
Hyth
IC,G
Anterior commissure
Ammon’s horn
Amygdaloid nucleus (complex)
Anterior nucleus of thalamus
Corpus callosum, genu
Corpus callosum, splenium
Cingulum
Cingulate gyrus
Corticohippocampal fibers
Dentate gyrus
Entorhinal cortex
Fornix
Gyrus rectus
Hippocampus
Hypothalamus
Internal capsule, genu
LT
MB
MedFCtx
MedTh
MTegTr
MtTr
NuAcc
OpCh
Pi
RSplCtx
SepNu
SMNu
Sub
TegNu
VmNu
Lamina terminalis
Mammillary body
Medial frontal cortex
Medial thalamus
Mammillotegmental tract
Mammillothalamic tract
Nucleus accumbens
Optic chiasm
Pineal
Retrosplenial cortex
Septal nuclei
Supramammillary nucleus
Subiculum
Tegmental nuclei
Ventromedial hypothalamic nucleus
Review of Blood Supply to Hip, MB, Hyth, and CingGy
STRUCTURES
Hip
MB, Hyth
AntNu
CingGy
ARTERIES
anterior choroidal (see Figure 5–38)
branches of circle of Willis (see Figure 2–21)
thalamoperforating (see Figure 5–38)
branches of anterior cerebral
Limbic System 233
Hippocampal Connections
CingGy
Cing
For
IC,G
CC,Spl
CC,G
RSplCtx
MedTh
AntNu
For
Pi
AC
SepNu
LT
GyRec
NuAcc
VmNu
Amy
OpCh
CorHip
MB
EnCtx
Hip
DenGy
AmHrn
Sub
CingGy
Cing
IC,G
For
MTTr
AntNu
For
AC
MTegTr
TegNu
SepNu
LT
Hyth
DenGy
OpCh
AmHrn
Sub
EnCtx
Amy
MB
Hip
234 Synopsis of Functional Components, Tracts, Pathways, and Systems
Amygdaloid Connections
7–33 The origin, course, and distribution of selected afferent and
efferent connections of the amygdaloid nuclear complex in sagittal (upper) and coronal (lower) planes. The amygdala receives input from,
and projects to, brainstem and forebrain centers via the stria terminalis
and the ventral amygdalofugal pathway. Corticoamygdaloid and amygdalocortical fibers interconnect the basal and lateral amygdaloid nuclei
with select cortical areas.
Neurotransmitters: Cells in the amygdaloid complex contain
vasoactive intestinal polypeptide (VIP, ⫹), neurotensin (NT), somatostatin (SOM,⫺), enkephalin (ENK,⫺), and substance P (SP, ⫹).
These neurons project, via the stria terminalis or the ventral amygdalofugal path, to the septal nuclei (VIP, NT), the bed nucleus of the
stria terminalis (NT, ENK, SP), the hypothalamus (VIP, SOM, SP), the
nucleus accumbens septi, and the caudate and putamen (NT). Serotonergic amygdaloid fibers originate from the nucleus raphe dorsalis
and the superior central nucleus, dopaminergic axons from the ventral
tegmental area and the substantia nigra-pars compacta, and noradrenalin-containing fibers from the locus ceruleus. Glutamate (⫹) is found
in olfactory projections to the prepiriform cortex and the amygdaloid
complex. Acetylcholine is present in afferents to the amygdala from the
substantia innominata, as well as from the septal area. In patients with
Alzheimer disease and the associated dementia, there is a marked loss
of acetylcholine-containing neurons in the basal nucleus of the substantia innominata, in the cortex, and in the hippocampus.
Clinical Correlations: Dysfunctions related to damage to the
amygdaloid complex are seen in patients with trauma to the temporal
lobes, herpes simplex encephalitis, bilateral temporal lobe surgery to treat
intractable epileptic activity, and in some CNS degenerative disorders
(such as Alzheimer disease and Pick disease). The behavioral changes seen
in individuals with amygdala lesions collectively form the Klüver-Bucy
syndrome. In humans these changes/deficits are 1) hyperorality; 2) visual,
tactile, and auditory agnosia; 3) placidity; 4) hyperphagia or other dietary
manifestations; 5) an intense desire to explore the immediate environment (hypermetamorphosis), and 6) what is commonly called hypersexuality. These changes in sexual attitudes are usually in the form of comments, suggestions, and attempts to make a sexual contact (such as
touching) rather than in actual intercourse or masturbation. These patients may also show aphasia, dementia, and amnesia.
Abbreviations
AC
Amy
AmyCor
AmyFugPath
AntHyth
Ba-LatNu
CaNu
Cen-MedNu
CorAmy
DVagNu
EnCtx
For
GP
Hyth
LT
LHAr
MedThNu
MGNu
MidTh
NuAcc
NuCen,s
NuCer
Anterior commissure
Amygdaloid nuclear complex
Amygdalocortical fibers
Amygdalofugal pathway
Anterior hypothalamus
Basal and lateral nuclei
Caudate nucleus
Central, cortical and medial nuclei
Corticoamygdaloid fibers
Dorsal motor vagal nucleus
Entorhinal cortex
Fornix
Globus pallidus
Hypothalamus
Lamina terminalis
Lateral hypothalamic area
Medial thalamic nuclei
Medial geniculate nucleus
Midline thalamic nuclei
Nucleus accumbens
Nucleus centralis, superior
Nucleus ceruleus
NuRa,d
NuRa,m
NuRa,o
NuRa,p
NuStTer
OlfB
OpCh
PAG
PBrNu
PfNu
Pi
POpNu
PPriCtx
Put
SepNu
SNpc
SolNu
StTer
Sub
Subln
VenTegAr
VmNu
Nucleus raphe, dorsalis
Nucleus raphe, magnus
Nucleus raphe, obscurus
Nucleus raphe, pallidus
Nucleus of the stria terminalis
Olfactory bulb
Optic chiasm
Periaqueductal (central) gray
Parabrachial nuclei
Parafascicular nucleus
Pineal
Preoptic nucleus
Prepiriform cortex
Putamen
Septal nuclei
Substantia nigra, pars compacta
Solitary nucleus
Stria terminalis
Subiculum
Substantia innominata
Ventral tegmental area
Ventromedial hypothalamic nucleus
Review of Blood Supply to Amy and Related Centers
STRUCTURES
Amy
Hyth
Brainstem
Thalamus
ARTERIES
anterior choroidal (see Figure 5–38)
branches of circle of Willis (see Figure 5–38)
(see Figures 5–14, 5–21, and 5–27)
thalamoperforating, thalamogeniculate (see Figure 5–38)
Limbic System 235
Amygdaloid Connections
MedThNu
MidTh, PfNu, MGNu
StTer
Put, CaNu
StTer
NuAcc
VmNu, LHAr
NuStTer
OpCh
OlfB
PAG
AC
LT
AntHyth
POpNU
SNpc, VenTegAr
NuRa,d
NuCer
Amy
AmyFugPath
Cen-MedNu
PBrNu
Ba-LatNu
NuCen,s
PPriCtx
Sub
NuRa,m
EnCtx
SolNu
NuRa,p
NuRa,o
DVagNu
Prefrontal cortex
Cingulate gyrus
CaNu
NuStTer
Insula
StTer
Temporal
lobe
Put
NuAcc
GP
SepNu
For
AmyCor
CorAmy
Hyth
POpNu
Parahippocampal
gyrus
to StTer
SubIn
AmyFugPath
Cen-MedNu
Ba-LatNu
Amy
236 Synopsis of Functional Components, Tracts, Pathways, and Systems
7–34 Blank master drawing for limbic pathways. This illustration
is provided for self-evaluation of limbic pathways or connections, for
the instructor to expand on aspects of these pathways not covered in
the atlas, or both.
Limbic System 237
CHAPTER
8
Anatomical–Clinical Correlations:
Cerebral Angiogram, MRA, and MRV
240 Anatomical–Clinical Correlations
A
Callosomarginal branch
(of ACA)
Parietal branches
(of MCA)
Pericallosal branch
(of ACA)
Angular branch
(of MCA)
Anterior cerebral artery
(ACA)
Middle cerebral artery
(MCA)
Internal carotid artery
B
Internal carotid artery
(petrous part)
Ophthalmic artery
Internal carotid artery
(cavernous part)
Internal carotid artery
(cerebral part)
8-1 Internal carotid angiogram (left lateral projection, arterial
phase) showing the general patterns of the internal carotid, middle,
and anterior cerebral arteries (A, B) and an image with especially good
filling of the ophthalmic artery (B). The ophthalmic artery leaves the
cerebral part of the internal carotid and enters the orbit via the optic
canal. This vessel gives rise to the central artery of the retina, which is
an important source of blood supply to the retina. Occlusion of the
ophthalamic artery may result in blindness in the eye on that side. The
terminal branches of the ophthalamic artery will anastomose with superficial vessels around the orbit. Compare with Figures 2-12 (page 19)
and 2-25 (page 27).
Cerebral Angiogram, MRA, and MRV 241
A
Superior sagittal sinus
Superior cerebral veins
Inferior sagittal sinus
Straight sinus
Thalamostriate vein
Transverse sinus
Internal cerebral vein
Great cerebral vein (of Galen)
Venous angle
Sigmoid sinus
Inferior cerebral veins
Inferior anastomotic vein
(of Labbé)
Superficial middle
cerebral vein
Basal vein
(of Rosenthal)
B
Superior cerebral
veins
Superior anastomotic vein
(of Trolard)
Straight sinus
Inferior anastomotic vein
(of Labbé)
Superficial middle
cerebral vein
8-2 Two internal carotid angiograms (left lateral projection, venous phase). Superficial and deep venous structures are clear in A, but
B shows a particularly obvious vein of Trolard. The thalamostriate vein
(A) at this location can also be called the superior thalamostriate vein.
The junction of the superior thalamostriate vein with the internal cere-
bral vein is called the venous angle (A). The interventricular foramen
is located immediately rostral to this point.Compare these images with
the drawings of veins and sinuses in Figures 2-13 (page 19) and 2-28
(page 29).
242 Anatomical–Clinical Correlations
Middle cerebral artery
(M4–cortical branches)
Anterior cerebral artery
(A4, A5)
Anterior cerebral artery
(A3)
(A2)
Middle cerebral artery
(M2–insular branches)
Lenticulostriate branches
(of M1)
(A1)
Middle cerebral artery
(M1)
Internal carotid artery
(cavernous part)
Internal carotid artery
(cerebral part)
Internal carotid artery
(petrous part)
8-3 Internal carotid angiogram (anterior–posterior projection,
arterial phase). Note general distribution patterns of anterior and
middle cerebral arteries and the location of lenticulostriate
branches. The A1 segment of the anterior cerebral artery is located
between the internal carotid bifurcation and the anterior communicating artery. The distal portion of the anterior cerebral artery
(ACA) immediately rostral to the anterior communicating artery
and inferior to the rostrum of the corpus callosum is the A2 segment
(infracallosal). The portion of the ACA arching around the genu of
the corpus callosum is the A3 segment (precallosal) and the A4
(supracallosal) and A5 (postcallosal) segments are located superior
(above) the corpus callosum.
The M1 segment of the middle cerebral artery is located between
the internal carotid bifurcation and the point at which this vessel
branches into superior and inferior trunks on the insular cortex. As
branches of the middle cerebral artery pass over the insular cortex they
are designated as M2, as M3 when these branches are located on the inner surface of the frontal, parietal, and temporal opercula, and as M4
where they exit the lateral sulcus and fan out over the lateral aspect of
the cerebral hemisphere. Compare with Figure 2-21 on page 25.
Cerebral Angiogram, MRA, and MRV 243
Arachnoid villi
Superior cerebral veins
Superior sagittal sinus
Superior sagittal
sinus
Inferior sagittal sinus
Confluence of sinuses
Transverse
sinus
Transverse sinus
Sigmoid sinus
8-4 Internal carotid angiogram (anterior–posterior projection, venous phase). The patient’s head is tilted slightly; this shows the arching shapes of the superior and inferior sagittal sinuses to full advantage.
Note the other venous structures in this image and compare with the
arterial phase shown in Figure 8-3 on page 242 and the images in Figures 8-5 and 8-6 on pages 244 and 245. Also compare with Figure 228 on page 29.
244 Anatomical–Clinical Correlations
A
Superior sagittal sinus
Superficial cerebral veins
B
Superior sagittal sinus
Confluence of sinuses
Transverse sinus
Sigmoid sinus
Jugular bold
Internal jugular vein
8-5 Digital subtraction image of an internal carotid angiogram (anterior–posterior projection, venous phase). Image A is early in the venous phase (greater filling of cortical veins), whereas image B is later
in the venous phase (greater filling of the sinuses and jugular vein). Both
images are of the same patient.
The jugular bulb is a dilated portion of internal jugular vein (IJV)
in the jugular fossa at the point where the sigmoid sinus is continuous with the IJV; this continuity is through the jugular foramen. The
jugular foramen also contains the roots of cranial nerves IX, X, and
XI, the continuation of inferior petrosal sinus with the IJV and several small arteries. Compare with Figures 2-16 (page 21) and 2-19
(page 23).
Cerebral Angiogram, MRA, and MRV 245
B
ACA
PCA
A
SSS
MCA
SCA
TS
ICA
CS
AICA
BA
VA
Anterior cerebral artery (ACA)
A2 segment
C
A1 segment
Anterior communicating artery
Posterior cerebral artery (PCA)
Superior sagittal sinus (SSS)
Superior cerebellar artery (SCA)
Middle cerebral artery (MCA)
M2 segment
M1 segment
Internal carotid artery (ICA):
Cerebral part
Sigmoid sinus
Cavernous part
Petrous part
Transverse sinus (TS)
Anterior inferior cerebellar artery (AICA)
Confluence of sinuses (CS)
Basilar artery (BA)
8-6 Magnetic resonance angiography (MRA) is a noninvasive
method for imaging cerebral arteries, veins, and sinuses simultaneously. A 3-D phase contrast MRA (A) and an inverted video image
window (B) of the same view show major vessels and sinuses from an-
Vertebral artery (VA)
terior to posterior. C shows the relative position of the major vessels
and dural sinuses as imaged in A and B. The superior sagittal sinus, as
seen in A and B, is usually continuous with the right transverse sinus at
the confluence of sinuses.
246 Anatomical–Clinical Correlations
A
Parieto-occipitial
branches (of PCA)
Thalamogeniculate arteries
Calcarine branch
(of PCA)
Posterior choroidal arteries
Posterior cerebral arteries
(PCA)
Thalamoperforating
arteries
Basilar bifurcation
Posterior communicating
artery
Superior cerebellar
artery (SCA)
Posterior inferior
cerebellar artery
(PICA)
Basilar artery (BA)
Vertebral artery
(VA)
B
Parieto-occipital
branches
Calcarine branch
PCA
Basilar bifurcation
SCA
BA
Anterior inferior
cerebellar artery
8-7 A vertebral artery angiogram (left lateral
projection, arterial phase) is shown in A, and the
same view, but in a different patient, is shown in B,
using digital subtraction methods. Note the characteristic orientation of the major vessels. Compare
with Figure 2-21 on page 25.
PICA
VA
Cerebral Angiogram, MRA, and MRV 247
PCA, Cortical branches
PCA
Thalamoperforating
arteries
SCA
AICA
Basilar artery (BA)
Vertebral artery (VA)
B
Posterior cerebral artery,
Cortical branches
Posterior cerebral arteries
(PCA)
Superior cerebellar artery
(SCA)
BA
Thalamoperforating arteries
(of the basilar bifurcation)
SCA
AICA
Anterior inferior cerebellar
artery (AICA)
PICA
Posterior inferior cerebellar
artery (PICA)
VA
8-8 A vertebral artery angiogram (anterior–posterior projection,
arterial phase) is shown in A; the same view, but in a different patient,
is shown in B, using digital subtraction methods. Even though the injection is into the left vertebral, there is bilateral filling of the vertebral
arteries and of branches of the basilar artery. The thalamoperforating
arteries are important branches of P1 that generally serve rostral portions of the diencephalon.
The root of the oculomotor (IIIrd) nerve, after exiting the inferior
aspect of the midbrain, characteristically passes through the interpeduncular cistern and between the superior cerebellar and posterior
cerebral arteries en route to its exit from the skull through the superior orbital fissure. In this position the IIIrd nerve may be damaged by
large aneurysms that impinge on the nerve root. Compare with Figures
2-40 (page 39) and 2-41 (page 40).
248 Anatomical–Clinical Correlations
A
Middle cerebral
artery (MCA):
Anterior cerebral artery:
A3 segment
M1 segment
A2 segment
M2 segment
A1 segment
MCA, Insular branches
Basilar artery (BA)
Posterior cerebral
artery (PCA)
MCA, Cortical branches
(M4 segment)
PCA, Temporal branch
Internal cerebral vein
Superior petrosal sinus
Lateral ventricular
vein
Great cerebral vein
(of Galen)
Straight sinus (SS)
Transverse sinus (TS)
TS
B
Superior sagittal sinus
Anterior cerebral artery:
Great cerebral vein
(A3)
(A2)
MCA, M2 segment
SS
Internal carotid
artery
Posterior communicating artery
BA
PCA
TS
Superior cerebellar
artery
8-9 MRA images arteries, veins, and sinuses simultaneously, based
on the movement of fluid in these structures. These are inverted video
images of 3-D phase contrast MRA images as viewed from the dorsal
to ventral (A) and from the lateral aspect (B). The distal portion of the
anterior cerebral artery (ACA) immediately rostral to the anterior
communicating artery and inferior to the rostrum of the corpus callo-
sum is the A2 segment (infracallosal). The portion of the ACA arching
around the genu of the corpus callosum is the A3 segment (precallosal)
and the A4 (supracallosal) and A5 (postcallosal) segments are located
superior to (above) the corpus callosum. Compare these images with
arteries and veins as depicted in Figures 2-18 and 2-19 (page 23), 2-21
(page 25), and 2-23 (page 27).
Cerebral Angiogram, MRA, and MRV 249
Anterior cerebral artery:
Cortical branches
A1 segment
A
Internal carotid artery
Branches of middle
cerebral artery
Middle cerebral artery:
Branches on insula (M2)
Internal carotid artery
M1 segment
Posterior communicating
artery
Cortical branches (M4)
Posterior cerebral artery:
Posterior communicating
artery
P2 segment
P1 segment
Posterior cerebral artery
Parieto-occiptal artery
Calcarine artery
Anterior communicating artery
B
Orbit
Anterior cerebral artery
(A1 segment)
Ophthalmic artery
Cavernosus sinus
(containing internal
carotid artery)
Petrosal segment of
internal carotid artery
Superior cerebellar artery
Middle cerebral artery
(branches on insula)
Middle cerebral artery
(M1 segment)
Tumor (vestibular schwannoma)
Posterior communicating
artery
Posterior cerebral artery
Basilar artery
Calcarine artery
Posterior cerebral
artery
Vertebral arteries
8-10 MRA images of the vessels at the base of the brain forming
much of the cerebral arterial circle (of Willis) (A, B). Note the anterior, middle, and posterior cerebral arteries as they extend outward
from the circle. The upper image is from a normal individual, and the
lower image is from a patient with a vestibular schwannoma. Descriptions of the segments of the anterior, middle, and posterior cerebral
arteries are found on pages 25 and 242.
250 Anatomical–Clinical Correlations
Superior sagittal sinus
A
Callosomarginal branch
of ACA
Superficial cerebral veins
Pericallosal branch
of ACA
Anterior cerebral artery
(ACA)
Internal cerebral vein
Middle cerebral
artery
Great cerebral vein (of Galen)
Ophthalmic artery
or vein
Straight sinus
Carotid artery
(cavernous portions)
Vein of Labbé
Transverse sinus
Basal vein (of Rosenthal)
Confluence of sinuses
Sigmoid sinus
Basilar artery
Internal jugular vein
B
Superficial cerebral veins
Superficial cerebral
veins
Superior sagittal sinus
Middle cerebral artery
on insular cortex
Superficial cerebral vein
Transverse sinus
Confluence of sinuses
Basilar artery
Sigmoid sinus
Internal carotid artery
Inferior petrosal sinus
Internal jugular vein
8-11 Magnetic resonance venography (MRV) primarily demonstrates veins and venous sinuses although arteries (seen in A and B) will
also sometimes be visualized. Many veins and venous sinuses can be
seen in this lateral view (A) and in the anterior-posterior view (B).
Vertebral artery
Note that the continuation of the superior sagittal sinus is most prominent into the right transverse sinus (B, compare with Figure 8-6 on
page 245). Compare with Figures 2-13 (page 19), 2-16 (page 21), 219 (page 27), and 2-28 (page 29).
Blood Supply to the Choroid Plexi 251
A
Choroid plexus (CP) in
body of lateral ventricle
CP in atrium of lateral ventricle
CP in roof of third ventricle
CP in temporal horn
of lateral ventricle
CP in fourth ventricle
Anterior choroidal artery
AICA
PICA
Posterior communicating artery
Lateral posterior choroidal artery
BA
VA
Medial posterior choroidal artery
B
Medial striate artery
Internal carotid artery
A1
P1
Middle cerebral artery (M1)
Anterior choroidal artery
Anterior choroidal artery
Posterior communicating artery
Posterior cerebral artery (P2)
Lateral posterior choroidal artery
Superior cerebellar artery
Medial posterior choroidal artery
Basilar artery (BA)
Anterior inferior cerebellar artery (AICA)
AICA branch to choroid plexus
at the foramen of Luschka
Vertebral artery (VA)
Posterior inferior cerebellar artery (PICA)
PICA branch to choroid plexus
in the fourth ventricle
8-12 Blood supply to the choroid plexus of the lateral, third, and
fourth ventricles. Those branches of the vertebrobasilar system and of the
internal carotid artery and P2 segment of the posterior cerebral artery that
supply the choroid plexus are accentuated by appearing in a darker red
shade. In A, a representation of these vessels (origin, course, termination)
is shown from the lateral aspect. Anterior, posterior medial, and poste-
rior lateral choroidal arteries serve the plexuses of the lateral and third
ventricles. The choroid plexus in the fourth ventricle and the clump of
choroid plexus protruding out of the foramen of Luschka are served by
posterior inferior and anterior inferior cerebellar arteries, respectively. In
B, the origins of these branches from their main arterial trunks are shown.
See also Figures 2-21 (page 25), 2-24 (page 27), and 2-35 (page 35).
252 Anatomical–Clinical Correlations
A
Anterior cerebral artery
Middle cerebral artery
A2
M2
A1
M1
Internal carotid artery
Cerebral part
Basilar artery
Cavernous part
Anterior inferior
cerebellar artery
Petrosal part
Cervical part
Vertebral artery
Posterior inferior
cerebellar artery
Maxillary artery
(br. of external
carotid artery)
Internal carotid artery
Vertebral artery
External carotid artery
Common carotid artery
B
Posterior cerebral artery
Superior cerebellar artery
Position of
occulomotor nerve
Basilar artery
Anterior inferior
cerebellar artery
Vertebral artery
(intercranial portion)
Anterior inferior
cerebellar artery
Vertebral artery
(passing caudally and
medially around the lateral
mass of the atlas)
Vertebral artery
(passing through transverse
foramen of the atlas)
8-13 Overview (A) of the arteries in the neck that serve the brain (internal carotid and vertebral) and of their main terminal branches (anterior
cerebral artery and middle cerebral artery, vertebrobasilar system) as seen
in an MRA (anterior-posterior view). In approximately 40–45% of individuals the left vertebral artery is larger, as seen here, and in about 5–10%
of individuals one or the other of the vertebral arteries may be hypoplastic as seen here on the patient’s right. The MRI in B is a detailed view of
the vertebrobasilar system from the point where the vertebral arteries exit
the transverse foramen to where the basilar artery bifurcates into the posterior cerebral arteries. Compare this image with Figure 2-21 on page 25.
The vertebral artery (VA) is generally described as being composed
of 4 segments sometimes designated as V1 to V4. The first segment (V1)
is between the VA origin from the subclavian artery and the entrance
of VA into the first transverse foramen (usually C6); the second segment V2 is that part of VA ascending through the transverse foramen
of C6 to C2; the third segment (V3) is between the exit of VA from the
transverse foramen of the axis and the dura at the foramen magnum
(this includes the loop of the VA that passes through the transverse
foramen of C1/the atlas); the fourth segment (V4) pierces the dura and
joins its counterpart to form the basilar artery.
CHAPTER
9
Q & A’s: A Sampling of Study and Review
Questions, Many in the USMLE Style,
All With Explained Answers
D. E. Haines and J. A. Lancon
There are two essential goals of a student studying human neurobiology, or, for that matter, the student of any of the medical
sciences. The first is to gain the knowledge base and diagnostic
skills to become a competent health care professional. Addressing the medical needs of the patient with insight, skill, and compassion is paramount. The second is to successfully negotiate
whatever examination procedures are used in a given setting.
These may be standard class examinations, Subject National
Board Examination (now used/required in many courses), the
USMLE Step 1 Examination (required of all U.S. medical students), or simply the desire, on the part of the student, for selfassessment.
The questions in this chapter are prepared in two general
styles. First, there are study or review questions that test general knowledge concerning the structure of the central nervous system. Many of these have a functional flavor. Second,
there are single one best answer questions in the USMLE style
that use a patient vignette approach in the stem. These questions have been carefully reviewed for clinical accuracy and
relevance as used in these examples. At the end of each explained answer, page numbers appear in parentheses that
specify where the correct answer, be it in a figure or in the text,
may be found. In order to make this a fruitful learning exer-
cise, some answers may contain additional relevant information to extend the educational process.
In general, the questions are organized by individual chapters,
although chapters 1 and 2 and chapters 3 and 4 are combined. Reference to the page (or pages) containing the correct answer are
usually to the chapter(s) from which the question originated.
However, recognizing that neuroscience is dynamic and three-dimensional, some answers contain references to chapters other
than that from which the question originated. This provides a
greater level of integration by bringing a wider range of information to bear on a single question.
Correct diagnosis of the neurologically compromised patient
not only requires integration of information contained in different chapters but may also require inclusion of concepts gained in
other basic science courses. In this regard a few questions, and
their answers, may include such additional basic concepts.
This is not an all-inclusive list of questions, but rather a
sampling that covers a wide variety of neuroanatomical and
clinically relevant points. There is certainly a much larger variety of questions that could be developed from the topics covered in this atlas. It is hoped that this sample will give the user
a good idea of how basic neuroscience information correlates
with a range of clinically relevant topics.
254 Q & A’s: A Sampling of Study and Review Questions with Explained Answers
Review and Study Questions for
Chapters 1 and 2
1. A 71-year-old man complains to his family physician that his face
“feels funny.” The examination reveals numbness on his face and
on the same side of his tongue. MRI shows a lesion in the cerebral
cortex. This man’s lesion is most likely located in which of the following cortical regions?
䊊 (A) Anterior paracentral
䊊 (B) Lateral one-third of the postcentral
䊊 (C) Lateral one-third of the precentral
䊊 (D) Middle one-third of the postcentral
䊊 (E) Posterior paracentral
2. A 41-year-old woman complains to her family physician about recurring episodes of sharp pain that seem to originate from around
her mouth and cheek. The pain is so intense that she is unable to
eat, brush her teeth, or apply make-up. Which of the following
cranial nerves is the most likely source of this pain?
䊊 (A) Facial (VII)
䊊 (B) Glossopharyngeal (IX)
䊊 (C) Hypoglossal (XII)
䊊 (D) Trigeminal (V)
䊊 (E) Vagus (X)
3. The labyrinthine artery is an important source of blood supply to
the inner ear. Which of the following arteries represents the major vessel from which this branch usually arises?
䊊 (A) Anterior inferior cerebellar
䊊 (B) Basilar
䊊 (C) Posterior inferior cerebellar
䊊 (D) Superior cerebellar
䊊 (E) Vertebral
4. The quadrigeminal artery in a 20-year-old man is occluded by a fat
embolus originating from a compound fracture of the humerus.
Which of the following structures does this occluded vessel most
directly affect?
䊊 (A) Superior cerebellar peduncle
䊊 (B) Mammillary bodies
䊊 (C) Medial and lateral geniculate bodies
䊊 (D) Pineal and habenula
䊊 (E) Superior and inferior colliculi
Questions 5 and 6 are based on the following patient.
A 63-year-old man has hearing loss, tinnitus (ringing or buzzing sounds
in the ear), vertigo, and unsteady gait; all of these have developed over
several years. MRI reveals a large tumor (3 cm in diameter) at the cerebellopontine angle, most likely a vestibular schwannoma (sometimes
incorrectly called an acoustic neuroma).
5. What additional deficit could this patient also have?
䊊 (A) Anosmia
䊊 (B) Hemianopsia
䊊 (C) Numbness on the face
䊊 (D) Visual field deficits
䊊 (E) Weakness of the tongue
6. In addition to the vestibulocochlear nerve, which of the following
structures would most likely also be affected by the tumor in this
man?
䊊 (A) Anterior inferior cerebellar artery
䊊 (B) Facial nerve
䊊 (C) Glossopharyngeal nerve
䊊 (D) Posterior inferior cerebellar artery
䊊 (E) Vagus nerve
7. A 67-year-old man complains to his family physician of severe
headaches. The examination reveals visual deficits in both eyes,
and MRI shows a lesion in the cerebral cortex. Which of the following cortical structures represents the most likely location of
this lesion?
䊊 (A) Angular gyrus
䊊 (B) Cingulate gyrus
䊊 (C) Lingual gyrus
䊊 (D) Parahippocampal gyrus
䊊 (E) Precuneus
8. A sagittal MRI of a 23-year-old woman is located at, or immediately adjacent to, the midline. Which of the following spaces or
structures would be in the image and would indicate a midline
plane?
䊊 (A) Cerebral aqueduct
䊊 (B) Corpus callosum
䊊 (C) Interpeduncular fossa
䊊 (D) Interventricular foramen
䊊 (E) Superior colliculus
9. A 20-year-old man is brought to the emergency department from
the site of a motorcycle accident. He is unconscious and has a broken femur, humerus, and extensive facial injuries. Axial CT shows
a white layer on the lateral aspect of the left hemisphere that is approximately 5 mm thick and extends for 12 cm. This observation
most likely represents:
䊊 (A) Epidural hemorrhage/hematoma
䊊 (B) Parenchymatous hemorrhage in the cortex
䊊 (C) Subarachnoid hemorrhage
䊊 (D) Subdural hemorrhage/hematoma
䊊 (E) Ventricular hemorrhage
10. Which of the following portions of the ventricular system does not
contain choroid plexus?
䊊 (A) Cerebral aqueduct
䊊 (B) Fourth ventricle
䊊 (C) Lateral ventricle
䊊 (D) Interventricular foramen
䊊 (E) Third ventricle
11. A 47-year-old man presents with an intense pain on his face arising from stimulation at the corner of his mouth. This is characteristic of trigeminal neuralgia (tic douloureux). MRI shows a vessel
compressing the root of the trigeminal nerve. Which of the following vessels would most likely be involved?
䊊 (A) Anterior inferior cerebellar artery
䊊 (B) Basal vein (of Rosenthal)
䊊 (C) Basilar artery
䊊 (D) Posterior cerebral artery
䊊 (E) Superior cerebellar artery
Q & A’s: A Sampling of Study and Review Questions with Explained Answers 255
12. Which of the following cranial nerves contain the afferent and efferent limbs of the corneal reflex?
䊊 (A) II and III (optic and oculomotor)
䊊 (B) III, IV, VI (oculomotor, trochlear, abducens)
䊊 (C) V and VII (trigeminal, facial)
䊊 (D) VIII and IX (vestibulocochlear, glossopharyngeal)
䊊 (E) IX and X (glossopharyngeal, vagus)
13. A 73-year-old man is brought to the emergency department after
being found in his garage in a state of confusion. CT shows an infarct involving much of the superior frontal gyrus. Which of the
following vessels is most likely occluded in this patient?
䊊 (A) Angular artery
䊊 (B) Callosomarginal artery
䊊 (C) Lenticulostriate arteries
䊊 (D) Middle cerebral artery, M4 segments
䊊 (E) Posterior cerebral artery, P4 segments
14. The MRI of a 49-year-old woman shows a tumor located immediately superior to the corpus callosum. This lesion is most likely located in which of the following lobes?
䊊 (A) Frontal
䊊 (B) Limbic
䊊 (C) Occipital
䊊 (D) Parietal
䊊 (E) Temporal
15. A 69-year-old woman is brought to the emergency department.
The daughter reports that her mother suddenly seemed to be unable to speak. The examination reveals that the woman has a nonfluent (Broca) aphasia. A sagittal MRI shows a lesion in which of
the following gyri?
䊊 (A) Angular
䊊 (B) Inferior frontal
䊊 (C) Lateral one-third of the precentral
䊊 (D) Middle frontal
䊊 (E) Supramarginal
16. Which of the following Brodmann areas represents the primary
somatosensory cortex?
䊊 (A) Areas 3, 1, 2
䊊 (B) Area 4
䊊 (C) Area 17
䊊 (D) Area 22
䊊 (E) Area 40
17. A 64-year-old man awakens with a profound weakness of his right
hand. The man is transported by ambulance to a major medical
center, a distance of 240 miles and taking several hours. About 2.5
hours after his arrival, an MRI shows a small lesion in the cerebral
cortex. Which of the following gyri represents the most likely location of this lesion?
䊊 (A) Anterior paracentral
䊊 (B) Medial one-third of precentral
䊊 (C) Middle frontal
䊊 (D) Middle one-third of precentral
䊊 (E) Lateral one-third of precentral
18. A lumbar puncture, commonly called a “lumbar tap,” consists of a
needle being inserted through an intervertebral space into the
lumbar cistern to retrieve a sample of cerebrospinal fluid. Which
of the following is the most likely level for the insertion of the needle?
䊊 (A) L1–L2
䊊 (B) L2–L3
䊊 (C) L4–L5
䊊 (D) S1–S2
䊊 (E) T12–L1
19. A 59-year-old man complains of persistent headache. An MRA
(Magnetic Resonance Angiography) shows an aneurysm in the interpeduncular fossa (and cistern) arising from the basilar tip.
Which of the following cranial nerves would be most directly affected by this aneurysm?
䊊 (A) Abducens (VI)
䊊 (B) Oculomotor (III)
䊊 (C) Optic (II)
䊊 (D) Trigeminal, V1 (V)
䊊 (E) Trochlear (IV)
20. A 71-year-old man presents with a Broca (nonfluent) aphasia. MRI
reveals a lesion in Brodmann area 44. As this lesion expands, due
to edema, and impinges on the immediately adjacent cortical areas, which of the following deficits would most likely be seen?
䊊 (A) Loss of hearing in one ear
䊊 (B) Numbness and prickly sensation on the hand
䊊 (C) Visual field deficits in both eyes
䊊 (D) Weakness of facial muscles
䊊 (E) Weakness of the upper extremity
21. A 47-year-old woman presents with seizures and ill-defined neurologic complaints. The examination reveals a bruit on the lateral
aspect of the head immediately rostral and superior to the ear. A
CT shows a large arteriovenous malformation in the area of the lateral sulcus. The feeding artery(ies) is M4 branches. Which of the
following most likely represents the major draining vein?
䊊 (A) Inferior sagittal sinus
䊊 (B) Internal cerebral vein
䊊 (C) Ophthalmic vein
䊊 (D) Superficial middle cerebral vein
䊊 (E) Superior petrosal sinus
22. The collection of posterior and anterior roots that occupy the lumbar cistern are collectively known as which of the following?
䊊 (A) Cauda equina
䊊 (B) Conus medullaris
䊊 (C) Denticulate ligament
䊊 (D) Filum terminale externum
䊊 (E) Filum terminale internum
23. Which of the following Brodmann areas represents the primary
somatomotor cortex?
䊊 (A) Areas 3,1,2
䊊 (B) Area 4
䊊 (C) Area 5
䊊 (D) Area 6
䊊 (E) Area 7
256 Q & A’s: A Sampling of Study and Review Questions with Explained Answers
24. A 39-year-old woman complains of weakness in her right lower
extremity. The history suggests that this deficit has developed
slowly, perhaps over several years. MRI shows a meningioma imposing on the cerebral cortex. Which of the following gyri is most
likely involved in this patient?
䊊 (A) Anterior paracentral
䊊 (B) Lateral part of precentral
䊊 (C) Medial part of precentral
䊊 (D) Medial part of postcentral
䊊 (E) Posterior paracentral
30. The abducens nerve exits the brainstem at the pons-medulla junction generally in line with the preolivary sulcus and passes rostrally
just lateral to, and in the same cistern as, the basilar artery. Which
of the following cisterns contains the abducens nerve and basilar
artery?
䊊 (A) Ambient
䊊 (B) Inferior cerebellopontine
䊊 (C) Premedullary
䊊 (D) Prepontine
䊊 (E) Superior cerebellopontine
25. A 71-year-old woman presents with motor and sensory deficits affecting her face and upper extremity. CT shows a hemorrhage that
is confined largely to the cortex and adjacent subcortical areas.
Which of the following vessels/segments are most likely involved?
䊊 (A) A1
䊊 (B) M2
䊊 (C) M3
䊊 (D) M4
䊊 (E) P4
31. An 81-year-old woman is brought to the emergency department
by her son with a complaint of weakness on the same side of her
body and face. CT shows a hemorrhage in the territory of the
lenticulostriate arteries. Which of the following represents the
most likely origin of these vessels?
䊊 (A) A1
䊊 (B) M1
䊊 (C) M2
䊊 (D) P1
䊊 (E) P2
26. A 22-year-old man is brought to the emergency department with
a gunshot wound to the head. He is decorticate but soon becomes
decerebrate. This change in status is due to uncal herniation.
Which of the following most specifically describes the position of
the uncus prior to herniation?
䊊 (A) At the temporal lobe
䊊 (B) Caudal aspect of the cingulate gyrus
䊊 (C) Caudal aspect of the gyrus rectus
䊊 (D) Medial edge of occipitotemporal gyri
䊊 (E) Rostromedial aspect of the parahippocampal gyrus
32. The MRI of a 27-year-old woman shows a meningioma impinging
on the gyrus rectus in axial and coronal MRI. This lesion is located
on which of the following lobes of the cerebral hemisphere?
䊊 (A) Frontal
䊊 (B) Insular
䊊 (C) Occipital
䊊 (D) Parietal
䊊 (E) Temporal
27. A 73-year-old woman presents with visual deficits in both eyes.
No other cranial nerve deficits or motor or sensory deficits are
seen. CT shows a hemorrhage in the cerebral cortex. Which of the
following vessels/segments is most likely involved in this hemorrhage?
䊊 (A) A1
䊊 (B) M3
䊊 (C) M4
䊊 (D) P2
䊊 (E) P4
33. A 51-year-old man presents with visual field deficits in both eyes
and a right-sided weakness of the upper and lower extremities.
MRI shows a lesion in the optic tract that has spread into a structure located immediately adjacent to this tract. Based on its
anatomical relationship, which of the following structures is most
likely involved in a lesion spreading from the optic tract?
䊊 (A) Left basilar pons
䊊 (B) Left crus cerebri
䊊 (C) Left pyramid
䊊 (D) Right crus cerebri
䊊 (E) Right optic nerve
28. The CT of a 77-year-old man shows a calcified tuft of choroid
plexus, the glomus choroideum. Which of the following represents the location of this part of the choroid plexus?
䊊 (A) Anterior horn of the lateral ventricle
䊊 (B) Atrium of the lateral ventricle
䊊 (C) Body of the lateral ventricle
䊊 (D) Caudal roof of the third ventricle
䊊 (E) Temporal horn of the lateral ventricle
29. Which of the following represents the most common cause of
blood in the subarachnoid space (subarachnoid hemorrhage)?
䊊 (A) Bleeding from an arteriovenous malformation
䊊 (B) Bleeding from a meningioma
䊊 (C) Bleeding from a tumor
䊊 (D) Rupture of an aneurysm
䊊 (E) Trauma to the brain
34. A 19-year-old man presents with significant paralysis of movement in his left eye and a dilated pupil. No other deficits are
seen. Suspecting some type of lesion on the root or along the
intracranial course of the oculomotor (III) nerve, the neurologist orders an MRI. Which of the following describes the appearance of the subarachnoid and ventricular spaces in a T2weighted image?
䊊 (A) Black (hypointense)
䊊 (B) Dark grey
䊊 (C) Light grey
䊊 (D) Medium grey
䊊 (E) White (hyperintense)
Q & A’s: A Sampling of Study and Review Questions with Explained Answers 257
35. A 49-year-old woman presents with ill-defined neurologic deficits
that have persisted over several months. As part of the evaluation,
the neurologist orders an MRI. Which of the following describes
the appearance of CSF in the ventricular spaces, and consequently
the outline and shape of the ventricles, in a T1-weighted image?
䊊 (A) Black (hypointense)
䊊 (B) Dark grey
䊊 (C) Light grey
䊊 (D) Medium grey
䊊 (E) White (hyperintense)
36. A 71-year-old morbidly obese man is brought to the emergency
department by his son. The son reports that the man complained
of a sudden excruciating headache and then became stuporous.
Suspecting a ruptured aneurysm the physician orders a CT. Which
of the following describes the appearance of acute blood in the
subarachnoid space in CT?
䊊 (A) Black (hypodense)
䊊 (B) Black to grey
䊊 (C) Light grey
䊊 (D) Medium grey
䊊 (E) White (hyperdense)
37. Which of the following cranial nerves exits the brainstem via the
preolivary sulcus?
䊊 (A) Abducens (VI)
䊊 (B) Facial (VII)
䊊 (C) Hypoglossal (XII)
䊊 (D) Vagus (X)
䊊 (E) Trigeminal (V)
38. A 29-year-old woman becomes acutely ill with high fever, a stiff
neck, and stupor. A lumbar puncture reveals cloudy cerebrospinal
fluid from which organisms are cultured. Which of the following
represents the most frequently seen organisms in cases of adult
bacterial meningitis?
䊊 (A) Escherichia coli
䊊 (B) Haemophilus influenzae
䊊 (C) Herpes simplex
䊊 (D) Listeria monocytogenes
䊊 (E) Streptococcus pneumoniae
39. Which of the following cranial nerves exits the posterior (dorsal)
aspect of the brainstem?
䊊 (A) Abducens (VI)
䊊 (B) Hypoglossal (XII)
䊊 (C) Trigeminal (V)
䊊 (D) Trochlear (IV)
䊊 (E) Vestibulocochlear (VIII)
40. Which of the following cranial nerves passes between the posterior cerebral artery and the superior cerebellar artery as it exits the
brainstem?
䊊 (A) Abducens
䊊 (B) Oculomotor
䊊 (C) Optic
䊊 (D) Trigeminal
䊊 (E) Vestibulocochlear
41. The MRI of an 11-year-old boy shows a tumor in the pontine portion of the fourth ventricle. The rostral edge of which of the following structures represents the border between the medullary
and pontine parts of the fourth ventricle?
䊊 (A) Facial colliculus
䊊 (B) Hypoglossal trigone
䊊 (C) Medial eminence
䊊 (D) Stria medullares
䊊 (E) Vagal trigone
42. A 61-year-old man presents with a tremor and unsteady gait; these
problems are on the same side of his body. Sagittal MRI shows a
lesion in the anterior lobe of the cerebellum. Which of the following represents the fissure separating the anterior and posterior
lobes of the cerebellum?
䊊 (A) Horizontal fissure
䊊 (B) Posterior superior fissure
䊊 (C) Posterolateral fissure
䊊 (D) Primary fissure
䊊 (E) Secondary fissure
43. The MRI of a 49-year-old woman with a brain tumor shows tonsillar herniation. Based on its anatomical position, which of the following portions of the brainstem would be most adversely affected
by tonsillar herniation?
䊊 (A) Caudal midbrain
䊊 (B) Caudal pons
䊊 (C) Medulla
䊊 (D) Rostral midbrain
䊊 (E) Rostral pons
44. A 4-year-old boy is brought to the emergency department by his
mother who explains that the boy fell off a porch onto a concrete
sidewalk. The examination reveals that the boy has a parietal scalp
laceration, is stuporous, and has reactive pupils. Suspecting that
this boy may have a possible skull fracture with some type of intracranial bleeding, which of the following imaging tests would be
most immediately (and appropriately) useful?
䊊 (A) CT
䊊 (B) MRI, gadolinium enhanced
䊊 (C) MRI, T1-weighted
䊊 (D) MRI, T2-weighted
䊊 (E) PET (Positron Emission Tomography)
45. A sagittal MRI of a 52-year-old man clearly shows a small tumor
in the area of the long and short gyri. These gyri are characteristically found in which of the following lobes?
䊊 (A) Frontal
䊊 (B) Insular
䊊 (C) Limbic
䊊 (D) Occipital
䊊 (E) Parietal
46. A lesion involving the root of which of the following nerves would
most likely have an effect on the gag reflex?
䊊 (A) Accessory
䊊 (B) Facial
䊊 (C) Glossopharyngeal
䊊 (D) Hypoglossal
䊊 (E) Trigeminal
258 Q & A’s: A Sampling of Study and Review Questions with Explained Answers
Answers for Chapters 1 and 2
1. Answer B: Numbness on the face, resulting from a lesion in the
cerebral cortex, indicates a lesion in the lateral one-third of the
postcentral gyrus (face area of the somatosensory cortex). The anterior paracentral gyrus and the precentral gyrus are somatomotor
areas of the cerebral cortex. The upper extremity is represented in
the middle one-third of the postcentral gyrus and the lower extremity is represented in the posterior paracentral gyrus. (p. 15)
2. Answer D: Tic douloureux (trigeminal neuralgia) is a lancinating pain that originates from the territories of the trigeminal
nerve, primarily its V2 or V3 territories. The trigger zone is frequently around the corner of the mouth. There is a geniculate neuralgia (related to the ear) and a glossopharyngeal neuralgia (related
to the throat or palate), but neither of these originates from the
surface of the face near the oral cavity. The hypoglossal nerve is
the motor for the tongue and the vagus is the motor for most of
the pharynx and larynx, visceromotor for much of the gut, and
contains viscerosensory fibers from the gut. (p. 41)
3. Answer A: In most cases (85–100%), the labyrinthine artery,
also called the internal auditory artery, originates from the anterior inferior cerebellar artery. It enters the internal acoustic meatus, serves bone and dura of the canal, the nerves of the canal, and
vestibular and cochlear structures. In a few cases (15% or less),
this artery originates from the basilar artery. None of the other
choices gives rise to vessels that serve the inner ear. (p. 25, 27)
4. Answer E: The quadrigeminal artery is the primary blood supply to the superior and inferior colliculi: this vessel originates from
P1. The geniculate bodies receive their blood supply from the thalamogeniculate arteries, and the pineal and habenula from the posterior medial choroidal artery. The superior cerebellar peduncle
receives its blood supply via the medial branch of the superior
cerebellar artery, and branches of the cerebral circle (of Willis)
serve the mammillary bodies. (p. 25, 35)
5. Answer C: Vestibular schwannomas larger than 2.0 cm in diameter may impinge on the root of the trigeminal nerve and cause numbness on the same side of the face. Although the other deficits listed
are not seen in these patients, diplopia (involvement of oculomotor,
abducens or trochlear nerves, singularly or in combination) may be
present, but in fewer than 10% of these individuals. (p. 42)
6. Answer B: The internal acoustic meatus contains the vestibulocochlear nerve, the facial nerve, and the labyrinthine artery, a
branch of the anterior inferior cerebellar artery. A vestibular
schwannoma located in the meatus would likely affect the facial
nerve and result in facial weakness. The vagus and glossopharyngeal nerves exit the skull via the jugular foramen (along with the
accessory nerve). The cerebellar arteries originate within the skull
and distribute to structures within the skull. (p. 42)
7. Answer C: The lingual gyrus is the lower bank of the calcarine sulcus; the upper (cuneus) and lower banks of this sulcus are the location of the primary visual cortex. The precuneus is the medial aspect
of the parietal lobe, and the angular gyrus is a portion of the inferior
parietal lobule on the lateral aspect of the hemisphere. The cingulate and parahippocampal gyri are located on the medial aspect of the
hemisphere and are parts of the limbic lobe. (p. 13–15, 28)
8. Answer A: The cerebral aqueduct is about 1.5–2.0 mm in diameter, and connects the third ventricle with the fourth ventricle.
When this part of the ventricular system appears in a sagittal MRI,
the plane of the scan is at the midline. Neither the interventricular foramen nor the superior colliculus are on the midline. Both
the interpeduncular fossa and the corpus callosum are on the midline, but extend off the midline well beyond the width of the cerebral aqueduct. (p. 28–31, 49, 50, 52)
9. Answer D: Trauma may cause epidural hemorrhage, subdural
hemorrhage, or subarachnoid hemorrhage. Acute subdural hemorrhage/hematoma will appear white in CT and will usually present as a comparatively thin but long defect. Epidural hemorrhage
will usually be seen as a shorter but thicker lesion and may appear
loculated (have some sort of internal structure). The structure
(shape) of this lesion does not conform to hemorrhage into the
substance of the brain (brain parenchyma), into the subarachnoid
space (or cisterns), and certainly not to hemorrhage into the ventricles. (p. 46, 48, 51)
10. Answer A: The only portion of the ventricular system that does
not contain choroid plexus is the cerebral aqueduct. The choroid
plexus in the lateral ventricle is continuous from the inferior horn
into the atrium and into the body of the ventricle, and through the
interventricular foramen with the choroid plexus located along
the roof of the third ventricle. There is a tuft of choroid plexus in
the fourth ventricle, a small part of which extends into the lateral
recess and through the lateral foramen (of Luschka) into the subarachnoid space at the cerebellopontine angle. (p. 52–53)
11. Answer E: Branches of the superior cerebellar artery are most
frequently involved in cases of trigeminal neuralgia that are presumably of vascular origin. The posterior cerebral artery and its
larger branches serve the midbrain-diencephalic junction or join
the medial surface of the hemisphere. The basilar artery serves the
basilar pons and the anterior inferior cerebellar artery serves the
caudal midbrain, inner ear, and the inferior surface of the cerebellar surface. The basal vein drains the medial portions of the
hemisphere and passes through the ambient cistern to enter the
great cerebral vein (of Galen). (p. 41)
12. Answer C: The afferent limb of the corneal reflex is via the ophthalmic division of the trigeminal nerve (V); the cell body of origin is in the trigeminal ganglion and the central terminations in the
pars caudalis of the spinal trigeminal nucleus. The efferent limb
originates in the motor nucleus of the facial nerve (VII) and distributes to the facial muscles around the eye. None of the other
choices contains fibers related to the corneal reflex. (p. 42)
13. Answer B: The callosomarginal artery, a branch of the anterior
cerebral artery, serves the medial aspect of the superior frontal
gyrus and that portion of this gyrus on the superior and lateral aspects of the hemisphere. M4 segments of the middle cerebral
artery serve the lateral aspects of the hemisphere; P4 segments of
the posterior cerebral artery serve the medial aspects of the hemisphere caudal to the parietoccipital sulcus, and the angular artery
(an M4 branch) serves the angular gyrus of the inferior parietal lobule. The lenticulostriate arteries are branches of M1 that serve internal structures of the hemisphere. (p. 17, 29)
14. Answer B: The limbic lobe, consisting primarily of the cingulate gyrus and the parahippocampal gyrus, is located on the most
Q & A’s: A Sampling of Study and Review Questions with Explained Answers 259
medial aspect of the hemisphere; the cingulate gyrus is located immediately adjacent to the corpus callosum. None of the other
lobes of the cerebral cortex borders directly on the corpus callosum. (p. 13, 28)
15. Answer B: The inferior frontal gyrus consists of the pars orbitalis (Brodmann area 47), pars triangularis (area 45), and pars
opercularis (area 44). A lesion located primarily in areas 44 and
45 in the dominant hemisphere will result in a nonfluent (Broca)
aphasia. The supramarginal (area 40) and angular (area 39) gyri
represent what is called the Wernicke area, and the middle frontal
gyrus contains areas 6 and 8. The lateral one-third of the precentral gyrus is the face area of the somatomotor cortex. (p. 14)
16. Answer A: Areas 3, 1, 2 collectively represent the primary somatosensory cortex. Area 4 is the primary somatomotor cortex,
area 17 the primary visual cortex, and area 22 the primary auditory cortex. Area 40 is in the supramarginal gyrus, a large part of
which is called the Wernicke area. (p. 14)
17. Answer D: The body is represented in the somatomotor cortex
(precentral gyrus, anterior paracentral gyrus) in the following pattern: the face in about the lateral one-third of the precentral gyrus
above the lateral sulcus; the hand and upper extremity in about its
middle third; and the trunk and hip in about its medial third. The
lower extremity and foot are represented in the anterior paracentral gyrus. Caudal portions of the middle frontal gyrus are the location of the frontal eye field. (p. 15)
18. Answer C: The L4-L5 interspace is commonly used for a lumbar puncture. The L3-L4 space may also be used. Levels T12 to
L2-L3 are too high. Because the caudal end of the spinal cord (the
conus medullaris) may be as low as L2 in some individuals, levels
T12-L1 to L2-L3 are not used, as this would most likely result in
damage to the spinal cord. The S1-S2 vertebrae are fused so there
is no intervertebral space through which a needle can pass. Furthermore, the dural sac ends at about S2. (p. 12)
19. Answer B: The oculomotor nerve (III) exits from the medial aspect of the midbrain into the interpeduncular fossa/cistern. It traverses this space, courses through the lateral wall of the cavernous
sinus to eventually enter (along with the trochlear [IV] and abducens [VI] nerves) the superior orbital fissure. Cranial nerves IV,
VI, and V1 (the ophthalmic portion of the trigeminal nerve), along
with III, pass through the cavernous sinus. Cranial nerve II (optic)
is quite rostral to the interpeduncular fossa. (p. 24, 30, 40)
20. Answer D: A lesion in area 44 (the pars opercularis) that spreads
will affect the lower portions of the precentral gyrus in which the
face is represented. This will result in weakness of facial muscles,
accompanied by other cranial nerve deficits. The cortical areas for
hearing and vision are far separated from area 44. Also, a lesion in
the primary auditory cortex will not result in a hearing loss in one
ear. The hand area of the sensory cortex and the upper extremity
area of the motor cortex are not adjacent to Brodmann area 44.
(p. 14)
21. Answer D: The superficial middle cerebral vein is located on the
surface of the cerebral hemisphere in the immediate vicinity of the
lateral sulcus and, of the choices, is the most likely candidate. The
deep middle cerebral vein is located on the surface of the insular
lobe. The inferior sagittal sinus primarily drains the medial aspect
of the hemisphere immediately superior to the corpus callosum.
The internal cerebral vein (to the great cerebral vein) drains the
internal parts of the hemisphere; the ophthalmic vein connects the
orbit with the cavernous sinus; and the superior petrosal sinus
connects the cavernous sinus with the sigmoid sinus at its junction
with the transverse sinus. (p. 19, 23, 29)
22. Answer A: As they descend in the dural sac from their origin
from the spinal cord to their exit at their respective intervertebral
foramen, the anterior and posterior roots form the cauda equina.
The conus medullaris is the most caudal end of the spinal cord, and
the filum terminale internum is the strand of pia that extends from
the conus caudally to attach to the inner aspect of the dural sac at
about S2. The denticulate ligament anchors the spinal cord laterally to the inner surface of the dural sac, and the filum terminale
externum anchors the dural sac caudally to the inner aspect of the
coccyx. (p. 12, 85, 87)
23. Answer B: The primary somatomotor cortex consists of the
precentral gyrus and the anterior paracentral gyrus; area 4 is found
in these structures. Areas 3, 1, and 2 are the primary somatosensory cortex; areas 5 and 7 make up the superior parietal lobule and
the precuneus; and area 6 is located rostral to area 4. Portions of
area 6 in the caudal region of the middle frontal gyrus are the
frontal eye field. (p. 15)
24. Answer A: In this patient, the meningioma is located in the falx
cerebri and is impinging on the anterior paracentral gyrus correlating with her motor deficit. The lower extremity is represented
in the anterior paracentral gyrus (somatomotor) and in the posterior paracentral gyrus (somatosensory). The precentral gyrus contains the motor representation for the face (lateral part) and the
trunk and hip (medial part). The postcentral gyrus is part of the
somatosensory cortex. (p. 15)
25. Answer D: The M4 segments of the middle cerebral artery serve
the lateral aspect of the cerebral hemisphere. The named M4 vessels that serve the pre- and postcentral gyri (hemorrhage into approximately the lower two-thirds of these gyri explain the motor
and sensory deficits) are the precentral branches (prerolandic),
central branches (Rolandic branches), and anterior parietal
branches. The M2 segment serves the insular cortex, and the M3
segment serves the inner surface of the frontal, parietal, and temporal opercula. The A1 segment serves hypothalamic structures,
the subcallosal and septal areas, and adjacent structures. P4 serves
the medial aspect of the occipital lobe (visual cortex). (p. 19, 29)
26. Answer E: The uncus is a small elevation at the rostral and medial aspect of the parahippocampal gyrus adjacent to the crus cerebri of the midbrain. In addition to the catastrophic effect of decerebration, herniation of the uncus may also affect corticospinal
and corticonuclear (corticobulbar) fibers in the crus cerebri and
the root of the oculomotor nerve. None of the other areas of the
forebrain listed as choices is related to uncal herniation. (p. 20, 22)
27. Answer E: The P4 segments of the posterior cerebral artery consist of the parieto-occipital and calcarine branches; the latter being
located in the calcarine sulcus and a primary blood supply to the
primary visual cortex. M3 and M4 segments of the middle cerebral
are located, respectively, on the inner aspect of the frontal, parietal, and temporal opercula and on the lateral aspect of the cere-
260 Q & A’s: A Sampling of Study and Review Questions with Explained Answers
bral hemisphere. The P2 segment of the posterior cerebral artery is
located just distal to the posterior communicating–posterior cerebral intersection and gives rise to medial and lateral posterior
choroidal and to thalamogeniculate arteries. The A1 segment is located between the internal carotid and anterior communicating
artery and gives rise to branches that serve anterior hypothalamic
structures, septal areas, and the optic chiasm. (p. 21, 29)
28. Answer B: The glomus choroideum is found in the atrium of the
lateral ventricle. This part of the choroid plexus is rostrally continuous with that in the body of the lateral ventricle and continuous
anteroinferiorly with that in the temporal horn. The roof of the
third ventricle has a small portion of choroid plexus that is continuous with that in the body of the ventricle via the interventricular
foramen. The anterior horn contains no choroid plexus. (p. 52)
29. Answer E: Trauma is the most common cause of subarachnoid
hemorrhage (SAH). The most common cause of spontaneous (also
called nontraumatic) SAH is bleeding from a ruptured aneurysm
(about 75% of all spontaneous cases). Bleeding from an arteriovenous malformation (AVM) is an infrequent cause of SAH (about
5% of cases), and bleeding from brain tumors into the subarachnoid space is rare. Meningiomas are usually slow-growing tumors
that may have a rich vascular supply but rarely hemorrhage spontaneously. (p. 46)
30. Answer D: The prepontine cistern is located external to the
basilar pons and contains the abducens nerve, basilar artery, origin of the anterior inferior cerebellar artery, and small perforating
arteries and veins. The ambient cistern is located on the lateral aspect of the midbrain and contains the trochlear nerve and several
major arteries. The premedullary cistern is located at the anterior
surface of the medulla and contains the anterior spinal artery. The
inferior cerebellopontine cistern contains the glossopharyngeal,
vagus, and accessory nerves. The superior cerebellopontine cistern contains the trigeminal, facial, and vestibulocochlear nerves
plus a short segment of the trochlear nerve. (p. 50, 51)
31. Answer B: Lenticulostriate arteries, also called the lateral striate arteries, originate from the M1 segment of the middle cerebral
artery and serve much of the lenticular nucleus and adjacent parts
of the internal capsule. A1 branches serve the anterior hypothalamus and optic chiasm, and M2 branches serve the insular cortex.
The P1 and P2 segments give rise to many small perforating
branches and to the thalamoperforating and quadrigeminal arteries (P1), medial and lateral posterior choroidal arteries, and the
thalamogeniculate artery (P2). (p. 25, 49, 242)
32. Answer A: The gyrus rectus is located on the inferior and medial aspect of the frontal lobe. It is separated from the orbital gyri
by the olfactory sulcus in which the olfactory bulb and tract is located. None of the other lobes has a direct relationship to the gyrus
rectus. (p. 20, 22)
33. Answer B: The optic tract lies immediately on the surface of the
crus cerebri, a relationship frequently seen in MRI. The fact that this
patient has a right-sided weakness of the extremities specifies that the
lesion is in the left crus cerebri. The bilateral visual deficits correlate
with damage to the left optic tract. Lesions of the left basilar pons and
pyramid would result in a right-sided weakness but no visual deficits.
A lesion in the right optic nerve would result in blindness in that eye
but no weakness of the extremities. (p. 20, 26, 40, 220–221)
34. Answer E: Cerebrospinal fluid in the ventricles, and throughout
the subarachnoid space, appears very white in T2-weighted MRI
images. Structures located in, or traversing the subarachnoid
space (such as vessels or cranial nerve roots, including the oculomotor nerve) appear grey to black against a white background. (p.
46–47, 49, 51, 54)
35. Answer A: Cerebrospinal fluid, and other fluids, appear black in
T1-weighted MRI images. Consequently, the ventricles, and more
obvious parts of the subarachnoid space, appear black. Changes in
ventricular shape (i.e., enlargement, midline shift), or obliterated
sulci, or even subarachnoid space, most likely represent a potentially serious clinical issue. (p. 2–4, 33 as one example)
36. Answer E: Patients who experience rupture of an intracranial
aneurysm frequently complain of an intense, sudden headache
(“the most horrible headache I have ever had”). Acute blood in the
subarachnoid space will appear white to very white on CT. This
will contrast with the medium grey of the brain and the black of
cerebrospinal fluid (CSF) in the ventricles. The degree of white
may vary somewhat, based on the relative concentration of blood,
from very white (concentrated blood) to white (mostly blood,
some CSF), to very light grey (mixture of blood and CSF). (p.
46–47, 51)
37. Answer C: The hypoglossal nerve exits the medulla via the preolivary sulcus of the medulla immediately (and laterally) adjacent
to the pyramid. The abducens nerve exits in line with the preolivary sulcus, but, at the caudal edge of the pons, and the trigeminal nerve exits the lateral aspect of the pons. The vagus nerve exits the lateral aspect of the medulla via the postolivary sulcus, and
the facial nerve in line with this sulcus, but at the pons-medulla
junction. (p. 24, 44)
38. Answer E: Approximately one-half of cases of bacterial meningitis in adults are caused by S. pneumoniae. E. coli and L. monocytogenes are causative agents in neonates and children, although the
latter (L. monocytogenes) is present in less than 10% of cases. While
H. influenzae was a major cause of bacterial meningitis in children,
the use of a vaccine has reduced this bacterium as a causative agent
to well under 10% of cases. H. simplex is a virus. (p. 46)
39. Answer D: The trochlear nerve exits the posterior (dorsal) aspect of the brainstem just caudal to the inferior colliculus and
passes around the lateral aspect of the midbrain in the ambient cistern, en route to its exit from the skull via the superior orbital fissure. The abducens nerve exits at the caudal edge of the pons in
line with the preolivary fissure, and the hypoglossal exits from the
medulla via this fissure. The trigeminal nerve exits the lateral aspect of the pons, and the vestibulocochlear nerve exits at the most
lateral aspect of the pons-medulla junction. (p. 26, 33, 34)
40. Answer B: As it exits the anterior (ventral) surface of the midbrain, the oculomotor nerve passes between the superior cerebellar artery (which is caudal to the nerve root) and the P1 segment
of the posterior cerebral artery (which is rostral to the nerve root).
The trigeminal root is adjacent to more distal portions of the superior cerebellar artery; the labyrinthine artery accompanies the
vestibulocochlear nerve as it enters the internal acoustic meatus;
and the ophthalmic artery accompanies the optic nerve along part
of its extent. The abducens nerve passes rostrally adjacent to the
basilar artery in the prepontine cistern. (p. 25, 39, 40)
Q & A’s: A Sampling of Study and Review Questions with Explained Answers 261
41. Answer D: The rostral edge of the striae medullares (of the
fourth ventricle) is regarded as the border between the pontine
and medullary portions of the fourth ventricle. These fibers pass
from the median fissure in the floor of the ventricle laterally into
the lateral recess where they arch up into the cerebellum. The facial colliculus and median eminence are located in the floor of the
pontine portion of the ventricle, and the vagal and hypoglossal
trigones are found in the medial floor of the medullary portion of
the fourth ventricle. (p. 34, 35, 36)
42. Answer D: The primary fissure is the deepest fissure in the cerebellum and it separates the anterior lobe from the posterior lobe
and extends from the vermis to the lateral cerebellar margin. The
posterolateral fissure is located between the flocculonodular lobe
and the posterior lobe. The horizontal, secondary, and posterior
superior fissures are all located within the posterior lobe. (p. 32,
33)
43. Answer C: The tonsil of the cerebellum is found on the caudal
and inferior aspect of the cerebellar hemisphere, adjacent to the
midline and immediately posterior (dorsal) to the medulla. The
cisterna magna is located in this area. Sudden tonsillar herniation
may compress the medulla and damage respiratory and cardiac
centers resulting in sudden death. The tonsil herniates downward
through the foramen magnum. Consequently, no other part of the
brainstem is directly affected. (p. 32, 44)
Review and Study Questions for
Chapters 3 and 4
1. A 47-year-old woman presents with signs of increased intracranial
pressure (vomiting, headache, lethargy). MRI shows a large tumor
invading the head of the caudate nucleus, the rostral portion of the
putamen and involving a fiber bundle located between these two
structures. This fiber bundle is most likely the
䊊 (A) anterior commissure
䊊 (B) anterior limb of the internal capsule
䊊 (C) column of the fornix
䊊 (D) external capsule
䊊 (E) posterior limb of the internal capsule
2. A 76-year-old woman is diagnosed as having “probable”
Alzheimer’s disease based on a steady decline in cognitive function. It is likely that this woman has cell dropout in the nucleus accumbens. Which of the following most specifically describes the
location of this cell group?
䊊 (A) At the junction of the caudate head and putamen
䊊 (B) At the junction of the pallidum and putamen
䊊 (C) At the junction of the pallidum and substantia nigra
䊊 (D) In the anterior wall of the temporal horn
䊊 (E) Internal to the uncus
44. Answer A: A CT is a fast method, does not require sedation of
young patients, and shows bone fractures and acute intracranial
blood in detail. MRI (T1- and T2-weighted) does not show acute
blood or bone fracture to advantage, takes much longer to do, and
may require sedation in a child. Enhanced MRI is uniquely useful
for tumors, and PET is useful in identifying metabolic activity of
brain tissue, not anatomic detail. (p. 4–6)
3. Which of the following structures is located in the medial wall of
the temporal horn of the lateral ventricle and, if severely damaged, may result in memory deficits?
䊊 (A) Amygdaloid complex
䊊 (B) Calcar avis
䊊 (C) Hippocampus
䊊 (D) Pulvinar
䊊 (E) Tail of the caudate
45. Answer B: The long and short gyri (gyri longi et breves) are
components of the insular lobe. This lobe is located deep to the
lateral sulcus, has a central sulcus that separates the short gyri (rostral to this sulcus) from the long gyri (caudal to this sulcus). The
cortex of the insular lobe is separated from the adjacent frontal,
parietal, and temporal opercula by the circular sulcus of the insula.
None of the other lobes has gyri that are specifically named long
and short gyri. (p. 13, 45, 56)
4. Which of the following represents the fibers that fan out from the
internal capsule into the white matter of the hemisphere?
䊊 (A) Cingulum
䊊 (B) Corona radiata
䊊 (C) Genu of the corpus callosum
䊊 (D) Superior longitudinal fasciculus
䊊 (E) Uncinate fasciculus
46. Answer C: The glossopharyngeal nerve contains the afferent
limb of the gag reflex and, through its innervation of the stylopharyngeus muscle, is an important part of the efferent limb of this
reflex. The nucleus ambiguus, the location of the motor neurons
serving the stylopharyngeus, also contributes to the innervation of
muscle served by the vagus nerve and, therefore, to the efferent
limb of the gag reflex. The trigeminal and facial nerves participate
in the afferent and efferent limbs (respectively) of the corneal reflex. The accessory nerve innervates the ipsilateral trapezius and
sternocleidomastoid muscles, and the hypoglossal nerve innervates the ipsilateral genioglossus muscle. (p. 24, 43)
5. The lamina of white matter located immediately internal to the
cortex of the insula is the:
䊊 (A) Arcuate fasciculus
䊊 (B) External capsule
䊊 (C) Extreme capsule
䊊 (D) Internal capsule
䊊 (E) Tapetum
6. A 48-year-old man presents with a movement disorder (chorea)
and mental deterioration. MRI shows the loss of a structure in the
wall of the anterior horn of the lateral ventricle. Which of the following is most likely lost in this patient?
䊊 (A) Anterior thalamic nucleus
䊊 (B) Body of the caudate nucleus
䊊 (C) Column of the fornix
䊊 (D) Dorsomedial nucleus
䊊 (E) Head of the caudate nucleus
262 Q & A’s: A Sampling of Study and Review Questions with Explained Answers
7. A 76-year-old man presents with a resting tremor, bradykinesia,
and stooped posture. These observations suggest loss of a prominent population of cells in the brain. Which of the following structures is most likely affected in this patient?
䊊 (A) Lateral cerebellar nucleus
䊊 (B) Locus ceruleus
䊊 (C) Red nucleus
䊊 (D) Substantia nigra
䊊 (E) Subthalamic nucleus
8. Which of the following represents the larger, more laterally located portion of the basal nuclei (also called the basal ganglia)?
䊊 (A) Caudate nucleus
䊊 (B) Globus pallidus
䊊 (C) Putamen
䊊 (D) Subthalamic nucleus
䊊 (E) Substantia nigra
9. The MRI of a 59-year-old woman shows a large arteriovenous
malformation (AVM) located between the lenticular nucleus and
the dorsal thalamus. Based on its location, this AVM most likely
involves which of the following structures?
䊊 (A) Anterior limb of the internal capsule
䊊 (B) Crus cerebri
䊊 (C) External capsule
䊊 (D) Posterior limb of the internal capsule
䊊 (E) Retrolenticular limb of the internal capsule
10. A 29-year-old man is brought to the emergency department with
a severe and persistent headache. MRI shows a large tumor of the
pineal gland. Based on its location, this pineal lesion would most
likely impinge on which of the following structures?
䊊 (A) Anterior thalamic nucleus
䊊 (B) Body of the caudate nucleus
䊊 (C) Globus pallidus
䊊 (D) Pulvinar nucleus(i)
䊊 (E) Ventral posteromedial nucleus
11. The hippocampal commissure contains fibers from one hippocampal formation that cross the midline to distribute to targets on the
opposite side of the hemisphere. Which of the following structures is directly adjacent to this commissure?
䊊 (A) Body of the corpus callosum
䊊 (B) Genu of the corpus callosum
䊊 (C) Splenium of the corpus callosum
䊊 (D) Spiral fibers of the hippocampus
䊊 (E) Precommissural fornix
12. An 85-year-old woman is brought to the emergency department
by her family because she suddenly became confused and lethargic. CT shows a hemorrhage into the medial and lateral geniculate
bodies. Which of the following structures would also likely be involved in this lesion due to its apposition to the geniculate bodies?
䊊 (A) Anterior thalamic nucleus
䊊 (B) Rostral dorsomedial nucleus
䊊 (C) Globus pallidus
䊊 (D) Pulvinar nucleus(i)
䊊 (E) Subthalamic nucleus
13. A 29-year-old woman presents with neurologic deficits that wax and
wane over time suggestive of multiple sclerosis. MRI (especially T2weighted) shows small, demyelinated areas at several locations in
her brain, one of these being the mammillothalamic tract. Which of
the following structures is most intimately associated with this tract?
䊊 (A) Anterior thalamic nucleus
䊊 (B) Centromedian nucleus
䊊 (C) Dorsomedial nucleus
䊊 (D) Ventral anterior thalamic nucleus
䊊 (E) Ventral lateral thalamic nucleus
14. Which of the following structures is a primary target of the optic
tract as it passes caudally from the optic chiasm?
䊊 (A) Lateral geniculate nucleus
䊊 (B) Mammillary body
䊊 (C) Medial geniculate nucleus
䊊 (D) Pulvinar
䊊 (E) Ventral posterolateral nucleus
15. An 82-year-old man presents with a severe motor deficit (resting
tremor) and dementia. The former correlates with degenerative
changes in the putamen and globus pallidus and the latter with degenerative changes in the ventral striatum and ventral pallidum.
Which of the following structures separates these two areas in the
basal forebrain?
䊊 (A) Anterior commissure
䊊 (B) Lamina terminalis
䊊 (C) Massa intermedia
䊊 (D) Posterior commissure
䊊 (E) Septum pellucidum
16. A 23-year-old man is brought to the emergency department by
emergency medical personnel after an automobile collision. CT
shows bilateral damage to the temporal pole and the uncus. Which
of the following structures is also most likely damaged in this patient?
䊊 (A) Amygdaloid complex
䊊 (B) Anterior thalamic nucleus
䊊 (C) Cingulum
䊊 (D) Gracile nucleus
䊊 (E) Hippocampal formation
17. The optic radiations are closely associated with which of the following spaces?
䊊 (A) Anterior horn of the lateral ventricle
䊊 (B) Body of the lateral ventricle
䊊 (C) Cisterns adjacent to the midbrain
䊊 (D) Posterior horn of the lateral ventricle
䊊 (E) Third ventricle
18. A 31-year-old man presents with ill-defined neurologic complaints (persistently tired, headache, confusion). CT shows an arteriovenous malformation occupying most of the dorsomedial nucleus (DM) of the thalamus. Which of the following structures
separates the DM from the lateral thalamic nuclei and encompasses the centromedial nucleus?
䊊 (A) Ansa lenticularis
䊊 (B) External medullary lamina
䊊 (C) Internal medullary lamina
䊊 (D) Lamina terminalis
䊊 (E) Stria medullaris thalami
Q & A’s: A Sampling of Study and Review Questions with Explained Answers 263
19. A 48-year-old woman presents with violent, flailing movements
of her left upper extremity. CT shows a small hemorrhage in the
subthalamic nucleus. Which of the following structures is located
directly adjacent to the subthalamic nucleus?
䊊 (A) Centromedian nucleus
䊊 (B) Globus pallidus
䊊 (C) Medial geniculate nucleus
䊊 (D) Putamen
䊊 (E) Substantia nigra
5. Answer C: The layer of white matter located internal to the insular cortex, and external to the claustrum, is the extreme capsule. The external capsule is found between the claustrum and the
putamen, and the internal capsule is a large bundle of fibers located
primarily between the lenticular nucleus on one side and the head
of the caudate and the diencephalon on the other side. The tapetum is located in the lateral wall of the posterior horn of the lateral ventricle. Arcuate fasciculi are small bundles of fibers passing
between gyri. (p. 65, 67–68, 77)
20. Which of the following structures is located immediately caudal to
the anterior commissure and appears as a distinct black spot in a
T2-weighted axial MRI?
䊊 (A) Anterior limb of internal capsule
䊊 (B) Column of the fornix
䊊 (C) Crus of the fornix
䊊 (D) Lenticular fasciculus
䊊 (E) Mammillothalamic tract
6. Answer E: The large bulge in the lateral wall of the anterior horn
of the lateral ventricle is the head of the caudate nucleus. The position of the interventricular foramen represents the point at which
the head of the caudate becomes the body of the caudate. The dorsomedial nucleus borders on the third ventricle; the anterior thalamic nucleus is located at the rostral end of the diencephalon and is
caudomedial to the interventricular foramen; and the column of
the fornix is rostromedial to this foramen. (p. 64–65, 76)
Answers for Chapters 3 and 4
1. Answer B: The anterior limb of the internal capsule is insinuated between the head of the caudate nucleus and the rostral aspect of the lenticular nucleus, mostly the putamen. The posterior
limb is between the lenticular nucleus and the thalamus; the column of the fornix is rostromedial to the interventricular foramen;
and the anterior commissure traverses the midline at the level of
the genu of the internal capsule. The external capsule is a thin
sheet of white matter lateral to the lenticular nucleus and medial
to the claustrum. (p. 64–65, 76–77)
2. Answer A: The nucleus accumbens is located in the rostral and
basal forebrain at the point where the head of the caudate is continuous with the putamen. The amygdaloid nucleus is located internal to the uncus and in the anterior wall of the temporal horn.
The pallidum (globus pallidus) and the substantia nigra do not have
a continuum with the nucleus accumbens. (p. 64, 78)
3. Answer C: The hippocampal formation, commonly called the
hippocampus, is located in the medial wall of the temporal (inferior) horn of the lateral ventricle. Damage to the hippocampus
may result in memory problems. The amygdaloid complex is located in the rostral wall of the temporal horn, the tail of the caudate in its lateral wall, and the calcar avis (also called the calcarine
spur, a ridge in the wall of the posterior horn indicating the depth
of the calcarine sulcus) is in the medial wall of the posterior horn
of the lateral ventricle. The pulvinar is part of the diencephalon.
(p. 58, 68–71)
4. Answer B: The corona radiata (radiating crown) are those fibers
of the internal capsule that fan out in all directions from its superior edge. These fibers contain a variety of fibers traveling to and
from the cerebral cortex. The superior longitudinal and uncinate
fasciculi are organized bundles of corticocortical fibers on the ipsilateral side, and the cingulum is a fiber bundle located internal to
the cingulate cortex. The fibers of the genu of the corpus callosum contain corticocortical fibers that pass between the cerebral
hemispheres. (p. 57, 65–69)
7. Answer D: These deficits are characteristic of Parkinson’s disease and are directly correlated with loss of the dopamine (and
melanin)-containing cells of the substantia nigra of the midbrain.
The locus (nucleus) ceruleus, also called the nucleus pigmentosus
pontis, also contains cells with melanin, but loss of these cells does
not cause motor deficits. The other choices do not contain pigmented cells, but damage to these structures does cause a different series of motor deficits. (p. 68–69, 78)
8. Answer C: The putamen is the most lateral part of the basal nuclei; taken together, the putamen and the globus pallidus comprise
the lenticular nucleus. The caudate nucleus, specifically its head
and body portions, is located medial to the internal capsule. While
the subthalamic nucleus and the substantia nigra function in concert with the basal nuclei, these structures are medially located
and are not part of the basal nuclei. (p. 65–68, 76)
9. Answer D: The posterior limb of the internal capsule, containing important cortical afferent and efferent fibers, is located between the lenticular nucleus and the dorsal thalamus. Damage to
this structure may result in sensory and/or motor deficits on the
opposite side of the body. The anterior limb is located between
the head of the caudate and the putamen, while the retrolenticular limb is found caudal to the lenticular nucleus. The crus cerebri
is on the inferolateral aspect of the midbrain. The external capsule
is lateral to the putamen. (p. 67–69, 76–77)
10. Answer D: The pineal gland is located in the quadrigeminal cistern, superior to the colliculi, and between the pulvinar nuclei of
the thalamus. At this location, the lesion would potentially involve
the colliculi and pulvinar. The other thalamic nuclei are not adjacent to the pineal, the globus pallidus is lateral to the posterior
limb of the internal capsule, and the body of the caudate is located
in the lateral wall of the body of the lateral ventricle. (p. 71)
11. Answer C: The hippocampal commissure is located immediately inferior to the splenium of the corpus callosum; the crossing
of these fibers takes place at this point. Other parts of the corpus
callosum are not related to the hippocampal commissure, and the
spiral fibers of the hippocampus are bundles within the hippocampal formation in the temporal lobe. Some of the fibers in the hippocampal commissure enter the precommissural fornix, but by no
means all. (p. 72)
264 Q & A’s: A Sampling of Study and Review Questions with Explained Answers
12. Answer D: The geniculate bodies are tucked-up under the caudal and inferior aspect of the pulvinar. The groove between the
medial geniculate body and the pulvinar contains the brachium of
the superior colliculus. The geniculate bodies and the pulvinar
have a common blood supply from the thalamogeniculate artery,
a branch of P2. None of the other choices have a close apposition
with the geniculate bodies. The anterior thalamic, rostral dorsomedial, and subthalamic nuclei do not share a common blood supply with the pulvinar. (p. 58–59, 70)
13. Answer A: The mammillothalamic tract extends from the
mammillary bodies to the anterior nucleus of the thalamus; the
cells of origin are in the mammillary nuclei and the axons terminate in the anterior nucleus. This tract is frequently visible in axial T2-weighted MRI. The ventral anterior nucleus is laterally adjacent to the mammillothalamic tract, but does not receive input
therefrom. The other choices are nuclei located more caudally in
the diencephalon. (p. 67, 77)
14. Answer A: Many of the fibers contained in the optic tract terminate in the lateral geniculate nucleus. Some of these fibers bypass this nucleus to traverse the brachium of the superior colliculus and a few enter the suprachiasmatic nucleus. The medial
geniculate nucleus receives input via the brachium of the inferior
colliculus (auditory); the pulvinar has interconnections with the
visual cortex and superior colliculus; and the ventral posterolateral nucleus receives input from the anterolateral system and the
medical lemniscus. The mammillary body is located rostral to the
interpeduncular fossa and medial to the optic tract. (p. 58, 59)
15. Answer A: The anterior commissure, as it passes laterally from
the midline, separates the dorsal basal nuclei (putamen and globus
pallidus) from the ventral striatum and ventral pallidum. The posterior commissure is located at the caudal aspect of the third ventricle just above the opening of the cerebral aqueduct, and the
Massa intermedia bridges the third ventricle in about 80% of individuals. The rostral wall of the third ventricle is formed by the
lamina terminalis and the septum pellucidum forms the medial
wall of the anterior horn of the lateral ventricle. (p. 65, 152–153)
16. Answer A: The amygdaloid complex is located immediately internal to the uncus. Bilateral damage to rostral portions of the
temporal lobe may include the amygdala and result in a constellation of deficits known as the Klüver-Bucy syndrome. The hippocampal formation is internal to the cortex of the parahippocampal gyrus, and the anterior thalamic nucleus is internal to
the anterior thalamic tubercle. The cingulate gyrus overlies the
longitudinally oriented fibers of the cingulum and the gracile tubercle is the external elevation formed by the gracile nucleus. (p.
58–59, 65–66, 78, 170)
17. Answer D: The optic radiations are located in the lateral wall of
the posterior horn of the lateral ventricle as they pass through the
retrolenticular limb of the internal capsule from the lateral geniculate nucleus to the primary visual cortex. A thin layer of white
matter, the tapetum, separates the optic radiations from the wall
of the ventricle. The cisterns at the midbrain on the basal aspect of
the hemisphere contain the optic tract. The other ventricular
spaces listed have no direct relationship to the optic radiations. (p.
71–72, 77, 138–141, 162)
18. Answer C: The internal medullary lamina is a vertically oriented sheet of fibers that extends from the rostral portion of the
thalamus caudally to surround the centromedian nucleus; this nucleus is frequently referred to as “in” the internal medullary lamina due to its position. This lamina separates the dorsomedial nucleus from the laterally adjacent ventral anterior, ventral lateral,
and ventral posterolateral nuclei. The external medullary lamina
is located between the thalamus and the posterior limb of the internal capsule, and the lamina terminalis is the rostral wall of the
third ventricle. The stria medullaris is a small bundle of fibers passing rostrocaudally along the upper medial edge of the thalamus
from the general location of the interventricular foramen to the
habenula, and the ansa lenticularis contains pallidothalamic fibers.
(p. 68–69, 76, 144–149, 162)
19. Answer E: The subthalamic nucleus is separated from the substantia nigra by only a thin layer of myelinated fibers; these two
structures are directly adjacent to each other. Damage to the subthalamic nucleus gives rise to hemiballistic movements (described
in this question) while loss of cells in the substantia nigra results in
the motor deficits seen in Parkinson’s disease. The putamen,
globus pallidus, and the medial geniculate nucleus are all lateral to
the internal capsule; the subthalamic nucleus is medial. The centromedial nucleus is separated from the subthalamic nucleus by
other thalamic nuclei. (p. 68–69, 148–149)
20. Answer B: The column of the fornix is that portion of this
fiber bundle that arches around the rostromedial end of the thalamus. As it does so, the column joins its counterpart on the opposite side and “leans against” the anterior commissure. The column of the fornix also signifies, in cross section or axial planes,
the laterally adjacent interventricular foramen and genu of the
internal capsule. The mammillothalamic tract is located caudal
to the fornix, and the crus of the fornix is found along the midline superior to the thalamus. The anterior limb of the internal
capsule is found between the head of the caudate nucleus and the
lenticular nucleus (mainly the putamen). The lenticular fasciculus contains pallidothalamic fibers and traverses the posterior
limb of the internal capsule en route to the dorsal thalamus. (p.
31, 77, 163–164)
Review and Study Questions for
Chapter 5
1. A 16-year-old boy is brought to the emergency department following a diving accident at a local quarry. The examination reveals
a bilateral loss of motor and sensory function in the trunk and
lower extremities. At 36 hours after the accident the boy is able
to dorsiflex his toes, barely move his right lower extremity at the
knee, and is able to perceive pinprick stimulation of the perianal
skin (sacral sparing). Which of the following most specifically describes the spinal cord lesion in this patient?
䊊 (A) Central cord
䊊 (B) Complete
䊊 (C) Hemisection
䊊 (D) Incomplete
䊊 (E) Large syringomyelia
Q & A’s: A Sampling of Study and Review Questions with Explained Answers 265
2. A 54-year-old morbidly obese and hypertensive man is brought to
the emergency department after experiencing sudden onset of
weakness of his left upper and lower extremities. CT shows an infarcted area in the medulla. Damage to which of the following
tracts or fiber bundles of the medulla would most likely explain
this deficit?
䊊 (A) Anterolateral system
䊊 (B) Corticospinal fibers
䊊 (C) Medial lemniscus
䊊 (D) Rubrospinal tract
䊊 (E) Vestibulospinal fibers
3. A 78-year-old healthy, active woman experiences a sudden weakness of her right upper extremity during an angiogram to determine the patency of her carotid bifurcation. The immediate examination reveals weakness of both extremities on the right and a
partial loss of vision in both eyes (homonymous hemianopsia).
These observations suggest an embolic stroke resulting in a lesion
involving motor and visual structures. The infarcted area in CT
points to the occlusion of one vessel. Which of the following vessels is most likely occluded?
䊊
䊊
䊊
䊊
䊊
(A)
(B)
(C)
(D)
(E)
Anterior cerebral artery
Anterior choroidal artery
Ophthalmic artery
Lateral posterior choroidal artery
Posterior cerebral artery (P3 and P4 segments)
4. A 69-year-old man is brought to the emergency department by his
wife after complaining of a bad headache and becoming stuporous.
CT shows a hemorrhage into the head of the caudate nucleus that
has ruptured into the anterior horn of the lateral ventricle. This
hemorrhage has most likely originated from which of the following vessels?
䊊
䊊
䊊
䊊
䊊
(A)
(B)
(C)
(D)
(E)
Anterior choroidal artery (branch of internal carotid)
Lenticulostriate branches (of M1)
Medial posterior choroidal artery (branch of P2)
Medial striate artery (branch of A2)
Thalamoperforating artery(ies)
Questions 5 and 6 are based on the following patient.
A 23-year-old man is brought to the emergency department from the
site of an automobile collision. The neurologic examination reveals
weakness of the right lower extremity and a loss of pain and thermal
sensations on the left side beginning at the level of the umbilicus. CT
shows a fracture of the vertebral column with displacement of bone
fragments into the vertebral canal.
5. Damage to which of the following tracts would correlate with
weakness of the lower extremity in this man?
䊊
䊊
䊊
䊊
䊊
(A)
(B)
(C)
(D)
(E)
Left lateral corticospinal tract
Reticulospinal fibers on the right
Right lateral corticospinal tract
Right rubrospinal tract
Vestibulospinal fibers on the right
6. Which of the following represents the most likely level of damage
to the spinal cord resulting from the fracture to the vertebral column in this man?
䊊 (A) T6 on the left
䊊 (B) T8 on the left
䊊 (C) T8 on the right
䊊 (D) T10 on the left
䊊 (E) T10 on the right
7. The artery of Adamkiewicz is an especially large spinal medullary
artery supplementing the arterial blood supply to the spinal cord.
Which of the following represents the most consistent location of
this vessel?
䊊 (A) At C7–C8 on the left
䊊 (B) At L5–S1 on the left
䊊 (C) At L5–S1 on the right
䊊 (D) At T6–T7 on the right
䊊 (E) At T12–L1 on the left
8. The CT of a 73-year-old woman shows an infarcted area in the rostral portions of the dorsomedial nucleus, the anterior nucleus, and
the ventral anterior nucleus. Which of the following arteries supply blood to this area of the brain?
䊊 (A) Anterior choroidal
䊊 (B) Lateral striate (lenticulostriate)
䊊 (C) Medial striate
䊊 (D) Thalamogeniculate
䊊 (E) Thalamoperforating
9. Which of the following structures is insinuated between the external and extreme capsules and is functionally related to the insular cortex?
䊊 (A) Claustrum
䊊 (B) External medullary lamina
䊊 (C) Lamina terminalis
䊊 (D) Putamen
䊊 (E) Stria terminalis
10. An 83-year-old man is brought to the emergency department by his
daughter, who explains that her father started having “fits”. The examination reveals an alert, otherwise healthy, man who frequently
has uncontrollable flailing movements of his left arm. Which of the
following structures is most likely involved in this lesion?
䊊 (A) Cerebellar cortex plus nuclei
䊊 (B) Lenticular nucleus
䊊 (C) Subthalamic nucleus
䊊 (D) Ventral lateral nucleus
䊊 (E) Ventral posterolateral nucleus
11. A 17-year-old girl presents with a bilateral loss of pain and thermal sensations at the base of the neck (C3 dermatome) and extending over the upper extremity and down to the level of the nipple (C4 to T4 dermatomes). MRI shows a cavitation in the spinal
cord at these levels. Damage to which of the following structures
would most likely explain this deficit?
䊊 (A) Anterior white commissure
䊊 (B) Left anterolateral system
䊊 (C) Medial longitudinal fasciculus
䊊 (D) Posterior columns
䊊 (E) Right anterolateral system
266 Q & A’s: A Sampling of Study and Review Questions with Explained Answers
12. Which of the following structures is located within the territory
of the medulla that is served by the anterior spinal artery?
䊊
䊊
䊊
䊊
䊊
(A)
(B)
(C)
(D)
(E)
Anterolateral system
Gracile fasciculus
Medial lemniscus
Rubrospinal tract
Spinal trigeminal tract
13. A 59-year-old man complains to his family physician that he has
trouble chewing. The examination reveals a weakness of masticatory muscles on the left side. Which of the following nuclei is
specifically related to the deficit seen in this man?
䊊
䊊
䊊
䊊
䊊
(A)
(B)
(C)
(D)
(E)
Left facial motor
Left hypoglossal
Left trigeminal motor
Right facial motor
Right trigeminal motor
14. A 15-year-old boy with signs of increased intracranial pressure
(stupor, vomiting, headache) is referred to a neurologist. The examination reveals a paralysis of upward gaze, and MRI shows a
large tumor of the pineal gland. Damage to which of the following structures would be most specifically related to the gaze
deficit?
䊊
䊊
䊊
䊊
䊊
(A)
(B)
(C)
(D)
(E)
Exit of the trochlear nerve
Inferior colliculus
Occlusion of the great cerebral vein
Posterior commissure
Superior colliculus
15. A 61-year-old man is brought to the emergency department after
a fall from his garage roof. The examination reveals a hemiplegia
on the left, a loss of vibratory sense on the left, and a loss of pain
and thermal sensation on the right side involving the upper and
lower extremities. These deficits are characteristically seen in
which of the following syndromes?
䊊
䊊
䊊
䊊
䊊
(A)
(B)
(C)
(D)
(E)
Benedikt
Brown-Séquard
Claude
Wallenberg
Weber
16. Based on their relative locations in the spinal cord, which of the
following tracts or fiber bundles would most likely be involved in
a lesion located in the immediate vicinity of the lateral corticospinal tract?
䊊
䊊
䊊
䊊
䊊
(A)
(B)
(C)
(D)
(E)
Anterolateral system
Anterior spinocerebellar tract
Gracile fasciculus
Medial longitudinal fasciculus
Rubrospinal tract
17. A 92-year-old woman is brought to the emergency department by
her caregiver. The woman had suddenly become drowsy and confused. The examination revealed no cranial nerve deficits and agenormal motor function, but a loss of pain, thermal, vibratory, and
discriminative touch sensations on one side of the body excluding
the head. CT shows a small infarcted area. Which of the following
structures is the most likely location of this lesion?
䊊 (A) Anterolateral system
䊊 (B) Medial geniculate nucleus
䊊 (C) Subthalamic nucleus
䊊 (D) Ventral posterolateral nucleus
䊊 (E) Ventral posteromedial nucleus
18. In its location immediately internal to the anterior spinocerebellar tract, which of the following fiber bundles would most likely
be damaged in a lesion to this area of the spinal cord?
䊊 (A) Anterolateral system
䊊 (B) Anterior corticospinal tract
䊊 (C) Anterior white commissure
䊊 (D) Cuneate fasciculus
䊊 (E) Lateral corticospinal tract
19. A 37-year-old man is brought to the emergency department with
a severe head injury. Within a few hours he is decerebrate (upper
and lower extremities extended) and comatose. The extension of
his extremities indicates a dominant input to extensor motor neurons through vestibulospinal and reticulospinal fibers/tracts.
Which of the following most specifically describes the position of
these activated fibers within the spinal cord?
䊊 (A) Anterolateral area (area of anterolateral system)
䊊 (B) Posterolateral area (area of lateral corticospinal tract)
䊊 (C) Posterior columns
䊊 (D) Posterolateral (dorsolateral) tract
䊊 (E) Intermediate zone
Question 20 and 21 are based on the following patient.
A 71-year-old woman presents to her family physician with the complaint that “food dribbles out of my mouth when I eat”. The examination reveals a unilateral weakness of muscles around the eye (palpebral
fissure) and the opening of the mouth (oral fissure). She also has a loss
of pain and thermal sensations on the opposite side of the body excluding the head. CT shows an infarcted area in the lateral portion of
the pontine tegmentum.
20. Damage to which of the following nuclei would most likely explain the muscle weakness experienced by this woman?
䊊 (A) Abducens
䊊 (B) Arcuate
䊊 (C) Facial motor
䊊 (D) Hypoglossal
䊊 (E) Trigeminal motor
21. The loss of pain and thermal sensations experienced by this woman
would most likely correlate with a lesion involving which of the
following structures?
䊊 (A) Anterior (ventral) trigeminothalamic tract
䊊 (B) Anterolateral system
䊊 (C) Lateral lemniscus
䊊 (D) Medial lemniscus
䊊 (E) Spinal trigeminal tract
Q & A’s: A Sampling of Study and Review Questions with Explained Answers 267
22. A 77-year-old woman is discovered slumped on the floor in the grocery store; emergency medical personnel transport her to a local
hospital. The examination reveals a drowsy somewhat stuporous
woman who is difficult to arouse. CT shows a large hemorrhage
within the brain medial to the internal medullary lamina. Which of
the following structures is most likely involved in this lesion?
䊊 (A) Anterior thalamic nucleus
䊊 (B) Dorsomedial nucleus
䊊 (C) Globus pallidus
䊊 (D) Ventral lateral and anterior nuclei
䊊 (E) Ventral posterolateral nucleus
28. A 67-year-old woman is brought to the emergency department.
She is stuporous and has signs that suggest a lesion in the brainstem; CT confirms this. Her right pupil is constricted (small)
when compared with the left. Damage to which of the following
tracts or fiber bundles in the pons or medulla would most likely
explain this observation?
䊊 (A) Anterolateral system
䊊 (B) Hypothalamospinal fibers
䊊 (C) Medial longitudinal fasciculus
䊊 (D) Reticulospinal fibers
䊊 (E) Vestibulospinal fibers
23. A 78-year-old man presents with deficits suggesting an occlusion
of the posterior spinal artery at spinal cord levels C4-T2. Which
of the following structures are in the territory served by this vessel at these levels?
䊊 (A) Anterolateral system
䊊 (B) Cuneate fasciculus
䊊 (C) Gracile nucleus
䊊 (D) Lateral corticospinal tract
䊊 (E) Medial longitudinal fasciculus
29. In addition to the medial and lateral geniculate nuclei, which of the
following structures is also served by the thalamogeniculate
artery, a branch of P2?
䊊 (A) Anterior thalamic nucleus
䊊 (B) Globus pallidus
䊊 (C) Pulvinar nucleus(i)
䊊 (D) Substantia nigra
䊊 (E) Ventral anterior thalamic nucleus
24. Based partially on their embryological origin from a common
group of cells, which of the following combinations of structures
appear to be the same shade of grey in a T1- weighted MRI?
䊊 (A) Dorsomedial nucleus and Globus pallidus
䊊 (B) Globus pallidus and Caudate
䊊 (C) Globus pallidus and Putamen
䊊 (D) Putamen and Caudate nucleus
䊊 (E) Putamen and Pulvinar
25. Which of the following portions of the trigeminal nuclear complex
is found in lateral areas of the brainstem between the level of the
obex and the spinal cord-medulla junction and is the source of
trigeminothalamic fibers conveying pain and thermal information
originating from the face and oral cavity?
䊊 (A) Mesencephalic nucleus
䊊 (B) Principal sensory nucleus
䊊 (C) Spinal trigeminal nucleus, pars caudalis
䊊 (D) Spinal trigeminal nucleus, pars interpolaris
䊊 (E) Spinal trigeminal nucleus, pars oralis
26. Which of the following structures is located within the territory
served by branches of the posterior inferior cerebellar artery
(commonly called PICA by clinicians)?
䊊 (A) Corticospinal fibers
䊊 (B) Hypoglossal root
䊊 (C) Medial lemniscus
䊊 (D) Nucleus raphe magnus
䊊 (E) Solitary nucleus
27. Space-occupying lesions within the posterior cranial fossa, or
events that increase pressure within this infratentorial region, may
result in herniation of a portion of the cerebellum through the
foramen magnum. Which of the following parts of the cerebellum
is most likely involved in this event?
䊊 (A) Anterior lobe
䊊 (B) Flocculus
䊊 (C) Nodulus
䊊 (D) Simple lobule
䊊 (E) Tonsil
30. A 71-year- old man is brought to the emergency department by
his wife. She explains that he suddenly became weak in his left
lower extremity. She immediately rushed him to the hospital,
a trip of about 20 minutes. The examination reveals an alert
man who is obese and hypertensive. He has no cranial nerve
deficits, is slightly weak on his left side, and has no sensory
deficits. Within 2 hours the weakness has disappeared. An MRI
obtained the following day shows no lesions. Which of the following most specifically describes this man’s medical experience?
䊊 (A) Central cord syndrome
䊊 (B) Small embolic stroke
䊊 (C) Small hemorrhagic stroke
䊊 (D) Syringobulbia
䊊 (E) Transient ischemic attack
Questions 31 and 32 are based on the following patient.
A 41-year-old man is brought to the emergency department after an
accident at a construction site. The examination reveals a weakness
(hemiplegia) and a loss of vibratory sensation and discriminative touch
all on the left lower extremity, and a loss of pain and thermal sensations on the right lower extremity. CT shows a fracture of the vertebral column adjacent to the T8 level of the spinal cord.
31. Damage to which of the following fiber bundles or tracts would
most likely explain the loss of vibratory sensation in this man?
䊊 (A) Anterolateral system on the right
䊊 (B) Cuneate fasciculus on the left
䊊 (C) Cuneate fasciculus on the right
䊊 (D) Gracile fasciculus on the left
䊊 (E) Gracile fasciculus on the right
32. The loss of pain and thermal sensation in this man reflects damage
to which of the following fiber bundles or tracts?
䊊 (A) Anterolateral system on the left
䊊 (B) Anterolateral system on the right
䊊 (C) Cuneate fasciculus on the left
䊊 (D) Gracile fasciculus on the left
䊊 (E) Posterior spinocerebellar tract on the left
268 Q & A’s: A Sampling of Study and Review Questions with Explained Answers
33. Which of the following is the prominent population of melanincontaining cells located immediately internal to the crus cerebri?
The loss of these cells may result in motor deficits.
䊊 (A) Locus ceruleus
䊊 (B) Pontine nuclei
䊊 (C) Red nucleus
䊊 (D) Reticular formation
䊊 (E) Substantia nigra
34. Which of the following structures receives visceral sensory input
and is located immediately inferior to the medial and spinal
vestibular nuclei at medullary levels?
䊊 (A) Cochlear nuclei
䊊 (B) Inferior salivatory nucleus
䊊 (C) Nucleus ambiguus
䊊 (D) Spinal trigeminal nucleus
䊊 (E) Solitary nucleus
35. Which of the following groups of visceromotor (autonomic) cell
bodies is located lateral to the abducens nucleus, directly adjacent
to the exiting fibers of the facial nerve, and sends its axons out of
the brainstem via this cranial nerve?
䊊 (A) Dorsal motor nucleus
䊊 (B) Edinger-Westphal nucleus
䊊 (C) Inferior salivatory nucleus
䊊 (D) Intermediolateral cell column
䊊 (E) Superior salivatory nucleus
36. A 56-year-old woman presents to her family physician with persistent headache and nausea. MRI shows a tumor in the fourth ventricle impinging on the facial colliculus. Which of the following
nuclei is found immediately internal to this elevation?
䊊 (A) Abducens
䊊 (B) Facial
䊊 (C) Hypoglossal
䊊 (D) Trigeminal
䊊 (E) Vestibular
Questions 37 through 39 are based on the following patient.
An 88-year-old man is brought to the emergency department by his
daughter. She indicates that he complained of weakness of his “arm”
and “leg” (upper and lower extremities) on the right side and of “seeing two of everything” (double vision—diplopia). CT shows an infarcted area in the medial area of the pons at the pons-medulla junction. The infarcted area is consistent with the vascular territory served
by paramedian branches of the basilar artery.
37. Weakness of the extremities on the right can be explained by damage to which of the following structures?
䊊 (A) Corticospinal fibers on the left
䊊 (B) Corticospinal fibers on the right
䊊 (C) Middle cerebellar peduncle on the left
䊊 (D) Rubrospinal fibers on the left
䊊 (E) Rubrospinal fibers on the right
38. The diplopia (double vision) this man is having is most likely the
result of damage to which of the following structures?
䊊 (A) Abducens nerve root
䊊 (B) Facial nerve root
䊊 (C) Oculomotor nerve root
䊊 (D) Optic nerve
䊊 (E) Trochlear nerve or root
39. Recognizing that this patient’s lesion involves the territory served
by paramedian branches of the basilar artery, which of the following structures is also most likely included in the area of infarction?
䊊 (A) Anterolateral system
䊊 (B) Facial motor nucleus
䊊 (C) Hypoglossal nucleus
䊊 (D) Medial lemniscus
䊊 (E) Spinal trigeminal tract
40. A 77-year-old man presents with a weakness of his right upper and
lower extremities and he is unable to abduct his left eye on attempted gaze to the left. Which of the following most specifically
describes this deficit?
䊊 (A) Alternating hemianesthesia
䊊 (B) Hemihypesthesia
䊊 (C) Inferior alternating hemiplegia
䊊 (D) Middle alternating hemiplegia
䊊 (E) Superior alternating hemiplegia
41. In axial MRI which of the following structures is an important
landmark that separates the third ventricle (rostral to this point)
from the quadrigeminal cistern (caudal to this point)?
䊊 (A) Lamina terminalis
䊊 (B) Habenular nucleus
䊊 (C) Massa intermedia
䊊 (D) Pulvinar
䊊 (E) Superior colliculus
42. A 77-year-old woman presents with deficits that suggest a lesion
involving long tracts and a cranial nerve. CT shows an infarct in
the region served by the penetrating branches of the basilar bifurcation. Which of the following structures is most likely located in
this vascular territory?
䊊 (A) Abducens nerve
䊊 (B) Anterolateral system
䊊 (C) Corticospinal fibers in pyramid
䊊 (D) Medial lemniscus
䊊 (E) Red nucleus
Questions 43 through 46 are based on the following patient.
A 69-year-old man is brought to the emergency department with the
complaint of a sudden loss of sensation. The history reveals that the
man is overweight, hypertensive, and does not regularly take medication. When the man speaks his voice is gravely and hoarse. The examination further reveals a loss of pain and thermal sensations on the right
side of his body and on the left side of his face. CT shows an infarcted
area in the medulla.
43. Damage to which of the following structures would most likely
explain the man’s hoarse, gravely voice?
䊊 (A) Facial nucleus
䊊 (B) Gracile nucleus
䊊 (C) Hypoglossal nucleus
䊊 (D) Nucleus ambiguus
䊊 (E) Spinal trigeminal nucleus
Q & A’s: A Sampling of Study and Review Questions with Explained Answers 269
44. Injury to which of the following structures in this man is most
specifically related to the loss of pain and thermal sensations on the
body below the neck?
䊊 (A) Anterolateral system
䊊 (B) Cuneate fasciculus
䊊 (C) Gracile fasciculus
䊊 (D) Medial lemniscus
䊊 (E) Spinal trigeminal tract
45. Damage to which of the following structures would most specifically explain the loss of pain and thermal sensations on the man’s
face?
䊊 (A) Anterolateral system
䊊 (B) Medial lemniscus
䊊 (C) Medial longitudinal fasciculus
䊊 (D) Solitary tract
䊊 (E) Spinal trigeminal tract
46. The CT shows an infarcted area in the medulla in this man. Based
on the deficits described, and the corresponding structures involved, which of the following vessels is most likely occluded?
䊊 (A) Anterior spinal artery
䊊 (B) Posterior spinal artery
䊊 (C) Posterior inferior cerebellar artery
䊊 (D) Anterior inferior cerebellar artery
䊊 (E) Penetrating branches of the vertebral artery
47. A 77-year-old man presents with an ataxic gait. There are no other
deficits. CT shows an infarcted area in the medulla in the territory
served by the posterior inferior cerebellar artery. Damage to
which of the following structures would most likely explain the
symptoms experienced by this man?
䊊 (A) Anterolateral system
䊊 (B) Corticospinal tract
䊊 (C) Nucleus ambiguus
䊊 (D) Restiform body
䊊 (E) Vestibular nuclei
48. Which of the following cranial nerve nuclei is located in the anterior (ventral or inferior) and medial portion of the periaqueductal
grey at the cross-sectional level of the superior colliculus?
䊊 (A) Abducens
䊊 (B) Mesencephalic
䊊 (C) Oculomotor
䊊 (D) Trigeminal motor
䊊 (E) Trochlear
49. A 53-year-old woman presents with motor deficits that the examining neurologist describes as a superior alternating hemiplegia.
Which of the following cranial nerve roots is most likely involved
in this lesion?
䊊 (A) Abducens
䊊 (B) Hypoglossal
䊊 (C) Oculomotor
䊊 (D) Trigeminal
䊊 (E) Trochlear
50. An 82-year-old woman presents to the emergency department
with difficulty swallowing (dysphagia). Which of the following nuclei of the medulla contain motor neurons that innervate muscles
involved in swallowing?
䊊
䊊
䊊
䊊
䊊
(A)
(B)
(C)
(D)
(E)
Dorsal motor vagal
Hypoglossal
Inferior salivatory
Medial vestibular
Nucleus ambiguus
Questions 51 through 53 are based on the following patient.
A 73-year-old man is brought to the emergency department after losing consciousness at his home. CT shows a hemorrhage into the right
hemisphere. The man regains consciousness, but is not fully alert. After 3–4 days the man begins to rapidly deteriorate: his pupils are large
(dilated) and respond slowly to light, eye movement becomes restricted, there is weakness in the extremities on the left side, and the
man becomes comatose. Repeat CT shows an uncal herniation.
51. Based on its location, which of the following parts of the brainstem
is most likely to be directly affected by uncal herniation, especially
in the early stages?
䊊 (A) Diencephalon/thalamus
䊊 (B) Mesencephalon/midbrain
䊊 (C) Myelencephalon/medulla
䊊 (D) Pons and cerebellum
䊊 (E) Pons only
52. Damage to corticospinal fibers in which of the following locations
would most likely explain the weakness in his extremities?
䊊 (A) Left basilar pons
䊊 (B) Left crus cerebri
䊊 (C) Right basilar pons
䊊 (D) Right crus cerebri
䊊 (E) Right posterior limb of the internal capsule
53. The dilated, and slowly responsive, pupils in this man are most
likely explained by damage to fibers in which of the following?
䊊 (A) Abducens nerve
䊊 (B) Corticonuclear fibers in the crus
䊊 (C) Oculomotor nerve
䊊 (D) Optic nerve
䊊 (E) Sympathetic fibers on cerebral vessels
54. The sagittal MRI of a 26-year-old man shows a dark shadow in the
midbrain tegmentum on the midline at the cross-sectional level of
the inferior colliculus. Which of the following structures does this
dark area represent?
䊊 (A) Central portions of the red nucleus
䊊 (B) Compact and reticular parts of the substantia nigra
䊊 (C) Decussation of the superior cerebellar peduncle
䊊 (D) Decussation of trigeminothalamic fibers
䊊 (E) Motor (pyramidal) decussation
270 Q & A’s: A Sampling of Study and Review Questions with Explained Answers
55. The CT of a 39-year-old man with untreated hypertension shows
a small hemorrhage in the brainstem. This lesion encompasses the
brachium of the inferior colliculus and the brain substance immediately internal to this structure. Which of the following structures is also most likely involved in this lesion?
䊊 (A) Anterolateral system
䊊 (B) Central tegmental tract
䊊 (C) Corticospinal fibers
䊊 (D) Mesencephalic tract
䊊 (E) Oculomotor nerve
56. A 69-year-old man complains of difficulty walking. The examination reveals no weakness, but does reveal a loss of discriminative
touch and vibratory sense on the left lower extremity. MRI shows
a small infarcted area in the midbrain. Which of the following
structures is most likely involved in the infarcted area?
䊊 (A) Anterolateral system
䊊 (B) Corticospinal fibers
䊊 (C) Lateral part of the medial lemniscus
䊊 (D) Medial part of the medial lemniscus
䊊 (E) Rubrospinal fibers
57. Which of the following nuclei containing visceromotor (autonomic)
cell bodies is located immediately inferior to the medial vestibular
nucleus, medial to the solitary tract and nucleus, and has axons that
exit the brainstem on the glossopharyngeal nerve?
䊊 (A) Dorsal motor nucleus
䊊 (B) Edinger-Westphal nucleus
䊊 (C) Inferior salivatory nucleus
䊊 (D) Intermediolateral cell column
䊊 (E) Superior salivatory nucleus
58. An 81-year-old woman is brought to the emergency department
by her adult grandson. He explains that during dinner she slumped
off of her chair, did not lose consciousness, but had trouble speaking. The examination reveals weakness of the upper and lower extremities on the left and deviation of the tongue to the right on
protrusion. Which of the following most specifically describes this
deficit in this elderly patient?
䊊 (A) Alternating hemianesthesia
䊊 (B) Hemihypesthesia
䊊 (C) Inferior alternating hemiplegia
䊊 (D) Middle alternating hemiplegia
䊊 (E) Superior alternating hemiplegia
Questions 59 and 60 are based on the following patient.
A 79-year-old woman is brought to the emergency department after a
fall in her home from which she was unable to get up. The examination
reveals a deviation of the tongue to the left on protrusion, a pronounced
weakness of the right upper and lower extremities, and a loss of position and vibratory sense and discriminative touch on the right side of the
body below the neck. CT shows an infarcted area in the medulla.
59. Which of the following represents the best localizing sign in this
patient?
䊊 (A) Deviation of the tongue
䊊 (B) Motor loss on lower extremity
䊊 (C) Motor loss on upper extremity
䊊 (D) Sensory loss on lower extremity
䊊 (E) Sensory loss on upper extremity
60. Damage to which of the following tracts or fiber bundles would
most likely give rise to the sensory deficits experienced by this patient?
䊊 (A) Anterolateral system
䊊 (B) Medial lemniscus
䊊 (C) Medial longitudinal fasciculus
䊊 (D) Solitary tract
䊊 (E) Spinal trigeminal tract
61. The MRI of a 12-year-old boy reveals a cavity within the medulla.
Which of the following terms most specifically describes this condition?
䊊 (A) Brown-Séquard syndrome
䊊 (B) Central cord syndrome
䊊 (C) Hydromyelia
䊊 (D) Syringobulbia
䊊 (E) Syringomyelia
62. Which of the following cell groups within the white matter of the
cerebellum characteristically appears as a long undulating line,
looking somewhat like the principle olivary nucleus in the medulla?
䊊 (A) Dentate nucleus
䊊 (B) Emboliform nucleus
䊊 (C) Fastigial nucleus
䊊 (D) Globose nucleus
䊊 (E) Red nucleus
Answers for Chapter 5
1. Answer D: Although this patient initially presented with complete motor and sensory losses, some function had returned by 36
hours; in this case the lesion is classified as an incomplete lesion of
the spinal cord. Patients with no return of function at 24⫹ hours
and no sacral sparing have suffered a lesion classified as complete
and it is unlikely that they will recover useful neurologic function.
In a central cord and a large syringomyelia there is sparing of posterior column sensations and in a hemisection the loss of motor
function is unilateral. (p. 94–95)
2. Answer B: A medullary lesion that results in weakness of the
extremities on one side indicates involvement of the corticospinal
fibers located in the pyramid on the contralateral side; these fibers
largely cross in the pyramidal (motor) decussation. Rubrospinal
and vestibulospinal fibers influence the activity of spinal motor
neurons, but isolated lesions of these fibers would not result in a
unilateral weakness of upper and lower extremities. The anterolateral system and the medial lemniscus are sensory tracts. (p.
98–108, 110–111)
3. Answer B: The anterior choroidal artery serves the optic tract
(homonymous hemianopsia) and the inferior portions of the posterior limb of the internal capsule (weakness of the extremities).
The ophthalmic artery, via its central retinal branch, serves the
retina; the anterior cerebral artery serves the lower extremity areas of the motor and sensory cortices; and distal segments of the
posterior cerebral artery serve the medial temporal cortex and the
visual cortex. The lateral posterior choroidal artery serves the
choroid plexus in the lateral ventricle and some adjacent structures. (p. 21, 25, 29, 35, 158–159)
Q & A’s: A Sampling of Study and Review Questions with Explained Answers 271
4. Answer D: The head of the caudate nucleus is located in the lateral wall of the anterior horn of the lateral ventricle and receives
its blood supply from the medial striate artery (also called the
artery of Heubner). This vessel also serves much of the anterior
limb of the internal capsule. The lenticulostriate arteries serve a
large part of the lenticular nucleus and portions of the surrounding internal capsule, and thalamoperforating arteries serve anterior portions of the dorsal thalamus. The anterior choroidal artery
provides blood supply to inferior portions of the internal capsule,
optic tract, and structures in the medial portions of the temporal
lobe. The medial posterior choroidal artery serves choroid plexus
in the lateral and third ventricles and adjacent areas of the lateral
midbrain and caudomedial thalamus. (p. 154–158)
5. Answer C: In this patient the weakness of the right lower extremity is related to a lesion of lateral corticospinal tract fibers on
the right side of the spinal cord. The left corticospinal tract serves
the left side of the spinal cord and the left lower extremity.
Rubrospinal, reticulospinal, and vestibulospinal fibers influence
the activity of spinal motor neurons; however, the deficits related
to corticospinal tract damage (significant weakness) will dominate
over the lack of excitation to flexor or extensor motor neurons in
the spinal cord via these tracts. (p. 86–88, 94)
6. Answer C: The loss of pain and thermal sensations beginning at
the level of the umbilicus (T10 dermatome) on the left side results
from damage to fibers of the anterolateral system at about the T8
level on the right. These fibers ascend 1 to 2 levels as they cross
the midline. Damage at the T6 level would result in a loss beginning at the T8 level on the contralateral side and damage at the T10
level would result in a loss beginning at about the T12 level. (p.
88–89, 94)
7. Answer E: The artery of Adamkiewicz is usually located at the
T12-L1 spinal cord levels and is more frequently (about 65% of
the time) seen on the left side. The other cord levels listed may
have small spinal medullary arteries but not the large diameter
vessel characteristic of Adamkiewicz. (p. 94)
8. Answer E: The thalamoperforating arteries serve the more rostral portions of the dorsal thalamus. These vessels may originate as
a single trunk or as several vessels from the P1 segment of the posterior cerebral artery. The anterior choroidal artery serves the optic tract, inferior portions of the internal capsule, choroid plexus
in the temporal horn, and structures in the medial region of the
temporal lobe. The thalamogeniculate artery supplies blood to the
caudal thalamus, the medial striate arteries to the head of the caudate nucleus, and the lateral striate arteries to much of the lenticular nucleus. (p. 25, 158–159)
9. Answer A: The claustrum is the thin layer of grey matter that is
located between the extreme and external capsules. It is generally
regarded as being functionally related to the insular cortex. The
external medullary lamina is found at the interface of the lateral
portions of the thalamus with the internal capsule and the lamina
terminalis is the thin structure forming the rostral wall of the third
ventricle. The putamen is located medial to the external capsule
and lateral to the globus pallidus and the stria terminalis is a fiber
bundle in the groove between the body of the caudate nucleus and
the dorsal thalamus. (p. 144–153, 162)
10. Answer C: Wild flailing movements of the extremities, especially the upper, are hemiballistic movements (hemiballismus);
these are characteristic of a lesion in the subthalamic nucleus.
Damage to the cerebellar cortex and nuclei and the lenticular nucleus will result in motor deficits, but these are usually described
as involving tremor, ataxia, and related motor problems. The ventral lateral nucleus is a thalamic relay center for motor information and the ventral posterolateral nucleus is a sensory relay nucleus. Lesions of these nuclei will result in motor (but not
hemiballismus) and sensory deficits. (p. 146–149, 158)
11. Answer A: Fibers conveying pain and thermal sensations cross
the midline in the anterior white commissure. Consequently, a lesion of this structure, as in syringomyelia, would result in a bilateral loss of these sensations, reflecting the levels of the syrinx.
Damage to fibers of the anterolateral system results in a loss of
these sensations on the contralateral side and the posterior
columns convey proprioception, discriminative touch, and vibratory sense. The medial longitudinal fasciculus does not contain
fibers conveying sensory input. (p. 90–91, 94)
12. Answer C: The anterior spinal artery serves the medial portion
of the medulla, an area that encompasses the medial lemniscus, exiting roots of the hypoglossal nerve, and the corticospinal fibers in
the pyramid. The anterolateral system, spinal trigeminal tract, and
rubrospinal tract are in the territory of the posterior inferior cerebellar artery (commonly called PICA by clinicians). The posterior
spinal artery in the caudal medulla and spinal cord serves the
gracile fasciculus. (p. 110–111)
13. Answer C: The masticatory muscles receive their motor innervation via the motor neurons located in the trigeminal motor nucleus on the ipsilateral side; this excludes the right trigeminal nucleus. Facial motor neurons innervate the muscles of facial
expression on the ipsilateral side and the hypoglossal nucleus innervates the ipsilateral side of the tongue. (p. 120–121, 124)
14. Answer E: A pineal tumor impinging on the superior colliculus
may result in a paralysis of upward gaze (Parinaud syndrome). The
inferior colliculus is related to the auditory system, trochlear fibers
innervate the ipsilateral superior oblique muscle, and the posterior
commissure contains fibers related to the pupillary light pathway.
Occlusion of the great cerebral vein may cause serious neurologic
deficits but not specifically a paralysis of upward gaze. (p. 136)
15. Answer B: Alternating sensory losses accompanied by a motor
deficit on the same side as the loss of vibratory sensation are characteristics of the Brown-Séquard syndrome (also commonly called
a spinal cord hemisection). The Wallenberg syndrome is seen in
lesions of the medulla, and the Benedikt, Claude, and Weber syndromes are seen in lesions of the midbrain. In these brainstem syndromes there are usually characteristic cranial nerve and long tract
signs and symptoms. (p. 90–91, 94, 110, 136)
16. Answer E: The rubrospinal tract lies immediately anterior
(ventral) to, and partially overlaps with, the lateral corticospinal
tract. The anterolateral system is in the anterolateral area of the
spinal cord and is spatially separated from the lateral corticospinal
tract. The gracile fasciculus is in the posterior columns, the medial longitudinal fasciculus is in the ventral funiculus, and the anterior spinocerebellar tract is located on the anterolateral surface
of the spinal cord. (p. 90–91, 94–95, 100)
272 Q & A’s: A Sampling of Study and Review Questions with Explained Answers
17. Answer D: The ventral posterolateral nucleus of the thalamus
receives the pathways (medial lemniscus and anterolateral system)
that relay the information lost as a result of the lesion in this
woman. The ventral posteromedial nucleus relays comparable information from the face and the medial geniculate nucleus is related to the auditory system. Lesions in the subthalamic nucleus
result in hemiballismus. The anterolateral system relays pain and
thermal sense; this is only part of the sensory deficits experienced
by this woman. (p. 142, 158–159)
23. Answer B: Penetrating branches of the posterior spinal artery
serve the posterior columns (gracile and cuneate fasciculi) of the
spinal cord at all levels. Branches of the posterior spinal artery also
serve the gracile nucleus, but this structure is in the medulla, not
in the spinal cord. The lateral corticospinal tract and the anterolateral system are served by the arterial vasocorona on the surface
of the cord and the internal branches of the anterior spinal artery.
The medial longitudinal fasciculus is in the territory of the anterior spinal artery. (p. 95, 111)
18. Answer A: The anterolateral system is located internal to the
position of the anterior spinocerebellar tract; damage to this area
of the spinal cord would most likely result in a loss of pain and
thermal sensations on the contralateral side of the body below the
lesion. The lateral corticospinal tract is located internal to the posterior spinocerebellar tract, the anterior white commissure and
the anterior corticospinal tract are located in the anterior funiculus of the cord, and the cuneate fasciculus is in the posterior column medial to the posterior horn at upper thoracic and cervical
levels. (p. 88–91, 95)
24. Answer D: The putamen and the caudate nucleus originate from
the same group of developing neurons, are collectively referred to
as the neostriatum, and appear in the same shade of grey in a T1weighted MRI. In general, the globus pallidus and pulvinar are distinctly lighter than the putamen and the dorsomedial nucleus frequently appears dark in a shade of grey distinctly different from
that of the globus pallidus. (p. 151, 153, 155, 162)
19. Answer A: Reticulospinal fibers (medial and lateral) and lateral
vestibulospinal fibers are found predominately in the anterolateral
area of the spinal cord; medial vestibulospinal fibers are located in
the medial longitudinal fasciculus. In the decerebrate patient, the
descending influence of rubrospinal fibers on spinal flexor motor
neurons is removed, and descending influence on extensor motor
neurons is predominant. The posterior columns, posterolateral
area of the cord, and the posterolateral tract do not contain
vestibulospinal or reticulospinal fibers. The intermediate zone, a
part of the spinal cord grey matter, contains some of the terminals
of these fibers but not the descending tracts in toto. (p. 86, 88, 90,
95)
20. Answer C: Weakness of the muscles of the face, particularly
when upper and lower portions of the face are involved, indicate
a lesion of either the facial motor nucleus or the exiting fibers of
the facial nerve; both are located in the lateral pontine tegmentum
at caudal levels. The hypoglossal nucleus innervates muscles of the
tongue, the trigeminal nucleus innervates masticatory muscles,
and the abducens nucleus innervates the lateral rectus muscle, all
on the ipsilateral side. The arcuate nucleus is a group of cells located on the surface of the pyramid. (p.106, 116–120, 124)
21. Answer B: The fibers of the anterolateral system are located in
the lateral portion of the pontine tegmentum anterior (ventral) to
the facial motor nucleus; these fibers convey pain and thermal inputs. The spinal trigeminal tract and the anterior trigeminothalamic tract also convey pain and thermal input but from the ipsilateral and contralateral sides of the face, respectively. The lateral
lemniscus is auditory in function and the medial lemniscus conveys
proprioception, vibratory sense, and discriminative touch. (p.
116–120, 124)
22. Answer B: The dorsomedial nucleus is located medial to the internal medullary lamina and, through its connections, one if its
functions is to participate in arousal of the cerebral cortex. The
other choices are in (anterior nucleus) or lateral to the internal
medullary lamina, or, in the case of the globus pallidus, lateral to
the internal capsule. (p. 144–149)
25. Answer C: The pars caudalis portion of the spinal trigeminal nucleus is located in the lateral medulla adjacent to the spinal trigeminal tract in cross-sectional levels between the obex and the C1
level of the spinal cord. This portion of the spinal trigeminal nucleus is responsible for relaying pain and thermal information originating from the face and oral cavity on one side to the ventral posteromedial nucleus on the contralateral side. The pars interpolaris
is found at levels between the obex and the rostral end of the hypoglossal nucleus and the pars oralis between the interpolaris and
the principal sensory nucleus. The principal sensory nucleus is in
the pons and the mesencephalic nucleus is in the midbrain. (p.
98–106, 120, 130)
26. Answer E: The solitary nucleus receives general visceral afferent (GVA) and special visceral afferent information (SVA, this input is taste) and is located in the region of the medulla served by
posterior inferior cerebellar artery. All of the other choices are in
the territory served by the anterior spinal artery. (p. 111)
27. Answer E: The tonsil of the cerebellum is located close to the
midline and immediately above the medulla: its position relative to
the cerebellum is caudal, medial, and inferior. Tonsillar herniation
may compress the medulla, and if sudden, may result in death. The
other portions of the cerebellum do not herniate. (p. 110, 123)
28. Answer B: In addition to other signs or symptoms, lesions in lateral areas of the brainstem may also interrupt hypothalamospinal
fibers descending from the hypothalamus to the intermediolateral
cell column in upper thoracic levels of the spinal cord. In this case
the patient may present with a Horner syndrome, part of which is
a small (constricted) pupil. In addition, the affected pupil may react slowly to reduced light. The anterolateral system conveys somatosensory input and fibers of the medial longitudinal fasciculus
(originating from the medulla) are primarily descending to spinal
cord levels. Reticulospinal and vestibulospinal tracts influence
spinal motor neurons. (p. 124)
29. Answer C: The pulvinar, geniculate nuclei, ventral posteromedial and posterolateral nuclei, centromedian, and some other adjacent nuclei are served by the thalamogeniculate artery. The anterior and ventral anterior thalamic nuclei receive their blood
supply from thalamoperforating arteries, the substantia nigra via
branches of P1 and P2, and globus pallidus from the lenticulostriate branches of M1. (p. 140–141, 158–159)
Q & A’s: A Sampling of Study and Review Questions with Explained Answers 273
30. Answer E: The short-term loss of function, frequently involving a specific part of the body, is characteristic of a transient ischemic attack (commonly called a TIA). The follow-up MRI
shows no lesion because there has been no permanent damage.
TIAs are caused by a brief period of inadequate perfusion of a localized region of the nervous system; recovery is usually rapid and
complete. However, TIAs, especially if repeated, may be indicative of an impending stroke. Hemorrhagic strokes frequently result in some type of permanent deficit, and the central cord syndrome has bilateral deficits. A small embolic stroke would be
visible on the follow-up MRI, and in this patient would have resulted in a persistent deficit. Syringobulbia may include long tract
signs as well as cranial nerve signs. (p.158)
31. Answer D: Damage to the gracile fasciculus on the left (at the
T8 level this is the only part of the posterior columns present) accounts for the loss of vibratory sensation (and discriminative
touch). Injury to the gracile fasciculus on the right would result is
this type of deficit on the right side. The level of the cord damage
is caudal to the cuneate fasciculi and the anterolateral system conveys pain and thermal sensations. (p. 86, 88, 90, 94)
32. Answer A: The loss of pain and thermal sensations on the right
side of the body correlates with a lesion involving the anterolateral
system on the left side of the spinal cord. A lesion of the right anterolateral system would result in a left-sided deficit. The gracile
and cuneate fasciculi convey discriminative touch, vibratory sensation, and proprioception. The posterior spinocerebellar tract
conveys similar information, but it is not perceived/recognized as
such (consciously) by the brain. (p. 88, 90, 94)
33. Answer E: The substantia nigra contains a large population of
melanin-containing cells, is located in the midbrain just internal to
the crus cerebri, and the loss of these cells gives rise to the motor
deficits characteristic of Parkinson disease. The neurotransmitter
associated with these cells is dopamine. The reticular formation is
in the core of the brainstem and the pontine nuclei are in the basilar pons; neither of these contain cells with melanin. The red nucleus is in the midbrain, but its reddish tone is related to a rich vascular supply, not to cells containing a pigment. (p. 128–133)
34. Answer E: The solitary nucleus is located immediately inferior
(ventral) to the medial and spinal vestibular nuclei and is the only
nucleus in the choices to receive a general visceral afferent (GVA)
and special visceral afferent (SVA-taste) input. The inferior salivatory nucleus and the nucleus ambiguus are visceromotor (general
visceral efferent [GVE] and special visceral efferent [SVE], respectively) and the spinal trigeminal and cochlear nuclei are sensory
(general somatic afferent [GSA] and special somatic afferent [SSA],
respectively). (p. 104, 106, 174–175)
35. Answer E: The superior salivatory nucleus lies adjacent to the
exiting fibers of the facial nerve in a position just lateral to the abducens nucleus in caudal levels of the pons. The preganglionic axons originating from these cells distribute on peripheral branches
of the facial nerve. The dorsal motor and inferior salivatory nuclei
are in the medulla and associated, respectively, with the vagus and
glossopharyngeal nerves. The Edinger-Westphal nucleus is related
to the oculomotor nucleus and the intermediolateral cell column
is located primarily in thoracic levels of the spinal cord. (p.116,
203)
36. Answer A: The facial colliculus is an elevation in the floor of the
fourth ventricle located medial to the sulcus limitans and formed
by the underlying abducens nucleus and fibers (internal genu)
originating from the facial nucleus. The vestibular area, indicating
the position of the vestibular nuclei, is lateral to the sulcus limitans and the hypoglossal nucleus is internal to the hypoglossal
trigone in the medial floor of the ventricle in the medulla. The
trigeminal and facial nuclei are located in the pontine tegmentum
and do not border on the ventricular space. (p. 34–36, 114–117)
37. Answer A: In this case the weakness of the upper and lower extremities on the right reflects damage to corticospinal fibers on the
left side of the basilar pons. A lesion of these fibers on the right side
of the pons would produce a left-sided weakness. Rubrospinal
fibers are not located in the territory of paramedian branches of
the basilar artery. Also, lesions of rubrospinal fibers and of the
middle cerebellar peduncle do not cause weakness but may cause
other types of motor deficits. (p. 24, 116, 124, 190–191)
38. Answer A: The exiting fibers of the abducens nerve (on the left)
are in the territory of the paramedian branches of the basilar artery
and are laterally adjacent to corticospinal fibers in the basilar pons.
Diplopia may result from lesions of the oculomotor and trochlear
nerves, but these structures are not in the domain of the paramedian basilar branches. A lesion of the optic nerve results in blindness in that eye and damage to the facial root does not affect eye
movement but may cause a loss of view of the external world if the
palpebral fissure is closed due to facial muscle weakness. (p. 24,
116, 124)
39. Answer D: At caudal pontine levels most, if not all, of the medial lemniscus is located within the territory served by paramedian
branches of the basilar artery. Penetrating branches of the anterior
spinal artery serve the hypoglossal nucleus. The other choices are
generally in the territories of short or long circumferential
branches of the basilar artery. (p. 124–125)
40. Answer D: Weakness of the extremities accompanied by paralysis
of the lateral rectus muscle (innervated by the abducens nerve) on the
contralateral side indicates a lesion in the caudal and medial pons involving the abducens nerve root and corticospinal fibers. This is a
middle alternating hemiplegia. Inferior alternating hemiplegia specifies involvement of the hypoglossal root and the pyramid, and superior alternating hemiplegia indicates damage to the oculomotor root
and the crus cerebri. Alternating (or alternate) hemianesthesia and
hemihypesthesia are sensory losses. (p. 116, 124)
41. Answer B: The prominent elevation formed on the caudal and
medial wall of the third ventricle, at the general level of the posterior commissure, represents the location of the habenular nucleus. This is an excellent landmark to use in axial MRI when designating the separation between the third ventricle (rostral to this
point on the midline) and the quadrigeminal cistern (caudal to this
point). The pulvinar is lateral to the quadrigeminal cistern, the
lamina terminalis forms the rostral wall of the third ventricle, and
the massa intermedia bridges the space of the third ventricle.
When present (in about 80% of patients) the Massa intermedia appears as a shadow in T2-weighted MRI bridging the third ventricle. The superior colliculus is a mesencephalic structure found in
the quadrigeminal cistern. (p. 76, 138–143, 162)
274 Q & A’s: A Sampling of Study and Review Questions with Explained Answers
42. Answer E: The red nucleus, exiting fibers of the oculomotor
nerve, portions of the corticospinal fibers in the crus cerebri, and
a number of other medially located structures are found in the territory of the penetrating branches of the basilar bifurcation. The
paramedian branches of the basilar artery and the corticospinal
fibers in the pyramid serve the abducens nerve by branches of the
anterior spinal artery. The anterolateral system and the medial
lemniscus are mainly, if not entirely, in the region of the midbrain
served by branches of the quadrigeminal and posterior medial
choroidal arteries. (p. 137)
43. Answer D: The vocalis muscle (this muscle is actually the medial portion of the thyroarytenoid muscle) is innervated, via the
vagus nerve, by motor neurons located in the nucleus ambiguus.
The gracile nucleus conveys sensory input from the body and the
spinal trigeminal nucleus relays sensory input from the face. The
hypoglossal nucleus is motor to the tongue and the facial nucleus
is motor to the muscles of facial expression. (p. 100–106, 110)
44. Answer A: Fibers comprising the anterolateral system convey
pain and thermal sensations from the body, excluding the face.
These fibers are located in lateral portions of the medulla adjacent
to the spinal trigeminal tract; this latter tract relays pain and thermal sensations from the face. The gracile and cuneate fasciculi convey proprioception, discriminative touch, and vibratory sense in
the spinal cord and the medial lemniscus conveys this same information from the medulla to the dorsal thalamus. (p. 100, 102,
104, 106, 110)
45. Answer E: The loss of pain and thermal sensations on one side
of the face correlates with damage to the spinal trigeminal tract;
in this case the loss is ipsilateral to the lesion. The anterolateral system relays pain and thermal sensations from the contralateral side
of the body, the solitary tract conveys visceral sensory input (especially taste), and the medial lemniscus contains fibers relaying
information related to position sense and discriminative touch.
The medial longitudinal fasciculus does not contain sensory fibers.
(p. 100–108, 110)
46. Answer C: The posterior inferior cerebellar artery (commonly
called PICA by clinicians) serves the posterolateral portion of the
medulla, which encompasses the anterolateral system, spinal trigeminal tract, and nucleus ambiguus. The anterior and medial areas of the
medulla (containing the pyramid, medial lemniscus, and hypoglossal
nucleus/nerve) are served by the anterior spinal artery and the anterolateral area of the medulla (the region of the olivary nuclei) is
served by penetrating branches of the vertebral artery. The posterior
spinal artery serves the posterior column nuclei in the medulla and
the anterior inferior cerebellar artery (commonly called AICA)
serves caudal portions of the pons and cerebellum. (p. 111)
47. Answer D: The restiform body is a large fiber bundle located in
the posterolateral area of the medulla in the region served by posterior inferior cerebellar artery (PICA). This structure contains a
variety of cerebellar afferent fibers including those of the posterior
spinocerebellar tract. Damage to the vestibular nuclei will result
in a tendency to fall to the ipsilateral side but will also produce
diplopia (double vision) and nausea; symptoms not experienced
by this patient. The anterolateral system is sensory, the nucleus
ambiguus is motor to muscles of the throat (including the vocalis),
and the corticospinal tract is not in the PICA territory. (p. 104,
106, 110–111)
48. Answer C: The oculomotor nucleus (containing general somatic
efferent [GSE] cell bodies), along with the Edinger-Westphal (containing general visceral efferent [GVE] cell bodies) nucleus, is
found in the most anterior and medial portion of the periaqueductal grey at the superior colliculus level. The trochlear nucleus is
found at a comparable position, but at the cross-sectional level of
the inferior colliculus. The mesencephalic nucleus is found in the
lateral area of the periaqueductal grey, and the trigeminal and abducens nuclei are located in the pons. (p. 130–133, 201)
49. Answer C: A superior alternating (or alternate) hemiplegia is characterized by a loss of most eye movement (damage to oculomotor
nerve fibers) on the ipsilateral side and weakness of the upper and
lower extremities (damage to corticospinal fibers in the crus cerebri)
on the contralateral side. The abducens nerve is the cranial nerve involved in a middle alternating hemiplegia and the hypoglossal is that
nerve involved in an inferior alternating hemiplegia. The trigeminal
nerve innervates the muscles of mastication and the trochlear nerve
innervates the superior oblique muscle. (p. 132, 136, 200)
50. Answer E: Motor neurons in the nucleus ambiguus innervate,
primarily through the vagus nerve, the muscles of the throat that
move a bolus of food from the oral cavity to the esophagus. The
tongue, via the hypoglossal nucleus and nerve, may move food
around in the mouth and toward the back of the oral cavity, but
the actual act of swallowing is through the action of pharyngeal and
laryngeal musculature. The dorsal motor vagal and inferior salivatory nuclei are both visceromotor (autonomic) nuclei, and the medial vestibular nucleus is involved in the regulation of eye movement and in balance and equilibrium. (p. 100–106, 110)
51. Answer B: The uncus is at the rostral and medial aspect of the
parahippocampal gyrus, and, in this position, is directly adjacent to
the anterolateral aspect of the midbrain. The diencephalon is rostral to this point and the medulla, the most caudal part of the brainstem, is located in the posterior fossa. Late stages of uncal herniation may, but not always, result in damage to the rostral pons; this
is especially so if the patient becomes decerebrate. The cerebellum
is not involved in uncal herniation. (p. 20, 22, 24, 38, 78, 136)
52. Answer D: Uncal herniation compresses the lateral portion of
the brainstem, eventually resulting in compression of the corticospinal fibers in the crus cerebri. Weakness on the patient’s left
side indicates damage to corticospinal fibers in the right crus. In situations of significant shift of the midbrain due to the herniation, the
contralateral crus may also be damaged resulting in bilateral weakness. While all other choices contain corticospinal fibers, none of
these areas are directly involved in uncal herniation. (p, 136)
53. Answer C: The root of the oculomotor nerve conveys GSE
fibers to four of the six major extraocular muscles and GVE
parasympathetic preganglionic fibers to the ciliary ganglion from
which postganglionic fibers travel to the sphincter muscle of the
iris. Pressure on the oculomotor root, as in uncal herniation, will
usually compress the smaller diameter, and more superficially located GVE fibers first. Optic nerve damage results in blindness in
that eye, injury to sympathetic fibers to the eye results in constriction of the pupil, and an abducens root injury results in an inability to abduct that eye. A lesion of corticonuclear fibers in the
crus results primarily in motor deficits related to the facial, hypoglossal, and accessory nerves. (p. 136, 201, 221)
Q & A’s: A Sampling of Study and Review Questions with Explained Answers 275
54. Answer C: The decussation of the superior cerebellar peduncle
is a prominent fiber bundle located in the tegmentum of the midbrain directly on the midline at the level of the inferior colliculus.
This bundle is made up of cerebellar efferent fibers. The red nucleus is located in the midbrain tegmentum, but not on the midline. Decussating trigeminothalamic fibers are found in the
medulla and do not form a visible structure on the midline. The
motor decussation is a compact bundle on the midline, but it is in
the medulla, not the midbrain. The main parts of the substantia nigra are in the midbrain, are seen in sagittal MRI, but they are definitely not on the midline. (p. 128, 163, 211)
55. Answer A: The anterolateral system is located just internal to
the brachium of the inferior colliculus in the lateral portions of the
midbrain tegmentum. This tract conveys pain and thermal sensations from the contralateral side of the body excluding the face.
Corticospinal fibers are located in the crus cerebri, the mesencephalic tract at the lateral edge of the periaqueductal (central)
grey, and the central tegmental tract is, as its name indicates, in
the central part of the tegmentum. Oculomotor fibers within the
midbrain leave the nucleus, arch through the tegmentum, and exit
on the medial surface of the basis pedunculi into the interpeduncular cistern. (p. 128–131)
56. Answer C: Fibers conveying discriminative touch, vibratory
sensations, and proprioception are located in the lateral lemniscus; those from the contralateral upper extremity are medial while
those from the contralateral lower extremity are lateral. This man
has difficulty walking due to a lesion of fibers conveying position
sense from the lower extremity, not due to a lesion influencing descending fibers passing to spinal motor neurons. Fibers of the anterolateral system convey pain and thermal sensation. Rubrospinal
and corticospinal are motor in function; however this man has no
weakness. (p. 126–132, 178–179)
57. Answer C: The inferior salivatory nucleus is located in the rostral medulla, medial to the solitary tract and nuclei and inferior
to the medial vestibular nucleus. Preganglionic axons that originate from these cells distribute on branches of the glossopharyngeal nerve. The dorsal motor nucleus is in the medulla, its axons travel on the vagus nerve. The superior salivatory nucleus is
in the caudal pons and is associated with the facial nerve. Cells
of the Edinger-Westphal nucleus are associated with the oculomotor nucleus of the midbrain and the intermediolateral cell
column is located primarily in thoracic levels of the spinal cord.
(p. 106, 203)
58. Answer C: Weakness of the extremities accompanied by
paralysis of muscles on the contralateral side of the tongue (seen
as a deviation of the tongue to that side on protrusion) indicates
a lesion in the medulla involving the corticospinal fibers in the
pyramid and the exiting hypoglossal roots. This is an inferior alternating hemiplegia. Middle alternating hemiplegia refers to a
lesion of the pontine corticospinal fibers and the root of the abducens nerve, and superior alternating hemiplegia specifies
damage to the oculomotor root and crus cerebri. Alternating
(alternate) hemianesthesia and hemihypesthesia are sensory
losses. (p. 102, 110)
59. Answer A: The deviation of the tongue to the left on attempted
protrusion is the best localizing sign in this woman. This is especially the case when the deviation of the tongue is seen in concert
with the motor and sensory losses described for this patient. This
clearly indicates a lesion in the medial medulla encompassing the
corticospinal fibers, medial lemniscus, and exiting fibers on the
hypoglossal nerve. Motor and sensory losses, without the cranial
nerve sign, could suggest a lesion at several different levels of the
neuraxis. (p. 83, 110–111)
60. Answer B: All of the sensory deficits seen in this woman reflect
a lesion in the medial lemniscus, which is located in the medial
medulla in the territory of the anterior spinal artery. The anterolateral system and the spinal trigeminal tract convey pain and thermal sensations from the body (sans face) and face, respectively.
The solitary tract is made up of the central processes of viscerosensory fibers and the medial longitudinal fasciculus at this
level contains descending fibers that influence spinal motor neurons. (p. 100–108, 110–111)
61. Answer D: Syringobulbia is a cavitation within the medulla. A
cavitation in this location may communicate with a cavity in cervical levels of the spinal cord (syringomyelia). Hydromyelia refers
to a cavity of the spinal cord that is lined with ependymal cells. The
central cord and Brown-Séquard syndromes are lesions of the
spinal cord that give rise to characteristic motor and sensory
losses. (p. 110)
62. Answer A: The dentate nucleus appears as a long thin undulating line within the white matter core of the cerebellar hemisphere.
It is frequently described as having the three-dimensional shape of
a crumpled bag with its hilus (the opening of the bag) directed rostromedially. The other cerebellar nuclei (fastigial, globose, emboliform) are small clumps of cells, and the red nucleus is found
in the midbrain, not in the cerebellum. (p. 112–115)
Review and Study Questions for
Chapter 6
1. The MRI of a 66-year-old man shows a tumor 2.0 cm in diameter
located in the lateral wall of the atrium of the lateral ventricle.
Which of the following structures does this lesion most likely
damage?
䊊 (A) Corticonuclear (corticobulbar) fibers
䊊 (B) Corticospinal fibers
䊊 (C) Optic radiations
䊊 (D) Pulvinar nucleus
䊊 (E) Splenium of the corpus callosum
2. Which of the following structures is clearly seen in coronal and axial brain slices, and in many MRIs, in planes extending from the
midline laterally through the basal nuclei?
䊊 (A) Anterior commissure
䊊 (B) Column of the fornix
䊊 (C) Genu of the internal capsule
䊊 (D) Optic chiasm
䊊 (E) Posterior commissure
276 Q & A’s: A Sampling of Study and Review Questions with Explained Answers
3. The MRI of a 49-year-old woman with movement and personality
disorders and with cognitive dysfunction shows a large anterior
horn of the lateral ventricle. The attending physician suspects that
her disease has resulted in loss of brain tissue in the lateral wall of
the anterior horn. A loss of which of the following structures would
result in this portion of the ventricular system being enlarged?
䊊 (A) Body of the caudate nucleus
䊊 (B) Head of the caudate nucleus
䊊 (C) Lenticular nucleus
䊊 (D) Pulvinar nucleus (i)
䊊 (E) Septum pellucidum and fornix
4. The axial MRI of a 54-year-old man shows an arteriovenous malformation located between the thalamus and the lenticular nucleus. Which of the following structures is probably most affected
by this malformation?
䊊 (A) Anterior commissure
䊊 (B) Anterior limb of the internal capsule
䊊 (C) Extreme capsule
䊊 (D) Retrolenticular limb of the internal capsule
䊊 (E) Posterior limb of the internal capsule
5. In a sagittal MRI, and in a sagittal brain slice, both taken just off
the midline (2–4 mm), which of the following structures would
be clearly evident immediately caudal to the anterior commissure?
䊊 (A) Column of the fornix
䊊 (B) Lamina terminalis
䊊 (C) Mammillothalamic tract
䊊 (D) Optic chiasm
䊊 (E) Precommissural fornix
6. The coronal MRI of a 15-year-old boy shows a 2.0 cm-diameter
tumor in the rostral tip of the temporal (inferior) horn of the lateral ventricle. It is possibly arising from the choroid plexus in this
area of the ventricle. In addition to the hippocampus, this tumor
is most likely impinging on which of the following structures?
䊊 (A) Amygdaloid nucleus
䊊 (B) Body of the caudate nucleus
䊊 (C) Hypothalamus
䊊 (D) Optic radiations
䊊 (E) Putamen
7. Which of the following structures is located immediately internal
to the crus cerebri and appears as a dark shade of grey (hypointense) in a sagittal T1-weighted MRI?
䊊 (A) Brachium of the inferior colliculus
䊊 (B) Periaqueductal grey
䊊 (C) Pretectal area
䊊 (D) Red nucleus
䊊 (E) Substantia nigra
8. An 81-year-old man is brought to the emergency department following a fall while walking in the park. The examination reveals
mild confusion and memory loss, but no obvious motor or sensory
deficits. MRI shows an old infarct in the territory of the thalamus
served by the thalamoperforating artery. Which of the following
nuclei is most likely involved in this lesion?
䊊 (A) Centromedian
䊊 (B) Medial geniculate
䊊 (C) Ventral anterior
䊊 (D) Ventral posterolateral
䊊 (E) Ventral posteromedial
9. Which of the following nuclei is located within the internal
medullary lamina and may be visible in an axial MRI in either T1or T2-weighted images?
䊊 (A) Centromedian
䊊 (B) Dorsomedial
䊊 (C) Pulvinar
䊊 (D) Ventral anterior
䊊 (E) Ventral lateral
10. The sagittal MRI of a 23-year-old woman shows a mass in the right
interventricular foramen (possibly a colloid cyst); the right lateral
ventricle is enlarged. Based on its location, this mass is most likely
impinging on which of the following structures?
䊊 (A) Anterior nucleus of thalamus
䊊 (B) Posterior limb of internal capsule
䊊 (C) Habenular nucleus
䊊 (D) Head of caudate nucleus
䊊 (E) Lamina terminalis
11. The sagittal MRI of a 42-year-old woman taken adjacent to the
midline shows a round structure immediately rostral to the interpeduncular fossa on the inferior surface of the hemisphere.
Which of the following most likely represents this elevation?
䊊 (A) Anterior commissure
䊊 (B) Basilar pons
䊊 (C) Lamina terminalis
䊊 (D) Mammillary body
䊊 (E) Optic chiasm
12. Which of the following structures is located immediately inferior
to the pulvinar and, in the sagittal plane (MRI or brain section),
forms a distinct elevation immediately adjacent to the lateral aspect of the crus cerebri?
䊊 (A) Mammillary nuclei
䊊 (B) Medial geniculate nucleus
䊊 (C) Optic tract
䊊 (D) Subthalamic nucleus
䊊 (E) Uncus
Answers for Chapter 6
1. Answer C: The optic radiations are located in the lateral wall of
the atrium of the lateral ventricle, represent projections from the
lateral geniculate nucleus to the calcarine cortex, pass through the
retrolenticular limb of the internal capsule, and are separated from
the ventricular space by a thin layer of fibers called the tapetum.
The pulvinar and splenium are located rostromedial and medial,
respectively, to the atrium. Corticonuclear and corticospinal
fibers are found in the genu, and the posterior limb of the internal
capsule within the hemisphere. (p. 76, 77, 138, 162)
2. Answer A: The anterior commissure is a mediolaterally oriented
bundle of fibers that crosses the midline and extends laterally, immediately inferior to the basal nuclei. In sagittal section, or in a sagittal MRI, this bundle can be followed into planes of the hemisphere
that include the most lateral portions of the thalamus and the lenticular nucleus. The column of the fornix and optic chiasm are located
immediately adjacent to the midline. The posterior commissure is
located at the caudal aspect of the third ventricle and immediately
Q & A’s: A Sampling of Study and Review Questions with Explained Answers 277
superior to the opening of the cerebral aqueduct. The genu of the
internal capsule is medial to the lenticular nucleus and rostrolateral
to the anterior nucleus of the thalamus. (p. 163, 165, 167, 169, 171)
3. Answer B: The head of the caudate nucleus forms a prominent
bulge in the lateral wall of the anterior horn of the lateral ventricle. In Huntington’s disease, this elevation disappears, and the
wall of the ventricle may become concave laterally; the result being an enlarged anterior horn (hydrocephalus ex vacuo). The body
of the caudate is located in the lateral wall of the body of the lateral ventricle. The lenticular nucleus lies within the hemisphere
and does not border on any ventricular space. The septum and the
fornix are located in the medial wall of the ventricle, and the pulvinar borders on the superior cistern. (p. 75, 76, 152–156, 162)
4. Answer E: The posterior limb of the internal capsule is located
between the lenticular nucleus, which is lateral, and the thalamus,
which is medial. This large fiber bundle contains thalamocortical
projections related to motor and sensory function and descending
corticospinal fibers. The anterior limb of the internal capsule is located between the head of the caudate and the lenticular nucleus,
and the retrolenticular limb is found caudal to the lenticular nucleus. The anterior commissure is in the rostroventral portion of
the hemisphere, and the extreme capsule is immediately internal
to the insular cortex. (p. 162, 164, 166)
5. Answer A: The column of the fornix, commonly called the postcommissural fornix, lies caudal to, and against, the anterior commissure as it arches around the interventricular foramen and the anterior tubercle of the thalamus. The precommissural fornix is a
diffuse bundle of fibers rostral to the anterior commissure, and the
mammillothalamic tract is located between the mammillary body
and the anterior nucleus of the thalamus. The lamina terminalis and
the optic chiasm are inferior to the anterior commissure. (p. 163)
6. Answer A: The amygdaloid nucleus is in the rostral wall of the
temporal horn of the lateral ventricle. In this position the amygdala is separated from the rostral tip of the hippocampus (the hippocampus occupies the medial and inferior wall of the temporal
horn) by a narrow space of the ventricle. The optic radiations are
in the lateral wall of the temporal horn, but are quite caudal to its
rostral tip. The other choices do not have direct structural relationship to the rostral portions of the temporal horn. (p. 170, 171)
7. Answer E: The substantia nigra is located internal to the crus
cerebri and, in T1-weighted MRI, appears a darker shade of grey
(hypointense) than does the crus. The red nucleus and the periaqueductal grey are located in the midbrain, but do not border on
the crus cerebri. The brachium of the inferior colliculus is found
on the lateral surface of the midbrain, and the pretectal area is adjacent to the cerebral aqueduct at the midbrain-diencephalic junction. (p. 165, 167)
8. Answer C: The ventral anterior nucleus is located in the rostral
portions of the thalamus, is in the territory of the thalamoperforating artery, and projects to large regions of the frontal lobe. An
occlusion of the vessels serving this portion of the thalamus may
result in a decreased level of alertness. The other choices are in
caudal regions of the thalamus, are not in the territory served by
the thalamoperforating artery, and, with the exception of the centromedian nucleus, do not relate to the cortex of the frontal lobe.
(p. 159, 162, 164)
9. Answer A: The centromedian nucleus is found within the internal medullary lamina in a position just rostral to the pulvinar. The
ventral anterior and ventral lateral nuclei are lateral to the internal medullary lamina, the dorsomedial nucleus is medial to this
lamina, and the pulvinar is the large nucleus forming the caudal
part of the dorsal thalamus. (p. 76, 142–143, 162, 164)
10. Answer A: The interventricular foramen is the space formed
between the column of the fornix (located somewhat rostromedially) and the anterior nucleus of the thalamus (located somewhat caudolaterally). The anterior nucleus is located internal to
the anterior tubercle of the thalamus. The head of the caudate is
found in the lateral wall of the anterior horn of the lateral ventricle, and the posterior limb is located in the hemisphere between the thalamus and the lenticular nucleus. The lamina terminalis extends from the anterior commissure inferiority to the
upper edge of the optic chiasm. The habenula is a small elevation
in the caudal and medial wall of the third ventricle. (p. 76, 162,
164)
11. Answer D: The mammillary body forms an obvious elevation
on the inferior aspect of the hemisphere rostral to the interpeduncular fossa/cistern; this small bulge is clearly evident in MRI.
The optic chiasm and the basilar pons are both on the inferior aspect of the brain at the midline. The former is rostral to the infundibulum (and the mammillary body) and the latter is caudal to
the interpeduncular fossa. The lamina terminalis forms the rostral
end of the third ventricle and the anterior commissure is adjacent
to the column of the fornix. (p. 31, 163, 170)
12. Answer B: The medial and lateral geniculate nuclei are located
inferior to the pulvinar, and form elevations on the surface of the
dorsal thalamus; the medial geniculate is adjacent to the lateral
edge of the crus cerebri. The subthalamic nucleus is located internally, the mammillary nuclei (medial and lateral) are on the inferior aspect of the thalamus, and the uncus is on the medial portion
on the temporal pole. The optic tract lies on the surface of the crus
cerebri, but it does not form a distinct elevation on the brain surface inferior to the pulvinar; rather, it has a structural relationship
to the lateral geniculate nucleus. (p. 26, 59, 169)
Review and Study Questions for
Chapter 7
1. A 15-year-old boy is brought to the emergency department after
an accident on his father’s farm. The examination reveals weakness of the left lower extremity, but no frank paralysis. There is a
loss of pinprick sensation on the right side beginning at the T8 dermatome (about half way between the nipple and umbilicus), and
dorsiflexion of the great toe in response to plantar stimulation.
Based on this examination, which of the following represents the
most likely approximate location of this lesion?
䊊 (A) T6 on the left side
䊊 (B) T6 on the right side
䊊 (C) T8 on the left side
䊊 (D) T8 on the right side
䊊 (E) T10 on the left side
278 Q & A’s: A Sampling of Study and Review Questions with Explained Answers
2. A 47-year-old man is transported to the emergency department
from the site of an automobile collision. The examination reveals
a paralysis of both lower extremities. Which of the following most
specifically identifies this clinical picture?
䊊 (A) Alternating hemiplegia
䊊 (B) Hemiplegia
䊊 (C) Monoplegia
䊊 (D) Quadriplegia
䊊 (E) Paraplegia
3. A 68-year-old woman presents with a complaint of difficulty swallowing. Which of the following most specifically identifies this
condition in this patient?
䊊 (A) Dysarthria
䊊 (B) Dysmetria
䊊 (C) Dysphagia
䊊 (D) Dyspnea
䊊 (E) Dysdiadochokinesia
4. A 37-year-old man presents to his family physician with a complaint of pain on his face. The examination shows that gentle stimulation of the cheek and corner of the mouth precipitates a severe,
sharp, lancinating pain. A consulting neurologist orders an MRI
(T2-weighted), which reveals a vascular loop that appears to be
pressing on the trigeminal root proximal to the ganglion. Which
of the following vessels is most likely involved?
䊊 (A) Anterior inferior cerebellar artery
䊊 (B) Posterior cerebral artery
䊊 (C) Posterior inferior cerebellar artery
䊊 (D) Quadrigeminal artery
䊊 (E) Superior cerebellar artery
5. Which of the following brainstem structures receives input from
the frontal eye field (in the caudal part of the middle frontal gyrus,
areas 6 and 8) and is regarded as a vertical gaze center?
䊊 (A) Abducens nucleus
䊊 (B) Edinger-Westphal nucleus
䊊 (C) Oculomotor nucleus
䊊 (D) Paramedian pontine reticular formation (PPRF)
䊊 (E) Rostral interstitial nucleus of the medial longitudinal
fasciculus (MLF)
6. A newborn girl baby is unable to suckle. The examination reveals that
muscles around the oral cavity and of the cheek are poorly developed
or absent. A failure in proper development of which of the following
structures would most likely contribute to this problem for this baby?
䊊 (A) Head mesoderm
䊊 (B) Pharyngeal arch 1
䊊 (C) Pharyngeal arch 2
䊊 (D) Pharyngeal arch 3
䊊 (E) Pharyngeal arch 4
7. Which of the following neurotransmitters is associated with hypothalamic fibers that project to the cerebellar cortex (hypothalamocerebellar fibers)?
䊊 (A) Gamma aminobutyric acid
䊊 (B) Glutamate
䊊 (C) Histamine
䊊 (D) Noradrenalin
䊊 (E) Serotonin
Questions 8 through 9 are based on the following patient.
A 62-year-old woman presents with tremor and ataxia on the right side
of the body excluding the head, and with a loss of most eye movement
on the left; the woman’s eye is rotated slightly down and out at rest.
The left pupil is dilated. There are no sensory losses on her face or
body.
8. Based on the deficits seen in this woman, which of the following
represents the most likely location of the causative lesion?
䊊 (A) Cerebellum on the left
䊊 (B) Cerebellum on the right
䊊 (C) Midbrain on the left
䊊 (D) Midbrain on the right
䊊 (E) Rostral pons on the right
9. The dilated pupil in this woman is most likely a result of which of
the following?
䊊 (A) Intact parasympathetic fibers on the left
䊊 (B) Intact parasympathetic fibers on the right
䊊 (C) Intact sympathetic fibers on the left
䊊 (D) Intact sympathetic fibers on the right
䊊 (E) Interrupted hypothalamospinal fibers on the left
10. Which of the following nuclei are the primary target of cerebellar
efferent fibers that arise in the dentate, emboliform, and globose
nuclei on the left side?
䊊 (A) Ventral anterior nucleus on the right
䊊 (B) Ventral lateral nucleus on the left
䊊 (C) Ventral lateral nucleus on the right
䊊 (D) Ventral posterolateral nucleus on the left
䊊 (E) Ventral posterolateral nucleus on the right
11. A 22-year-old man presents to his family physician with motor
deficits. The examination reveals that the man has jerky up-down
movements of his upper extremities especially noticeable in his
hands when his arms are extended. Which of the following most
specifically designate this abnormal movement?
䊊 (A) Akinesia
䊊 (B) Asterixis
䊊 (C) Dystonia
䊊 (D) Intention tremor
䊊 (E) Resting tremor
12. A 59-year-old man is brought to his family physician by his wife.
He complains of frequent and severe headaches. His wife states
that he does not seem to understand what she is saying when she
talks to him. The examination reveals that the man can speak fluently and clearly, can read notes written on paper, can hear noise,
but has great difficulty understanding or interpreting sounds. MRI
shows a tumor in the temporal lobe. This man is most likely suffering from which of the following?
䊊 (A) Agnosia
䊊 (B) Agraphia
䊊 (C) Alexia
䊊 (D) Aphasia
䊊 (E) Aphonia
Q & A’s: A Sampling of Study and Review Questions with Explained Answers 279
13. A 47-year-old man is brought to the emergency department by local law enforcement personnel. The man is thin, undernourished,
somnolent, and clearly intoxicated. Other indicators, such as a
lack of personal hygiene, suggest that the man’s condition has been
long-term. When the physician asks the man his name and where
he lives the man give a nonsensical response. This man is most
likely suffering from which of the following?
䊊 (A) Broca aphasia
䊊 (B) Klüver-Bucy syndrome
䊊 (C) Korsakoff syndrome
䊊 (D) Munchausen syndrome
䊊 (E) Pick disease
Questions 14 through 15 are based on the following patient.
A 69-year-old man is diagnosed with dysarthria. The history reveals
that the man has had this problem for several weeks. MRI shows an infarcted area in the brainstem on the right side.
14. Damage to which of the following structures would most likely
explain this deficit in this man?
䊊 (A) Cuneate nucleus
䊊 (B) Nucleus ambiguus
䊊 (C) Solitary tract and nuclei
䊊 (D) Spinal trigeminal tract
䊊 (E) Vestibular nuclei
15. Assuming that the infarcted area in the brain of this man is the result of a vascular occlusion, which of the following arteries is most
likely involved?
䊊 (A) Anterior inferior cerebellar
䊊 (B) Labyrinthine
䊊 (C) Posterior inferior cerebellar
䊊 (D) Posterior spinal
䊊 (E) Superior cerebellar
16. Which of the following neurotransmitters is associated with the
cells in the somatomotor cortex that project to the spinal cord as
corticospinal fibers?
䊊 (A) Acetylcholine
䊊 (B) Dopamine
䊊 (C) Gamma aminobutyric acid
䊊 (D) Glutamate
䊊 (E) Serotonin
17. A 77-year-old woman presents with a loss of pain and thermal sensations on the right side of her face and on the left side of her body.
Which of the following most specifically describe this deficit in this
woman?
䊊 (A) Alternating hemianesthesia
䊊 (B) Epidural anesthesia
䊊 (C) Facial hemiplegia
䊊 (D) Hemifacial spasm
䊊 (E) Superior alternating hemiplegia
18. During a busy day in the emergency department, the neurology
resident sees three patients with brainstem lesions. The first is an
83-year-old woman with a lesion in the territory of the midbrain
served by the quadrigeminal and lateral posterior choroidal arteries. The second is a 68-year-old man with a posterior inferior cerebellar artery (lateral medullary or Wallenberg) syndrome. The
third is a 47-year-old woman with a presumptive glioblastoma
multiforme invading the mid- to lateral portions of the pontine
tegmentum and adjacent portions of the middle cerebellar peduncle. Which of the following would most likely be seen in all three
patients assuming a thorough neurologic examination?
䊊 (A) Claude syndrome
䊊 (B) Contralateral hemiplegia
䊊 (C) Facial hemiplegia
䊊 (D) Horner syndrome
䊊 (E) Medial medullary syndrome
19. Which of the following structures serves as an important landmark
in the placement of the intentional division of the spinal cord
(myelotomy) in an anterolateral cordotomy?
䊊 (A) Anterior median sulcus
䊊 (B) Anterolateral sulcus
䊊 (C) Denticulate ligament
䊊 (D) Posterior intermediate sulcus
䊊 (E) Posterolateral sulcus
20. A 17-year-old boy is brought to the emergency department from
a high school football game. The examination reveals a loss of vibratory sensation and discriminative touch on the left lower extremity and to the level of the umbilicus. CT shows a vertebral
fracture with bone displacement into the vertebral canal. Which
of the following indicates the most likely level of damage to the
spinal cord in this boy?
䊊 (A) T7–8 on the left
䊊 (B) T10 on the left
䊊 (C) T12 on the left
䊊 (D) T8–9 on the right
䊊 (E) T10 on the right
21. During the neurologic examination of a 52-year-old man, the
physician decides to test the gag reflex. Which of the following difficulties does this man have that would cause the physician to decide to test this particular reflex?
䊊 (A) Dysgeusia
䊊 (B) Dysmetria
䊊 (C) Dysphagia
䊊 (D) Dyspnea
䊊 (E) Gustatory agnosia
22. A 57-year-old woman presents with the main complaint of difficulty speaking. The examination reveals that the woman’s tongue
deviates to the right on attempted protrusion. When she says “Ah”
her soft palate elevates slightly on the left and the uvula deviates
to the same side. This combination of deficits would most likely
indicate a small lesion in which of the following?
䊊 (A) Crus cerebri on the right
䊊 (B) Genu of the internal capsule on the left
䊊 (C) Genu of the internal capsule on the right
䊊 (D) Lateral medulla on the right
䊊 (E) Medial medulla on the right
280 Q & A’s: A Sampling of Study and Review Questions with Explained Answers
23. A 36 year-old-woman is diagnosed with myasthenia gravis. Which
of the following deficits are seen first in about one-half of patients
with this disease and is present in most at some time during its
course?
䊊 (A) Diplopia
䊊 (B) Dysmetria
䊊 (C) Lower extremity weakness
䊊 (D) Tremor
䊊 (E) Upper extremity weakness
Questions 24 through 26 are based on the following patient.
An 80-year-old woman is brought to the emergency department from
an assisted care facility. The woman, who is in a wheelchair, complains
of not feeling well, of numbness on her face, and of being hoarse, although she claims not to have a cold. The examination reveals a loss of
pain and thermal sensations on the right side of her face and on the left
side of her body. CT shows an infarcted area in the lateral portion of
the medulla.
24. A lesion of which of the following structures in this woman would
explain the loss of pain and thermal sensations on her body excluding the head?
䊊 (A) Anterolateral system on the left
䊊 (B) Anterolateral system on the right
䊊 (C) Medial lemniscus on the left
䊊 (D) Spinal trigeminal nucleus on the left
䊊 (E) Spinal trigeminal tract on the left
25. The hoarseness in this woman is most likely due to which of the
following?
䊊 (A) Lesion of the facial nucleus
䊊 (B) Lesion of the hypoglossal nucleus/nerve
䊊 (C) Lesion of the nucleus ambiguus
䊊 (D) Lesion of the spinal trigeminal tract
䊊 (E) Lesion of the trigeminal nucleus
26. Assuming this woman suffered a vascular occlusion, which of the
following vessels is most likely involved?
䊊 (A) Anterior inferior cerebellar artery
䊊 (B) Anterior spinal artery
䊊 (C) Posterior inferior cerebellar artery
䊊 (D) Posterior spinal artery
䊊 (E) Superior cerebellar artery
27. In the course of a neurologic examination of a 23-year-old man,
the physician places her index finger on the midline of the
mandible and taps it with a percussion hammer stimulating the afferent limb of the jaw ( jaw-jerk) reflex. Collateral fibers from
which of the following brainstem nuclei enter the trigeminal motor nucleus to initiate the motor response?
䊊 (A) Hypoglossal
䊊 (B) Mesencephalic
䊊 (C) Principal sensory
䊊 (D) Spinal trigeminal, pars caudalis
䊊 (E) Spinal trigeminal, pars interpolaris
28. A 45-year-old-man is brought to his family physician by his wife.
The man’s main complaint is that he feels “ real dizzy” and a little
nauseated. The examination reveals that the man has a disease of
his semicircular canals. While sitting still the man perceives that
his body is actually moving around the room. Which of the following most specifically describes this condition?
䊊 (A) Ataxia
䊊 (B) Hysterical vertigo
䊊 (C) Nystagmus
䊊 (D) Objective vertigo
䊊 (E) Subjective vertigo
Questions 29 and 30 are based on the following patient.
A 37-year-old-man is brought to the emergency department from the
site of an automobile collision. He was unrestrained and, as a result,
has extensive injuries to his face and head. CT shows numerous fractures of the facial bones and skull and blood in the rostral areas of the
frontal lobes and in the rostral 3–4 cm of the temporal lobes, bilaterally. After several weeks of recovery the man is moved to a long-term
care facility. His behavior is characterized by (1) difficulty recognizing
sounds such as music or words; (2) a propensity to place inappropriate
objects in his mouth; (3) a tendency to eat excessively or to eat nonfood items such as the leaves on the plant in his room; and (4) a tendency to touch his genitalia.
29. Which of the following most specifically describes the tendency of
this man to eat excessively?
䊊 (A) Aphagia
䊊 (B) Dysphagia
䊊 (C) Dyspnea
䊊 (D) Hyperorality
䊊 (E) Hyperphagia
30. Based on the totality of this man’s deficits he is most likely suffering from which of the following?
䊊 (A) Klüver-Bucy syndrome
䊊 (B) Korsakoff syndrome
䊊 (C) Senile dementia
䊊 (D) Wallenberg syndrome
䊊 (E) Wernicke aphasia
31. A 31-year-old woman is examined by an otolaryngologist pursuant to her complaint of hearing difficulties. The physician places
a tuning fork against the woman’s mastoid bone until she no longer
perceives sound, then moves the prongs to her external ear where
a faint sound is again heard. This maneuver is best described as:
䊊 (A) A negative (abnormal) Rinne test
䊊 (B) A normal Binet test
䊊 (C) A normal Weber test
䊊 (D) A positive (normal) Rinne test
䊊 (E) Weber test localizing to the deaf side
Q & A’s: A Sampling of Study and Review Questions with Explained Answers 281
32. A 64-year-old man is brought to a rural health clinic by a neighbor. The history reveals that the man is a recluse, lives by himself,
and does not regularly visit a physician. The examination reveals
that the man has difficulty walking, chorea and dystonia, and is suffering dementia. The neighbor believes that the man’s father died
from a similar disease. A tentative diagnosis of Huntington’s disease is made. Absence of which of the following structures in an
MRI of this man would be consistent with this diagnosis?
䊊 (A) Anterior lobe of cerebellum
䊊 (B) Head of the caudate
䊊 (C) Lateral thalamic nuclei
䊊 (D) Substantia nigra
䊊 (E) Subthalamic nucleus
33. A 23-year-old man is brought to the emergency department from an
accident at a construction site. CT shows a fracture of the left mastoid bone with total disruption of the stylomastoid foramen. Which
of the following deficits would most likely be seen in this man?
䊊 (A) Alternating hemianesthesia
䊊 (B) Alternating hemiplegia
䊊 (C) Central seven
䊊 (D) Facial hemiplegia
䊊 (E) Hemifacial spasm
34. Cell bodies located in which of the following ganglia of the head
supply postganglionic fibers to the parotid gland?
䊊 (A) Ciliary
䊊 (B) Intramural
䊊 (C) Otic
䊊 (D) Pterygopalatine
䊊 (E) Submandibular
Questions 35 and 36 are based on the following patient.
A 23-year-old man is brought to the emergency department from the
site of an automobile collision. CT shows fractures of the facial bones
and evidence of bilateral trauma to the temporal lobes (blood in the
substance of the brain).
35. As this man recovers, which of the following deficits is most likely
to be the most obvious in this man?
䊊 (A) A bilateral sensory loss in the lower body
䊊 (B) A loss of immediate and short-term memory
䊊 (C) A loss of long-term (remote) memory
䊊 (D) Dementia
䊊 (E) Dysphagia and dysarthria
36. Assuming that this man has also sustained bilateral injury to the
Meyer-Archambault loop, which of the following deficits would
this man also most likely have?
䊊 (A) Bitemporal hemianopsia
䊊 (B) Bilateral inferior quadrantanopia
䊊 (C) Bilateral superior quadrantanopia
䊊 (D) Left superior quadrantanopia
䊊 (E) Right superior quadrantanopia
37. A 59-year-old man, who is a family physician, confides in a neurology colleague that he believes he has early stage Parkinson’s disease. The neurologic examination reveals a slight resting tremor
of the left hand, a slow gait, and a lack of the normal range of facial expression for this man. Which of the following is the most
likely location of the degenerative changes at this stage of this
physician’s disease?
䊊 (A) Bilateral substantia nigra
䊊 (B) Left globus pallidus
䊊 (C) Left substantia nigra
䊊 (D) Right globus pallidus
䊊 (E) Right substantia nigra
38. A 14-year-old boy is brought to the emergency department after
an accident on his BMX bicycle. The examination reveals that the
boy has severe facial injuries. Craniofacial CT shows fracture of facial bones and probable crushing of the structures traversing the
superior orbital fissure. Damage to which of the following structures passing through this fissure would result in diplopia when attempting to look down and in?
䊊 (A) Abducens nerve
䊊 (B) Oculomotor nerve
䊊 (C) Ophthalmic nerve
䊊 (D) Ophthalmic vein
䊊 (E) Trochlear nerve
Questions 39 through 41 are based on the following patient.
A 67-year-old man is brought to the emergency department by his
wife. She explains that he fell suddenly, could not get up, and complained of feeling sick. The examination revealed a left-sided weakness
of the upper and lower extremities, a lack of most movement of the
right eye, and a dilated pupil on the right. MRI shows an infarcted area
in the brainstem.
39. The weakness of this man’s extremities is explained by damage to
the axons of cell bodies that are located in which of the following
regions of the brain?
䊊 (A) Left somatomotor cortex
䊊 (B) Right anterior paracentral gyrus
䊊 (C) Right crus cerebri
䊊 (D) Right precentral gyrus
䊊 (E) Right somatomotor cortex
40. This man’s dilated pupil is due to damage to which of the following fiber populations?
䊊 (A) Preganglionic fibers from the Edinger-Westphal nucleus
䊊 (B) Preganglionic fibers from the inferior salivatory nucleus
䊊 (C) Postganglionic fibers from the ciliary ganglion
䊊 (D) Postganglionic fibers from the geniculate ganglion
䊊 (E) Postganglionic fibers from the superior cervical ganglion
41. Which of the following descriptive phrases best describes the constellation of signs and symptoms seen in the man?
䊊 (A) Alternating hemianesthesia
䊊 (B) Brown-Séquard syndrome
䊊 (C) Inferior alternating hemiplegia
䊊 (D) Middle alternating hemiplegia
䊊 (E) Superior alternating hemiplegia
282 Q & A’s: A Sampling of Study and Review Questions with Explained Answers
42. Which of the following structures contains the cell bodies of origin for fibers conveying taste information from the anterior twothirds of the tongue?
䊊 (A) Ciliary ganglion
䊊 (B) Geniculate ganglion
䊊 (C) Superior ganglion of the vagus nerve
䊊 (D) Superior ganglion of the glossopharyngeal nerve
䊊 (E) Trigeminal ganglion
46. The facial sensory deficits experienced by this woman are explained by a lesion to the axons of cell bodies located in which of
the following structures?
䊊 (A) Anterior trigeminothalamic fibers on the left
䊊 (B) Left trigeminal ganglion
䊊 (C) Principal sensory nucleus on the left
䊊 (D) Right trigeminal ganglion
䊊 (E) Spinal trigeminal nucleus on the right
43. During a screening neurologic examination of a 39-year-old man,
the physician taps the supraorbital ridge, stimulating the supraorbital nerve, and elicits a motor response. Which of the following
most likely represents the motor response in this man?
䊊 (A) Constriction of the masticatory muscles
䊊 (B) Constriction of the orbicularis oculi muscle
䊊 (C) Constriction of the pupil
䊊 (D) Dilation of the pupil
䊊 (E) Horizontal nystagmus
47. The loss of pain and thermal sensations experienced by this woman
on the right side of her body (excluding the face) is most likely the
result of damage to which of the following structures?
䊊 (A) Anterolateral system fibers on the left
䊊 (B) Anterolateral system fibers on the right
䊊 (C) Anterior trigeminothalamic fibers on the left
䊊 (D) Medial lemniscus on the left
䊊 (E) Medial lemniscus on the right
44. A 67-year-old man has a bilateral anterolateral cordotomy at T10
for intractable pelvic pain. Four months after this procedure the
man begins to experience pain sensations. Which of the following
would most likely explain this apparent recurrence of pain in this
man?
䊊 (A) Activation of postsynaptic posterior column and spinocervicothalamic pathways
(B)
Activation of recurrent corticospinal fibers
䊊
(C)
Activation
of spinoreticular-reticulothalamic-thalamo䊊
cortical pathways
䊊 (D) Regeneration of anterolateral system fibers in the spinal
cord
(E)
Regeneration of anterolateral system fibers into the
䊊
posterior column system
45. An 84-year-old woman presents to her physician with the complaint of difficulty walking. The examination reveals that the
woman has an unsteady gait and tends to forcibly slap her feet to
the floor as she walks. She has no other deficits. The physician concludes that the woman has sensory ataxia. Degenerative changes
in which of the following would most likely explain this deficit?
䊊 (A) Anterolateral system fibers
䊊 (B) Corticospinal fibers
䊊 (C) Posterior column fibers
䊊 (D) Posterior root fibers
䊊 (E) Vestibulospinal and reticulospinal fibers
Questions 46 through 48 are based on the following patient.
A 70-year-old woman is brought to the emergency department by her
daughter after becoming ill during a trip to the mall. The woman is
conscious but lethargic, and she has trouble speaking and swallowing.
The examination reveals a loss of pain and thermal sensation on the left
side of the face and a hoarse gravely voice (as if the woman has a sore
throat). Movements of the extremities are normal for the woman’s
age, but she has a loss of pain and thermal sensations on the right side
of her body. The corneal reflex is absent on the left side. MRI shows
an infarcted area in the brainstem.
48. Taking into account all the deficits experienced by this woman,
which of the following characterizes the syndrome, and the side,
in this patient?
䊊 (A) Benedikt syndrome on the left
䊊 (B) Lateral medullary syndrome on the left
䊊 (C) Lateral medullary syndrome on the right
䊊 (D) Parinaud syndrome (bilateral)
䊊 (E) Weber syndrome on the right
49. A 17-year-old boy from a poor rural community is diagnosed with
hepatolenticular degeneration (Wilson’s disease). Which of the
following is accumulating in certain tissues of his body and producing health problems?
䊊 (A) Arsenic
䊊 (B) Copper
䊊 (C) Lead
䊊 (D) Magnesium
䊊 (E) Mercury
50. Which of the following represents the location of the postganglionic fibers that influence the dilator pupillae muscle of the iris
on the ipsilateral side?
䊊 (A) Ciliary ganglion
䊊 (B) Edinger-Westphal nucleus
䊊 (C) Hypothalamus
䊊 (D) Intermediolateral cell column
䊊 (E) Superior cervical ganglion
51. A 37-year-old man presents with vertigo, nystagmus, ataxia, and
hearing loss in his right ear. MRI shows a tumor in the cerebellopontine angle. A biopsy specimen of this tumor indicates that this
mass most likely originated from myelin-forming cells on the root
of the vestibulocochlear nerve. Which of the following terms most
correctly identifies this tumor?
䊊 (A) Acoustic neuroma
䊊 (B) Ependymoma
䊊 (C) Glioblastoma multiforme
䊊 (D) Meningioma
䊊 (E) Vestibular schwannoma
Q & A’s: A Sampling of Study and Review Questions with Explained Answers 283
52. An inherited (autosomal recessive) disorder may appear early in
the teenage years. These patients have degenerative changes in the
spinocerebellar tracts, posterior columns, corticospinal fibers,
cerebellar cortex, and at select places in the brainstem. The symptoms of these patients may include ataxia, paralysis, dysarthria,
and other clinical manifestations. This constellation of deficits is
most characteristically seen in which of the following?
䊊 (A) Friedreich ataxia
䊊 (B) Huntington disease
䊊 (C) Olivopontocerebellar degeneration (atrophy)
䊊 (D) Parkinson disease
䊊 (E) Wallenberg syndrome
57. An 11-year-old girl is brought to the family physician by her
mother. The mother explains that the girl has been complaining
that her hands and arms “feel funny”. In fact, the mother states that
the girl cut her little finger, but did not realize it until she saw
blood. The examination reveals a bilateral loss of pain and thermal
sensation on the upper extremities and shoulder. Which of the following is the most likely cause of this deficit in this girl?
䊊 (A) Brown-Séquard syndrome
䊊 (B) Posterior inferior cerebellar artery syndrome
䊊 (C) Tabes dorsalis
䊊 (D) Syringobulbia
䊊 (E) Syringomyelia
53. A 45-year-old man complains to his family physician that there seems
to be something wrong with his mouth. The examination reveals a
weakness of the masticatory muscles, a deviation of the jaw to the left
on closure, and a sensory loss on the same side of the lower jaw. MRI
shows a tumor, presumably a trigeminal schwannoma, in the foramen ovale. Compression of which of the following structures would
most likely be the cause of the deficits experienced by this man?
䊊 (A) Maxillary and mandibular nerves and motor fibers on
the left
䊊 (B) Motor fibers and mandibular nerve on the left
䊊 (C) Motor fibers and mandibular nerve on the right
䊊 (D) Motor fibers and maxillary nerve on the left
䊊 (E) Motor fibers and maxillary nerve on the right
58. A 57-year-old obese man is brought to the emergency department
by his wife. The examination reveals that cranial nerve function is
normal but the man has bilateral weakness of his lower extremities.
He has no sensory deficits. MRI shows a small infarcted area in the
general region of the cervical spinal cord-medulla junction. Which
of the following represents the most likely location of this lesion?
䊊 (A) Caudal part of the pyramidal decussation
䊊 (B) Lateral corticospinal tract on the left
䊊 (C) Pyramids bilaterally
䊊 (D) Pyramid on the right
䊊 (E) Rostral part of the pyramidal decussation
54. A 49-year-old man visits his ophthalmologist with what the man
interprets as “trouble seeing”. The history reveals that the man had
a sudden event a few days before in which he felt sick and was nauseated. The man said his trouble “seeing” started after this sudden
sickness. The examination reveals a loss of abduction and adduction of the right eye and a loss of adduction of the left eye. MRI
confirms an infarcted area in the caudal and medial pontine
tegmentum. Which of the following most specifically identifies
this man’s clinical problem?
䊊 (A) Horizontal gaze palsy
䊊 (B) Internuclear ophthalmoplegia
䊊 (C) One-and-a-half syndrome
䊊 (D) Parinaud syndrome
䊊 (E) Vertical gaze palsy
55. Collaterals of ascending anterior (ventral) trigeminothalamic
fibers that contribute to the vomiting reflex would most likely
project into which of the following brainstem structures?
䊊 (A) Dorsal motor vagal nucleus
䊊 (B) Facial nucleus
䊊 (C) Nucleus ambiguus
䊊 (D) Superior salivatory nucleus
䊊 (E) Trigeminal motor nucleus
56. The topographical arrangement of fibers in the medial lemniscus at
mid-olivary levels is such that the sensory information being conveyed by those fibers located most anterior (ventral) in this bundle
will eventually terminate in which of the following structures?
䊊 (A) Anterior paracentral gyrus
䊊 (B) Lateral one-third of the postcentral gyrus
䊊 (C) Medial one-third of the postcentral gyrus
䊊 (D) Middle one-third of the postcentral gyrus
䊊 (E) Posterior paracentral gyrus
Questions 59 through 61 are based on the following patient.
A 34-year-old woman presents with the complaint of seeing “two of
everything” (diplopia). The history reveals that the woman becomes
tired during the workday to the point where she frequently must leave
her workplace early. The woman said that her vision problems appeared first, and later she noticed that, when she drank, it would “go
down the wrong pipe”. The examination reveals weakness of the ocular muscle, difficulty in swallowing (dysphagia), and mild weakness of
the upper extremities. Sensation is normal. Further laboratory tests
indicate that the woman has a neurotransmitter disease.
59. Based on the history and symptoms experienced by this woman,
which of the following is the most likely cause of her medical condition?
䊊 (A) Amyotrophic lateral sclerosis
䊊 (B) Huntington disease
䊊 (C) Myasthenia gravis
䊊 (D) Multiple sclerosis
䊊 (E) Parkinson disease
60. Which of the following represents the most likely location of the
neurotransmitter dysfunction in this woman?
䊊 (A) At the termination of corticonuclear fibers
䊊 (B) At the termination of corticospinal fibers
䊊 (C) At the neuromuscular junction
䊊 (D) Within the basal nuclei
䊊 (E) Within the cerebellum
61. Which of the following represents the neurotransmitter most
likely affected in this woman?
䊊 (A) Acetylcholine
䊊 (B) Dopamine
䊊 (C) Glutamate
䊊 (D) GABA
䊊 (E) Serotonin
284 Q & A’s: A Sampling of Study and Review Questions with Explained Answers
62. A 39-year-old woman complains to her family physician that
“sometimes I see two of everything, but not always”. The examination reveals that the woman can abduct both eyes and can adduct
her left eye but cannot adduct her right eye. All other eye movement is normal. MRI shows a small lesion suggesting an area of demyelination in the pons. Which of the following represents the
most likely location of this lesion?
䊊 (A) Left abducens nucleus
䊊 (B) Left medial longitudinal fasciculus
䊊 (C) Right abducens nucleus
䊊 (D) Right medial longitudinal fasciculus
䊊 (E) Right PPRF
63. A 20-year-old man is brought to the emergency department from
the site of a motorcycle accident. The examination reveals multiple head injuries and a broken humerus. Cranial CT shows a basal
skull fracture extending through the jugular foramen. Assuming
that the nerve or nerves that traverse this opening are damaged,
which of the following deficits would most likely be seen in this
man?
䊊 (A) Deviation of the tongue to the injured side on protrusion
䊊 (B) Diplopia and ptosis
䊊 (C) Drooping and difficulty elevating the shoulder
䊊 (D) Drooping of the face on the ipsilateral side
䊊 (E) Loss of the efferent limb of the corneal reflex
64. A 17-year-old boy is brought to the pediatrician by his mother. The
examination reveals that the boy has rigidity, athetoid movements
(athetosis), and difficulty speaking. His ophthalmologist reports
that the boy has a greenish-brown ring at the corneoscleral margin.
This boy is most likely suffering from which of the following?
䊊 (A) Huntington disease
䊊 (B) Parkinson disease
䊊 (C) Pick disease
䊊 (D) Sydenham chorea
䊊 (E) Wilson disease
65. A 32-year-old woman complains to her gynecologist that her
breasts are tender and a white fluid issues from her nipples. The
examination reveals that the woman is not pregnant (she had her
ovaries removed at age 28 resultant to a diagnosis of ovarian cancer), that a milky substance can be expressed from her nipples, and
that she has a visual field deficit. MRI shows a tumor impinging on
the midline portion of the optic chiasm. Based on the position of
this tumor which of the following visual deficits would most likely
be seen in this woman?
䊊 (A) Bitemporal hemianopsia
䊊 (B) Left homonymous hemianopsia
䊊 (C) Left superior quadrantanopia
䊊 (D) Right homonymous hemianopsia
䊊 (E) Right superior quadrantanopia
66. Which of the following portions of the cerebellum have a close
structural and functional relationship with the vestibular apparatus and the vestibular nuclei?
䊊 (A) Dentate nucleus and interposed nuclei
䊊 (B) Dentate nucleus only
䊊 (C) Fastigial nucleus and flocculonodular lobe
䊊 (D) Hemisphere of the posterior lobe
䊊 (E) Interposed nuclei and hemisphere of the anterior lobe
67. A 17-year-old boy presents with the major complaint that he is
having trouble playing baseball on the high school varsity team.
The examination reveals a healthy, well-nurtured, athletic boy
with normal motor and sensory function. The visual examination reveals a superior right quadrantanopia. MRI shows a small
lesion in a position consistent with the visual field loss. Which of
the following represents the most likely location of the lesion in
this boy?
䊊 (A) Crossing fibers in the optic chiasm
䊊 (B) Lower portions of the optic radiations in the left temporal lobe
䊊 (C) Lower portions of the optic radiations in the right temporal lobe
䊊 (D) Upper portions of the optic radiations in the left parietal lobe
䊊 (E) Upper portions of the optic radiations in the right parietal lobe
68. A 68-year-old man is brought to the emergency department by his
daughter. She explains that he unexpectedly began to have sudden
movements of his left “arm”. The examination reveals a slender
man with hypertension and with periodic, uncontrollable flailing
movements of his left upper extremity suggestive of hemiballismus. Assuming this to result from a vascular occlusion, MRI
would most likely show an infarction in which of the following
structures?
䊊 (A) Left substantia nigra
䊊 (B) Left subthalamic nucleus
䊊 (C) Right motor cortex
䊊 (D) Right substantia nigra
䊊 (E) Right subthalamic nucleus
Questions 69 through 72 are based on the following patient.
A 67-year-old man visits his family physician with the complaint that
he is not able to “do things like I used to”. The examination reveals that
the man is not able to perform rapid alternating movements with his
left upper extremity, and is not able to touch his left index finger to his
nose because of a tremor that worsens as the finger approaches the
nose. He is able to do these movements on the right. When he walks,
he is unsteady with a tendency to fall to the left. He has no sensory
deficits.
69. Which of the following terms specifically designates the inability
of this man to perform rapid alternating movements?
䊊 (A) Dysarthria
䊊 (B) Dysdiadochokinesia
䊊 (C) Dysmetria
䊊 (D) Intention tremor
䊊 (E) Resting tremor
70. Which of the following terms specifically designates this man’s inability to touch his nose with his index finger?
䊊 (A) Dysmetria
䊊 (B) Intention tremor
䊊 (C) Rebound phenomenon
䊊 (D) Resting tremor
䊊 (E) Static tremor
Q & A’s: A Sampling of Study and Review Questions with Explained Answers 285
71. The MRI of this man shows an infarcted area in the brain. Based
on the deficits this man is experiencing, which of the following
represents the most likely location of this lesion?
䊊 (A) Basal nuclei on the left side
䊊 (B) Basal nuclei on the right side
䊊 (C) Cerebellar cortex and nuclei on the left side
䊊 (D) Cerebellar cortex and nuclei on the right side
䊊 (E) Midbrain on the right side
72. Assuming this lesion to be the result of the occlusion of an artery,
which of the following is the most likely candidate?
䊊 (A) Left anterior inferior cerebellar artery
䊊 (B) Left superior cerebellar artery
䊊 (C) Lenticulostriate arteries on the left
䊊 (D) Right anterior inferior cerebellar artery
䊊 (E) Right superior cerebellar artery
73. A 61-year-old woman complains to her family physician that the
muscles of her face sometimes twitch. The examination reveals
that the woman has irregular and intermittent contractions of facial muscles; sometimes these are painful. MRI shows an aberrant
loop of an artery that appears to be compressing the facial nerve
root. Which of the following is most likely the offending vessel in
this case?
䊊 (A) Anterior inferior cerebellar artery
䊊 (B) Anterior spinal artery
䊊 (C) Posterior inferior cerebellar artery
䊊 (D) Posterior spinal artery
䊊 (E) Superior cerebellar artery
74. An 81-year-old man presents with a loss of pain, thermal sensations, discriminative touch, and vibratory sense on the right side
of his body excluding his head. CT shows a comparatively small
infarct representing the territory of one vessel. Based on the positions and relationships of the pathways conveying the sensations
lost in this man, which of the following represents the most likely
location of this lesion?
䊊 (A) Caudal pons
䊊 (B) Midbrain
䊊 (C) Mid-medulla
䊊 (D) Rostral medulla
䊊 (E) Upper cervical spinal cord
75. The MRI of a 70-year-old man shows an infarcted area in the medial medulla at a mid-olivary level on the left. This correlates
with a loss of position sense from the man’s upper right extremity. Which of the following represents the location of the cell
bodies of origin of those fibers damaged in this patient in the
medulla?
䊊 (A) Cuneate nucleus on the left
䊊 (B) Cuneate nucleus on the right
䊊 (C) Gracile nucleus on the left
䊊 (D) Gracile nucleus on the right
䊊 (E) Posterior root ganglia on the left
76. A 39-year-old woman presents with sustained and oscillating muscle contractions that have twisted her trunk and extremities into
unusual and abnormal postures. This woman is most likely suffering from which of the following?
䊊 (A) Dysarthria
䊊 (B) Dysmetria
䊊 (C) Dysphagia
䊊 (D) Dyspnea
䊊 (E) Dystonia
77. A 21-year-old man is brought to the emergency department from
the scene of an automobile collision. He has a compound fracture
of the humerus, a fractured tibia, various cuts and bruises, and significant facial trauma. Cranial CT shows fractures of the bones of
the face and orbit on the left, and a total collapse of the optic canal
on that side with probable transection of the optic nerve. Following an initial recovery period, which of the following would most
likely be seen during an ophthalmologic examination?
䊊 (A) A loss of both the direct and consensual pupillary response when the light is shown in the right eye
䊊 (B) A loss of only the consensual pupillary response when
the light is shown in the right eye
䊊 (C) A loss of the direct but not the consensual pupillary response when a light is shown in the left eye
䊊 (D) Direct and consensual pupillary responses are intact
when light is shown in the left eye
䊊 (E) Direct and consensual pupillary responses are intact
when light is shown in the right eye
78. A 27-year-old man presents with athetosis (athetoid movements),
rigidity, and dysarthria. He also has a flapping tremor. The man
has an obvious greenish-brown ring at the corneoscleral margin.
A tentative diagnosis of advanced Wilson disease is made. MRI
showing which of the following would provide further, if not conclusive evidence, of this disease?
䊊 (A) Atrophy of gyri of the frontal and temporal lobes
䊊 (B) Degeneration and cavitation of the putamen
䊊 (C) Lacunae in the thalamus and internal capsule
䊊 (D) Loss of cells in the substantia nigra
䊊 (E) Loss of the caudate nucleus
79. A 77-year-old man complains to his family physician that he is having trouble picking up his coffee cup, shaving with a safety razor,
and picking up the checkers when playing with his grandson. The
examination reveals that the man is unable to control the distance,
power, or accuracy of a movement as the movement is taking
place. He undershoots or overshoots that target. Which of the following most specifically describes this condition?
䊊 (A) Bradykinesia
䊊 (B) Dysarthria
䊊 (C) Dysdiadochokinesia
䊊 (D) Dysmetria
䊊 (E) Dysphagia
Questions 80 through 82 are based on the following patient.
A 70-year-old woman is brought to the emergency department by
members of the volunteer fire department of a small town. She primarily complains of weakness. The examination reveals a hemiplegia
involving the left upper and lower extremities, sensory losses (pain,
thermal sensations, and proprioception) on the left side of the body and
286 Q & A’s: A Sampling of Study and Review Questions with Explained Answers
face, and a visual deficit in both eyes. MRI shows an area of infarction
consistent with the territory served by the anterior choroidal artery.
80. Which of the following visual deficits is seen in this woman?
䊊 (A) Left homonymous hemianopsia
䊊 (B) Left nasal hemianopsia
䊊 (C) Left superior quadrantanopia
䊊 (D) Right homonymous hemianopsia
䊊 (E) Right superior quadrantanopia
81. Which of the following most specifically identifies the pattern of
sensory deficits experienced by this woman?
䊊 (A) Alternating hemianesthesia
䊊 (B) Hemianesthesia
䊊 (C) Paresthesia
䊊 (D) Sensory level
䊊 (E) Superior alternating hemiplegia
82. The weakness of the extremities in this woman is most likely due
to damage to which of the following?
䊊 (A) Corticospinal fibers on the left
䊊 (B) Corticospinal fibers on the right
䊊 (C) Somatomotor cortex on the right
䊊 (D) Thalamocortical fibers to motor cortex on the right
䊊 (E) Thalamocortical fibers to sensory cortex on the right
83. A 16-year-old boy is brought to the family physician by his
mother. The mother explains that her son is having trouble in
school even though he is a hard worker and is well behaved. The
examination reveals that the boy has a sensorineural hearing loss
in his right ear. He has no other deficits. Which of the following
represents the most likely location of the lesion in this boy?
䊊 (A) Auditory cortex
䊊 (B) Cochlea
䊊 (C) External ear
䊊 (D) Inferior colliculus
䊊 (E) Middle ear
84. Which of the following laminae of the lateral geniculate nucleus
receive input from the contralateral retina?
䊊 (A) 1, 2
䊊 (B) 1, 3, 5
䊊 (C) 1, 4, 6
䊊 (D) 2, 3, 5
䊊 (E) 3, 4, 5, 6
85. A 12-year-old girl is brought to the pediatrician by her mother
who explains that the girl has started to “act funny”. The history
reveals that the girl was treated for a hemolytic streptococcus infection 4 weeks before the appearance of her symptoms; the
mother states that the girl has had this problem for 3 weeks. The
examination reveals a well-nurtured girl with brisk, flowing, and
irregular movements of her face, neck, and upper extremities.
This girl is most likely suffering from which of the following?
䊊 (A) Huntington disease
䊊 (B) Parkinson disease
䊊 (C) Senile chorea
䊊 (D) Sydenham chorea
䊊 (E) Weber syndrome
Answers for Chapter 7
1. Answer A: The combination of weakness on one side (corticospinal involvement) and a loss of pain sensation on the opposite
side specifies components of a Brown-Séquard syndrome. The
motor loss is ipsilateral to the damage and the sensory loss is contralateral; second order fibers conveying pain information cross in
the anterior white commissure ascending one to two spinal segments in the process. In this patient, the lesion is on the left side
at about the T6 level; this explains the loss of pain sensation on the
right beginning at the T8 dermatome level. Lesions at T8 or T10
would result in a loss of pain sensation beginning, respectively, at
dermatome levels T10 or T12 on the contralateral side. (p.
180–181)
2. Answer E: The paralysis of both lower extremities is paraplegia.
Monoplegia specifies paralysis of one extremity, hemiplegia of
both extremities on the same side, and quadriplegia of all four extremities. An alternating hemiplegia is the combination of a motor cranial nerve deficit on one side and a hemiplegia on the contralateral side; this is a brainstem lesion not a spinal cord lesion.
(p. 190–193)
3. Answer C: While the causes of swallowing difficulties may be
central or peripheral (and multiple), this particular problem is
called dysphagia. Dysmetria is an inability to control the distance
or power of a movement and is commonly seen in cerebellar disease. Dysarthria is difficulty in speaking, and dyspnea is a difficulty
in breathing; the latter is usually associated with diseases of the
lungs or heart. Dysdiadochokinesia, an inability to perform rapid
alternating movements, is seen most commonly in cerebellar disease. (p. 190, 202)
4. Answer E: One possible cause of trigeminal neuralgia (tic
douloureux) is compression of the trigeminal root by the superior
cerebellar artery or its main branches; surgical relocation of the
aberrant vessel (neurovascular decompression) relieves the symptoms. Hemifacial spasm may be caused by compression of the facial nerve by the anterior inferior cerebellar artery (commonly
called AICA). The other choices do not cause trigeminal neuralgia
and are not a principal cause of cranial nerve dysfunction via root
compression. (p. 41, 184–185)
5. Answer E: The rostral interstitial nucleus of the medial longitudinal fasciculus receives cortical input from the frontal eye field on
the ipsilateral side and projects to the ipsilateral (heavy) and contralateral (light) oculomotor and trochlear nuclei. This nucleus is
regarded as the vertical gaze center. The paramedian pontine
reticular formation is the horizontal gaze center. The oculomotor
and abducens nuclei do not receive direct input from the frontal
eye field and the Edinger-Westphal is a visceromotor nucleus containing preganglionic parasympathetic cell bodies. (p. 192–193)
6. Answer C: The absence of, or the aberrant development of,
muscle around the oral cavity and over the cheek (muscles of facial expression, innervated by the facial [VII] nerve) indicate a failure of proper differentiation of the second (2nd) pharyngeal arch.
Arch 2 also gives rise to the stapedius, buccinator, stylohyoid,
platysma, and posterior belly of the digastric. Mesoderm of the
head outside of the pharyngeal arches gives rise to the extraocular
muscles and muscles of the tongue. The muscles of mastication
Q & A’s: A Sampling of Study and Review Questions with Explained Answers 287
(plus the tensor tympani, tensor veli palati, mylohyoid, anterior
belly of the digastric) arise from arch 1, the stylopharyngeus from
arch 3, and striated muscles of the pharynx, larynx, and upper
esophagus from arch 4. (p. 202–203)
7. Answer C: Hypothalamocerebellar fibers that project to the
cerebellar nuclei and cortex contain histamine. GABA is found in
several neurons that are located in the cerebellar cortex, and in
Purkinje cells glutamate is found in many pontocerebellar fibers
and in granule cells of the cerebellar cortex; and noradrenalin is
found in ceruleocerebellar fibers. Serotonin is found in cells of the
reticular formation and in some raphe cells that project to the
cerebellum. (p. 206–207)
8. Answer C: The best localizing sign in this patient is the paucity
of eye movement and dilated pupil on the left; this indicates a lesion of the midbrain on the left at the level of the exiting oculomotor fibers. The red nucleus is found at the same level and, more
importantly, immediately lateral to the red nucleus is a compact
bundle of cerebellothalamic fibers. The ataxia and tremor are related primarily to damage to these cerebellar efferent fibers. The
motor deficit is contralateral to the lesion because the corticospinal fibers, through which the deficit is expressed, cross at the
motor (pyramidal) decussation. Lesions at the other choices
would not result in a paucity of eye movement and are, therefore,
not potential candidates. (p. 132–133, 208–211)
9. Answer C: The lesion on the exiting oculomotor fibers (on the
left) damages the preganglionic fibers from the Edinger-Westphal
nucleus and removes their influence on the pupil. Consequently,
the intact postganglionic sympathetic fibers from the ipsilateral superior cervical ganglion predominate, and the pupil dilates.
Choices on the right are on the incorrect side. Damage to hypothalamospinal fibers would remove sympathetic influence at the
intermediolateral cell column, and the pupil would constrict
(parasympathetic domination). (p. 200–201, 208–211, 220–221)
10. Answer C: Cerebellar efferent fibers exit the cerebellum via the
superior cerebellar peduncle, cross in its decussation, and terminate primarily in the ventral lateral nucleus (VL). Consequently,
the cerebellar nuclei on the left project to the right VL. The ventral anterior nucleus does not receive significant cerebellar input.
While the ventral posterolateral nucleus receives a limited amount
of cerebellar input, its major role is the relay of somatosensory information to the primary somatosensory cortex (postcentral
gyrus). (p. 210–211)
11. Answer B: The jerking movements of the upper extremity (asterixis) are also called a flapping tremor and are seen in patients
with hepatolenticular degeneration (Wilson disease). Akinesia is
lack of movement. Resting tremor is seen in patients with disease
of the basal nuclei, such as Parkinson disease, and an intention
tremor is a characteristic of patients with cerebellar lesions. Dystonia is the result of sustained muscle contractions that twist the
extremities, trunk, and neck into distorted and abnormal postures. (p. 214–215)
12. Answer A: This man is unable to recognize or comprehend the
meaning of sounds; although he is able to hear sounds, he is not
able to put meaning to the sounds; this man is suffering from auditory agnosia. Agraphia is the inability to write in a person with
no paralysis, and alexia is the inability to comprehend the mean-
ing of written or printed words. Aphonia is a loss of the voice frequently due to disease of, or injury to, the larynx. Aphasia is seen
in individuals with a lesion in the dominant hemisphere, and is
manifest as an inability to comprehend the meaning of spoken,
written, or various other types of input. (p. 226–227)
13. Answer C: The Korsakoff syndrome is a constellation of deficits
the include memory loss, confabulation, amnesia, and dementia
that is seen in chronic alcoholics; the manifestations are related, in
part, to excessive alcohol consumption and malnutrition. Therapeutic doses of thiamine are used to treat this disease. Broca aphasia (nonfluent or expressive aphasia) results from lesions in the
dominant hemisphere. The Klüver-Bucy syndrome is related to
bilateral lesions to the amygdaloid complex, and Pick disease is dementia related to atrophy of the frontal and temporal lobes. Munchausen syndrome is the fabrication or feigning of illness or disease
to gain attention or control. (p. 232–233)
14. Answer B: Cell bodies in the nucleus ambiguus innervate muscles of the pharynx and larynx, including what is commonly called
the vocalis muscle. A lesion of this nucleus is one cause of
dysarthria. The solitary tract and nuclei are concerned with visceral afferent information including taste, and the spinal trigeminal tract is made the central processes of primary sensory fibers
conveying general somatic afferent (GSA) information from the
ipsilateral side of the face and oral cavity. Proprioceptive information from the ipsilateral upper extremity is transmitted via the
cuneate nucleus; the vestibular nuclei are related to balance, equilibrium, and control of eye movement. (p. 202–203)
15. Answer C: The area of the brainstem that contains the nucleus
ambiguus is served by branches of the posterior inferior cerebellar artery (PICA). Occlusion of this vessel usually gives rise to the
PICA (lateral medullary or Wallenberg) syndrome. The anterior
inferior cerebellar artery (AICA) serves the lateral and inferior
cerebellar surface and the superior cerebellar artery serves the superior surface and much of the cerebellar nuclei. The labyrinthine
artery, a branch of AICA, serves the inner ear. The posterior
spinal artery serves the posterior columns and their nuclei. (p.
110–111, 202–203)
16. Answer D: Glutamate is found in many efferent fibers of the
cerebral cortex including those of the corticospinal tract. Consequently, there are many glutaminergic terminals in the spinal
cord. Acetylcholine is found at many central nervous system
(CNS) sites and at the neuromuscular junction, and dopamine is
found mainly in cells of the substantia nigra-pars compacta and in
their nigrostriatal terminals. Gamma aminobutyric acid (GAMA)
is an inhibitory neurotransmitter and is found in many interneurons in the CNS. Serotonin is found in CNS areas such as the hypothalamus, basal nuclei, and the raphe nuclei. (p. 190)
17. Answer A: The loss of sensation on one side of the face and the
opposite side of the body is an alternating hemianesthesia (also
called an alternate hemianesthesia or a crossed hemianesthesia).
Epidural anesthesia refers to anesthesia resultant to injection of an
appropriate agent into the epidural space. The other choices are
motor abnormalities. (p. 180–181, 186–187)
18. Answer D: Lesions in the lateral portions of the brainstem damage descending projections from the hypothalamus to the ipsilateral intermediolateral cell column at spinal levels T1-T4, these be-
288 Q & A’s: A Sampling of Study and Review Questions with Explained Answers
ing the hypothalamospinal fibers. The result is Horner syndrome
on the side ipsilateral to the lesion. Horner syndrome may also be
seen following cervical spinal cord lesions. A contralateral hemiplegia is not seen in lesions in lateral areas of the brainstem. The
other choices are syndromes or deficits specific to medial brainstem areas or to only a particular level. (p. 110, 124, 136,
220–221)
19. Answer C: The denticulate ligament is located on the lateral aspect of the spinal cord at a midpoint in the posterior-anterior extent of the spinal cord. The anterolateral system, the tract divided
in the anterolateral cordotomy, is located in the anterolateral portion of the spinal cord just inferior to the position of the denticulate ligament. The posterolateral sulcus is the entrance point for
sensory fibers of the posterior roots; the anterolateral sulcus is the
exit point for motor fibers of the anterior root; and the posterior
intermediate sulcus separates the gracile and cuneate fasciculi. The
anterior median sulcus is located on the anterior midline and contains the anterior spinal artery. (p. 182–183)
20. Answer B: Damage to the gracile fasciculus on the left at T10,
the level of the umbilicus, will result in the deficits experienced by
this boy. The gracile fasciculus contains uncrossed ascending fibers
conveying vibratory sensation, discriminative touch, and proprioception; consequently, the deficits will be seen beginning at the
level of the lesion and extending caudally on the same side. These
fibers are the central processes of primary sensory neurons whose
cell bodies are located in the ipsilateral posterior root ganglion.
The other choices are either on the wrong side (right) or at the
wrong level. (p. 178–179)
21. Answer C: The gag reflex is not regularly tested. However, in
patients with dysphagia (difficulty swallowing) or dysarthria (difficulty speaking), the gag reflex should be evaluated. Dysmetria is
a movement disorder associated with cerebellar lesions; dysgeusia
is the perception of an abnormal taste or of a tastant when there is
none; and dyspnea is difficulty breathing, usually associated with
disease of the lung or heart. Gustatory agnosia is the inability to
recognize food or distinguish between different food items. (p.
186–187)
22. Answer B: The combination of a deviation of the tongue to one
side (right) and the uvula to the opposite side (left) indicates a lesion in the genu of the internal capsule on the left involving corticonuclear (corticobulbar) fibers. Corticonuclear fibers to the hypoglossal nucleus are crossed and the tongue deviates toward the
weak side on protrusion. These fibers to the nucleus ambiguus are
also crossed resulting in weakness of the contralateral side of the
palate. However, on attempted phonation (say “Ah”), the strong
side of the palate will contract and elevate, and the uvula will deviate to the intact side (opposite to the tongue). Lesions in the
right genu would result in deficits on the opposite sides. Lesions
in the medial medulla on the right would include the tongue, exclude the uvula but also show a left-sided hemiplegia. Lesions of
the right lateral medulla could include the uvula, but exclude the
tongue. A lesion in the crus would include a number of additional
deficits and would have to be on the left, not the right. (p.
192–193)
23. Answer A: Deficits of eye movement (resulting in diplopia and
ptosis) are seen first in about 50% of all patients with myasthenia
gravis and are eventually seen in approximately 85% of all patients
with this disease. Weakness of the extremities may be seen, but
this almost always follows ocular movement disorders. Dysmetria
is most commonly seen in cerebellar disease and may be present
in patients with lesions involving corticospinal fibers. Tremor is
commonly seen in diseases or lesions of the basal nuclei and the
cerebellum. (p. 200–201)
24. Answer B: The lesion in this woman is in the medulla, and the
sensory loss on the body (excluding the head) is on her left side; a
lesion in the medulla on the right side, involving fibers of the anterolateral system (ALS), accounts for this sensory deficit. A lesion of the ALS on the left side of the medulla would result in sensory deficits on the right side of the body. The spinal trigeminal
tract and nucleus convey pain and thermal sensations from the ipsilateral side (right side in this case) of the face, and the medial
lemniscus conveys vibratory and discriminative touch sensations
from the contralateral side of the body. (p. 180–181, 184–185)
25. Answer C: The woman is hoarse because the lesion involves the
region of the medulla that includes the nucleus ambiguus. These
motor neurons serve, via the glossopharyngeal (IX) and vagus (X)
nerves, the muscles of the larynx and pharynx, including the medial portion of the thyroarytenoid, also called the vocalis muscle.
Paralysis of the vocalis on one side will cause hoarseness of the
voice. Hypoglossal nucleus or nerve, or facial nucleus lesions may
cause difficulty with speech but not hoarseness. The spinal trigeminal tract conveys sensory input from the ipsilateral side of the
face. There are no historical or examination findings to support a
diagnosis of upper respiratory viral findings (cold or flu). (p.
180–181, 202–203)
26. Answer C: The posterior inferior cerebellar artery (PICA)
serves the lateral area of the medulla that contains the anterolateral system, spinal trigeminal tract (loss of pain and thermal sensations from the ipsilateral side of the face), and the nucleus ambiguus. Many patients that present with a PICA (Wallenberg or
lateral medullary) syndrome also have involvement of the vertebral artery on that side. The posterior spinal artery serves the posterior column nuclei in the medulla, and the anterior spinal artery
serves the pyramid, medial lemniscus, and exiting roots of the hypoglossal nerve. The anterior inferior cerebellar artery and the superior cerebellar artery distribute to the pons and midbrain, respectively, plus significant portions of the cerebellum. (p.
110–111, 180)
27. Answer B: The mesencephalic nucleus, a part of the trigeminal
complex, has peripheral processes attached to neuromuscular
spindles in the masticatory muscles, unipolar cell bodies in the rostral pons and midbrain, and central collaterals that distribute bilaterally to the trigeminal motor nucleus. Through these connections, stretching of the spindle initiates a motor response. The
principal sensory and spinal trigeminal nuclei relay touch and
pain/thermal sensations respectively. The hypoglossal nucleus is
motor to the ipsilateral side of the tongue. (p. 184–185)
28. Answer E: The patient’s perception that his body is moving
around the room when he is actually sitting or laying still is subjective vertigo. Objective vertigo is the perception, on the part of
the patient, that he is still and objects in the room are moving. As
its name clearly implies, hysterical vertigo is a psychosomatic disorder. Nystagmus is abnormal rhythmic movements of the eyes,
usually with fast and slow components. Ataxia is an inability to co-
Q & A’s: A Sampling of Study and Review Questions with Explained Answers 289
ordinate muscle activity resulting in an unsteady gait or other uncoordinated movements. (p. 228–229)
29. Answer E: Excessive eating (gluttony), which may include a
propensity to attempt to eat things not considered food items, is
hyperphagia. Dysphagia is difficulty in swallowing, and aphagia is
the inability to eat. Hyperorality is the tendency to put items in
the mouth or to appear to be examining objects by placing them
in the oral cavity. Dyspnea is difficulty breathing. (p. 234–235)
30. Answer A: The constellation of deficits experienced by this man
is characteristic of the Klüver-Bucy syndrome; this may be seen
following bilateral damage to the temporal poles that includes the
amygdaloid complex. The Korsakoff syndrome is seen, for example, in chronic alcoholics, and senile dementia is a loss of cognitive
and intellectual function associated with neurodegenerative diseases of the elderly (such as Alzheimer). Wernicke (receptive or
fluent) aphasia is seen in patients with a lesion in the area of the inferior parietal lobule, and the Wallenberg syndrome results from
a lesion in the medulla characterized by alternating hemisensory
losses and, depending on the extent of the damage, other deficits.
(p. 234–235)
31. Answer D: Hearing a sound in the ipsilateral ear with the application of a tuning fork to the mastoid bone (actually the mastoid
process of the temporal bone), and then hearing the sound again at
the external ear by moving the prongs to the external ear after the
sound disappears at the mastoid is a normal Rinne test. In a negative Rinne test, the sound is not heard at the external meatus after
it has disappeared from touching the mastoid. In a normal Weber
test, sound is heard equally in both ears with application of a tuning fork to the midline of the forehead. A localizing Weber test indicates that sound is heard in the normal ear, but not in the ear with
disease or lesion. The Binet is an intelligence test. (p. 226–227)
32. Answer B: In Huntington disease, especially in advanced stages,
there is a loss of the caudate nucleus and ex vacuo enlargement of
the ventricles. The most obvious portion of the caudate missing in
MRI coronal or axial planes is the head. The anterior lobe of the
cerebellum is diminished in size in alcoholic cerebellar degeneration, but not so in Huntington disease. Lesions of the subthalamic
nucleus result in hemiballismus, and degenerative changes in the
substantia nigra result in the motor deficits seen in Parkinson disease. One of the main responsibilities of the lateral thalamic nuclei
is to convey input to the somatomotor and somatosensory cortices. (p. 214–215)
33. Answer D: The paralysis of facial muscles on one side of the face
(left in this case) with no paralysis of the extremities is a facial
hemiplegia; this is also commonly known as Bell palsy or facial
palsy. Hemifacial spasms are irregular contractions of the facial
muscles, and a central seven refers to paralysis of muscles on the
lower half of the face contralateral to a lesion in the genu of the internal capsule. Alternating hemiplegia describes a motor loss related to a cranial nerve on one side of the head and motor deficits
of the extremities on the contralateral side of the body. A similar
pattern of sensory losses is called an alternating hemianesthesia.
(p. 202–203)
34. Answer C: The otic ganglion receives preganglionic parasympathetic fibers from the inferior salivatory nucleus (associated with
the glossopharyngeal [IX] nerve) and sends postganglionic fibers to
the parotid gland. The ciliary receives from the Edinger-Westphal
nucleus and sends to the pupil; the pterygopalatine and submandibular receives from the superior salivatory nucleus (associated with the facial [VII] nerve) and send, to the lacrimal, submandibular, and sublingual glands, respectively. Intramural
ganglia are located in the gut and receive input from the dorsal motor vagal nucleus. (p. 202–203)
35. Answer B: Bilateral damage to the temporal lobes, as in an automobile collision, may result in damage to the hippocampus.
While remote memory, the ability to recall events that happened
years or decades ago, is intact, the man will have difficulty “remembering” recent or immediate events. That is, he will find it
difficult, if not impossible, to turn a new experience into longerterm memory (something that can be recalled in its proper context at a later time). Dysphagia (difficulty swallowing) and
dysarthria (difficulty speaking) are deficits usually seen in brainstem lesions. Bilateral sensory losses of the lower portion of the
body could be seen with bilateral damage to the posterior paracentral gyri (falcine meningioma) or to the anterior white commissure of the spinal cord. Dementia is a multiregional symptom
that usually involves several areas of the brain, cortical as well as
subcortical. (p. 232–233)
36. Answer C: The Meyer-Archambault loop is composed of optic
radiation fibers that loop through the temporal lobe; these fibers,
on each side, convey visual input from the contralateral superior
quadrant of the visual field. Consequently, a bilateral lesion of
these fibers results as a bilateral superior quadrantanopia. Bilateral
inferior quadrantanopia is seen in bilateral lesions that would involve the superior portion of the optic radiations. Right or left superior quadrantanopia is seen in cases of unilateral damage to, respectively, the left or right Meyer-Archambault loop. A
bitemporal hemianopsia results in a lesion of the optic chiasm. (p.
220, 223)
37. Answer E: Degenerative changes in the dopamine-containing
cells of the substantia nigra pars compacta on the right side correlate with a left-sided tremor. The altered message through the
lenticular nucleus and thalamus and on to the motor cortex on the
side of the degenerative changes will result in tremor on the opposite (right) side via altered messages traveling down the corticospinal tract. The initial symptoms of Parkinson disease appear
on one side in about 80% of patients and extend to bilateral involvement as the disease progresses. Bilateral changes in the substantia nigra correlate with bilateral deficits. The globus pallidus
does not receive direct nigral input but rather input via a nigrostriatal-striatopallidal circuit. (p. 214–215)
38. Answer E: Damage to the trochlear nerve will cause diplopia on
gaze inward and downward on the side of the injury. Abducens
damage will result in an inability to look laterally on the side of the
lesion, and oculomotor injury will result in the loss of most eye
movement on that side; the eye will be deviated slightly down and
out. The ophthalmic nerve is sensory. (p. 200–201)
39. Answer E: The combination of eye movement disorders and a
contralateral hemiplegia localizes this lesion to the midbrain on the
side of the ocular deficits (right side). This also specifies that corticospinal fibers on the right (in the crus) are damaged, and places
the location of the cells of origin for these fibers in the somatomotor cortex on the right side. The right crus contains the axons
290 Q & A’s: A Sampling of Study and Review Questions with Explained Answers
of these fibers but not the neuronal cell bodies. The left somatomotor cortex influences the right extremities. The right precentral gyrus does not contain cells projecting to the left lumbosacral
spinal cord (left lower extremity), and the right anterior paracentral gyrus does not contain the cells that project to the left cervical spinal cord (left upper extremity). (p. 15, 190–193)
40. Answer A: The lesion in this man is central (brainstem) and involves the IIIrd nerve. Consequently, the damage is to the preganglionic parasympathetic fibers in the root of the oculomotor
(III) nerve; this removes the parasympathetic influence (pupil constriction) that originates from the Edinger-Westphal nucleus.
Fibers from the superior cervical ganglion are intact, hence the dilated pupil. Fibers from the geniculate ganglion and inferior salivatory nucleus distribute on the facial (VII) and glossopharyngeal
(IX) nerves respectively. Postganglionic fibers from the ciliary
ganglion, while involved in this pathway, are not damaged in this
lesion. (p. 200–201)
41. Answer E: The loss of most eye movement on one side (oculomotor nerve root involvement) coupled with a paralysis of the extremities on the contralateral side is a superior alternating hemiplegia (this is also Weber syndrome): superior because it is the most
rostral of three; alternating because it is a cranial nerve on one side
and the extremities on the other; and hemiplegia because one-half
of the body below the head is involved. A middle alternating hemiplegia involves the abducens (VI) nerve root and adjacent corticospinal fibers, and an inferior alternating hemiplegia involves the
hypoglossal (XII) nerve root and corticospinal fibers in the pyramid. Alternating hemianesthesia is a sensory loss, and a BrownSéquard syndrome is a spinal cord lesion with no cranial nerve
deficits. (p. 200–201)
42. Answer B: Taste fibers (special visceral afferent, SVA) that
serve the anterior two-thirds of the tongue on the ipsilateral side
are conveyed on the facial nerve and have their cell bodies of origin in the geniculate ganglion. The trigeminal ganglion contains
cell bodies that convey general sensation (general somatic afferent, GSA), and the ciliary ganglion contains visceromotor cell
bodies (general visceral efferent GVE, postganglionic, parasympathetic). The superior ganglion of the glossopharyngeal contains
cell bodies for taste from the posterior one-third of the tongue,
and the superior ganglion of the vagus nerve contains cell bodies
for taste from the root of the tongue. (p. 187)
43. Answer B: Stimulation of the supraorbital nerve (Vth nerve, afferent limb of the supraorbital reflex) results in contraction of the
orbicularis oculi muscle (VIIth nerve, efferent limb of the supraorbital reflex). Changes in pupil size relate to the third nerve, the
pupillary light reflex, and the distribution of postganglionic fibers
from the superior cervical ganglion. Contraction of masticatory
muscles is seen in the jaw-jerk reflex, and nystagmus usually results from cerebellum or brainstem lesions or disease of the
vestibular apparatus. (p. 184–185, 202–203)
44. Answer A: Fibers in the postsynaptic posterior column and in
the spinocervicothalamic pathways are spared in an anterolateral
cordotomy. These pathways originate from those laminae of the
posterior horn that also contribute to the anterolateral system. It
is possible that these pathways remodel to transmit pain and thermal sensations in the absence of the normal anterolateral system
(ALS) pathway. Regeneration to a functional state probably does
not normally take place in the human nervous system; spinoreticular fibers are in the divided ALS; and corticospinal fibers function
in the motor sphere. (p. 182–183)
45. Answer C: The ataxia seen in patients with lesions of posterior
column fibers is due to the loss of proprioceptive input and the resultant inability of the patient to accurately judge the relative position of the extremity. Thus, the extremity is forcibly slapped to
the floor partially in an attempt to “create” the missing input. Anterolateral system fibers convey pain and thermal sensations, and
posterior root fibers convey these sensations plus those related to
the posterior columns. Corticospinal, vestibulospinal, and reticulospinal fibers function in the motor sphere. (p. 178–179)
46. Answer B: The axons of cell bodies located in the left trigeminal ganglion collect inside the brainstem to form the spinal trigeminal tract on the left (this tract is made up of the central processes
of primary sensory fibers on the trigeminal [V] nerve). A lesion of
these fibers on the left side of the medulla will result in a loss of
pain and thermal sensations on the left side of the face. Lesions of
the right trigeminal ganglion, trigeminothalamic fibers on the left,
and the right spinal trigeminal nucleus would all result in pain and
thermal losses on the right side of the face. The principal sensory
nucleus conveys touch information. (p. 184–185)
47. Answer A: Recognizing that this woman has a sensory loss on
the left side of her face, damage to fibers of the anterolateral system on the left correlates with the loss of pain and thermal sensations on the right side of her body. These anterolateral system
(ALS) fibers cross in the spinal cord within about two levels of
where they enter. Lesions of ALS on the right would result in a
left-sided deficit on the body. Damage to anterior trigeminothalamic fibers on the left would produce a corresponding right-sided
deficit on the face. The medial lemniscus conveys vibratory, discriminative touch, and proprioceptive sensations. (p. 180–181,
184–185)
48. Answer B: This patient has a lateral medullary syndrome (also
commonly called a posterior inferior cerebellar artery, or PICA
syndrome) on the left; this correlates with the left-sided sensory
loss on the face and right-sided sensory loss on the body. A lateral
medullary lesion on the right would result in the same deficits, but
on the opposite sides. The Parinaud, Weber, and Benedikt syndromes are all associated with lesions in the midbrain. (p.
180–181, 184–185, see also p. 136)
49. Answer B: Wilson disease (hepatolenticular degeneration) is an
inherited error of copper metabolism. Plasma levels of copper are
decreased; urinary levels are increased; and copper accumulates in
the liver, lenticular nuclei, and kidneys. Wilson disease can be
treated by reducing the level of dietary copper and administering
a copper-chelating agent. Maintenance can be achieved by taking
zinc, and treatment must be life-long. Ingestion of the other
choices can cause serious illness and death. However, none of
these is the causative agent in hepatolenticular degeneration. (p.
214–215)
50. Answer E: The dilator pupillae muscle of the iris is innervated
by postganglionic sympathetic fibers whose cell bodies of origin
are located in the ipsilateral superior cervical ganglion. Preganglionic sympathetic cell bodies are found in the intermediolateral
cell column. Preganglionic parasympathetic cell bodies are found
Q & A’s: A Sampling of Study and Review Questions with Explained Answers 291
in the Edinger-Westphal nucleus; axons of these cells terminate in
the ciliary ganglion, which, in turn, innervates the sphincter pupillae muscle of the iris. The hypothalamus is the origin of hypothalamospinal fibers that project to the intermediolateral cell column.
(p. 220–21)
51. Answer E: The deficits described for the man are consistent
with a tumor on the root of the vestibulocochlear (VIII) nerve;
these are correctly called a vestibular schwannoma because they
arise from the Schwann cells on the root of the vestibular portion
of the VIIIth nerve. Acoustic neuroma is an earlier, and now incorrect, designation for this lesion. Meningiomas arise primarily
from the arachnoid layer, ependymomas from the cells lining the
ventricular spaces, and a glioblastoma multiforme arises from astrocytes within the substance of the brain. (p. 228–229)
52. Answer A: This inherited disease is Friedreich ataxia; it initially
appears in children in the age range of 8–15 years and has the characteristic deficits described. Huntington disease is inherited, but
appears in adults; olivopontocerebellar atrophy is an autosomal
dominant disease and gives rise to a different set of deficits. The
cause of Parkinson disease is unclear, but it is probably not inherited; the Wallenberg syndrome is a brainstem lesion resulting
from a vascular occlusion. (p. 204–205)
53. Answer B: A tumor in the foramen would damage the motor
root of the trigeminal nerve and the mandibular root (sensory) of
the Vth nerve. In this patient, the jaw deviates to the left and the
sensory loss is on the left; this indicates that the tumor is on the
left. The deviation of the jaw to the left is due to the action of the
intact pterygoid muscles on the right (unlesioned side). Motor
fibers on the trigeminal (V) nerve travel in association with the
mandibular root and through the foramen ovale. Maxillary fibers
are sensory for the upper jaw and cheek area of the face. (p.
202–203)
54. Answer C: The loss of abduction and adduction in one eye and
of adduction in the opposite eye (the one-and-a-half syndrome) indicates a lesion in the area of the paramedian pontine reticular formation and abducens nucleus (in this case on the right side) and the
adjacent medial longitudinal fasciculus (MLF). The lesion damages
the ipsilateral abducens motor neurons, internuclear neurons
passing to the contralateral MLF, and internuclear axons in the ipsilateral MLF coming from the contralateral abducens nucleus.
Parinaud syndrome is a paralysis of upward gaze, and gaze palsies
tend to be toward one side and may result from cortical lesions.
Internuclear ophthalmoplegia is a deficit of medial gaze in one eye,
assuming a one-sided lesion. (p. 192–193)
55. Answer A: Anterior trigeminothalamic collaterals that project
into the dorsal motor nucleus of the vagus are an important link in
the reflex pathway for vomiting. The superior salivatory nucleus
is involved in the tearing or lacrimal reflex, the nucleus ambiguus
in the sneezing reflex, and the facial nucleus in the corneal reflex.
Collaterals of primary afferent fibers to the mesencephalic nucleus
that branch to enter the trigeminal motor nucleus mediate the jaw
reflex. (p. 184–184)
56. Answer E: The most anterior (ventral) portion of the medial
lemniscus at mid-olivary levels contains second order fibers conveying discriminative touch, vibratory sense, and proprioception from the contralateral lower extremity. These axons will
terminate in the lateral parts of the ventral posterolateral nucleus and, from there, be relayed to the posterior paracentral
gyrus (the lower extremity area of the primary somatomotor
cortex). The postcentral gyrus is the primary sensory cortex for
the face (approximately the lateral one-third), upper extremity
(middle one-third), and the trunk (medial). The anterior paracentral gyrus is the somatomotor cortex for the lower extremity. (p. 179–180)
57. Answer E: Syringomyelia is a cavitation in central areas of the
spinal cord that results in damage to fibers conveying pain and
thermal sensation as they cross the midline in the anterior white
commissure. The loss is bilateral since fibers from both sides are
damaged as they cross. Tabes dorsalis presents as posterior column
deficits and lancinating pain; syringobulbia (cavitation within the
brainstem) may have long tract signs and cranial nerve deficits; and
PICA syndrome characteristically has alternating sensory losses
(one side of face, opposite side of body). The Brown-Séquard syndrome has both sensory (anterolateral system and posterior column) and motor (corticospinal) deficits. (p. 180–181)
58. Answer A: There are basically only two areas where a relatively
restricted lesion would result in weakness of both lower extremities. One is in caudal parts of the pyramidal decussation (damage
to decussating corticospinal fibers traveling to the lumbosacral
cord levels), and the other would be a lesion in the falx cerebri
(such as a meningioma) damaging the lower extremity areas on the
somatomotor cortex bilaterally. Decussating fibers in the rostral
part of the pyramidal decussation terminate in cervical levels of
the spinal cord. Damage to either the pyramid or the lateral corticospinal tract would result in a hemiplegia (pyramid-contralateral,
lateral corticospinal tract-ipsilateral). Damage to the pyramids bilaterally would result in quadriplegia. (p. 190–191)
59. Answer C: The fatigability (progressive weakness), involvement of ocular muscles initially, followed by other muscle weakness, is characteristic of myasthenia gravis. Amyotrophic lateral
sclerosis is an inherited disease that affects spinal and/or brainstem
motor neurons and may result in upper or lower motor neuron
symptoms; this disease is usually fatal within a few years. Multiple
sclerosis is a demyelinating disease; Parkinson and Huntington diseases are neurodegenerative conditions that eventually have a dementia component. (p.190, 202)
60. Answer C: The history and the combination of signs and symptoms seen in this woman indicate a probable diagnosis of myasthenia gravis and, consequently, a neurotransmitter disease at the
neuromuscular junction. Damage to corticospinal and corticonuclear terminals and to synaptic contacts within the basal nuclei and
the cerebellum would result in motor deficits but not in the pattern seen in this woman. (p. 190, 202)
61. Answer A: The neurotransmitter at the neuromuscular junction
is acetylcholine; a blockage of postsynaptic nicotinic acetylcholine
receptors is the cause of the motor deficits characteristically seen
in patients with myasthenia gravis. A loss of dopamine results in
Parkinson disease, motor deficits that are not seen in this woman.
Glutamate and GABA are found in many pathways involved in motor function but are not located at the neuromuscular junction.
Serotonin is found in pathways related to the basal nuclei, raphe
nuclei, and the hypothalamus. (p. 190, 202)
292 Q & A’s: A Sampling of Study and Review Questions with Explained Answers
62. Answer D: A lesion in the medial longitudinal fasciculus (MLF)
on the right interrupts axons of the interneurons that arise from
the left abducens nucleus and pass to oculomotor motor neurons
on the right innervating the medial rectus muscle (internuclear
ophthalmoplegia). Damage to the abducens nucleus will indeed
destroy these interneurons, but will also result in an inability to
abduct the eye on the ipsilateral side. Injury to the MLF on the left
would result in an inability to adduct the left eye, and a lesion in
the PPRF would most likely produce a bilateral horizontal gaze
palsy. (p. 192–193, 200–201)
63. Answer C: A fracture through the jugular foramen would potentially damage the glossopharyngeal (IX), vagus (X), and spinal
accessory (XI) nerves. The major observable deficit would be a
loss of the efferent limb of the gag reflex and a paralysis of the ipsilateral trapezius and sternocleidomastoid muscles (drooping of
the shoulder, difficulty elevating the shoulder especially against resistance, difficulty turning the head to the contralateral side). Involvement of facial muscles would suggest damage to the internal
acoustic or stylomastoid foramina; this would also be the case for
the efferent limb of the corneal reflex. Diplopia and ptosis would
suggest injury to the superior orbital fissure, as all three nerves
controlling ocular movement traverse this space. The hypoglossal
nerve (which supplies muscles of the tongue) passes through the
hypoglossal canal. (p. 200–201)
64. Answer E: The constellation of signs and symptoms experienced by this boy are characteristic of Wilson disease, also called
hepatolenticular degeneration. These may include movement disorders, tremor, the Kayser-Fleischer ring at the corneoscleral
margin, and eventual cirrhosis of the liver. Huntington and
Parkinson diseases are predominately motor problems in the early
stages and Pick disease is a degenerative disease of the cerebral cortex affecting mainly the frontal and temporal lobes; dementia is
the primary deficit. Sydenham chorea is seen in children following an infection with hemolytic streptococcus; after treatment
for the infection, the choreiform movements usually resolve. (p.
214–215)
65. Answer A: A tumor impinging on the midline of the optic chiasm would damage crossing fibers from both eyes that are coming
from the nasal retinae and would reflect a loss of all, or part of both
temporal retinal fields. Between 60% and 70% of pituitary adenomas are prolactin-secreting tumors. Right or left homonymous
hemianopsia (the nasal visual field of one eye and the temporal visual field of the other eye) are seen following lesions of, respectively, the left and right optic tracts. Quadrantanopsias result from
lesions in the optic radiations. (p. 220–221)
66. Answer C: The flocculonodular lobe and the fastigial nucleus receive input from the vestibular apparatus (primary vestibulocerebellar fibers) and from the vestibular nuclei (secondary vestibulocerebellar fibers). In turn, the Purkinje cells of the flocculonodular
cortex and cells of the fastigial nucleus project to the vestibular nuclei as cerebellar corticovestibular and cerebellar efferent fibers,
respectively. While other areas of the cerebellar cortex may have
a small projection to the vestibular nuclei, this is not significant
compared to that of the flocculonodular lobe. (p. 228–229)
67. Answer B: Quadrantanopia, a loss of approximately one quarter (a quadrant) of the visual field, is seen in lesions in the optic ra-
diations (geniculocalcarine radiations). The visual loss is in the visual field contralateral to the side of the lesion. Lesions in the
lower portions of the radiations result in deficits in the contralateral superior quadrants, while lesions in the upper portions of the
radiations result in deficits in the contralateral lower quadrants.
Consequently, in this boy (with a superior right quadrantanopia),
the lesion is in the lower portions of the optic radiations in the left
temporal lobe (Meyer-Archambault loop). The lesion in the chiasm would result in a bitemporal hemianopsia. (p. 220–223)
68. Answer E: Hemiballismus is the result of a lesion largely confined to the subthalamic nucleus on the side contralateral to the
deficit. These movements are violent, flinging, unpredictable, and
uncontrollable. The abnormal movements are contralateral to the
lesion because the expression of the lesion is through the corticospinal tract. Lesions in the left subthalamic nucleus would result
in a right-sided problem. Damage in the motor cortex would be
seen as a contralateral weakness, and cell loss in the substantia nigra would result in motor deficits characteristic of Parkinson disease (resting tremor, bradykinesia, stooped posture, festinating
gait). (p. 216–217)
69. Answer B: The inability to perform a rapid alternating movement, such as pronating and supinating the hand on the knee, is
dysdiadochokinesia. This is one of several cardinal signs of cerebellar disease or stroke. Dysmetria is an inability to judge power,
distance, and accuracy during a movement, and dysarthria is difficulty speaking. A resting tremor is seen in diseases of the basal nuclei, and an intention tremor is seen in cerebellar lesions. (p.
208–211)
70. Answer B: The tremor that worsens as this man attempts to
bring his index finger to his nose is called an intention tremor,
sometimes referred to as a kinetic tremor. This type of tremor is
one cardinal sign of cerebellar lesions. A resting tremor is seen in
diseases of the basal nuclei and a static tremor (postural tremor) is
seen in the trunk and extremities in a static position. Dysmetria is
an inability to judge distance, power, or accuracy during a movement. The rebound phenomenon is an inability of agonist and antagonist muscles to rapidly adapt to changes in load. (p. 208–211)
71. Answer C: The signs and symptoms in this man clearly indicate
a lesion in the cerebellum on the left side. The cerebellar nuclei
on the left (lesion side) project to the contralateral thalamus
(right) and from here to motor cortical areas (also right). The motor cortex projects, via the corticospinal tract and its decussation,
back to the side of the body, excluding the head, on which the lesion is located (left cerebellum). The motor expression of the
cerebellar deficit is through the corticospinal tract. The man’s leftsided deficits are not consistent with a right cerebellar lesion, and
the deficits are not consistent with a midbrain lesion. Lesions of
the basal nuclei would result in a different set of motor disorders.
(p. 208–211)
72. Answer B: The superior cerebellar artery serves the cortex on
the superior surface of the cerebellum and most of the cerebellar
nuclei on the same side; in this case, it is the left artery. The anterior inferior cerebellar artery serves the cortex on the lateral inferior surface of the cerebellum and a small caudal tip of the dentate
nucleus. A lesion that involves primarily the cerebellar cortex will
not result in long-term deficits. A lesion that involves cortex plus
nuclei or primarily nuclei, especially in an older patient (as in this
Q & A’s: A Sampling of Study and Review Questions with Explained Answers 293
man), is likely to result in long-term deficits. The lenticulostriate
arteries serve the basal nuclei. (p. 208–211)
its head) is seen in Huntington disease. Lacunae are usually seen in
patients who have had small strokes. (p. 214–215)
73. Answer A: One cause of hemifacial spasm (intermittent and abnormal contractions of the facial muscles) is compression of the facial root by a loop of the anterior inferior cerebellar artery, or perhaps one of its larger branches. Aberrant loops of the superior
cerebellar artery may compress the trigeminal root (trigeminal neuralgia), and the posterior inferior cerebellar artery serves the lateral
medulla and medial regions of the cerebellum. The anterior and posterior spinal arteries serve areas of the medulla. (p. 202–203)
79. Answer D: The inability of this man to control the distance,
power, and accuracy of a movement is dysmetria; this is characteristically seen in cerebellar lesions. Dysphagia is difficulty swallowing, and dysarthria is difficulty speaking. The inability to perform rapid alternating movements is dysdiadochokinesia, and
bradykinesia is a slowness to initiate movement. The latter is characteristic of individuals with disease of the basal nuclei. (p.
208–211)
74. Answer B: The anterolateral system and the medial lemniscus
are adjacent to each other in lateral portions of the midbrain and
are served largely by the same vessel(s), these being penetrating
branches of the quadrigeminal artery. This area may also receive
some blood supply from the posterior choroidal arteries.
Throughout the spinal cord, medulla, and into about the mid- to
more rostral pons, these fiber bundles are spatially separated from
each other and have separate blood supplies. (p. 137, 178–181)
80. Answer A: The territory served by the anterior choroidal artery
includes the optic tract, inferior portions of the posterior limb of
the internal capsule, thalamocortical radiations within the posterior limb, and structures in the temporal lobe. The left-sided
deficits indicate a lesion on the right side. A lesion of the right optic tract results in a loss of vision in the opposite (left) visual fields;
this being the temporal visual field of the left eye and the nasal visual field of the right eye (left homonymous hemianopsia). This
constellation of deficits is known as the anterior choroidal artery
syndrome. Quadrantanopia specifies a lesion in a portion of the
optic radiations, and a nasal hemianopsia indicates a small lesion in
the lateral aspect of the optic chiasm on one side. (p. 158–159,
220–223)
75. Answer B: Fibers in the left medial lemniscus conveying position sense from the right upper extremity originate from cell bodies located in the right cuneate nucleus. These cuneate neurons
give rise to axons that form the internal arcuate fibers that arch
towards the midline, cross, and collect to form the contralateral
medial lemniscus. The left cuneate nucleus sends axons to the
right medial lemniscus, and the gracile nucleus (right or left) conveys information from the lower extremity. Posterior root ganglia
neurons project to the gracile or cuneate nuclei. (p. 178–179)
76. Answer E: Dystonia is a movement disorder characterized by
abnormal, sometimes intermittent, but frequently sustained, contractions of the muscles of the trunk and extremities that force the
body into a twisted posture. Dystonia may be seen in patients with
diseases of the basal nuclei. Dysmetria is the inability to judge the
distance and trajectory of a movement. Dyspnea is difficulty
breathing; this may result from heart and/or lung disorders as well
as from neurologic disorders. Dysphagia is difficulty swallowing,
and dysarthria is difficulty speaking. (p. 124–215)
77. Answer E: Transection of the optic nerve (on the left in this
man) eliminates the afferent limb of the pupillary light reflex, but
the efferent limb, via the oculomotor nerve, is intact. Consequently, there is a loss of both the direct response (in the blind eye)
and the consensual response (in the good eye) when light is shined
in the blind eye, because the afferent limb is eliminated and no input is getting to the center from which the efferent limb originates. On the other hand, light shined into the good eye (right in
this man) results in a direct pupillary response (in the good eye)
and a consensual pupillary response in the blind eye because the
efferent limb of this reflex is not damaged for the blind eye. Other
combinations of responses may occur as a result of lesions in other
portions of the nervous system. (p. 220–221)
78. Answer B: In addition to the motor deficits characteristic of this
disease, MRI would reveal a spongy degeneration (with cavitations) of the lenticular nucleus most noticeable in the putamen.
There may also be a spongy degeneration in areas of the cerebral
cortex. Atrophy of frontal and temporal lobe gyri is seen in Pick
disease; loss of nigral cells is characteristic of Parkinson disease;
and loss of the caudate nucleus (especially noticeable as absence of
81. Answer B: This woman has sensory losses on the left side of her
body and face that include pain/thermal sensations and the general category of proprioception (discriminative touch, vibratory
and position sense); this is a hemianesthesia, a loss of sensation on
one side of the body. This is a result of damage to thalamocortical
fibers projecting from the ventral posteromedial and ventral posterolateral thalamic nuclei to the somatosensory cortex. Alternating hemianesthesia refers to a sensory loss on one side of the face
and on the contralateral side of the body. A sensory level is a characteristic of lesions in the spinal cord, and paresthesia refers to an
abnormal spontaneous sensation not a loss. A superior alternating
hemiplegia is a motor deficit. (p. 158–159, 178–181, 220–223)
82. Answer B: The corticospinal fibers traversing the inferior portions of the posterior limb of the internal capsule are damaged by
an occlusion of the anterior choroidal artery; a left-sided deficit
correlates with a lesion on the right side, especially when taking
into consideration the concurrent visual loss. Damage to corticospinal fibers on the left would result in a right-sided deficit. The
somatomotor cortex is not involved in the lesion. While thalamocortical fibers are certainly damaged in this lesion, the deficits
related to corticospinal fiber involvement predominate. (p.
190–191)
83. Answer B: Sensorineural hearing loss, also called nerve deafness, results from lesions or diseases that involve the cochlea or
the cochlear portion of the vestibulocochlear nerve. Obstructions
of the external ear or diseases of the middle ear result in conductive deafness (conductive hearing loss). Lesions in the inferior colliculus, auditory cortex, or other areas within the brain may result
in difficulty localizing, interpreting, or understanding sound but
do not result in total deafness in one ear. (p. 226–227)
84. Answer C: Laminae 1, 4, and 6 receive input from the ganglion
cells in the contralateral retina. Laminae 2, 3, and 5 receive an ipsilateral input; laminae 1 and 2 are the magnocellular layers of the
294 Q & A’s: A Sampling of Study and Review Questions with Explained Answers
lateral geniculate nucleus; and laminae 3, 4, 5, and 6 are its parvocellular layers. (p. 222)
85. Answer D: Sydenham chorea is a disease of childhood thought
to be an autoimmune disorder seen in children as a sequel to a hemolytic streptococcus infection. In most children the disease is
self-limiting and the patient recovers with no permanent deficits.
Huntington disease, Parkinson disease, and senile chorea present
with motor deficits that partially resemble those seen in this girl
but these are diseases of adults or the elderly. Weber syndrome (a
superior alternating hemiplegia) is a motor deficit involving the
oculomotor nerve on one side and the corticospinal tract on the
opposite side. (p. 214–215)
Review and Study Questions for
Chapter 8
1. Which of the following arteries is generally found in the area of the
cingulate sulcus and has branches that serve the lower extremity
areas of the somatomotor and somatosensory cortex?
䊊 (A) Callosomarginal
䊊 (B) Frontopolar
䊊 (C) Internal parietal
䊊 (D) Parietooccipital
䊊 (E) Pericallosal
2. A 44-year-old woman presents to her family physician with intermittent headache and the complaint that she can’t see in her left
eye. The examination reveals that the woman is blind in her left
eye. When a light is shined into her left eye there is no direct or
consensual pupillary light reflex. Magnetic resonance angiography
(MRA) shows a large aneurysm at the origin of the ophthalmic
artery. Which of the following represents the usual point of origin
of this vessel?
䊊 (A) Cavernous part of the internal carotid artery
䊊 (B) Cerebral part of the internal carotid artery
䊊 (C) First segment (A1) of the anterior cerebral artery
䊊 (D) First segment (M1) of the middle cerebral artery
䊊 (E) Petrous part of the internal carotid artery
3. The venous phase of an angiogram of a 52-year-old man suggests
a small tumor at what the neuroradiologist refers to as the venous
angle. Which of the following points most specifically describes
the position of the venous angle?
䊊 (A) Where the internal cerebral vein meets the great cerebral vein
䊊 (B) Where the superficial middle cerebral vein meets the
cavernous sinus
䊊 (C) Where the thalamostriate vein turns to form the internal cerebral vein
䊊 (D) Where the transverse sinus turns to form the sigmoid
sinus
䊊 (E) Where the vein of Labbé meets the vein of Trolard
4. The superficial middle cerebral vein forms a direct anastomotic
junction with which of the following venous structures on the lateral aspect of the cerebral hemisphere?
䊊 (A) Cavernous sinus
䊊 (B) Confluence of sinuses
䊊 (C) Superior sagittal sinus
䊊 (D) Transverse sinus
䊊 (E) Veins of Labbé and Trolard
5. The coronal MRI of a 69-year-old man reveals an infarcted area in
the region of the cerebral hemisphere lateral to the internal capsule but internal to the insular cortex. A comparison of coronal
and sagittal MRI suggests that the vessels involved are branches of
the middle cerebral artery. Which of the following branches or
segments of the middle cerebral artery are most likely involved in
this man?
䊊 (A) Anterior and polar temporal branches
䊊 (B) Insular branches
䊊 (C) Lenticulostriate branches
䊊 (D) Opercular segment
䊊 (E) Uncal artery
6. The anterior and middle cerebral arteries are the terminal
branches of which of the following vascular trunks?
䊊 (A) Basilar artery
䊊 (B) Cavernous part of the internal carotid
䊊 (C) Cerebral part of the internal carotid
䊊 (D) External carotid artery
䊊 (E) Petrous part of the internal carotid
7. The superior sagittal sinus, straight sinus, and transverse sinuses
converge at which of the following landmarks?
䊊 (A) Clivus
䊊 (B) Confluens sinuum
䊊 (C) Great cerebral vein
䊊 (D) Jugular foramen
䊊 (E) Venous angle
8. A 47-year-old woman is brought to the emergency department by
her husband. She has a severe headache, nausea, and is somnolent.
The examination reveals that the woman is hypertensive and has
papilledema. MRI shows evidence of cerebral edema, bilateral infarcted areas in the thalamus, and a large sinus thrombosis that is
blocking the egress of blood through the vascular system. This
thrombus is most likely located in which of the following venous
structures?
䊊 (A) Inferior sagittal sinus
䊊 (B) Left sigmoid sinus
䊊 (C) Right transverse sinus
䊊 (D) Straight sinus
䊊 (E) Superior sagittal sinus
Q & A’s: A Sampling of Study and Review Questions with Explained Answers 295
9. A 39-year-old man presents to his family physician with a complaint of difficulty swallowing. The history reveals that the man has
had severe recurring headaches over the last 5 days and suffered
several bouts of vomiting. The examination confirms the difficulty
swallowing, and reveals that the man’s voice is hoarse and gravely,
and that he is unable to elevate his left shoulder against resistance.
MRI shows a dural sinus thrombosis. Based on this man’s deficits,
which of the following represents the most likely location of this
thrombus?
䊊 (A) Left cavernous sinus
䊊 (B) Left jugular bulb
䊊 (C) Left transverse sinus
䊊 (D) Right jugular bulb
䊊 (E) Straight sinus
10. Which of the following vessels forms a characteristic loop in the
cisterna magna that is prominent on lateral angiograms and, in the
process, supplies blood to the choroid plexus of the fourth ventricle?
䊊 (A) Anterior inferior cerebellar artery
䊊 (B) Posterior inferior cerebellar artery
䊊 (C) Posterior spinal artery
䊊 (D) Superior cerebellar artery
䊊 (E) Vertebral artery
11. The MRI of a 42-year-old man shows a small tumor in the choroid
plexus of the third ventricle. Angiogram and MRA suggest that
this tumor contains numerous vascular loops. Which of the following represents the blood supply to this portion of the choroid
plexus?
䊊 (A) Anterior choroidal artery
䊊 (B) Choroidal branches of AICA
䊊 (C) Choroidal branches of PICA
䊊 (D) Lateral posterior choroidal artery
䊊 (E) Medial posterior choroidal artery
12. The angiogram of a 56-year-old woman shows an aneurysm originating from the lateral aspect of the basilar bifurcation and extending into the space between the posterior cerebral and superior
cerebellar arteries. Based on the structure(s) located at this point,
which of the following deficits would most likely be seen in this
woman?
䊊 (A) Constriction of the ipsilateral pupil
䊊 (B) Inability to look down and out with the ipsilateral eye
䊊 (C) Inability to look laterally with the ipsilateral eye
䊊 (D) Inability to look up, down, or medially with the ipsilateral eye
䊊 (E) Loss of pain and thermal sensation from the ipsilateral
side of the face
13. The position of the posterior communicating artery, as frequently
seen in MRA, is an important landmark that specifies the intersection of which of the following?
䊊 (A) A1 and A2 segments
䊊 (B) M1and M2 segments
䊊 (C) M2 and M3 segments
䊊 (D) P1 and P2 segments
䊊 (E) P2 and P3 segments
14. A 16-year-old boy with developmental delay has been followed
since birth by a pediatric neurologist. A recent MRA is done in
which major arteries and venous sinuses are visualized. It is concluded that the pattern of the boy’s venous sinuses is essentially
normal. Which of the following describes the usual pattern of the
superior sagittal sinus at the confluence of sinuses?
䊊 (A) Always drains equally into the right and left transverse
sinuses
䊊 (B) Always drains into the left transverse sinus
䊊 (C) Always drains into the right transverse sinus
䊊 (D) Usually drains into the left transverse sinus
䊊 (E) Usually drains into the right transverse sinus
Answers for Chapter 8
1. Answer A: The callosomarginal artery lies generally in the region of the cingulate sulcus and gives rise to branches (paracentral
branches) that distribute to the anterior and posterior paracentral
gyri. The pericallosal artery is located immediately superior to the
corpus callosum and the frontopolar artery serves the medial aspect of the frontal lobe. The internal parietal arteries are the terminal branches of the pericallosal artery; these vessels distribute
to the medial portion of the parietal lobe, the precuneus. The parietooccipital artery is one of the terminal branches (part of P4) of
the posterior cerebral artery. (p. 29, 240)
2. Answer B: In most instances (approximately 80–85%), the
ophthalmic artery originates from the cerebral portion of the internal carotid artery just as this parent vessel leaves the cavernous
sinus and passes through the dura. In a small percentage of cases
the ophthalmic artery may originate from other locations on the
internal carotid artery, including its cavernous portion. This vessel does not originate from the petrous portion of the internal
carotid or from anterior or middle cerebral arteries. (p. 25, 240)
3. Answer C: The point at which the thalamostriate vein (also
called the superior thalamostriate vein at this position) abruptly
turns 180⬚ to form the internal cerebral vein is called the venous
angle. This angle is located immediately caudal to the position of
the interventricular foramen and is, therefore, an important landmark. The thalamostriate vein is located in the groove between the
thalamus and the caudate nucleus. At the superior aspect of the
thalamus, this vein is the superior thalamostriate vein, and, on the
inferior surface, it is called the inferior thalamostriate vein. None
of the other choices is involved in the formation of the venous angle. (p. 241)
4. Answer E: The superficial middle cerebral vein is a comparatively obvious venous structure on the lateral surface of the hemisphere that communicates directly with the veins of Trolard (to
the superior sagittal sinus) and Labbé (to the transverse sinus). The
superficial middle cerebral vein also communicates with the cavernous sinus, but this sinus in not on the lateral aspect of the hemisphere as specified in the question. The other choices do not receive venous blood directly from the superficial middle cerebral
vein. (p. 19, 241)
5. Answer C: The position of this lesion is in that portion of the
hemisphere occupied by the lenticular nucleus; the lenticulostri-
296 Q & A’s: A Sampling of Study and Review Questions with Explained Answers
ate branches of the M1 segment of the middle cerebral artery serve
this structure. The uncal, anterior, and polar temporal branches
originate from the M1 segment but do not serve structures in the
area of the hemisphere described. Insular branches (M2) and opercular branches (M3) serve cortical structures. (p. 25, 242)
6. Answer C: As the internal carotid artery exits the cavernous sinus, it becomes the cerebral part of the internal carotid and, after
giving rise to three important small branches (ophthalmic, anterior choroidal, posterior communicating), bifurcates into the anterior and middle cerebral arteries. These two cerebral vessels are
the terminal branches of the cerebral part of the internal carotid
artery. In approximately 70–75% of specimens, the anterior cerebral artery is the smaller of these two terminal branches. None of
the other choices gives rise to the anterior and middle cerebral arteries. (p. 242)
7. Answer B: The superior sagittal sinus, straight sinus, the two
transverse, and the occipital sinus (when present) converge at the
confluence of sinuses (confluens sinuum), which is located internal to the external occipital protuberance. The venous angle is the
junction of the thalamostriate and the internal cerebral veins, and
the great cerebral vein (of Galen) receives the internal cerebral
veins and several smaller veins including the basal vein (of Rosenthal) and empties into the straight sinus. The jugular foramen contains the transition from the sigmoid sinus to the internal jugular
vein and the terminus of the inferior petrosal sinus. The clivus is
composed mainly of the basal part of the occipital bone; this is the
location of the basilar plexus. (p. 19. 23, 243–245)
8. Answer D: A key observation in this woman is the bilateral infarcted areas in the thalamus. The straight sinus receives venous
flow from both internal cerebral veins; a blockage of flow through
the straight sinus would adversely affect both thalami. Such a lesion would also cause potential damage to the medial temporal
lobe due to the disruption of flow through the basal vein (of Rosenthal). None of the other choices receives venous drainage directly
from the thalamus. (p. 29, 248, 250)
9. Answer B: The deficits experienced by this man (difficulty swallowing, hoarseness, inability to elevate the left shoulder against resistance) point to damage to the glossopharyngeal (IXth), vagus
(Xth), and spinal accessory (XIth) nerves or to their roots. All
three of these cranial nerves exit the jugular foramen along with
the continuity of the sigmoid sinus with the internal jugular vein
( jugular bulb or bulb of the jugular vein). In this case, the venous
thrombosis is at the left jugular bulb and impinging on these three
cranial nerve roots. Dural sinus thrombosis of the other choices
may cause certain deficits, but not those experienced by this man.
(p. 244, 250)
10. Answer B: The posterior inferior cerebellar artery (commonly
called PICA) originates from the vertebral artery, courses around
the lateral aspect of the medulla, loops sharply into the space of
the cisterna magna (giving off small branches to the choroid plexus
in the fourth ventricle), then joins the inferior and medial surface
of the cerebellum. None of the other choices forms prominent
vascular structures in the cisterna magna or serves the choroid
plexus of the fourth ventricle. (p. 246)
11. Answer E: The medial posterior choroidal artery originates from
the P2 segment of the posterior cerebral artery, arches around the
midbrain, and enters the caudal end of the third ventricle. The anterior choroidal artery serves the choroid plexus in the temporal
horn, and the lateral posterior choroidal artery serves the glomus
choroideum and extends into the plexi of the temporal horn and
the body of the ventricle. These patterns may be somewhat variable. Choroidal branches of anterior inferior cerebellar artery
(AICA) serve the choroid plexus extending through the foramen of
Luschka, and these branches from the posterior inferior cerebellar
artery (PICA) serve the plexus within the fourth ventricle. (p. 251)
12. Answer D: The oculomotor nerve (III) is located between the
posterior cerebral and superior cerebellar arteries and may be
damaged by aneurysms at this location. Most eye movement
would be lost (the trochlear (IV) and abducens (VI) nerves are intact) and the ipsilateral pupil would be dilated, not constricted.
Sensation from the face is carried on the trigeminal nerve. Movement deficits related to injury to the IVth nerve (looking down and
out) or the VIth nerve (looking laterally) are not affected. (p. 39,
40, 247, 252)
13. Answer D: The posterior communicating artery originates
from the cerebral part of the internal carotid artery and courses
caudally to join the posterior cerebral artery (PCA). The part of
the PCA medial to this intersection is the P1 segment and the part
of the PCA immediately lateral to this junction is the P2 segment.
Important branches arise from both of these parts of the PCA.
None of the other choices have any direct relationship to the points
of origin of the posterior communicating artery. (p. 25, 247, 249)
14. Answer E: The drainage pattern of the superior sagittal sinus at
the confluence of sinuses is variable, including about equal to both
transverse sinuses or mainly to the right or to the left. However,
the usual pattern is for the superior sagittal sinus to drain predominately into the right transverse sinus. (p. 245)
Sources and Suggested Readings
AbuRahma A, Bergan JJ. Noninvasive Vascular Diagnosis. London:
Springer-Verlag, 2000.
Afifi AK, Bergman RA. Functional Neuroanatomy, Text and Atlas.
New York: McGraw-Hill, 1998.
Airaksinen MS, Panula P. The histaminergic system in the guinea pig central nervous system: An immunocytochemical mapping study using
an antiserum against histamine. J Comp Neurol 1988;273:163–186.
Airaksinen MS, Flugge G, Fuchs E, Panula P. Histaminergic system in
the tree shrew brain. J Comp Neurol 1989;286:289–310.
Aminoff MJ, Greenberg DA, Simon RP. Clincial Neurology, 3rd ed.
Stanford, CT: Appleton & Lange, 1996.
Anderson KD, Reiner A. Extensive co-occurrence of substance P and
dynorphin in striatal projection neurons: An evolutionarily conserved feature of basal ganglia organization. J Comp Neurol 1990;
295:339–369.
Angevine JB, Mancall EL, Yakovlev PI. The Human Cerebellum: An
Atlas of Gross Topography in Serial Sections. Boston: Little,
Brown, 1961.
Beckstead RM, Morse JR, Norgren R. The nucleus of the solitary tract
in the monkey: Projections to the thalamus and brain stem nuclei. J
Comp Neurol 1980;190:259–282.
Benarroch EE, Westmoreland BF, Daube JR, Reagan TJ, Sandok BA.
Medical Neuroscience, An Approach to Anatomy, Pathology, and
Physiology by Systems and Levels, 4th Ed. Baltimore: Lippincott
Williams & Wilkins, 1999.
Bishop GA, Ho RH, King JS. Localization of immunoreactivity in the
opossum cerebellum. J Comp Neurol 1985;235:301–321.
Bobillier P, Seguin S, Petitjean F, Salvert D, Touret M, Jouvert M. The
raphe nuclei of the cat brainstem: A topographical atlas of their efferent projections as revealed by autoradiography. Brain Res 1976;
113:449–486.
Brodal A. Neurological Anatomy in Relation to Clinical Medicine, 3rd
ed. New York: Oxford University Press, 1981.
Brodal P. The Central Nervous System: Structure and Function. 2nd
ed. New York: Oxford University Press, 1998.
Broman J, Blomqvist A. Substance P-like immunoreactivity in the lateral cervical nucleus of the owl monkey (Aotus trivirgatus): A comparison with the cat and rat. J Comp Neurol 1989;289:111–117.
Broman J, Westman J, Ottersen OP. Spinocervical tract terminals are
enriched in glutamate-like immunoreactivity: An anterograde
transport-quantitative immunogold study in the cat. Neurosci Abstr
1989;15:941.
Brown AG. Organization in the Spinal Cord. Berlin: Springer-Verlag,
1981.
Buisseret-Delmas C, Batini C, Compoint C, Daniel H, Menetrey D.
The GABAergic neurones of the cerebellar nuclei: Projection to the
caudal inferior olive and to the bulbar reticular formation. In: P
Strata (ed). The Olivocerebellar System in Motor Control. (also Exp
Brain Res Ser 17:108–110). New York: Springer-Verlag, 1989.
Burt AM. Textbook of Neuroanatomy. Philadelphia: WB Saunders
Co, 1993.
Cassini P, Ho RH, Martin GF. The brainstem origin of enkephalin- and
substance-P-like immunoreactive axons in the spinal cord of the
North American opossum. Brain Behav Evol 1989;34:212–222.
Clemente CD. Anatomy: A Regional Atlas of the Human Body, 3rd
ed. Baltimore: Urban & Schwarzenberg, 1987.
Council of Biology Editions Style Manual Committee. Scientific Style
and Format—The CBE Manual for Authors, Editors, and Publishers, 6th Ed. Cambridge: Cambridge University Press, 1994.
Craig AD, Jr, Sailer S, Kniffki K-D. Organization of anterogradely labeled spinocervical tract terminations in the lateral cervical nucleus
of the cat. J Comp Neurol 1987;263:214–222.
Craig AD, Jr, Tapper DN. Lateral cervical nucleus in the cat: Functional
organization and characteristics. J Neurophysiol 1978;41:1511–1534.
Crosby EC, Humphrey T, Lauer EW. Correlative Anatomy of the
Nervous System. New York: Macmillan, 1962.
DeArmond SJ, Fusco MM, Dewev MM. Structure of the Human
Brain: A Photographic Atlas, 3rd ed. New York: Oxford University
Press, 1989.
deGroot J. Correlative Neuroanatomy of Computed Tomography
and Magnetic Resonance Imaging. Philadelphia: Lea & Febiger,
1984.
DeLacalle S, Hersh LB, Saper CB. Cholinergic innervation of the human cerebellum. J Comp Neurol 1993;328:364–376.
Dietrichs E, Haines DE. Interconnections between hypothalamus and
cerebellum. Anat Embryol 1989;179:207–220.
Donaghy M. Neurology. New York: Oxford, 1997.
Dublin AB, Dublin WB. Atlas of Neuroanatomy with Radiologic Correlation and Pathologic Illustration. St. Louis: Warren H. Green
Inc., 1982.
Duus P. Topical Diagnosis in Neurology: Anatomy, Physiology, Signs,
Symptons, 2nd ed. Stuttgart: Georg Thieme Verlag, 1989.
Duvernoy HM. The Human Brain Stem and Cerebellum, Surface,
Structure, Vascularization, and Three-Dimensional Sectional
Anatomy with MRI. Wein: Springer-Verlag, 1995.
England MA, Wakely J. Color Atlas of the Brain and Spinal Cord. St.
Louis: Mosby Year Book, 1991.
Federative Committee on Anatomical Terminology. Terminologia
Anatomica. Stuttgart and New York: Thieme, 1998.
Fischer HW, Ketonen L. Radiographic Neuroanatomy: A Working Atlas. New York: McGraw-Hill, 1991.
Fix JD. Atlas of the Human Brain and Spinal Cord. Rockville, MD: Aspen Publishers, 1987.
Gasser RF. Personal communication, 1989.
Gasser RF. Atlas of Human Embryos. New York: Harper & Row, 1975.
Giesler GJ, Jr, Bjorkeland M, Xu Q, Grant G. Organization of the
spinocervicothalamic pathway in the rat. J Comp Neurol 1989;268:
223–233.
297
298 Sources and Suggested Readings
Gilman S, Winans SS. Manter and Gatz’s Essentials of Clinical Neuroanatomy and Neurophysiology, 10th ed. Philadelphia: FA Davis
Company, 2003.
Giuffrida R, Rustioni A. Glutamate and aspartate immunoreactivity in
corticospinal neurons of rats. J Comp Neurol 1989;288:154–164.
Gluhbegovic N, Williams TH. The Human Brain: A Photographic
Guide. Hagerstown, MD: Harper & Row, 1980.
Greenberg MS. Handbook of Neurosurgery, 5th Ed. New York:
Thieme, 2001.
Groenewegen HJ, Berendse HW. Connections of the subthalamic nucleus with ventral striatopallidal parts of the basal ganglia in the rat.
J Comp Neurol 1990;294:607–622.
Haines DE (ed). Fundamental Neuroscience, 2nd Edition. Philadelphia: Churchill Livingstone/Elsevier Science, 2002.
Haines DE, Dietrichs E. On the organization of interconnections between the cerebellum and hypothalamus. In: JS King (ed), New
Concepts in Cerebellar Neurobiology, pp. 113–149. New York:
Alan R. Liss, 1987.
Haines DE, May PJ, Dietrichs E. Neuronal connections between the
cerebellar nuclei and hypothalamus in Macaca fascicularis: Cerebellovisceral circuits. J Comp Neurol 1990;299:106–122.
Hamilton WJ, Mossman HW. Human Embryology, 4th ed. Baltimore:
Williams & Wilkins, 1972.
Heimer L. The Human Brain and Spinal Cord: Functional Neuroanatomy
and Dissection Guide. New York: Springer-Verlag, 1995.
Herbert H, Saper CB. Cholecystokinin-, galamin-, and corticotropinreleasing factor-like immunoreactive projections from the nucleus
of the solitary tract to the parabrachial nucleus in the rat. J Comp
Neurol 1990;293:581–598.
Huang X-F, Törk I, Paxinos G. Dorsal motor nucleus of the vagus
nerve: A cyto-and chemoarchitectonic study in the human. J Comp
Neurol 1993;330:158–182.
Iverson MA et al. American Medical Association Manual of Style—A
Guide for Authors and Editors, 9th Ed. Baltimore: Williams &
Wilkins, 1998.
Jackson GD, Duncan JS. MRI Neuroanatomy: A New Angle on the
Brain. New York: Churchill Livingstone, 1996.
Jennes L, Traurig HH, Conn PM. Atlas of the Human Brain. Philadelphia: JB Lippincott Co, 1995.
Jones SL, Light AR. Serotoninergic medullary raphespinal projection
to the lumbar spinal cord in the rat: A retrograde immunohistochemical study. J Comp Neurol 1992;322:599–610.
Kandel ER, Schwartz JH, Jessell TM. Principles of Neural Sciences,
4th ed. McGraw-Hill: New York, 2000.
Keirman JA. Barr’s The Human Nervous System: An Anatomical
Viewpoint, 7th Ed. Philadelphia: Lippincott-Raven, 1998.
Kirkwood JR. Essentials of Neuroimaging. New York: Churchill Livingstone, 1990.
Krammer EB, Lischka MF, Egger TP, Reidl M, Gruber H. The motoneuronal organization of the spinal accessory nuclear complex.
Adv Anat Embryol Cell Biol 1987;103:1–62.
Kretschmann H-J, Weinrich W. Cranial Neuroimaging and Clinical
Neuroanatomy: Magnetic Resonance Imaging and Computed Tomography, 2nd ed. New York: Georg Thieme Verlag, 1993.
Larsell O, Jansen, J. The Comparative Anatomy and Histology of the
Cerebellum: The Human Cerebellum, Cerebellar Connections, and
Cerebellar Cortex. Minneapolis: University of Minnesota Press, 1972.
Leah J, Menetrey D, dePommery J. Neuropeptides in long ascending
spinal tract cells in the rat: Evidence for parallel processing of ascending information. Neuroscience 1988;24:195–207.
Leblanc A. The Cranial Nerves: Anatomy Imaging Vascularization,
2nd ed. Berlin: Springer-Verlag, 1995.
Lechtenberg R. Synopsis of Neurology. Philadelphia: Lea & Febiger,
1991.
Lehéricy S, Hirsch EC, Cervera-Pierot P, Hersh LB, Bakchine S, Piette
F, Duyckaerts C, Hauw J-J , Javoy-Agid F, Agid Y. Heterogeneity
and selectivity of the degeneration of cholinergic neurons in the
basal forebrain of patients with Alzheimer’s disease. J Comp Neurol 1993;330:15–31.
Ljungdahl A, Hökfelt T, Nilsson G. Distribution of substance P-like
immunoreactivity in the central nervous system of the rat I cell bodies and nerve terminals. Neuroscience 1978;3:861–943.
Lilly R, Cummings JL, Benson DF, Frankel M. The human KlüverBucy syndrome. Neurology 33:1141–1145, 1983.
Lu GW. Spinocervical tract-dorsal column postsynaptic neurons: A
double-projection neuronal system. Somatosens Mot Res 1989;6:
445–454.
Lufkin R, Flannigan BD, Bentson IR, Wilson GH, Rauschning W,
Hanafee W. Magnetic resonance imaging of the brainstem and cranial nerves. Surgical and Radiologic Anatomy 1986;8:49–66.
Magnusson KR, Clements JR, Larson AA, Madl JE, Beitz AJ. Localization of glutamate in trigeminothalamic projection neurons: A
combined retrograde transport-immunohistochemical study. Somatosens Res 1987;4:177–190.
Mai J, Assheurer J, Paxinos G. Atlas of the Human Brain. New York:
Academic Press, 1997.
Martin JH. Neuroanatomy: Text and Atlas, 2nd ed. Stanford, CT: Appleton & Lange, 1996.
Martin TJ, Corbett JJ. Neuro-Ophthalmology: The Requisites of Ophthalmology. St. Louis: Mosby, 2000.
McKusick VA. On the naming of clinical disorders, with particular reference to eponyms. Medicine 77:1–2 (1998a).
McKusick VA. Mendelian Inheritance in Man: A Catalog of Human
Genes and Genetic Disorders. 12th edition. Baltimore: Johns Hopkins University Press, 1998b.
Mihailoff GA. Cerebellar nuclear projections from the basilar pontine
nuclei and nucleus reticularis tegmenti pontis as demonstrated with
PHA-L tracing in the rat. J Comp Neurol 1993;330:130–146.
Miller RA, Burack E. Atlas of the Central Nervous System in Man, 3rd
ed. Baltimore: Williams & Wilkins, 1982.
Monaghan PL, Beitz AJ, Larson AA, Altschuler RA, Madl JE, Mullett
MA. Immunocytochemical localization of glutamate-, glutaminaseand aspartate aminotransferase-like immunoreactivity in the rat
deep cerebellar nuclei. Brain Res 1986;363:364–370.
Montemurro DG, Bruni JE. The Human Brain in Dissection, 2nd ed.
New York: Oxford University Press, 1988.
Nahin RL. Immunocytochemical identification of long ascending, peptidergic lumbar spinal neurons terminating in either the medial or
lateral thalamus in the rat. Brain Res 1988;443:345–349.
Nelson BJ, Mugnaini E. Origins of GABAergic inputs to the inferior olive.
In: P Strata (ed), The Olivocerebellar System in Motor Control. (also
Exp Brain Res Ser 17:86–107). New York: Springer-Verlag, 1989.
Newman DB, Hilleary SK, Ginsberg CY. Nuclear terminations of corticoreticular fiber systems in rats. Brain Behav Evol 1989;34:223–264.
Sources and Suggested Readings 299
Nicholls JG, Martin AR, Wallace BG. From Neuron to Brain, 3rd ed.
Sunderland, MA: Sinauer Associates, Inc, 1992.
Nieuwenhuys R. Chemoarchitecture of the Brain. Berlin: SpringerVerlag, 1985.
Nieuwenhuys R, Voogd J, van Huijzen C. The Central Nervous System: A Synopsis and Atlas, 3rd ed. Berlin: Springer-Verlag, 1988.
Noback CR, Strominger NL, Demarest RJ. The Human Nervous
System, Structure and Function, 5th ed. Baltimore: Williams &
Wilkins, 1996.
Nolte J. The Human Brain: An Introduction to its Functional
Anatomy, 5th ed. St. Louis: CV Mosby, 2002.
Nolte J, Angevine JB. The Human Brain in Photographs and Diagrams.
St. Louis: CV Mosby, 1995.
Nudo RJ, Sutherland DP, Masterton RB. Inter- and intra-laminar distribution of tectospinal neurons in 23 mammals. Brain Behav Evol
1993;42:1–23.
Olszewski J, Baxter D. Cytoarchitecture of the Human Brain Stem,
2nd ed. Basel: S. Karger, 1982.
Pansky B, Allen DJ, Budd GC. Review of Neuroscience, 2nd ed. New
York: Macmillan Publishing Company, 1988.
Parent A. Carpenters Human Neuroanatomy, 9th ed. Baltimore:
Williams & Wilkins, 1996.
Parent A, DeBellefeuille L. The pallidointralaminar and pallidonigral
projections in primate as studied by retrograde double-labeling
method. Brain Res 1983;278:11–27.
Platzer W (ed). Pernkopf Atlas of Topographic and Applied Human
Anatomy, 3rd ed. Volume I, Head and Neck. Baltimore: Urban &
Schwarzenberg, 1989.
Poritsky R. Neuroanatomical Pathways. Philadelphia: WB Saunders
Co, 1984.
Rasmussen AT. Atlas of Cross Section Anatomy of the Brain: Guide to
the Study of the Morphology and Fiber Tracts of the Human Brain.
New York: Blakiston Division, McGraw-Hill Book Company, Inc,
1951.
Reddy VK, Cassini P, Ho RH, Martin GF. Origins and terminations of
bulbospinal axons that contain serotonin and either enkephalin or
substance P in the North American opossum. J Comp Neurol
1990;294:96–108.
Reddy VK, Fung SJ, Zhuo H, Barnes CD. Localization of enkephalinergic neurons in the dorsolateral pontine tegmentum projecting to
the spinal cord of the cat. J Comp Neurol 1990;291:195–202.
Rhoton AL, Jr. The posterior cranial fossa: Microsurgical anatomy and
surgical approaches. Neurosurgery 2000; 47 (Supplement);
S7–S297.
Rhoton AL, Jr. The supratentorial cranial space: Microsurgical
anatomy and surgical approaches. Neurosurgery 2002; 51 (Supplement); S1–S410.
Rosenberg RN. Neurology, Volume 5 of The Science and Practice
of Clinical Medicine, JM Dietschy, Editor-in-Chief. New York:
Grune & Stratton, 1980.
Rowland LP (ed.). Merritt’s Textbook of Neurology, 10th ed. Baltimore: Lippincott Williams & Wilkins, 2000.
Rustioni A. Non-primary afferents to the nucleus gracilis from the
lumbar cord of the cat. Brain Res 1973;51:81–95.
Rustioni A. Non-primary afferents to the cuneate nucleus in the
brachial dorsal funiculus of the cat. Brain Res 1974;75:247–259.
Rustioni A, Kaufman AB. Identification of cells of origin of non-pri-
mary afferents to the dorsal column nuclei of the cat. Exp Brain Res
1977;27:1–14.
Salt TE, Hill RG. Neurotransmitter candidates of somatosensory primary afferent fibres. Neuroscience 1983;10:1083–1103.
Schnitzlein HN, Hartley EW, Murtagh FR, Grundy L, Fargher JT.
Computed Tomography of the Head and Spine: A Photographic
Color Atlas of CT, Gross, and Microscopic Anatomy. Baltimore:
Urban & Schwarzenberg, 1983.
Schnitzlein HN, Murtagh FR. Imaging Anatomy of the Head and Spine:
A Photographic Color Atlas of MRI, CT, Gross, and Microscopic
Anatomy in Axial, Coronal, and Sagittal Planes, 2nd ed. Baltimore:
Urban & Schwarzenberg, 1990.
Schwedtfeger WK, Buhl EH, Germroth P. Disynaptic olfactory input
to the hippocampus mediated by stellate cells in the entorhinal cortex. J Comp Neurol 1990;292:163–177.
Shepherd GM. Neurobiology, 3rd ed. New York: Oxford University
Press, 1994.
Siegel G, Agranoff BW, Albers RW, Molinoff PB. Basic Neurochemistry, Molecular, Cellular, and Medical Aspects, 6th ed. Philadelphia: Lippincott-Raven, 1999.
Singer M, Yakovlev PI. The Human Brain in Sagittal Section. Springfield, IL: Charles C Thomas, 1964.
Smith CG. Serial Dissections of the Human Brain. Baltimore: Urban &
Schwarzenberg, 1981.
Smith Y, Bolam JP. The output neurones and the dopaminergic
neurones of the substantia nigra receive a GABA-containing input
from the globus pallidus in the rat. J Comp Neurol 1990;296:
47–64.
Smith Y, Hazrati L-N, Parent A. Efferent projections of the subthalamic nucleus in the squirrel monkey as studied by the PHA-L
anterograde tracing method. J Comp Neurol 1990;294:
306–323.
Strata P (ed). The Olivocerebellar System in Motor Control. Berlin:
Springer-Verlag, 1989.
Sugiura K, Robinson GA, Stuart DG. Illustrated Guide to the Central
Nervous System. St. Louis: Ishiyaku EuroAmerica, Inc, 1989.
Swash M, Oxburgy J (eds). Clinical Neurology, volumes 1 and 2. New
York: Churchill Livingstone, 1991.
Tatu L, Moulin T, Bogousslavsky J, Duvernoy H. Arterial territories
of human brain: Brainstem and cerebellum. Neurology 1996;47:
1125–1135.
Terzian H, Ore GD. Syndrome of Klüver and Bucy reproduced in man
by bilateral removal of the temporal lobes. Neurology 1955;5:
373–380.
Tieman SB, Butler K, Neale JH. N-acetylaspartylglutamate: A neuropeptide in the human visual system. JAMA 1988;259:2020.
Victor M, Ropper AH. Adams and Victor’s Principles of Neurology,
7th Ed. New York: McGraw-Hill, 2001.
Walker JJ, Bishop GA, Ho RH, King JS. Brainstem origin of serotoninand enkephalin-immunoreactive afferents to the opposum’s cerebellum. J Comp Neurol 1988;276:481–497.
Walton L. Essentials of Neurology, 6th ed. New York: Churchill Livingstone, 1989.
Watson C. Basic Human Neuroanatomy: An Introductory Atlas, 5th
ed. Boston: Little, Brown, 1995.
Waxman SG, deGroot J. Correlative Neuroanatomy, 22nd ed. Norwalk, CT: Appleton & Lange, 1995.
300 Sources and Suggested Readings
Westlund KN, Carlton SM, Zhang D, Willis WD. Glutamate-immunoreactive terminals synapse on primate spinothalamic tract
cells. J Comp Neurol 1992;322:519–527.
Wicke L. Atlas of Radiologic Anatomy, 3rd ed. Baltimore: Urban &
Schwarzenberg, 1982.
Willard FH. Medical Neuroanatomy: A Problem-Oriented Manual
with Annotated Atlas. Philadelphia: JB Lippincott Co, 1993.
Willis WD, Jr. The Pain System: The Neural Basis of Nociceptive
Transmission in the Mammalian Nervous System, Volume 8, Pain
and Headache. Basel: S Karger, 1985.
Willis WD, Grossman RG. Medical Neurobiology: Neuroanatomical
and Neurophysiological Principles Basic to Clinical Neuroscience,
3rd ed. St. Louis: CV Mosby, 1981.
Witelson SF, Kigas DL. Sylvian fissure morphology and asymmetry in
men and women: Bilateral differences in relation to handedness in
men. J Comp Neurol 1992;323:326–340.
Woolsey TA, Hanaway J, Gado MH: The Brain Atlas: A Visual Guide
to the Human Central Nervous System, 2nd Ed. Hoboken: Wiley,
2003.
Yasargil MG. Microneurosurgery I Microsurgical Anatomy of the
Basal Cisterns and Vessels of the Brain, Diagnostic Studies, General Operative Techniques and Pathological Considerations of
the Intracranial Aneurysms. New York: Georg Thieme Verlag,
1984.
Zulegar S, Staubesand J. Atlas of the Central Nervous System in Sectional Planes. Baltimore: Urban & Schwarzenberg, 1977.